Você está na página 1de 208

Química

Química geral e
estequiometria
1
SISTEMA COC DE ENSINO
Direção-Geral: Sandro Bonás
Direção Pedagógica: Zelci C. de Oliveira
Direção Editorial: Roger Trimer
Gerência Pedagódica: Luiz Fernando Duarte
Gerência Editorial: Osvaldo Govone
Gerência Operacional: Danilo Maurin
Gerência de Relacionamento: Danilo Lippi
Ouvidoria: Regina Gimenes
Conselho Editorial: José Tadeu B.
Terra, Luiz Fernando Duarte, Osvaldo
Govone e Zelci C. de Oliveira
PRODUÇÃO EDITORIAL
Autoria: Luiz Eduardo G. Dias
Editoria: Fábio M. Agostinho e Mauricio T. Hayashi
Coordenação editorial: Luzia H. Fávero F. López
Assistente Editorial: George R. Baldim
Projeto gráfico e direção de
arte: Matheus C. Sisdeli
Preparação de originais: Marisa A. dos Santos
e Silva e Sebastião S. Rodrigues Neto
Iconografia e licenciamento de texto:
Cristian N. Zaramella, Marcela Pelizaro
e Paula de Oliveira Quirino.
Diagramação: BFS bureau digital
Ilustração: BFS bureau digital
Revisão: Flávia P. Cruz, Flávio R. Santos,
José S. Lara, Leda G. de Almeida e
Maria Cecília R. D. B. Ribeiro.
Capa: LABCOM comunicação total
Fechamento: Matheus C. Sisdeli

Rua General Celso de Mello Rezende, 301 – Tel.: (16) 3238·6300


CEP 14095-270 – Lagoinha – Ribeirão Preto-SP
www.sistemacoc.com.br
CAPÍTULO 01 PROPRIEDADES DA MATÉRIA 7
Sumário 1.
2.
Introdução
Propriedades da matéria
7
7
3. Estados físicos da matéria 9
4. Mudanças de estado físico 9

CAPÍTULO 02 SUBSTÂNCIAS 10
1. Substância Pura 10
2. Mistura 11
3. Sistemas homogêneos e heterogêneos 14
4. Fenômenos (transformações) 14
5. Separação de misturas heterogêneas 15
6. Separação de misturas homogêneas 17

CAPÍTULO 03 MASSA ATÔMICA 20


1. Unidade de massa atômica 20
2. Massa atômica (MA) 20
3. Massa de um elemento 20
4. Massa molecular 21
5. Constante de Avogadro 21
6. Conceito de mol 22
7. Massa molar 22
8. Número de mols (n) 23

CAPÍTULO 04 GASES 25
1. Introdução 25
2. Transformações gasosas 26
3. Equação geral dos gases 27
4. Hipótese de Avogadro 28
5. Condições normais de temperatura e pressão (CNTP ou TPN) 28
6. Volume molar (Vm) 28
7. Equação de Clapeyron 28
8. Densidade dos gases 29
9. Misturas gasosas 30
10. Pressão parcial 31
11. Difusão e Efusão gasosa 31

CAPÍTULO 05 LEIS PONDERAIS 34


1. Lei de Lavoisier (Lei da conservação das massas) 34
2. Lei de Proust (Lei das proporções fixas ou definidas) 34
CAPÍTULO 06 DETERMINAÇÃO DE FÓRMULAS 36
1. Fórmula percentual ou centesimal 36
2. Fórmula mínima ou empírica 36
3. Fórmula molecular 37
4. Balanceamento das Equações 39

CAPÍTULO 07 CÁLCULO ESTEQUIOMÉTRICO 40


1. Regras para a resolução de problemas de estequiometria 40
2. Formas diferentes de se encarar um mesmo cálculo estequiométrico 41

EXERCÍCIOS PROPOSTOS 49
Capítulo 01 51
Capítulo 02 57
Capítulo 03 88
Capítulo 04 105
Capítulo 05 142
Capítulo 06 148
Capítulo 07 155

GABARITO 177
Teoria
Química geral e estequiometria Química

Capítulo 01 PROPRIEDADES DA MATÉRIA


1. Introdução • Extensão
Todo tipo de material apresenta certas caracte- É o espaço ocupado pelo corpo. Todo corpo
rísticas que definem seu comportamento e suas que possui massa ocupa lugar no espaço.
aplicações. Essas características, assim como as • Impenetrabilidade
temperaturas de fusão e de ebulição e a densi-
É a incapacidade de dois corpos ocuparem o
dade, entre outras, são denominadas proprie-
mesmo lugar no espaço, simultaneamente.
dades da matéria. Essas propriedades podem
sofrer ações externas e, assim, sofrer modifica- • Divisibilidade
ções que alteram seu modo de apresentação. Todos os corpos podem ser divididos em por-
Dessa maneira, todos os compostos existentes ções menores e, por isso, todos os corpos são
podem sofrer transformações (fenômenos). divisíveis (incluindo o átomo!).
A Química, portanto, é a ciência que estuda a • Compressibilidade
constituição da matéria, sua estrutura interna,
as relações entre os diversos tipos de materiais Os corpos possuem a propriedade de poder
encontrados na natureza, além, é claro, da de- diminuir de tamanho sob a ação de uma força
terminação das propriedades desses materiais, externa.
sendo essas propriedades físicas ou químicas. • Elasticidade
Os corpos possuem a propriedade de voltar à
Vá além forma original, cessada a força que o defor-
Chamamos de matéria tudo que possui mou. Além disso, é possível exercer uma força
massa e que ocupa lugar no espaço e pode, capaz de estender seu tamanho.
portanto, de alguma forma ser medido. Por
exemplo: madeira, alumínio, ferro, água, ar B. Propriedades funcionais da matéria
etc. Um corpo é definido como sendo uma Quando um grupo de substâncias apresen-
porção limitada da matéria e objeto é um ta um mesmo conjunto de propriedades,
corpo fabricado para um determinado fim. denomina-se função química. Assim, ob-
Resumidamente, pode-se dizer que: servam-se sempre as mesmas propriedades
Matéria: ferro em todas as substâncias desse grupo. As
Corpo: barra de ferro principais funções são: ácidos, bases, sais e
Objeto: portão de ferro óxidos.

C. Propriedades específicas
da matéria
2. Propriedades da matéria São propriedades que permitem identificar
PV-13-11

uma determinada espécie de matéria, pois


As propriedades que a matéria pode apre- representam características inerentes de cada
sentar são classificadas de três formas dife- tipo de material. Podem-se destacar:
rentes.
• Propriedades físicas
A. Propriedades gerais da matéria Temperatura de fusão (Ponto de fusão – PF)
São as propriedades comuns a toda e qualquer É a temperatura necessária para que uma es-
espécie de matéria, independentemente da trutura na fase sólida passe para a fase líquida.
substância de que ela é constituída. Uma mesma estrutura, à mesma pressão, apre-
• Massa sentará um mesmo valor de temperatura para
Corresponde à quantidade absoluta de maté- passar novamente para a fase sólida.
ria que compõe aquele material. Todos os cor- Sólido ← PF → Líquido
pos possuem massa.

7
Química Química geral e estequiometria

Temperatura de ebulição (ponto de ebuli- Densidade (d)


ção – PE) É a relação entre a massa e o volume ocupa-
É a temperatura necessária para que uma es- do por uma amostra de matéria. Representa a
trutura na fase líquida passe para a fase gasosa. quantidade de matéria que ocupa uma deter-
Da mesma maneira, uma mesma estrutura, à minada unidade de volume, numa dada tem-
mesma pressão, apresentará um mesmo valor peratura e pressão.
de temperatura para passar novamente para a m
fase líquida. d = (g/mL)
V
Líquido ← PE → Gasoso

EXERCÍCIO RESOLVIDO
01. ENEM Com base no texto e na análise realizada pelo
Certas ligas estanho-chumbo com composição técnico, as amostras que atendem às normas
específica formam um eutético simples, o que internacionais são:
significa que uma liga com essas características a. I e II.
se comporta como uma substância pura, com b. I e III.
um ponto de fusão definido, no caso 183 oC. Essa c. II e IV.
é uma temperatura inferior mesmo ao ponto de
fusão dos metais que compõem esta liga (o es- d. III e V.
tanho puro funde a 232 oC e o chumbo puro, e. IV e V.
a 320 oC), o que justifica sua ampla utilização na Resolução
soldagem de componentes eletrônicos, em que
o excesso de aquecimento deve sempre ser evi- Multiplicando-se as porcentagens apresenta-
tado. De acordo com as normas internacionais, das na tabela pelas respectivas densidades dos
os valores mínimo e máximo das densidades materiais, estanho e chumbo, encontram-se as
para essas ligas são de 8,74 g/mL e 8,82 g/mL, densidades das ligas formadas nas proporções
respectivamente. As densidades do estanho e em massa:
do chumbo são 7,3 g/mL e 11,3 g/mL, respec-
tivamente. Densidades das
Amostra % de Sn % de Pb
ligas (g/mL)
Um lote contendo 5 amostras de solda estanho-
-chumbo foi analisado por um técnico, por meio I 0,60 0,40 8,90
da determinação de sua composição percentual II 0,62 0,38 8,82
em massa, cujos resultados estão mostrados no
quadro a seguir. III 0,65 0,35 8,70
PV-13-11

Porcentagem Porcentagem IV 0,63 0,37 8,78


Amostra
de Sn (%) de Pb (%)
V 0,59 0,41 8,94
I 60 40
Assim, pela comparação da última coluna com
II 62 38 o intervalo de densidades permitido pelas
III 65 35 normas internacionais, somente as ligas II e IV
atendem às normas.
IV 63 37
Resposta
V 59 41 C
Disponível em: http://www.eletrica.ufpr.br.

8
Química geral e estequiometria Química

• Propriedades organolépticas todo o volume disponível, podendo ser expandi-


Sabor, aroma, textura das indefinidamente. Também são comprimidas
com grande facilidade. Esse comportamento
Estão relacionadas à capacidade sensorial de pode ser explicado pelas forças de atração entre
identificação de substâncias através dos ór- as partículas que são muito fracas, promovendo,
gãos dos sentidos, como o gosto identificado assim a grande mobilidade dessas partículas.
pela boca, o cheiro aferido pelo nariz etc. Desse modo, apresentarão sempre a forma e o
• Propriedades químicas volume do recipiente que as contém.
Reações químicas
4. Mudanças de estado fí sico
Essas propriedades são associadas à capaci-
Quando o gelo está derretendo ou a água fer-
dade intrínseca que uma determinada subs-
vendo, observamos o fenômeno da mudan-
tância (ou um grupo de substâncias) possui de
ça de estado físico. Todo e qualquer material
interagir com outras através de processos de
pode sofrer a ação desses fenômenos: basta
quebra e formação de ligações químicas, bem
que ocorram as devidas alterações na tempe-
como aos mecanismos necessários para tais
ratura ou na pressão.
transformações e também aos fatores que in-
fluenciam esses mecanismos. Sólido Líquido Gás
Fusão Vaporização
3. Estados fí sicos da matéria
Solidificação Condensação
Macroscopicamente, as substâncias podem
(liquefação)
apresentar-se de diferentes maneiras em relação
ao seu aspecto, à forma de apresentação e até Sublimação
seu volume, dependendo da pressão imprimida
e da temperatura do sistema. Dessa maneira, po- Fusão – é a passagem do estado sólido para
dem ser expostos três estados físicos da matéria. o líquido.
• Estado sólido Solidificação – é a passagem do estado líquido
para o sólido.
As substâncias que estão no estado físico sólido
apresentam forma definida e seu volume não va- Vaporização – é a passagem do estado líquido
ria consideravelmente com variações de pressão para o gasoso. A vaporização pode ocorrer de
e temperatura. Pode-se considerar, portanto, três formas:
que seu volume independe do espaço disponí- Evaporação – é uma vaporização lenta e superfi-
vel pelo recipiente que o contém. As partículas cial do sistema. Pode ocorrer sem aquecimento,
que constituem o estado sólido encontram-se em função da baixa umidade do ar ambiente.
tão intimamente ligadas umas às outras que não
conseguem movimentar-se livremente. Ebulição – é uma vaporização em que todas as
moléculas do sistema estão na temperatura de
PV-13-11

• Estado líquido ebulição, criando uma situação de fervura.


As partículas que constituem o estado líquido Calefação – é uma vaporização intensa, qua-
não estão unidas fortemente, visto que deslizam se que instantânea. Ocorre devido ao fato de
umas sobre as outras, adaptando-se à forma do pouca quantidade de líquido estar sob forte e
recipiente que as contém. Entretanto, essas for- intensa fonte de calor.
ças de atração entre as partículas são suficiente- Condensação – é a passagem do estado gaso-
mente fortes para que não sofram variações no so para o estado líquido. Quando a passagem
volume em decorrência da mudança de recipien- de um gás para o estado líquido ocorre sob
te. Considera-se também que as partículas de forte pressão, ela é denominada liquefação.
um líquido dificilmente podem ser comprimidas.
Sublimação – é a passagem do estado sólido
• Estado gasoso diretamente para o estado gasoso. O processo
As substâncias apresentam densidade muito me- inverso era antigamente denominado ressubli-
nor que a dos sólidos e a dos líquidos, ocupando mação.

9
Química Química geral e estequiometria

CAPÍTULO 02 SUBSTÂNCIAS
Na análise de um determinado material, perce- Quando não há mais fase sólida, a temperatu-
be-se que este apresenta massa, ocupa um de- ra volta a subir e, com aquecimento contínuo,
terminado volume, podendo-se até sentir sua a temperatura continua a subir até que o líqui-
textura. É possível perceber seu cheiro, seu bri- do chegue a 100 oC, momento no qual o líqui-
lho e também sua cor. Sabe-se que a avaliação do começa a ferver (novamente, um patamar
dessas características só é possível em estrutu- constante de temperatura – linha horizontal
ras macroscópicas (vistas a olho nu ou não). Isso no gráfico). Durante todo o processo de ebu-
quer dizer que, quando são analisados sistemas lição, a temperatura permanece constante em
macroscópicos, deve-se crer que nesse sistema 100 oC, só voltando a subir quando não houver
existe uma quantidade muito grande de estru- mais fase líquida.
turas elementares que, para a constituição de Temperatura (°C)
um corpo, são as moléculas. Assim, como cada

a
corpo (porção definida da matéria) precisa ne-

gu

cessariamente ser constituído por uma quan-

rd
po
tidade muito grande de moléculas, define-se

Va
Ebulição (cte.)
substância como o agrupamento de moléculas, 100 L+V
podendo essas serem iguais ou diferentes.

ida
líqu
Substância → Agrupamento de moléculas

ua
Ág
Fusão (cte.)
Dessa forma, um corpo sempre será constituí- 0 S+L
do por substâncias!
lo
Ge

1. Substância pura Tempo (minutos)


É definida como todo sistema constituído por Se uma substância é encontrada em um único
moléculas iguais. Assim, qualquer amostra de estado físico, o sistema é homogêneo. Contudo,
uma substância pura analisada apresenta valo- nos momentos de mudança de estado físico (em
res constantes para suas temperaturas de fusão que a temperatura é constante), percebe-se que
e de ebulição e também para a sua densidade e o sistema é heterogêneo.
composição química. Com isso, à pressão cons-
tante, as mudanças de estado físico para uma Exemplo
substância pura ocorrem sem alteração da tem- O sistema água é substância pura, pois possui PF
peratura durante a transformação. e PE constantes. É homogêneo, pois apresenta as
mesmas propriedades em toda a sua extensão.
Acompanhe o processo de mudança de esta- No entanto, se for encontrada em mais de um
do a partir de um cubo de gelo retirado de um estado físico (água e gelo, por exemplo, já que
PV-13-11

congelador, ao nível do mar: gelo e água possuem superfície de separação,


Inicialmente, nota-se que a temperatura está o que indica mudança de fase), o sistema será
abaixo de 0 oC. Em seguida, observa-se que o heterogêneo.
cubo de gelo começa a receber calor do am-
biente, aquecendo-se até que a temperatura A. Substância pura simples
atinja 0 oC. Nesse momento, começa a ocorrer Nessa classificação, além de as moléculas
o processo de fusão, ou seja, surgem as pri- serem iguais, todos os átomos também serão
meiras gotinhas de fase líquida. Observando o iguais, não podendo, portanto, haver decom-
termômetro, nota-se que a temperatura per- posição em outras espécies de matéria.
manece em 0 oC até que todo o material sóli-
do se converta em líquido (patamar constante Exemplos
de temperatura – linha horizontal no gráfico). H2, N2, O2 etc.

10
Química geral e estequiometria Química

A.1. Alotropia b. Oxigênio


Alotropia é a propriedade na qual um mesmo oxigênio comum (O2 )
elemento químico pode formar substâncias pu- 
ras simples diferentes. Constituem exemplos de ozônio (O3 )
elementos que apresentam variedades alotrópi- A diferença entre O2 e O3 está na atomicidade.
cas: o carbono, o oxigênio, o fósforo e o enxofre. O2 é menos energético que O3, logo é mais
Os alótropos possuem as mesmas proprie- estável.
dades químicas entre si, ou seja, reagem da c. Fósforo
mesma forma com os mesmos reagentes, fósforo vermelho (Pn )
porém, por apresentarem estrutura ou ato- 
micidade distintas, suas propriedades físicas (mais estável)
serão diferentes. fósforo branco (P )
 4

Assim, as variedades alotrópicas podem diferir d. Enxofre


umas das outras em relação ao número de áto-
enxofre α ou rômbico (S8 )
mos constituintes – alótropos por atomicidade 
– ou em relação à organização dos átomos em (mais estável)
sua estrutura – alótropos estruturais. enxofre β ou monoclínico (S )
 8
Exemplos:
A.2. Substância pura composta
a. Carbono
Apesar de conter átomos diferentes em sua
grafite (Cn )
 constituição, as moléculas são todas iguais e
diamante (Cn ) podem ser decompostas, dando origem a ou-
n = número muito grande e indeterminado tras espécies mais simples de matéria. Assim,
por exemplo, a água pode ser decomposta em
A diferença entre Cgr e Cd está no arranjo cris- gás hidrogênio e gás oxigênio.
talino.
Exemplos
H2O, C6H12O6, NH3 etc.

2. Mistura
Quando um sistema é constituído por molé-
culas diferentes (vários componentes), temos
C = 0,67 nm

uma mistura que, ao ser analisada, sua tempe-


ratura de fusão e/ou ebulição e sua composi-
ção, além da densidade, apresentam-se como
propriedades variáveis. Com isso, as transfor-
mações de estado físico para uma mistura, à
PV-13-11

Grafite (C(graf.)) pressão constante, ocorrem com o valor da


temperatura durante a mudança de estado so-
Camadas hexagonais sobrepostas
frendo alteração em função do tempo.
Temperatura (°C)

Fim da Vapor
ebulição
Início da
ebulição Intervalo da ebulição
Líquido L+V
Diamante (C(diam.)) Fim da
fusão
Distribuição tetraédrica na forma de treliças Início Intervalo da fusão
da fusão Sólido S+L
O Cgr é menos energético que o Cd, logo é mais
estável. Tempo (minutos)

11
Química Química geral e estequiometria

Conclui-se, então, que tanto a fusão/solidifica- Outros exemplos


ção quanto a ebulição/condensação ocorrem Água e areia; água, areia e óleo; granito etc.
ao longo de um intervalo de temperatura.
Mistura 2 fases Mistura de 3 fases
A.1. Mistura homogênea (solução) (bifásico) (trifásico)
Caracteriza-se por apresentar-se visualmente Granito
uniforme. Essas misturas apresentam as mes- Água Óleo
mas propriedades físicas e a mesma composi- Água
ção química em toda a sua extensão. Apresen- Areia Areia
tam um aspecto visual único (uma única fase).
Exemplo
Observação
O sistema água + sal dissolvido é mistura, pois
a temperatura varia durante a fusão e a ebu- Existem misturas que, durante uma das mu-
lição. É homogêneo, pois apresenta as mes- danças de estado, se comportam como subs-
mas propriedades em toda a sua extensão. O tâncias puras. São elas as misturas eutéticas e
mesmo ocorre para as misturas água e álcool e azeotrópicas.
ouro 18 k (75% de ouro e 25% de cobre). a. Mistura eutética: mistura que apresen-
ta temperatura de fusão constante e
temperatura de ebulição variável. Exem-
Aliança plo: algumas ligas metálicas, dentre elas
Água Água de ouro
+ + a solda usada em eletrônica (37% de
18 k
Álcool Sal chumbo e 63% de estanho).
dissolvido Temperatura
Mistura
A.2. Mistura heterogênea eutética
Caracteriza-se por apresentar superfície de Ebulição
separação. Elas não apresentam as mesmas
propriedades em toda a sua extensão, carac- PF Fusão
terizando duas ou mais fases. constante
Exemplo
O sistema água + gelo + limalha de ferro é uma Tempo
mistura, sendo heterogêneo devido à mu- b. Mistura azeotrópica: mistura que apre-
dança de propriedades entre as fases “água”, senta temperatura de fusão variável e
“gelo” e “limalha”. temperatura de ebulição constante.
Exemplo: água e álcool na proporção
de 4% de água e 96% de álcool (álcool
PV-13-11

Gelo 96 °GL).
Água Temperatura
Mistura
Limalha de ferro PE azeotrópica
constante Ebulição
Saiba que componente é cada substância (tipo
de molécula) participante da mistura. Fusão

Saiba que fase é cada extensão do sistema que


apresenta as mesmas propriedades.
Tempo
Note que, em uma mistura, o número de fases Comparando-se as características de substân-
não é necessariamente igual ao número de cias puras e misturas, pode-se construir a se-
componentes. guinte tabela:

12
Química geral e estequiometria Química

Substância Mistura Mistura


Mistura
pura eutética azeutrópica

Exemplo Água Água e Sal Solda Álcool 96%

Temperatura durante a fusão Constante Varia Constante Varia

Temperatura durante a ebulição Constante Varia Varia Constante

EXERCÍCIO RESOLVIDO
01. ENEM Resolução
Em nosso cotidiano, utilizamos as palavras “ca- Pelo senso comum, conforme citado no texto,
lor” e “temperatura” de forma diferente de recebendo calor, a água deveria sempre elevar
como elas são usadas no meio científico. Na sua temperatura, fato esse que não se verifica
linguagem corrente, calor é identificado como na fervura da água.
“algo quente” e temperatura mede a “quanti- Resposta
dade de calor de um corpo”. Esses significados,
no entanto, não conseguem explicar diversas A
situações que podem ser verificadas na prática.
Do ponto de vista científico, que situação prá-
tica mostra a limitação dos conceitos corri-
queiros de calor e temperatura?
a. A temperatura da água pode ficar cons-
tante durante o tempo em que estiver
fervendo.
b. Uma mãe coloca a mão na água da ba-
nheira do bebê para verificar a tempe-
ratura da água.
c. A chama de um fogão pode ser usada
para aumentar a temperatura da água
em uma panela.
d. A água quente que está em uma caneca
é passada para outra caneca a fim de
diminuir sua temperatura.
PV-13-11

e. Um forno pode fornecer calor para


uma vasilha de água que está em seu
interior com menor temperatura do
que a dele.

13
Química Química geral e estequiometria

3. Sistemas homogêneos e • O sistema água e gelo é classificado


heterogêneos como substância pura, pois é constituí-
do pelo mesmo tipo de molécula. Toda-
Sistema é definido como uma porção da ma- via, apresenta superfície de separação,
téria considerada como o universo específico sendo considerado como sistema hete-
para análise, sendo então submetida a estudo rogêneo, pois não apresenta as mesmas
(investigação), podendo ser constituído por propriedades em toda a sua extensão (o
uma única substância (pura) ou por várias ao gelo flutua, devido ao fato de apresentar
mesmo tempo (mistura). menor densidade que sua fase líquida).
A. Sistema homogêneo • O sistema água e óleo é classificado
como mistura, pois é constituído por
É visualmente uniforme em toda a sua exten- tipos diferentes de substâncias, sen-
são, não apresentando superfície de separação. do considerado heterogêneo, pois não
É constituído por uma única fase (encontram-se apresenta as mesmas propriedades em
as mesmas propriedades em todos os pontos da toda a sua extensão. Nesse sistema, é
extensão do volume). observada superfície de separação.
 − substância pura
Homogêneos  4. Fenômenos (transformações)
 − mistura hom ogênea
São os processos em que se promove uma
B. Sistema heterogêneo alteração nas condições de apresentação de
Um sistema heterogêneo não apresenta uni- uma substância ou mesmo nas suas carac-
formidade visual, caracterizando-se por apre- terísticas particulares, podendo ou não ser
sentar superfície de separação. Dessa forma, reversíveis.
possui mais de uma fase (encontram-se duas
ou mais propriedades em todos os pontos da A. Fenômenos físicos
extensão do volume). São os processos de transformação da matéria
 − substância pura em nos quais é alterada apenas a forma de apre-
 sentação do material, sem alterar sua identi-
Heterogêneos  mudança de fase dade química. Isso quer dizer que não ocorrem
 − mistura h eterogênea quebras de ligações químicas entre os átomos.

Exemplos Exemplos
• O sistema água pura é classificado Mudanças de estado físico, expansão volumé-
como substância pura, pois, além de trica etc.
ser constituído pelo mesmo tipo de
molécula, apresenta propriedades físi-
cas constantes, como TE e TF. É consi-
derado homogêneo, pois apresenta as
PV-13-11

mesmas propriedades em toda a sua


extensão, não sendo observada ne-
nhuma superfície de separação. Barra de ouro Fio de ouro
• O sistema água e sal é classificado como
mistura, pois é constituído por tipos di-
ferentes de substâncias, apresentando B. Fenômenos químicos
propriedades físicas variáveis, como TE Nesse caso, a identidade química da substân-
e TF. É considerado homogêneo, pois cia sofre radical alteração, em virtude da que-
apresenta as mesmas propriedades em bra das ligações do material e da consequen-
toda a sua extensão, não sendo obser- te formação de novas ligações químicas. São
vada nenhuma superfície de separação. também chamados de reações químicas.

14
Química geral e estequiometria Química

Exemplos
Cozimento dos alimentos, queima de combus- Sifão
tíveis, ferrugem etc.

Líquido
Queima +
Líquido
Papel Cinza

Contato com Líquido


o ar e a
umidade B. Flotação
É um processo usado para separar as fases da
mistura entre dois sólidos. Consiste em adicio-
nar à mistura a ser desdobrada um líquido que
não reaja e não dissolva nenhum dos compo-
nentes e, mais importante, que apresente
Esponja de aço Ferrugem
densidade intermediária a deles. A fase de me-
nor densidade flutua e a de maior densidade
5. Separação de misturas sedimenta.
heterogêneas
Introdução
Análise imediata é o conjunto de processos me- Água
cânicos utilizados na separação das fases e/ou
dos componentes de uma mistura sem alteração Areia Serragem
das propriedades químicas originais. Dentre os +
Água
processos de separação, os mais usados são: serragem
Areia
A. Sifonação
É um processo usado para separar as fases da C. Levigação
mistura heterogênea sólido/líquido e líquido/
Processo usado para separar as fases da mis-
líquido.
PV-13-11

tura entre dois sólidos. É utilizado quando dois


componentes da mistura apresentam densida-
Sifão des diferentes, sendo que o de menor densi-
dade pode ser arrastado por uma corrente de
líquido. É utilizado em garimpos para separar o
ouro do cascalho.

D. Decantação
Líquido É o processo usado para separar as fases de
+ misturas heterogêneas pela ação da gravida-
sólido de, já que as fases apresentam densidades di-
ferentes.

Líquido

15
Química Química geral e estequiometria

Sólido e líquido Líquido e líquido


Exemplo Exemplo
Mistura água e areia Mistura água e óleo
Para separar (decantar) dois ou mais líquidos
imiscíveis de densidades diferentes, utiliza-se
o funil de decantação ou funil de bromo, ou
ainda funil de separação. A mistura é deixada
em repouso dentro do funil. O líquido mais
denso fica embaixo. Em seguida, a torneira
Ao deixar a mistura heterogênea sólido-líqui- é aberta, deixando escoar o líquido de maior
do em repouso, lentamente o componente densidade. Quando a superfície de separação
sólido (mais denso), pela ação da gravidade, atinge a torneira, esta é fechada, separando
deposita-se no fundo do recipiente, ocorren- assim as duas fases.
do a sedimentação. Quando a sedimentação
do componente sólido se completa, inclina-se Suporte Funil de decantação
o recipiente para escoar a fase líquida. (funil de bromo)

Óleo
(menos denso)
Água
(mais denso)

Béquer

Água
Observação
Para acelerar a sedimentação, faz-se uso de
uma centrífuga. Em laboratórios clínicos, a
parte sólida do sangue (hemácias, plaquetas e E. Filtração
glóbulos brancos) é separada da parte líquida É o processo de separação das fases de uma
(soro ou plasma) por meio de centrífugas. Nela, mistura heterogênea (sólido-líquido ou sólido-
o sangue gira em alta velocidade e o material -gasoso) por meio de uma superfície porosa
mais denso se deposita rapidamente. denominada filtro. Esse retém a fase sólida em
sua superfície, permitindo a passagem somen-
te da fase líquida ou gasosa.
PV-13-11

Filtração simples – sólido e gasoso


A mistura é lançada sobre um filtro (por suc-
ção), o qual permite somente a passagem do
componente gasoso.

Método de centrifugação

16
Química geral e estequiometria Química

Filtração simples – sólido e líquido a mistura com um solvente que dissolva ape-
nas um dos componentes. Em seguida, filtra-se
Exemplo
e, por evaporação do solvente, recupera-se o
Água e areia componente sólido dissolvido.
A areia fica retida no papel de filtro e é deno- Exemplo
minada resíduo. A água que atravessa o filtro
Mistura de sal e areia
é o filtrado.
• Dissolução – solubilização do sal
Bastão • Filtração – isolamento da areia
• Vaporização – isolamento do sal

G. Separação magnéti ca
É o processo utilizado para separar misturas
sólido-sólido, quando um dos componentes é
Resíduo atraído por um ímã.
Funil Exemplo
Mistura de limalha de ferro e enxofre
Béquer
ímã
Filtrado

Limalha
Filtração à pressão reduzida ou de ferro
fi ltração a vácuo – sólido e líquido
É utilizada para acelerar o processo de filtra- Mistura
ção quando a mistura sólido-líquido é muito limalha de Enxofre
pastosa (como é o caso da mistura de água e ferro e enxofre
farinha de trigo) ou quando o líquido tem alta
viscosidade. 6. Separação de misturas
Funil de homogêneas
Büchner
A. Desti lação
É o método de separação dos componentes
Kitassato de uma mistura homogênea, constituída por
sólido e líquido ou por dois ou mais líquidos.
PV-13-11

A trompa d´água (ou uma bomba de vácuo) Desti lação simples – sólido e líquido
produz rarefação do ar no interior do kitassato, Processo de separação utilizado para separar
diminuindo a pressão interna, fazendo com que os componentes de uma mistura homogênea
o líquido do funil de Büchner seja sugado, atra- constituída de um sólido e um líquido.
vessando rapidamente o papel de filtro, acele-
rando assim a filtração. Exemplo
Mistura água e sal
F. Dissolução fracionada – sólido
A solução entra em ebulição no balão, mas so-
e sólido mente o líquido se vaporiza e caminha pelo con-
É um processo de separação utilizado para se- densador. Ao entrar em contato com as paredes
parar dois ou mais sólidos. Consiste em tratar frias, condensa-se, voltando ao estado líquido.

17
Química Química geral e estequiometria

Termômetro fácil será sua separação. Para aumentar o grau


Saída de água de pureza do destilado, utilizamos a coluna de
Condensador fracionamento. No topo da coluna sai o líqui-
do mais volátil (menor ponto de ebulição) com
Balão de alto grau de pureza, enquanto o menos volátil
destilação
Mistura
Componente condensa em suas paredes.
já destilado
a ser Termômetro
destilada
(água e sal) Coluna de
Entrada fracionamento
de água Condensador

Saída
de água
Desti lação fracionada – líquido e Balão Entrada
líquido Mistura de água
É utilizada para separar os componentes de a ser Líquido mais
uma mistura homogênea constituída por dois destilada Fonte volátil que já
ou mais líquidos. Por aquecimento da solução, (água e éter) de calor foi destilado
os líquidos vão se destilando à medida que va-
porizam. Quanto maior for a diferença entre A destilação fracionada é o método utilizado
os pontos de ebulição dos componentes, mais na destilação do petróleo.

Gás
combustível

Gasolina de
aviação
Gasolina
comum
A temperatura aumenta

Querosene

Óleo diesel
Petróleo Óleo
bruto combustível
Óleos
PV-13-11

lubrificantes

Parafinas

Forno de Asfalto
vaporização
do petróleo

18
Química geral e estequiometria Química

B. Liquefação fracionada – gasoso C. Extração


e gasoso É utilizada para separar os componentes de
Para separar os componentes de uma mistura uma mistura homogênea ou heterogênea. A
gasosa (ar atmosférico, por exemplo), realiza-se separação ocorre em função da diferença de
a liquefação da mistura gasosa (por diminui- solubilidade, em um determinado líquido, dos
ção da temperatura e aumento da pressão) componentes da mistura. É utilizada para a ex-
e, em seguida, destila-se. O componente de tração da clorofila nos vegetais, para separar
menor ponto de ebulição será destilado em substâncias oleosas na indústria de perfumes
primeiro lugar. (extração de essências) etc.

EXERCÍCIO RESOLVIDO
02. ENEM Resolução
Em visita a uma usina sucroalcooleira, um A etapa I corresponde à separação magnética,
grupo de alunos pôde observar a série de pro- na qual são removidos os objetos metálicos; a
cessos de beneficiamento da cana-de-açúcar, etapa II corresponde à extração do caldo açu-
entre os quais se destacam: carado; a etapa III corresponde à filtração, na
I. A cana chega cortada da lavoura por qual resíduos sólidos são separados da fase
meio de caminhões e é despejada em líquida.
mesas alimentadoras que a conduzem Resposta
para as moendas. Antes de ser esmaga-
C
da para a retirada do caldo açucarado,
toda a cana é transportada por esteiras
e passada por um eletroímã para a reti-
rada de materiais metálicos.
II. Após se esmagar a cana, o bagaço se-
gue para as caldeiras, que geram vapor
e energia para toda a usina.
III. O caldo primário, resultante do esma-
gamento, é passado por filtros e sofre
tratamento para transformar-se em
açúcar refinado e etanol.
Com base nos destaques da observação dos
alunos, quais operações físicas de separação
de materiais foram realizadas nas etapas de
PV-13-11

beneficiamento da cana-de-açúcar?
a. Separação mecânica, extração, decan-
tação
b. Separação magnética, combustão, fil-
tração
c. Separação magnética, extração, filtra-
ção
d. Imantação, combustão, peneiração
e. Imantação, destilação, filtração

19
Química Química geral e estequiometria

CAPÍTULO 03 MASSA ATÔMICA


1. Unidade de massa atômica Uma unidade de massa atômica (1 u), chama-
da de massa padrão, surgiu das análises rela-
Como determinar a massa de uma estrutura
tivas entre os átomos de todos os elementos
tão pequena como a do átomo?
com o átomo de 12C. Com isso, determinou-se
No dia a dia, além do hábito, temos a neces- que o átomo com a menor relação numérica
sidade de saber a medida das coisas. Quanto com o padrão de massa pertencia ao elemento
tempo demora para assar um frango, qual a hidrogênio, apresentando uma massa corres-
distância da cidade vizinha ou mesmo qual a pondente a 1/12 da massa de um átomo do
massa de um objeto. Nos três casos, não obte- isótopo 12 do carbono.
mos respostas absolutas, pois todas as nossas
Portanto:
medidas são relativas, ou seja, as respostas
são dadas sempre em relação a um referencial dividido 1u
predeterminado de tempo, distância e massa. em (unidade
As medidas são realizadas dessa forma para de massa)
12 partes
que possamos fazer comparações entre even- iguais
tos diferentes. 1 do átomo de 12C
12
Em geral, mesmo para uma mesma medida,
dependendo das dimensões, fazemos o uso O valor em gramas de 1 u (1/12 do 12C) é de
de adequações. Assim, para medir a massa 1,66 · 10–24 g, o que corresponde aproximada-
do corpo de uma pessoa, utiliza-se o quilogra- mente à massa de um próton ou de um nêutron.
ma (kg) (se você diz pesar 65 kg, isto significa
que você é 65 vezes mais pesado que a unida- 2. Massa atômica (MA)
de escolhida – 1 kg). O quilograma (kg) é uma
unidade prática, mas nem sempre é adequada Massa atômica é o número que indica quantas
para determinadas situações, como para in- vezes a massa de um átomo de um determina-
dicar a massa de um grão de areia, em que o do elemento é mais pesada que 1 u, ou seja,
padrão conveniente seria o miligrama (mg), ou mais pesado que 1/12 do átomo de 12C.
mesmo de um navio, em que tonelada (ton) Comparando-se a massa de um átomo de
seria o padrão mais adequado, mas nenhum um determinado elemento com a unidade
desses padrões citados seria utilizado para de massa atômica (1u), obtém-se a massa
medir a massa de um átomo. desse átomo. Portanto, a massa atômica é
Ainda assim, mesmo que seja a determina- uma análise relativa, não representando de
ção de algo tão pequeno como a estrutura forma absoluta essa dimensão.
atômica, faz-se necessária a definição de um Exemplo
padrão de massa. Como os átomos individu-
PV-13-11

Quando dizemos que a massa atômica do áto-


ais são muito pequenos para serem vistos e,
mo de 32S é igual a 32 u, concluímos que:
consequentemente, muito menos pesados,
podemos determinar o peso de um átomo – a massa atômica de um átomo de 32S é igual
comparando-o com o de um átomo de outro a 32 u;
elemento utilizado como padrão. – a massa atômica de um átomo de 32S é igual
Em 1961, na Conferência da União Interna- a 32 vezes a massa de 1/12 do átomo de 12C;
cional de Química Pura e Aplicada (IUPAC), – a massa de um átomo de 32S é igual a 2,7
adotou-se como padrão de massas atômicas vezes a massa de um átomo de 12C.
o isótopo 12 do elemento carbono (12C), ao
qual se convencionou atribuir o valor exato 3. Massa de um elemento
de 12 unidades de massa atômica. Portan-
to, considera-se o isótopo 12 do elemento C Um elemento químico é definido como o con-
como o padrão de massa. junto de átomos que apresentam o mesmo nú-

20
Química geral e estequiometria Química

mero atômico (mesmo número de prótons no Exemplo


núcleo), mas não necessariamente a mesma Quando dizemos que a massa molecular da
massa. Em função disso, a maioria dos elemen- água, H2O, é 18 u, concluímos que:
tos apresenta isótopos. O cloro, por exemplo, é
constituído por uma mistura de 2 isótopos de • a massa de uma molécula de H2O é
massas atômicas, respectivamente, 35 e 37. Eis igual a 18 u;
os isótopos do cloro e suas respectivas abun- • a massa de uma molécula de H2O é 18
dâncias na natureza: vezes mais pesada que 1/12 do átomo
de carbono-12;
 35 Cl → 75%
Elemento cloro  17 • a massa de uma molécula de água é 1,5
 17 Cl → 25%
37
vez mais pesada que um átomo de car-
bono-12.
A massa atômica do cloro é calculada utilizan-
do-se a média ponderada das massas isotópi- Observação
cas: Massa-fórmula – Para um composto iônico,
como a fórmula representa a proporção entre
35 ⋅ 75 + 37 ⋅ 25
MA Cl = = 35, 50 u cátions e ânions presentes, e não suas quanti-
100 dades absolutas, a massa é denominada massa-
Portanto, a massa atômica de um elemento é -fórmula. Assim, estaremos apenas calculando a
a média ponderada das massas atômicas dos massa da menor proporção quantitativa dos íons
isótopos naturais desse elemento. presentes na estrutura.
Sendo assim, a massa atômica de um elemen- Exemplo
to hipotético A, constituído dos isótopos natu- Quando dizemos que a massa-fórmula do clo-
rais A1, A2, ..., An, pode ser calculada por: reto de sódio, NaCl, é 58,5 u, concluímos que:
• a massa da menor proporção dos íons
A1 ⋅ %1 + A2 ⋅ %2 + An ⋅ %n presentes na estrutura do NaCl é igual
MA A =
100 a 58,5 u;
• a massa da menor proporção dos íons
presentes na estrutura do NaCl é 58,5
4. Massa molecular vezes mais pesada que 1/12 do átomo
de carbono-12;
Os átomos reúnem-se para formar moléculas
e, como as moléculas são caracterizadas por • a massa da menor proporção dos íons
uma quantidade definida de átomos, a massa presentes na estrutura do NaCl é 4,875
dessas moléculas é a soma das massas atômi- vezes mais pesada que um átomo de
cas dos átomos constituintes. carbono-12.
Como essas moléculas são formadas por um
5. Constante de Avogadro
PV-13-11

grupo de átomos ligados entre si, o padrão


usado como base para relacionar as massas Para que fosse possível trabalhar macroscopi-
dessas moléculas é o mesmo usado para os camente com certa quantidade de átomos e
átomos: a unidade de massa atômica (u). conseguir relacioná-la com a massa correspon-
dente, muitos cientistas se empenharam em
Portanto:
determinar um valor adequado. Sabia-se que,
• massa molecular é a soma das massas para que tivesse praticidade, a quantidade de
atômicas dos átomos que constituem a átomos envolvidos deveria ser muito, muito
molécula, grande (em função do limite extremo de seu
ou ainda, pequeno tamanho).
• massa molecular é o número que indica O valor encontrado para essa quantidade ficou
quantas vezes a massa de uma molécu- conhecido como Número de Avogadro (N), e é
la é mais pesada que 1 u, ou seja, que igual a: 6,02 · 1023 (equivalente a 600 trilhões
1/12 do átomo de 12C. de bilhões de alguma coisa).

21
Química Química geral e estequiometria

Esse valor foi aceito porque relaciona uma 7. Massa molar


massa macroscópica numericamente igual à
massa atômica relativa de cada átomo. Assim: A. Massa molar de um elemento
• se 1 átomo de enxofre tem massa atô- A massa molar de um elemento é a massa ex-
mica 32 u, significa que ele é 32 ve- pressa em gramas de 1 mol de átomos, ou seja,
zes mais pesado que a massa padrão 6,02 · 1023 átomos desse elemento. A massa
(1/12 do átomo de carbono-12); molar de um elemento é numericamente igual
• então, para 6,02 · 1023 átomos de en- à sua massa atômica.
xofre numa balança, a massa aferida Exemplo
será de 32 g, ou seja, a constante de
Avogadro corresponde ao número de AI (MA = 27 u)
1 mol de átomos de AI
átomos encontrados na massa atômi-
ca expressa em gramas de qualquer
elemento. contém pesa

Exemplos correspondem
6,02 · 1023 átomos 27 g
à massa de
12 g C (MAC = 12 u) contêm 6,02 · 1023 átomos
de C.
27 g Al (MAAl = 27 u) contêm 6,02 · 1023 átomos B. Massa molar de uma substância
de Al. A massa molar de uma substância é a massa
56 g Fe (MAFe = 56 u) contêm 6,02 · 10 áto-
23 em gramas de 1 mol de moléculas da referida
mos de Fe. substância. A massa molar de uma substância
é numericamente igual à sua massa molecular
expressa em gramas.
6. Conceito de mol
Exemplos
De acordo com a União Internacional de
Química Pura e Aplicada (IUPAC), mol é a 01. CO2 (MAc = 12 u; MAo = 16 u)
quantidade de matéria de um sistema que MM = 1 · 12 + 2 · 16
contém tantas entidades elementares quan- MM = 12 + 32 = 44 u
tos são os átomos contidos em 0,012 kg de 1 mol de
carbono-12. moléculas de CO2
As entidades elementares podem ser átomos,
moléculas, íons, elétrons etc. contém pesa
Unindo o conceito de mol com o número de correspondem
Avogadro, temos: 6,02 · 1023 moléculas 44 g
à massa de
1 mol contém 6,02 · 1023 partículas. 02. NaCl (MANa = 23 u; MACl = 35,5 u)
PV-13-11

MF = 1 · 23 + 1 · 35,5
Exemplos
MF = 23 + 35,5 = 58,5 u
1 mol de laranjas → 6,02 · 1023 laranjas
1 mol de
1 mol de moedas → 6,02 · 1023 moedas fórmulas de NaCl
1 mol de moléculas → 6,02 · 1023 moléculas
contém pesa
1 mol de átomos → 6,02 · 1023 átomos
correspondem
1 mol de íons → 6,02 · 1023 íons 6,02 · 1023 fórmulas 58,5 g
à massa de
1 mol de elétrons → 6,02 · 1023 elétrons

22
Química geral e estequiometria Química

C. Massa molar de um íon 1 mol de átomos de Fe  28 g


A massa molar de um íon é a massa de um n  280 g
mol de íons em gramas, que é numericamente 28 · n = 1 · 280
igual à massa do íon expressa em gramas.
n = 280/28 → n = 10 mol de átomos de Fe
Lembre-se de que a massa do elétron pode ser
considerada desprezível, por isso não há mo- Isso significa que, nessa amostra, estão pre-
dificação no mecanismo de cálculo em função sentes 10 vezes a quantidade de 6,02 · 1023
da entrada (ânions) ou da saída (cátions) de átomos de ferro, ou seja, 6,02 · 1024 átomos
elétrons. de ferro.
Exemplo Exemplo 2
CO2- Quantos mols de moléculas correspondem a
3 (MAC = 12 u; MAO = 16 u)
22 g de moléculas de CO2? (Dado: MAC = 12 u;
MM = 1 · 12 + 3 · 16 MAO = 16 u)
MM = 12 + 48 = 60 u 1 mol de moléculas de CO2  44 g
1 mol de n  22 g
íons CO 23−
44 · n = 1 · 22
contém pesa n = 22/44 → n = 0,5 mol de moléculas de CO2
correspondem Isso significa que, nessa amostra, está presen-
6,02 · 1023 íons 60 g
à massa de te a metade da quantidade correspondente a
6,02 · 1023 moléculas de CO2, ou seja, 3,02 · 1023
moléculas de CO2.
8. Número de mols (n)
Generalizando, pode-se construir uma equa-
A determinação do número de mols (quanti- ção para cálculos relativos ao número de mols,
dade de matéria ou quantidade em mols) de dado por n:
um determinado corpo serve para se estabele-
cer uma análise referencial a respeito do quan- m
to de partículas (átomos, moléculas ou íons) n=
M
existe nesse mesmo corpo comparada à quan-
tidade 1 mol, ou seja, 6,02 · 1023 partículas.
Acompanhe os exemplos. em que:
Exemplo 1 • n = número de mols (em mol)
• m = massa (em gramas)
Quantos mols de átomos correspondem a 280 g
de átomos de ferro? (Dado: MAFe = 56 u) • M = massa molar (em gramas/mol)
PV-13-11

23
Química Química geral e estequiometria

EXERCÍCIO RESOLVIDO
01. Fuvest-SP Resolução
O aspartame, um adoçante artificial, pode ser m
Aspartame: n = ⇒
utilizado para substituir o açúcar de cana. Bas- M
tam 42 miligramas de aspartame para produzir 42 ⋅ 10 −3
a mesma sensação de doçura que 6,8 gramas ⇒n= = 1, 4 ⋅ 10 −4 mol
300
de açúcar de cana. Sendo assim, quantas ve-
zes, aproximadamente, o número de molécu- m 6, 8
Açúcar: n = ⇒ n = = 2, 0 ⋅ 10 −2 mol
las de açúcar de cana deve ser maior do que o M 340
número de moléculas de aspartame para que nAçúcar 2, 0 ⋅ 10 −2
tenha o mesmo efeito sobre o paladar? Portanto ∴ = ≅ 140
nAspartame 1, 4 ⋅ 10 −4
Dados:
Resposta
Massas molares aproximadas (g/mol)
D
açúcar de cana: 340
adoçante artificial: 300
a. 30
b. 50
c. 100
d. 140
e. 200

PV-13-11

24
Química geral e estequiometria Química

CAPÍTULO 04 GASES
1. Introdução O estado em que se apresenta um gás, sob o
ponto de vista macroscópico, é caracterizado
Uma substância no estado gasoso apresenta
por três variáveis: pressão, volume e tempera-
características muito sensíveis para sua condi-
tura. São denominadas variáveis de estado de
ção, quando comparadas às características en-
um gás, por serem as condições de um sistema
contradas nos outros estados físicos, os quais
gasoso que podem sofrer alteração em seus
possuem análises mais precisas e determina-
valores. Assim, caracterizam-se como:
das. Pode-se comparar os três estados físicos,
avaliando suas apresentações em função do Temperatura
recipiente que os contém:
Pode ser considerada como a medida do grau de
agitação térmica ("energia") das partículas que
Forma Volume
constituem uma substância. Como um gás é ex-
Gasoso Variável Variável tremamente sensível a qualquer tipo de influên-
Aumento de

cia, deve-se utilizar a escala absoluta de tempe-


energia

Líquido Variável Constante ratura nas suas medidas, a escala kelvin (K).
Como, no Brasil, a escala usual é a escala Cel-
Sólido Constante Constante
sius (uma escala relativa), também chamada
de centígrada (°C), é necessária sempre a con-
Todo gás é constituído de partículas (moléculas) versão das unidades para kelvin.
que estão em contínuo movimento desordena-
do em decorrência da grande energia cinética T(K) = T(°C) + 273
contida nelas. Essa característica energética fez
com que fosse criada uma teoria que determi- Volume
na as propriedades de um gás: a teoria cinética
dos gases. O volume ocupado por qualquer substância é
definido como sendo o espaço ocupado por
Teoria cinética dos gases essa substância. No caso dos gases, o volume
• As moléculas de uma substância no es- de uma dada amostra é determinado pelo vo-
tado gasoso apresentam a maior quan- lume do recipiente que a contém. As unidades
tidade de energia em relação aos ou- usuais de volume são: litro (L), mililitro (mL),
tros estados físicos. metro cúbico (m3) e centímetro cúbico (cm3).
• Por apresentarem grande energia, 1 m3 = 1.000 L
movimentam-se aleatoriamente e de 1 dm3 = 1 L
forma desordenada, ocupando todo o 1 L = 1.000 mL
volume do recipiente que as contém. 1 mL = 1 cm3
PV-13-11

• Além de movimentos aleatórios e de- 1 L = 1.000 cm3


sordenados, as moléculas gasosas re-
alizam, individualmente, movimentos Pressão
retilíneos dentro de um recipiente. Fisicamente, a pressão é definida como a força
• As moléculas gasosas chocam-se entre realizada por unidade de área (pressão = for-
si e também com as paredes do reci- ça/área). No estado gasoso, a pressão é o re-
piente, sofrendo colisões perfeitamen- sultado das colisões de suas moléculas contra
te elásticas, ou seja, sem a ocorrência as paredes do recipiente que as contém.
da perda de energia. A medida da pressão de um gás é feita através
• Não existem relações intermoleculares de um equipamento chamado manômetro, sen-
para um sistema gasoso (as moléculas do as principais unidades de medida: atmosfera
comportam-se como se estivessem (atm), torricelli (torr), centrímetros de mercúrio
"sozinhas" no recipiente). (cmHg), milímetros de mercúrio (mmHg) etc.

25
Química Química geral e estequiometria

1 atm = 76 cmHg A curva obtida é uma hipérbole, cuja análise


1 atm = 760 mmHg matemática indica que P · V = constante. As-
sim, matematicamente, a lei das transforma-
1 mmHg = 1 torr
ções isotérmicas pode ser escrita por:
1 atm = 760 torr
P1 · V1 = P2 · V2
2. Transformações gasosas
Uma dada massa de gás sofre uma transfor- B. Transformação isobárica
mação quando ocorrem mudanças nas suas
variáveis de estado. Mantendo-se agora a pressão constante du-
rante todo um experimento, pode-se provocar
A. Transformação isotérmica uma variação da temperatura e avaliar como
consequência uma variação no volume do sis-
Mantendo a temperatura constante duran- tema gasoso.
te todo um experimento, pode-se provocar
uma variação do volume e avaliar como con- Lei de Charles/Gay-Lussac
sequência uma variação na pressão do siste- À pressão constante, o volume ocupa-
ma gasoso. do por uma massa fixa de gás é diretamente
Lei de Boyle-Mariotte proporcional à temperatura absoluta.
À temperatura constante, uma deter- P atm P atm
minada massa de gás ocupa um volume
inversamente proporcional à pressão exer- P = cte
cida sobre ele.
P1 atm P2 atm

V1 = V V2 = 2 V
T = cte
T1 = T T2 = 2 T

A lei de Charles/Gay-Lussac pode ser repre-


P1 = P P2 = 2 P
sentada por um gráfico temperatura x volume.
1
V1 = V V2 = V Nesse gráfico, a abscissa representa a tempe-
2 ratura absoluta de um gás, e a ordenada, o vo-
Experiência da Lei de Boyle-Mariotte lume ocupado.
A lei de Boyle-Marriotte pode ser represen- Graficamente, o processo é representado a
tada por um gráfico pressão x volume. Nesse seguir:
gráfico, a abscissa representa a pressão de um V
PV-13-11

gás, e a ordenada, o volume ocupado.


Graficamente:
V
2 V1

V1
V1

V1
2 T
T1 2 T1

O resultado é uma reta, cuja análise matemáti-


P ca indica que V/T = constante.
P1 2 P1

26
Química geral e estequiometria Química

Assim, matematicamente, a lei das transfor- O resultado desse experimento também é


mações isobáricas pode ser escrita por: uma reta, cuja análise matemática indica que
P/T = constante. Assim, matematicamente, a
V1 V2 lei das transformações isocóricas pode ser es-
= crita por:
T1 T2
P1 P2
=
C. Transformação isocórica, isométrica T1 T2
ou isovolumétrica
Finalmente, realizando o experimento em um
recipiente rígido (mantendo-se agora o volu- 3. Equação geral dos gases
me constante durante todo o experimento), Quando um sistema gasoso sofre uma trans-
pode-se provocar uma variação da temperatu- formação na qual as três variáveis se modifi-
ra e avaliar como consequência uma variação cam, utiliza-se a equação geral dos gases. Ela é
na pressão do sistema gasoso. obtida por meio da relação matemática entre
Lei de Charles/Gay-Lussac as transformações gasosas estudadas ante-
riormente.
Com volume constante, a pressão
exercida por uma determinada massa fixa Assim, obtém-se uma relação proporcional e
de gás é diretamente proporcional à tem- simultânea entre as três variáveis de estado
peratura absoluta. expressa por:

P⋅ V
V = cte = cons tan te ⇒ para determinada massa
T
fixa de gás
Consequentemente, as três transformações
P1 = P P2 = 2 P gasosas podem ser relacionadas a fim de se
obter uma única equação válida para qualquer
T1 = T T2 = 2 T transformação gasosa.
Isotérmica (T = cte)
A lei de Charles/Gay-Lussac pode ser represen- 
tada por um gráfico temperatura x pressão. P1 ⋅ V1 = P2 ⋅ V2 
Nesse gráfico, a abscissa representa a tem-
peratura absoluta de um gás, e a ordenada, a Isobárica (P = cte) 
pressão exercida. V1 V2  P ⋅ V P ⋅ V
= 
1 1
= 2 2
Graficamente, encontra-se a seguinte análise: T1 T2 T T2
 1
PV-13-11

P
Isocórica (V = cte) 

P1 P2 
= 
T1 T2 
2 P1

Observação
P1 Essas equações são utilizadas somente para
substâncias no estado gasoso e em quantida-
de constante. A temperatura deve ser na esca-
T la kelvin, obrigatoriamente.
T1 2 T1

27
Química Química geral e estequiometria

E, mesmo que as massas sejam diferentes,


Vá além pode-se concluir que:
Gás ideal e gás real: qual a diferença? N2 O3 He
Diz-se que um gás é ideal, também conhe-
cido como gás perfeito, quando obedece
de forma rigorosa às leis físicas dos gases
em quaisquer condições de temperatura
e pressão. Isso significa que, para um gás
submetido a uma transformação isotérmi- PVT PVT PVT
ca, por exemplo, tendo sua pressão dobra-
da, apresentará exatamente a metade de VN2 = VO3 = VHe
seu volume original. Entretanto, isso nunca
acontece na prática com tamanha exatidão,
pois, nessas situações, um gás não segue 5. Condições normais de
tal comportamento, daí ser caracterizado temperatura e pressão (CNTP ou TPN)
como gás real. Um gás real fica ainda mais
distante das propriedades de um gás ideal Existe um conjunto de dados de temperatura
principalmente quando está sob pressões e pressão que orientam sobre qual é o estado
muito elevadas e temperaturas muito bai- físico de uma determinada substância. Para o
xas. Isso decorre de o fato do volume sofrer estudo dos gases, em geral, um par de infor-
forte redução, ficando as partículas muito mações é sempre utilizado como referência de
próximas entre si, passando assim a interfe- análise. São as condições normais de tempe-
rir umas no movimento das outras. ratura e pressão, em que:
Um gás real aproxima-se do comportamen- P = 1 atm ou 760 mmHg
to de um gás ideal à medida que a pressão T = 0 °C = 273 K
diminui e a temperatura se eleva.
Observação
Condições ambientes de temperatura e pres-
4. Hipótese de Avogadro são (CA) – Nessas condições, a pressão é a
normal, ou seja, 1 atm, mas a temperatura
A partir da análise dos fatores propostos pela
considerada é de 25 °C (298 K) – atualmente
teoria cinética dos gases, verifica-se que cada
também é aceito o valor de 27 °C (300 K).
molécula individualmente é capaz de ocupar
um volume incrivelmente maior que seu tama-
nho individual. Assim, considerando que uma 6. Volume molar (Vm)
mesma quantidade de dois gases diferentes Corresponde ao volume ocupado por um mol
está submetida a uma mesma temperatura, de moléculas gasosas quaisquer em uma de-
pode-se concluir com muita precisão que am- terminada condição de temperatura e pres-
PV-13-11

bos ocuparão o mesmo volume, visto que os são. Esse valor é diretamente dependente de
tamanhos individuais de suas moléculas serão T e P e, dessa maneira, pode variar conforme
desprezíveis em relação à dimensão de seus esse gás muda suas condições.
respectivos espaços alcançados. Essa proposta
é relatada pela hipótese de Avogadro: Vm = 22,4 L/mol
1 mol de CNTP
Um mesmo número de moléculas de moléculas
gases quaisquer ocupará o mesmo volume gasosas CA
quando elas estiverem submetidas às mes- Vm = 24,6 L/mol
mas condições de temperatura e pressão.

Dessa forma, sendo n o número de mols de cada 7. Equação de Clapeyron


um dos gases relacionados abaixo, tem-se: Partindo-se da relação proporcional e simultâ-
nea entre as três variáveis de estado, expressa
nN2 = nO3 = nHe por:

28
Química geral e estequiometria Química

P⋅ V m
= cons tan te ⇒ para determinada massa d=
T V
fixa de gás,
Como o volume de um gás (e consequente-
verifica-se que a equação geral dos gases é es- mente a densidade) é dependente da tempe-
tabelecida para uma quantidade fixa de mo- ratura e da pressão, o cálculo da densidade
léculas gasosas. Isso significa que, se houver pode ser realizado utilizando-se as informações
alteração na quantidade de moléculas, as va- dessas condições, a partir da equação de Cla-
riáveis de estado também sofrerão alteração. peyron:
Consequentemente, também são dependen-
P ⋅ V = n⋅R ⋅ T
tes do número de mols (n).
P⋅ V m
Assim, como  é diretamente proporcio- P⋅ V = ⋅R ⋅ T
 T  M
nal a n, é lançada uma constante de proporcio- m
P ⋅M = ⋅R ⋅ T
nalidade k para que a igualdade seja estabele- V
cida. Dessa forma: m
como d =
P⋅ V V
= n⋅k
T P ⋅M = d⋅R ⋅ T
Reorganizando os fatores, temos uma equação Portanto, conclui-se que, para qualquer con-
que relaciona pressão, volume e temperatura dição de temperatura e pressão, a densidade
para uma determinada quantidade, em mols pode ser calculada por:
(n), de moléculas.
P ⋅M
P·V=n·R·T d=
R⋅T
em que:
P = pressão, em atm ou mmHg Notas:
V = volume, em litros (L) a. Comumente, a densidade é calculada
m contando com a unidade g/mL. Todavia,
n = quantidade em mols, em mol (mol): n =
M nesse caso, o cálculo estabelecido pela
R = constante universal dos gases densidade requer a unidade g/L.
• quando a pressão for dada em atm b. Observa-se que, conhecendo-se qual é
→ R = 0,082 atm · L · mol–1 · K–1 o gás, é possível determinar a sua den-
• quando a pressão for dada em mmHg sidade apenas com os dados de tempe-
→ R = 62,3 mmHg · L · mol–1 · K–1 ratura e pressão, sem a necessidade de
saber a massa da amostra ou o volume
T = temperatura absoluta, em kelvin (K)
ocupado.
PV-13-11

• lembre-se de que T(K) = T(°C) + 273


Na situação específica de se trabalhar nas CNTP
(P = 1 atm e T = 273 K), a equação da densidade
8. Densidade dos gases absoluta pode ser assim desenvolvida:
A densidade de um gás pode ser analisada sob
duas formas: a densidade absoluta e a densi- 1⋅M
d=
dade relativa. 0, 082 ⋅ 273

A. Densidade absoluta Daí, o cálculo da densidade de um gás qual-


quer nas CNTP pode ser resumido em:
A densidade absoluta á uma relação entre a
massa de uma determinada amostra de um
M
gás e o volume ocupado por essa mesma d=
quantidade de gás, considerando uma deter- 22, 4
minada condição de temperatura e pressão.

29
Química Química geral e estequiometria

B. Densidade relati va 9. Misturas gasosas


A densidade relativa é encontrada através da A mistura entre dois ou mais gases sempre
relação entre as densidades absolutas de dois constitui um sistema homogêneo.
gases, medidas nas mesmas condições de Consideremos inicialmente dois recipientes
temperatura e pressão. contendo, o primeiro, gás nitrogênio (N2) e o
P ⋅ M1 P ⋅ M1 segundo, gás hélio (He). Os dois gases são mis-
d1 =
R ⋅ T d1 = R ⋅ T turados em um terceiro recipiente, conforme
P ⋅ M2 d2 P ⋅ M2 o esquema representado abaixo.
d2 = N2 inicial He inicial
R⋅T R⋅T
d1 M1 M PN VN TN PH VH TH
= ou d1,2 = 1 2 2 2 e e e
d2 M2 M2 nN nH
2 e
Essa relação indicará quantas vezes um gás
é mais denso (“pesado”) ou menos denso
(“leve”) que outro gás. Assim, basta conhecer
as massas molares de cada um dos gases para P V T
saber qual ficará “em cima” e qual tenderá a Σn = nN + nH
ficar “em baixo”. 2 e

Observação Mistura
Esse mecanismo é muito utilizado para realizar
Para uma mistura gasosa (sem a ocorrência de
a comparação entre um gás e o ar atmosférico.
reação química), é possível estabelecer a se-
Como o ar é constituído por uma mistura gasosa,
guinte relação: a quantidade total de molécu-
faz-se a relação utilizando-se uma massa molar
las gasosas no recipiente da mistura é igual à
média do ar atmosférico, que é de 28,9 g/mol!
soma das quantidades de moléculas gasosas
dar = 28,9 g/mol de cada um dos gases.

nT = nN2 + nHe + ... Equação I


Basta agora conhecer a massa molar do gás e
compará-la com 28,9 g/mol. Com isso, entende- Para qualquer sistema gasoso pode ser utilizada
-se por que, quando sopramos em uma bexiga a equação de Clapeyron:
para enchê-la de gás carbônico (CO2 = 44 g/mol),
P·V=n·R·T
ela cai no chão ao soltarmos, e outra, cheia de
gás hélio (He = 4 g/mol), sobe espontaneamente.
P⋅ V
n= Equação II
He : 4 g/mol R⋅T
PV-13-11

Substituindo-se a Equação II em I, conclui-se


CH4 : 16 g/mol que a soma das quantidades em mols fica:
PN ⋅ VN PHe ⋅ VHe P⋅V
Ar atmosférico: 2 2
+ =
CO : 28 g/mol R ⋅ TN R ⋅ THe R⋅T
M = 28,9 g/mol 2

O2 : 32 g/mol Portanto, a equação geral para uma mistura


gasosa pode ser assim representada:
PN ⋅ VN PHe ⋅ VHe P⋅ V
CO2 : 44 g/mol
2 2
+ =
TN THe T
2

Solo

30
Química geral e estequiometria Química

Para a equação representada, utilizou-se a inicial: pN2 ⋅ VN2 = nN2 ⋅ R ⋅ TN2


mistura de dois gases; portanto, para uma mis-
tura qualquer, contendo dois ou mais gases, a mistura: pN2 ⋅ V = nN2 ⋅ R ⋅ T
equação genérica fica assim representada: Estabelecendo a igualdade:
pN ⋅ VN pN ⋅ V pN ⋅ VN pN ⋅ V
P1 ⋅ V1 P2 ⋅ V2 P ⋅ V 2 2
= 2 ⇒ 2 2
= 2
+ = R ⋅ TN R ⋅T TN T
T1 T2 T 2 2

Para o hélio, a equação fica:


em que P1, V1, T1, P2, V2, T2, ... representam a pHe ⋅ VHe pHe ⋅ V
=
situação inicial de cada gás. THe T
c. Relação entre pressão da mistura gaso-
10. Pressão parcial sa e pressão parcial
Utilizando o mesmo esquema anterior, temos: Inicialmente, definimos uma forma de concen-
N2 inicial He inicial tração denominada de fração molar (x).
PN VN TN PH VH TH A fração molar corresponde à razão entre a
2 2 2 e e e
quantidade em mols do gás presente na mis-
nN nH tura e a quantidade total, em mols, de gás.
2 e
Portanto, a equação fica:
nN n
xN = 2 e xHe = He
2
Σn Σn
P V T  − não possui unidade
Σn = nN + nH 
2 e
 − valor entre 0 e 1

Mistura em que x  − quando exp resso em porcenta-
 gem, é denomin ado de % molar
A pressão da mistura gasosa (P) corresponde à 
soma das pressões exercidas pelo hélio e pelo  − xN2 + xHe = 1
nitrogênio dentro do recipiente.
A pressão que cada gás exerce na mistura ga- Para estabelecer a relação entre as pressões,
sosa é chamada de pressão parcial. Portanto, recorremos à equação de estado:
podemos enunciar a lei de Dalton (das pres- pHe · V = nHe · R · T (pressão parcial)
sões parciais), que diz: a pressão total corres-
P · V = Σn · R · T (mistura gasosa)
ponde à soma das pressões parciais dos gases
componentes da mistura gasosa. Dividindo uma equação pela outra:
P = pHe + pN pHe ⋅ V nHe ⋅ R ⋅ T
2 =
PV-13-11

Para o cálculo da pressão parcial, podemos P⋅ V Σn ⋅ R ⋅ T


utilizar: encontramos:
a. Equação de estado pHe nHe p
= ou He = xHe
pN · V = nN · R · T e pHe · V = nHe · R · T, em que P Σn P
2 2 Finalmente: pHe = P · xHe
V e T são da mistura gasosa.
Para o nitrogênio: pN = P · xN
b. Equação geral 2 2

Como a quantidade em mols de cada gás não 11. Difusão e efusão gasosa
varia, podemos escrever:
nN = nN Difusão gasosa
2 2
Inicial Mistura É a capacidade que cada gás possui de se dis-
seminar por toda a extensão do volume de um
Utilizando a equação de estado, temos: outro sistema gasoso. Em razão dessa proprie-

31
Química Química geral e estequiometria

dade, todas as misturas gasosas são homogê- ma de um perfume quando abrimos o frasco
neas, não apresentando, portanto, superfície ou mesmo quando percebemos o cheiro de-
de separação. sagradável quando abrimos uma lata de lixo.
Assim, é em virtude da difusão gasosa que so- Contudo como saber de qual cheiro seremos
mos capazes de identificar rapidamente o aro- impregnados primeiro?

EXERCÍCIO RESOLVIDO
01. ENEM Resolução
As áreas numeradas no gráfico mostram a com- De acordo com o gráfico, teremos:
posição em volume, aproximada, dos gases na 100
atmosfera terrestre, desde a sua formação até 90
os dias atuais. 80

Composição/%
70
100 60 I II IV
90 50
80 40
Composição/%

70 30
60 I II IV
20
50 10 III V
40 VI
0
30 5 4 3 2 1 0
20 5% Tempo (bilhões de anos)
10 III V Data atual
VI
0
5 4 3 2 1 0
Tempo (bilhões de anos) Resposta
Data atual E
I. Metano e hidrogênio
II. Vapor-d´água
III. Amônia
IV. Nitrogênio
V. Gás carbônico
VI. Oxigênio
The Random House Encyclopedias,
3rd ed., 1990. Adaptado.

No que se refere à composição em volume da


PV-13-11

atmosfera terrestre há 2,5 bilhões de anos,


pode-se afirmar que o volume de oxigênio, em
valores percentuais, era de, aproximadamente:
a. 95%
b. 77%
c. 45%
d. 21%
e. 5%

32
Química geral e estequiometria Química

Com essa pergunta, o conceito mais impor-


tante a respeito da difusão gasosa é relativo V2 M1
à velocidade com a qual um gás consegue se Portanto: =
V1 M2
espalhar por todo volume de um outro gás:
velocidade de difusão.
Lei de Graham
Diferentemente de outras propriedades gaso-
sas, a velocidade de difusão não é dependen- As velocidades de difusão e de
dente da quantidade de matéria, e sim da mas- efusão gasosas são inversamente pro-
sa molecular do gás, ou dos gases, em questão. porcionais às raízes quadradas de suas
massas específicas, quando os gases es-
Portanto, considere dois gases quaisquer que es- tão submetidos à mesma pressão e tem-
tejam submetidos às mesmas condições de tem- peratura.
peratura e pressão e que habitem recipientes
iguais, colocados lado a lado sobre um bancada. Conclui-se, portanto, que a velocidade de difu-
Nesse momento, considera-se que a energia ci- são de um gás:
nética de cada um deles é exatamente igual. • depende da massa molar do próprio
Energia cinética1 = Energia cinética2 gás;
• é inversamente proporcional à raiz qua-
Ec1 = Ec2
drada de sua massa molar.
m1 ⋅ v12 m2 ⋅ v22
= Efusão gasosa
2 2
É definida como a capacidade que o gás pos-
m1 V22 sui de escapar de um recipiente, por meio de
= , e considerando
m2 V12 1 mol de cada gás um pequeno orifício, em direção a um meio de
menor pressão.
M1 V22
=
M2 V12
PV-13-11

33
Química Química geral e estequiometria

CAPÍTULO 05 LEIS PONDERAIS


No final do século XVIII, a Química se firma Ao utilizar obtinha 3 g 24 g de
como “ciência”, principalmente devido aos 27 g de ⇒ de hidrogênio e oxigênio
experimentos e às observações de cientistas água
como Lavoisier, Proust e Dalton. Esses expe-
rimentos foram realizados com base nas ob- Assim, Proust concluiu que:
servações das massas das substâncias que Independentemente da origem de
participavam dos fenômenos químicos, daí uma determinada substância pura, ela é
o nome leis ponderais. sempre formada pelos mesmos elemen-
tos químicos, combinados entre si na
1. Lei de Lavoisier (lei da conservação mesma proporção em massa.
das massas) Uma das consequências da lei de Proust é a
Antoine Laurent Lavoisier foi o primeiro cien- composição centesimal das substâncias, que
tista a dar conotação científica à Química. No indica a porcentagem, em massa, de cada ele-
final do século XVIII, ele fazia experiências nas mento que constitui a substância.
quais se preocupava em medir a massa total Exemplo
de um sistema, antes e depois de ocorrer a
transformação química. No caso da água, temos:
Exemplo 90 g de água fornecem 10 g de hidrogênio e 80
g de oxigênio.
 +
Hidrogênio  → água
oxigênio  Cálculo da
10 g 80 g 90 g
porcentagem 90 g água − 10 g hidrogênio

Massa total antes
da transformação = 90 g
Massa total após
a transformação = 90 g
do hidrogênio 100 g água − x
x = 11,11% de hidrogênio
Por meio da análise dos dados obtidos em
várias experiências, chegou à conclusão de que: Cálculo da 90 g água − 80 g oxigênio
porcentagem 
Num sistema fechado, a massa total do oxigênio 100 g água − y
das substâncias, antes da transformação
y = 88,89% de oxigênio
química, é igual à massa total após a
transformação ou na natureza nada se Outra consequência da lei de Proust é o cálculo
perde, nada se cria, tudo se transforma. estequiométrico.

Hidrogênio + oxigênio → água


2. Lei de Proust (lei das proporções
fixas ou definidas) 10 g 80 g 90 g
PV-13-11

Proust se preocupava em analisar a composi-


30 g x
ção das substâncias. Trabalhando com amos-
tras de água de várias procedências (água de
10 80
chuva, água de rio, água de lago, previamente = ⇒ x = 240 g
purificadas) e decompostas por eletrólise, ele 30 x
verificou que:
Para 10 g de hidrogênio, precisamos de 80 g
Ao utilizar obtinha 10 g 80 g de de oxigênio para reagir; em 30 g de hidrogê-
90 g de ⇒ de hidrogê- e oxigênio nio precisamos de 240 g de oxigênio. Logo, a
água nio proporção, em massa, com que o hidrogê-
Ao utilizar obtinha 1 g 8 g de nio reage com o oxigênio é a mesma nas
⇒ e
9 g de água de hidrogênio oxigênio duas reações.

34
Química geral e estequiometria Química

EXERCÍCIO RESOLVIDO
01. Unicamp-SP Resolução
Numa balança improvisada, feita com um cabi- a. O sólido colocado em A e B era palha de
de, como mostra a figura abaixo, nos recipien- aço. Quando a palha de aço (ferro) é queima-
tes (A e B) foram colocadas quantidades iguais da, ela absorve oxigênio, aumentando a sua
de um mesmo sólido, que poderia ou ser palha massa.
de ferro ou ser carvão. b. Fe(s) + 1 2 O2(g) → FeO(s) ou
Foi ateado fogo à amostra contida no recipien-
2 Fe(s) + 3 2 O2(g) → Fe2 O3(s)
te B. Após cessada a queima, o arranjo tomou
a seguinte disposição:
Figura I Figura II

A
A B
B
Recipientes com
amostras

a. Considerando o resultado do experi-


mento, decida se o sólido colocado em
A e B era palha de ferro ou carvão. Jus-
tifique.
b. Escreva a equação química da reação
que ocorreu.
PV-13-11

35
Química Química geral e estequiometria

CAPÍTULO 06 DETERMINAÇÃO DE FÓRMULAS


Introdução O cálculo da fórmula percentual pode ser feito
de duas formas diferentes:
Quando um químico se depara com um mate-
rial desconhecido, por exemplo, uma amostra a. quando se é informado da massa de
de uma pedra lunar ou de um meteorito, ou cada elemento que se combina para
ainda de uma matéria encontrada em uma ca- formar a substância. Por exemplo:
verna que está sendo explorada, ele procura, cálcio + oxigênio → óxido de cálcio
por meio de diversas técnicas físicas e quími-
cas, encontrar a composição desse material. 40 g + 16 g → 56 g
A primeira providência é fazer a análise ime-
diata desse material, isto é, separar através 56 g  100% 56 g  100%
de processos puramente físicos as diversas 40 g  x 16 g  y
substâncias presentes na amostra. Esses pro-
cessos baseiam-se no fato de que o conjunto x = 71,43% y = 28,57%
de características, como temperatura de fu-
cálcio oxigênio
são, temperatura de ebulição, densidade, so-
lubilidade, é diferente para cada substância e, b. quando se é informada a fórmula mole-
manejando criteriosamente essas diferenças, cular. Por exemplo:
consegue-se separá-las uma a uma.
Uma vez separadas as diversas substâncias da H=1·2=2
amostra, a próxima providência é fazer uma H2O 18 u (18 g/mol)
O = 16 · 1 = 16
análise elementar de cada uma delas.
A análise elementar divide-se em qualitativa, 18 g  100% 18 g  100%
cuja finalidade é descobrir os tipos de elemen- 2 g  x 16 g  y
tos que formam a substância, e quantitativa,
que tem por finalidade descobrir a proporção x = 11,11% y = 88,89%
em que esses elementos aparecem, ou seja,
descobrir a fórmula da substância. hidrogênio oxigênio
Resumidamente, pode-se propor o seguinte
esquema de ações: 2. Fórmula mínima ou empírica
Separam-se as
Material Submetido substâncias Submetido
Fórmula mínima é a representação que indica os
inicial à análise presentes à análise elementos formadores da substância e a menor
imediata no material qualitativa proporção inteira entre as quantidades dos áto-
mos dos diferentes elementos da substância.
PV-13-11

Descobrem-se O cálculo da fórmula mínima é realizado divi-


Descobrem-se
as quantidades Submetido
dos elementos à análise
os elementos dindo-se o número de átomos (ou número de
das substâncias mols) da fórmula molecular pelo máximo divi-
formadores quantitativa
sor comum entre eles.
Com base nesses dados, o químico já pode de-
Exemplo
terminar com muita precisão os três tipos de fór-
÷ MDC
mula para uma substância: a fórmula percentual, Fórmula molecular Fórmula
a fórmula empírica e a fórmula molecular. mínima
÷2
1. Fórmula percentual ou centesimal C2H6 (etano) CH3
Indica os elementos formadores de uma deter- ÷6
C6H12O6 (glicose) CH2O
minada substância, bem como a porcentagem
÷1
em massa de cada um desses elementos na H2SO4 (ácido sulfúrico) H2SO4
substância.

36
Química geral e estequiometria Química

Quando apenas as massas dos elementos que se 40 


combinam para formar a substância forem for- C= = 3, 33 : 3, 33 = 1 
necidas, deve-se calcular o número de mols de 12

6, 7 
cada um dos elementos presentes e, a seguir, se H= = 6, 7 : 3, 33 = 2  Fórmula
mínima : CH2 O
não forem obtidos os menores números inteiros 1 
possíveis, deve-se dividir todos os números pelo 53, 3 
menor deles. Assim, para uma substância que O= = 3, 33 : 3, 33 = 1
16 
apresenta 3,0 g de carbono, 0,5 g de hidrogênio
e 4,0 g de oxigênio, eis a sequência de cálculos:
carbono: 3,0 g 3. Fórmula molecular
7,5 g de uma
hidrogênio: 0,5 g A fórmula molecular é a representação que in-
substância X dica quais e quantos átomos de cada elemento
oxigênio: 4,0 g
químico exatamente constituem uma molécu-
3, 0  la de determinada substância.
C= = 0,25 : 0,25 = 1 
12 Exemplo

0, 5 
H= = 0, 5 : 0,25 = 2  Fórmula
mínima : CH2 O A fórmula molecular da glicose é C6H12O6.
1 
4, 0 Isso significa que 1 molécula de glicose é cons-

O= = 0,25 : 0,25 = 1 tituída de 6 átomos de carbono, 12 átomos de
16  hidrogênio e 6 átomos de oxigênio.
Em algumas situações, caso ainda não seja Os índices subscritos numa fórmula indicam
conseguida a sequência de números inteiros, tanto a proporção atômica com que os ele-
deve-se multiplicar todos os números por um mentos se combinam quanto à proporção mo-
mesmo valor. lar dos respectivos elementos.
Existe também a situação de se conhecer a com- Fórmula molecular da glicose:
posição percentual da substância. Aqui, o cami-
C6H12O6
nho matemático mais adequado é deduzir uma
massa de 100 g. Com isso, os valores percentuais Proporção atômica Proporção molar
passam a corresponder a valores em massa e, a 6 átomos de car- → 6 mol de átomos
partir daí, realiza-se a determinação da fórmula bono de carbono
empírica, como já demonstrado anteriormente. 12 átomos de → 12 mol de átomos
Acompanhe o exemplo: hidrogênio de hidrogênio
C = 40% em massa 6 átomos de oxi- → 6 mol de átomos
substância X H = 6,7% em massa gênio de oxigênio
O = 53,3% em massa Para a determinação da fórmula molecular,
independentemente das informações dispo-
PV-13-11

Veja que, para 100 g da substância X, encon- níveis, é necessário que se conheça a massa
tram-se 40 g de carbono, 6,7 g de hidrogênio e molecular da substância. Sabe-se, nesse caso,
53,3 g de oxigênio. que a massa molecular da glicose vale 180 u.

A. Partindo da fórmula mínima


Fórmula Massa Fórmula
Cálculos
mínima molecular molecular
Considerando-se a fórmula molecular (CH2O)n
Então: (CH2O)n = CnH2nOn
CnH2nOn
Somando-se as massas atômicas multiplicadas por n de cada
Para n = 6,
CH2O 180 u elemento, obtém-se massa molecular.
ficamos com
12n + 2n + 16n = 180 u
C6H12O6.
30n = 180
n=6

37
Química Química geral e estequiometria

B. Partindo da composição centesimal


Cálculos da
Fórmula Massa Cálculos da massa Fórmula
quantidade
percentual molecular em gramas molecular
em mols

100 g  40 g de C
72
180 g  x n= =6
x = 72 g 12

100 g  6,7 g de H 12
C40%H6,7%O53,3% 180 u 180 g  y n= = 12 C6H12O6
y = 12 g 1

100 g  53,3 g de O 96
180 g  z n= =6
z = 96 g 16

EXERCÍCIO RESOLVIDO
01. UFRJ b. O sulfato de amônio pode ser obtido
O nitrogênio é um dos elementos mais impor- industrialmente pela reação do carbo-
tantes para o desenvolvimento das plantas. nato de amônio com o sulfato de cálcio.
Escreva a equação que descreve esta
Apesar dos processos naturais de fornecimen- reação química.
to desse elemento, a grande parte necessária
para a agricultura é suprida através da adição Resolução
de fertilizantes. Tais fertilizantes são comercia- a. 1. Porcentagem de nitrogênio em mas-
lizados sob forma de ureia, sulfato de amônio sa em cada fertilizante:
e nitrato de amônio. 28
Ureia: ⋅ 100 = 47%
A tabela a seguir apresenta os preços desses 60
fertilizantes por tonelada. 28
Sulfato de amônio: ⋅ 100 = 21%
Dados: Massas molares (g/mol): H = 1,0; 132
N = 14,0; O = 16,0; S = 32,0 28
Nitrato de amônio: ⋅ 100 = 35%
80
Preço do produto m
2. Preço dos fertilizantes por mol: n =
Produto Fórmula (expresso em reais M
Ureia: 106/60 = 16.667 mol
PV-13-11

por tonelada)
R$ 230,00/16.667 = R$ 0,0138 por mol
Ureia NH2CONH2 230,00 Sulfato de amônio: 106/132 = 7.576 mol
R$ 210,00/7.576 = R$ 0,0277 por mol
Sulfato de
amônio
(NH4)2SO4 210,00 Nitrato de amônio: 106/80 = 12.500 mol
R$ 335,00/12.500 = R$ 0,0268 por mol
Nitrato de
NH4 NO3 335,00 Ou seja, o mais barato é a ureia, que, além de
amônio
ter o menor custo por mol, apresenta a maior
a. Com base na proporção (em massa) de porcentagem de nitrogênio. (Comentário: em
nitrogênio em cada um dos fertilizan- seguida, o nitrato de amônio, e por fim, o mais
tes, indique qual deles é o mais barato? caro, o sulfato de amônio.)
Justifique. b. (NH4)2 CO3 + CaSO4 → (NH4)2 SO4 + CaCO3

38
Química geral e estequiometria Química

4. Balanceamento das equações Consideremos a seguinte equação:


CH4 + O2 → CO2 + H2O
A. Método das tentativas
Para iniciarmos o balanceamento, devemos
A.1. Equação química procurar o elemento que se encontra uma úni-
É a maneira utilizada para representarmos a ca vez nos reagentes e nos produtos (carbono
reação química. Usamos símbolos e fórmulas: e hidrogênio).
Selecionar o elemento que possui maior índice
1
H2 + O2 → H2 O (hidrogênio).

2  Pr oduto
Re agentes 2 CH4 + O2 → CO2 + 4 H2O

Numa reação química, o total de átomos do


lado do reagente é igual ao total do lado do O índice do elemento do reagente torna-se coe-
produto. ficiente da substância em que se encontra esse
mesmo elemento nos produtos e vice-versa.
A.2. Balanceamento das equações
Prosseguir o balanceamento, igualando as
Acertar os coeficientes ou balancear uma quantidades dos demais elementos.
equação química significa igualar os átomos
2 CH4 + 4 O2 → 2 CO2 + 4 H2O
que se encontram nos reagentes com os áto-
mos que se encontram nos produtos. Se necessário, simplifique os coeficientes, ex-
primindo com os menores números inteiros
Vamos discutir um primeiro método chamado
possíveis.
de direto ou “por tentativas” e, mais tarde, dis-
cutiremos o método da oxirredução. CH4 + 2 O2 → CO2 + 2 H2O
PV-13-11

39
Química Química geral e estequiometria

CAPÍTULO 07 CÁLCULO ESTEQUIOMÉTRICO


É o cálculo das quantidades de reagentes e/ou Estequio: do grego Stoikheion (elemento ou
produtos das reações químicas. Essas quanti- substância)
dades podem ser em termos de massa, volu- Metria: do grego metron (medida)
me, quantidade em mols etc.
Os cálculos estequiométricos são de grande
Esses cálculos envolvem sempre uma propor- importância no cotidiano, tanto nas indústrias
ção, obtida a partir de uma equação química e nos laboratórios quanto nas cozinhas das do-
corretamente balanceada, o que é realizado nas de casa, pois possibilitam calcular teorica-
como consequência da lei de Proust. Lembre- mente a quantidade de reagentes (reagentes
se de que os coeficientes da equação indicam químicos ou mesmo quantidades de alimentos
a proporção da quantidade em mols existente para o almoço) a ser utilizada em um processo
entre os participantes da reação. de transformação, prevendo a quantidade de
A palavra estequiometria é de origem grega e produtos que será obtida em condições pre-
significa medida de uma substância. estabelecidas.

Alguns conceitos sobre relações conceituais básicas se fazem essenciais para o desenvolvimento
de um cálculo estequiométrico. Assim, cabe salientar:
• Para os elementos
(gás)
pesa contém ocupa
1 mol de átomos ⇒ (MA)g ⇒ 6, 02 ⋅ 1023 átomos ⇒ 22, 4 L
Volume
CNTP

Isso quer dizer que 1 mol de átomos apresenta massa correspondente ao valor numérico da
atômica expressa em gramas, um valor numericamente igual à massa de 6,02 · 1023 átomos do re-
ferido elemento que, se estiverem no estado gasoso e nas CNTP, ocuparão um volume de 22,4 L.
• Para as substâncias
(gás)
pesa contém ocupa
1 mol de moléculas ⇒ (MM)g ⇒ 6, 02 ⋅ 1023 moléculas ⇒ 22, 4 L
Volume
CNTP
Isso quer dizer que 1 mol de moléculas apresenta massa corresponde ao valor numérico da massa
molecular expressa em gramas, um valor numericamente igual à massa de 6,02 · 1023 moléculas da
referida substância que, se estiverem no estado gasoso e nas CNTP, ocuparão um volume de 22,4 L.
No estado gasoso e nas CNTP: 1 mol sempre ocupa um volume de 22,4 L.
PV-13-11

1. Regras para a resolução de problemas de estequiometria


Na estequiometria, os cálculos serão estabelecidos em função da lei de Proust e Gay-Lussac,
sendo, nesse caso, para reações envolvendo gases que estejam todos nas mesmas condições de
temperatura e pressão.
Com isso em mente, deve-se sempre tomar os coeficientes da equação química devidamente
balanceada como referência e, a partir deles, estabelecer a proporção em mols dos elementos ou
das substâncias da reação.
Uma sequência apropriada para os cálculos estequiométricos está descrita a seguir:
1º Escrever a equação relacionada com o problema.
2º Acertar os coeficientes da equação (balanceamento).
3º Relacionar cada coeficiente com a quantidade em mols da espécie participante (propor-
ção em mols entre reagentes e produtos).

40
Química geral e estequiometria Química

4º Conhecida a proporção em mols (coeficientes), pode-se chegar à proporção pedida no proble-


ma, que pode ser em mols, massa, átomos, moléculas, volume etc.
Exemplos
Síntese da amônia

Tipo de relação N2(g) + 3 H2(g) → 2 NH3(g)


  
 

Proporção
1 mol 3 mol 2 mol
em mols
↓ ↓ ↓
Proporção
28 g 3·2g 2 · 17 g
em massa
↓ ↓
Proporção
6,0 · 1023 3 · 6 · 1023 2 · 6 · 1023
em moléculas
↓ ↓ ↓
Proporção em volume
22,4 L 3 · 22,4 L 2 · 22,4 L
(CNTP)

Combustão do álcool etílico

Tipo de relação C2H6 O() + 3 O2(g) → 2 CO2(g) + 3 H2O()



    
  
  

Proporção
1 mol 3 mol 2 mol 3 mol
em mols
↓ ↓ ↓ ↓
Proporção
46 g 3 · 32 g 2 · 44 g 3 · 18 g
em massa
↓ ↓ ↓ ↓
Proporção
6 · 1023 3 · 6 · 1023 2 · 6 · 1023 3 · 6 · 1023
em moléculas
↓ ↓ ↓ ↓
PV-13-11

Proporção em volume
É líquido. 3 · 22,4 L 2 · 22,4 L É líquido.
(CNTP)

2. Formas diferentes de encarar um mesmo cálculo estequiométrico


A. Relação quantidade em mols – quantidade em mols
Os dados do problema e as quantidades incógnitas pedidas são expressos em termos de quanti-
dade em mols.
Exemplo:
Qual o número de mols de CO2 produzidos a partir de 3 mol de álcool etílico?

41
Química Química geral e estequiometria

Tipo de relação C2H6 O() + 3 O2(g) → 2 CO2(g) + 3 H2O()



    
  
  

Proporção
1 mol 3 mol 2 mol 3 mol
em mols
↓ ↓ ↓ ↓
Proporção
46 g 3 · 32 g 2 · 44 g 3 · 18 g
em massa
↓ ↓ ↓ ↓
Proporção
6 · 1023 3 · 6 · 1023 2 · 6 · 1023 3 · 6 · 1023
em moléculas
↓ ↓ ↓ ↓
Proporção em volume
É líquido. 3 · 22,4 L 2 · 22,4 L É líquido.
(CNTP)

1 mol C2H6O  2 mol CO2


3 mol C2H6O  x
x = 6 mol CO2

B. Relação quantidade em mols e massa


Os dados do problema são expressos em termos de quantidades em mols (ou massa) e a quanti-
dade incógnita é pedida em massa (ou em termos de quantidade em mols).
Exemplo
Qual a massa de CO2, em gramas, produzida a partir de 3 mol de álcool etílico?

Tipo de relação C2H6 O() + 3 O2(g) → 2 CO2(g) + 3 H2O()



    
  
  

Proporção
1 mol 3 mol 2 mol 3 mol
em mols
↓ ↓ ↓ ↓
Proporção
46 g 3 · 32 g 2 · 44 g 3 · 18 g
em massa
PV-13-11

↓ ↓ ↓ ↓
Proporção
6 · 1023 3 · 6 · 1023 2 · 6 · 1023 3 · 6 · 1023
em moléculas
↓ ↓ ↓ ↓
Proporção em volume
É líquido. 3 · 22,4 L 2 · 22,4 L É líquido.
(CNTP)

1 mol C2H6O  2 · 44 g CO2


3 mol C2H6O  x
x = 264 g CO2

42
Química geral e estequiometria Química

C. Relação massa e massa


Os dados do problema e as quantidades incógnitas pedidas são expressos em massa.
Exemplo:
Qual a massa de CO2, em gramas, produzida a partir de 138 g de álcool etílico?

Tipo de relação C2H6 O() + 3 O2(g) → 2 CO2(g) + 3 H2O()



    
  
  

Proporção
1 mol 3 mol 2 mol 3 mol
em mols
↓ ↓ ↓ ↓
Proporção
46 g 3 · 32 g 2 · 44 g 3 · 18 g
em massa
↓ ↓ ↓ ↓
Proporção
6 · 1023 3 · 6 · 1023 2 · 6 · 1023 3 · 6 · 1023
em moléculas
↓ ↓ ↓ ↓
Proporção em volume
É líquido. 3 · 22,4 L 2 · 22,4 L É líquido.
(CNTP)

46 g C2H6O  2 · 44 g CO2


138 g C2H6O  x
x = 264 g CO2

D. Relação massa e volume


Os dados do problema são expressos em massa (ou volume) e a quantidade incógnita é pedida
em volume (ou em massa).
Exemplo:
Qual o volume de CO2, em litros, produzido a partir de 138 g de álcool etílico, nas CNTP?

Tipo de relação C2H6 O() + 3 O2(g) → 2 CO2(g) + 3 H2O()



    
  
  
PV-13-11

Proporção
1 mol 3 mol 2 mol 3 mol
em mols
↓ ↓ ↓ ↓
Proporção
46 g 3 · 32 g 2 · 44 g 3 · 18 g
em massa
↓ ↓ ↓ ↓
Proporção
6 · 1023 3 · 6 · 1023 2 · 6 · 1023 3 · 6 · 1023
em moléculas
↓ ↓ ↓ ↓
Proporção em volume
É líquido. 3 · 22,4 L 2 · 22,4 L É líquido.
(CNTP)

43
Química Química geral e estequiometria

46 g C2H6O  2 · 22,4 L CO2


138 g C2H6O  x
x = 134,4 L CO2
Observação
Caso o sistema não esteja nas CNTP, deve-se calcular estequiometricamente a quantidade em
mols do gás e, a seguir, através da equação de estado (Clapeyron), determinar o volume corres-
pondente nas condições do sistema. Por exemplo:
Calcular o volume de CO2 produzido na reação de 16 g de gás oxigênio com monóxido de carbono,
numa temperatura de 27 °C e pressão de 1 atm.
Dado: constante universal dos gases:
0,082 atm · L · mol–1 · K–1
Resolução
2 CO(g) + O2(g) → 2 CO2(g)
   
1 mol 2 mol → proporção emmols

32 g  2 mol → relação do problema


16 g  n → dado e pergunta
n = 1,0 mol
Equação de estado: P · V = n · R · T
1 · V = 1 · 0,082 · 300
V = 24,6 L

E. Relação número de moléculas – massa, quantidade em mols ou volume


Os dados do problema são expressos em termos de número de moléculas ou número de átomos
e a quantidade incógnita é pedida em massa, quantidade em mols ou volume.
Exemplo
Qual é a massa de CO2, em gramas, produzida a partir de 1,8 · 1023 moléculas de álcool etílico?

Tipo de relação C2H6 O() + 3 O2(g) → 2 CO2(g) + 3 H2O()



    
  
  
PV-13-11

Proporção
1 mol 3 mol 2 mol 3 mol
em mols
↓ ↓ ↓ ↓
Proporção
46 g 3 · 32 g 2 · 44 g 3 · 18 g
em massa
↓ ↓ ↓ ↓
Proporção
6 · 1023 3 · 6 · 1023 2 · 6 · 1023 3 · 6 · 1023
em moléculas
↓ ↓ ↓ ↓
Proporção em volume
É líquido. 3 · 22,4 L 2 · 22,4 L É líquido.
(CNTP)

44
Química geral e estequiometria Química

6,0 · 1023 moléculas de C2H6O  2 · 44 g CO2 G. Problemas envolvendo


1,8 · 1023 moléculas de C2H6O  x reagentes em excesso
x = 26,4 g CO2 Quando o exercício fornece quantidades (mas-
sa, volume, mols etc.) de dois reagentes, de-
F. Reações envolvendo gases – Lei de vemos verificar se existe excesso de algum
Gay-Lussac (1.808) reagente.
Quando as substâncias envolvidas na reação As quantidades de substâncias que participam
forem gasosas e estiverem nas mesmas con- da reação química são sempre proporcionais
dições de temperatura e pressão, as relações aos coeficientes da equação. Se a quantidade
volumétricas serão equivalentes às respecti- de reagente estiver fora da proporção indicada
vas relações estequiométricas. pelos coeficientes da equação, reagirá somen-
Isso pode ser justificado pela lei de Gay-Lussac: te a parte que se encontra de acordo com a
proporção.
Os volumes das substâncias gaso-
sas participantes de uma reação química, Lembre-se: o excesso é uma quantidade de
estando nas mesmas condições de tem- reagente que não participa da reação. Não
peratura e pressão, guardam entre si uma sendo o excesso um reagente no estado gaso-
relação constante de números inteiros e so, ele permanece no meio após o término da
menores possíveis. reação (excesso + produto).

EXERCÍCIO RESOLVIDO
01. ENEM
A eutrofização é um processo em que rios, lagos e mares adquirem níveis altos de nutrientes,
especialmente fosfatos e nitratos, provocando posterior acúmulo de matéria orgânica em de-
composição. Os nutrientes são assimilados pelos produtores primários e o crescimento desses é
controlado pelo nutriente limítrofe, que é o elemento menos disponível em relação à abundância
necessária à sobrevivência dos organismos vivos. O ciclo representado na figura seguinte reflete
a dinâmica dos nutrientes em um lago.
Fosfatos N2 (atmosfera)
(solo; detergentes; esgoto) nitratos (solo; esgoto) CO2 (atmosfera)

P N C
CO2 (geralmente limitados) (possivelmente limitados) (geralmente abundante)

NO 3– (O2 suficiente)
Microelementos
PO 43– Produtores primários
(Fe, Mn, Cu etc.)
Algas e outros organismos fixadores de
PV-13-11

nitrogênio e fotossintéticos assimilam


C, N, P nas razões atômicas de 106 : 16 : 1

Decomposição bacteriana
de resíduos vegetais e animais

Crescimento de peixes e
outros produtores secundários
(O2 suficiente) Processo de
envelhecimento

Sedimentação
de resíduos vegetais e animais

45
Química Química geral e estequiometria

A análise da água de um lago que recebe a


descarga de águas residuais provenientes de
lavouras adubadas revelou as concentrações
dos elementos carbono (21,2 mol/L), nitrogê-
nio (1,2 mol/L) e fósforo (0,2 mol/L). Nessas
condições, o nutriente limítrofe é o:
a. C.
b. N.
c. P.
d. CO2.
e. PO4.
Resposta
B
H. Sistema em que os reagentes são substâncias impuras
Grau, teor ou % de pureza de uma amostra é a porcentagem da parte pura existente na amostra.
É a massa que vai reagir no problema de cálculo estequiométrico. Impurezas normalmente não
participam dos problemas.

NaOH impuro
80% pureza
mtotal = 200 g

No frasco acima, a massa total da amostra (pureza + impureza) é de 200 g.

Lembre-se: somente participa da reação a parte pura; a impureza é descartada (a menos que o
problema dê informações contrárias).

% pureza = 80% ⇒ Existem 160 g do com a proporção da reação química, dize-


de NaOH puro na amostra. mos que o rendimento não foi total.
 Rendimento de uma reação é o quociente
(É a massa que vai reagir).
PV-13-11

entre a quantidade de produto realmente



 obtida e a quantidade esperada, de acordo
mtotal = 200 g  com a proporção da equação química.
% impureza = 20% ⇒ Existem
40 g de impurezas na amostra. J. Problemas envolvendo reações

(Não reage, ou seja, não participa consecuti vas (mais de uma reação)
do problema). Neste caso, devemos escrever todas as rea-

ções envolvidas no problema e efetuar o ba-
I. Sistema em que o lanceamento. Observa-se que é possível efetu-
ar a soma algébrica dessas reações, bastando,
rendimento não é total para isso, multiplicar ou dividir as reações, a
Quando uma reação química não produz as fim de cancelarmos as substâncias intermedi-
quantidades de produto esperadas, de acor- árias e termos, então, uma única reação.

46
Química geral e estequiometria Química

A composição média de uma bateria automo- Segundo as condições do processo apresenta-


tiva esgotada é de aproximadamente 32% Pb, do para a obtenção de carbonato de chumbo
3% PbO, 17% PbO2 e 36% PbSO4. A média de (II) por meio da lixiviação por carbonato de
massa da pasta residual de uma bateria usada sódio e considerando uma massa de pasta re-
é de 6 kg, em que 19% são PbO2, 60% PbSO4 e sidual de uma bateria de 6 kg, qual quantida-
21% Pb. Entre todos os compostos de chum- de aproximada, em quilogramas, de PbCO3 é
bo presentes na pasta, o que mais preocupa obtida?
é o sulfato de chumbo (II), pois, nos processos a. 1,7 kg
pirometalúrgicos, em que os compostos de
b. 1,9 kg
chumbo (placas das baterias) são fundidos, há
a conversão de sulfato em dióxido de enxofre, c. 2,9 kg
gás muito poluente. d. 3,3 kg
Para reduzir o problema das emissões de SO2(g), e. 3,6 kg
a indústria pode utilizar uma planta mista, ou Resposta
seja, utilizar o processo hidrometalúrgico, para
a dessulfuração antes da fusão do composto de C
chumbo. Nesse caso, a redução de sulfato pre-
sente no PbSO4 é feita via lixiviação com solução
de carbonato de sódio (Na2CO3) 1 M a 45 °C, em
que se obtém o carbonato de chumbo (II) com
rendimento de 91%. Após esse processo, o ma-
terial segue para a fundição para obter o chum-
bo metálico.
PbSO4 + Na2CO3 → PbCO3 + Na2SO4
Dados: Massas molares em g/mol Pb = 207;
S = 32; Na = 23; O = 16; C = 12
ARAÚJO, R.V.V.; TINDADE, R.B.E.; SOARES, P.S.M.
Reciclagem de chumbo de bateria automotiva:
estudo de caso. Disponível em: <http://www.iqsc.
usp.br>. Acesso em: 17 abr. 2010. Adaptado.
PV-13-11

47
Química Química geral e estequiometria

ANOTAÇÕES

PV-13-11

48
Exercícios Propostos
Química geral e estequiometria Química

Capítulo 01
01. FAAP-SP Dessas afirmações, são corretas somente:
No texto: “Um escultor recebe um bloco retan- a. I e II.
gular de mármore e, habilmente, o transforma b. I e III.
na estátua de uma celebridade do cinema”, c. I e IV.
podemos identificar matéria, corpo e objeto e,
a partir daí, definir esses três conceitos. d. II e III.
I. Matéria (mármore): tudo aquilo que e. III e IV.
tem massa e ocupa lugar no espaço. 04. Cesgranrio-RJ
II. Corpo (bloco retangular de mármore): Determinou-se o ponto de fusão de uma subs-
porção limitada de matéria que, por tância X e encontrou-se um valor menor que o
sua forma especial, se presta a um de- tabelado para essa substância. Isso pode sig-
terminado uso. nificar que:
III. Objeto (estátua de mármore): porção a. a quantidade de substância utilizada
limitada de matéria. na determinação foi menor que o
Assinale: necessário.
a. se somente a afirmativa I é correta. b. a quantidade de substância utilizada
b. se somente a afirmativa II é correta. na determinação foi maior que o
necessário.
c. se somente a afirmativa III é correta.
c. uma parte da substância não fundiu.
d. se somente as afirmativas I e II são
corretas. d. a substância contém impurezas.
e. se as afirmativas I, II e III são corretas. e. a substância está 100% pura.
02. PUCCamp-SP 05. Mackenzie-SP
Em garimpos onde o ouro é encontrado em pó, A dureza de um mineral reflete a resistência
para separá-lo da areia acrescenta-se mercú- deste ao risco. Uma das escalas utilizadas para
rio líquido que forma liga metálica com ouro. verificar a dureza de um mineral é a escala de
Para separar os metais, a liga é aquecida até Mohs.
evaporação completa do mercúrio. Esse pro-
cedimento é possível porque dos dois metais, Escala de Mohs
(minerais em ordem crescente de dureza)
o mercúrio tem:
a. menor densidade. 1 - talco 6 - ortoclásio
b. menor massa molar. 2 - gesso 7 - quartzo
PV-13-14

c. menor temperatura de ebulição.


3 - calcita 8 - topázio
d. maior temperatura de fusão.
e. maior volume molar. 4 - fluorita 9 - coríndon

03. UFAL 5 - apatita 10 - diamante

Uma pessoa comprou um frasco de álcool ani-


dro. Para se certificar de que o conteúdo do De acordo com essa escala, é incorreto afir-
frasco não foi fraudado com a adição de água, mar que:
basta que ela determine, com exatidão: a. o diamante é o mineral mais duro.
I. a densidade; b. apenas o coríndon risca o diamante.
II. o volume; c. a apatita é riscada pelo quartzo.
III. a temperatura de ebulição; d. o topázio e a fluorita riscam a calcita.
IV. a massa. e. o mineral menos duro é o talco.

51
Química Química geral e estequiometria

06. Uespi a. Qual o volume ocupado pela “massa”


Um recipiente contendo 300 mL de água foi recém-preparada, correspondente a
esquecido em um freezer por cerca de 7 horas. uma receita?
Após este tempo, quando a porta do freezer b. Como se justifica o fato de a densidade
foi aberta, percebeu-se que: aparente da “massa” ser diferente da
a. o volume havia aumentado devido ao au- média ponderada das densidades apa-
mento da densidade da água quando no rentes dos constituintes?
estado sólido em relação à água líquida.
08. UFG-GO
b. o volume havia diminuído devido à di-
minuição da densidade da água quan- Analisando o diagrama de fases da água,
do no estado sólido em relação à água conclui-se que é possível liquefazer o gelo
líquida. por aumento de pressão. A 1,0 atm e –4 °C,
c. o volume havia diminuído devido ao por exemplo, essa pressão é da ordem de
aumento da densidade da água quan- 140 atm. Esse processo é apresentado, atra-
do no estado sólido em relação à água vés de um modelo simplificado, em livros
líquida. didáticos do Ensino Médio, quando se consi-
d. o recipiente permanecia com o mesmo dera, por exemplo, que um patinador desliza
volume, pois a densidade da água não no gelo com base apenas nesse fenômeno.
aumentou nem diminuiu.
Desse modo, considere um patinador sobre o
e. o volume havia aumentado devido à di- gelo usando um patim conforme a especifica-
minuição da densidade da água quan-
ção da figura abaixo
do no estado sólido em relação à água
líquida.
07. Unicamp-SP
Uma receita de biscoitinhos petit four de la-
ranja leva os seguintes ingredientes:
Quantidade/ Densidade
Ingredientes
Gramas aparente g/cm3

Farinha de trigo 360 0,65

Carbonato
ácido de 8 1,5
amônio 20 cm

Sal 1 2,0 Com base nas informações acima, calcule a


PV-13-14

Manteiga 100 0,85 massa, em kg, que o patinador deve ter, de


modo a liquefazer o gelo por pressão, e con-
Açúcar 90 0,90 firme se o modelo é, ou não, adequado.
Ovo 100 (2 ovos) 1,05 Dados:
Raspas de casca g = 10 m/s2
3 0,50
de laranja
1 atm = 105 N/m2
A densidade aparente da “massa” recém-pre- a. 11, não.
parada e antes de ser assada é de 1,10 g/cm3 . b. 40, sim.
Entende-se por densidade aparente a relação c. 80, sim.
entre a massa da “massa” ou do ingrediente,
na “forma” em que se encontra, e o respectivo d. 140, não.
volume ocupado. e. 280, não.

52
Química geral e estequiometria Química

09. Unicamp-SP c. I – fusão; II – sublimação; III – evapora-


Qual o estado físico (sólido, líquido ou gasoso) ção; IV – solidificação.
das substâncias da tabela a seguir, quando as mes- d. I – evaporação; II – solidificação; III – fu-
mas se encontram no deserto da Arábia, à tempe- são; IV – sublimação.
ratura de 50 °C (pressão ambiente = 1 atm)? e. I – evaporação; II – sublimação; III – fu-
são; IV – solidificação.
Substância TF TE
12. Mackenzie-SP
Clorofórmio –63 61 As fases de agregação para as substâncias a se-
guir, quando expostas à temperatura de 30 °C,
Éter etílico –116 34 são, respectivamente:

Etanol –117 78
Ponto de fusão Ponto de ebulição
Material
(°C) (1 atm) (°C) ( 1 atm)
Fenol 41 182

Pentano –130 36 Mercúrio –38,87 356,9

Amônia –77,7 –33,4


TF = temperatura de fusão em °C
Benzeno 5,5 80,1
TE = temperatura de ebulição em °C
(os dados da tabela estão em 1 atm) Naftaleno 80,0 217,0

10. UFRN a. sólido, líquido, gasoso e líquido.


Quitéria, para combater traças e baratas, foi b. líquido, sólido, líquido e gasoso.
aconselhada a colocar no guarda-roupa algu- c. líquido, gasoso, líquido e sólido.
mas bolinhas de naftalina (C10 H8). Com o pas-
sar do tempo, notou que as bolinhas diminu- d. gasoso, líquido, gasoso e sólido.
íam de tamanho. Buscando nos livros alguma e. sólido, gasoso, líquido e gasoso.
explicação para o curioso fato, encontrou que 13. Unesp
esse fenômeno é causado pela:
a. evaporação. Alquimia subterrânea transforma
b. sublimação. mina de carvão em mina de hidrogênio
c. fusão. Em uma área de mineração de carvão
localizada no sul da Polônia, um grupo de
d. condensação. cientistas está usando uma mina de carvão
11. para avaliar experimentalmente um método
alternativo para a produção de energia limpa
Observe os seguintes fatos:
PV-13-14

e, assim, oferecer uma utilização para pe-


I. Uma pedra de naftalina deixada no ar- quenos depósitos de carvão ou minas exau-
mário. ridas, que são tradicionalmente deixados de
II. Uma vasilha com água deixada no freezer. lado, representando passivos ambientais.
Na teoria e no laboratório, a injeção
III. Uma vasilha com água deixada no fogo. de oxigênio e de vapor no carvão resulta
IV. O derretimento de um pedaço de na produção de hidrogênio. No proces-
chumbo quando aquecido. so, oxigênio líquido é colocado em um
A esses fatos estão relacionados corretamente reservatório especial, localizado nas ga-
lerias da mina de carvão, onde se trans-
os seguintes fenômenos:
forma em oxigênio gasoso, começando
a. I – sublimação; II – solidificação; III – o processo denominado de gaseificação
evaporação; IV – fusão. de carvão.
b. I – sublimação; II – solidificação; III – fu- Disponível em: <http://www.
são; IV – evaporação. inovacaotecnologica.com.br>. Adaptado.

53
Química Química geral e estequiometria

A passagem do oxigênio líquido para oxigênio b. parte da água, ao passar pelos poros do
gasoso é uma transformação física: pote de barro, retira energia das molé-
a. exotérmica, classificada como fusão. culas do vapor d’água presentes fora do
pote. Neste processo, o vapor d’água,
b. exotérmica, classificada como ebulição. ao se condensar, absorve energia (ca-
c. endotérmica, classificada como lique- lor) das paredes do pote, esfriando-o.
fação. c. parte da água, ao atravessar os poros
d. endotérmica, classificada como eva- do pote de barro, reage com as partí-
poração. culas constituintes do barro. Como esta
reação é endotérmica, absorve energia
e. espontânea, classificada como subli-
(calor) das paredes do pote, esfriando-o.
mação.
d. parte da água, ao atravessar os poros do
14. UNIR-RO pote de barro, reage com as partículas
Considere os pontos de fusão (PF) e de ebuli- constituintes do barro. Como esta rea-
ção (PE) das seguintes substâncias: ção é exotérmica, absorve energia (ca-
lor) das paredes do pote, esfriando-o.
e. a água, por ficar muito tempo “parada”
Material PF (1 atm, °C) PE (1 atm, °C) dentro do pote, perde energia cinética,
resfriando-se.
Etanol –117 78 16. UFSCar-SP
Clorofórmio –63 61 Balões de festa de aniversário deixados sob sol
forte podem estourar porque o volume do gás
Iodo 113,5 184 contido em seu interior aumenta com o aumen-
to da temperatura e acaba rompendo a superfí-
Éter etílico –116 34
cie do balão depois que esta se estica até um ta-
manho máximo. Isso ocorre porque o aumento
Analisando os dados da tabela e considerando da temperatura eleva a energia das partículas
28 °C e 1 atm, pode-se afirmar que o estado que, com maior movimento, passam a ocupar
físico de cada substância é, respectivamente: um volume maior, no caso das moléculas dos
a. Sólido, gasoso, gasoso, líquido gases contidos no balão. A variação da energia
das partículas com a variação da temperatura
b. Líquido, líquido, gasoso, sólido também causa a compactação, que reduz o vo-
c. Líquido, líquido, sólido, líquido lume de substâncias ao se solidificarem com o
d. Gasoso, gasoso, líquido, sólido abaixamento da temperatura, quando a energia
das partículas diminui. Com a água é diferente:
e. Líquido, líquido, gasoso, líquido
ao passar do estado líquido para o estado sóli-
15. UESPI do, com o abaixamento da temperatura ocorre
aumento de volume, por isso não se deve co-
PV-13-14

Em algumas regiões do Brasil, principalmente


locar no congelador garrafa cheia com água e
aquelas com pouco acesso à energia elétrica,
lacrada. Assinale a alternativa que explica cor-
é comum as pessoas utilizarem potes de barro retamente o comportamento da água ao passar
para conservar água a uma temperatura um do estado líquido para estado sólido.
pouco mais fria que a do ambiente. Este fenô-
a. No estado sólido, as moléculas de água
meno, que para muitos ainda é considerado
formam pontes de hidrogênio com a
uma “crendice popular”, pode ser explicado, superfície interna da garrafa, que se
cientificamente, pelo fato de que: rompe devido ao efeito da pressão adi-
a. parte da água, ao atravessar os poros cional exercida.
do pote de barro, passa do estado lí- b. Na água líquida, há sais minerais dissol-
quido para o estado gasoso (evapora- vidos, que se cristalizam quando o gelo
ção). Neste processo, a água absorve é formado e ocupam maior espaço.
energia (calor) das paredes do pote, Assim, o volume do gelo é maior que o
esfriando-o. volume da água líquida.

54
Química geral e estequiometria Química

c. Na formação do gelo, as moléculas 18. UEPG-PR


de água assumem posições definidas As mudanças de estado físico, classificadas
numa estrutura organizada, mantida
como fenômenos físicos, ocorrem com a varia-
por ligações de hidrogênio. As molécu-
las nesta estrutura ocupam maior es- ção de entalpia (∆H). Sobre esses processos,
paço do que no estado líquido, onde se assinale o que for correto.
organizam de maneira diferente. 01. A fusão é um processo endotérmico
d. No estado sólido, as moléculas de água com ∆H > 0.
formam estrutura cristalina tridimen- 02. A produção de vapor a partir do estado
sional, com átomos unidos por ligação líquido é um processo exotérmico.
iônica em posições fixas, que ocupam 04. A condensação é um processo exo-
maior espaço que a geometria planar térmico.
das moléculas no estado líquido.
08. A variação de entalpia (∆H) é menor do
e. A autoionização da água, responsável que zero apenas quando na mudança
pelas espécies iônicas que são unidas de estado ocorre absorção de calor.
por ligações iônicas no estado líquido,
é desfavorecida com o abaixamento da 16. Na sublimação ocorre a passagem do
temperatura. Assim, a força de ligação estado sólido diretamente para o gaso-
das moléculas de água no gelo é mais so, com absorção de calor.
fraca e a distância entre elas aumenta, 19. Uncisal
aumentando o volume ocupado.
A rapadura, um produto sólido de sabor doce,
17. Unicamp-SP tradicionalmente consumida pela população do
Numa entrevista à Revista no163, um astrofí- Nordeste do Brasil, originou-se das crostas pre-
sico brasileiro conta que propôs, em um arti- sas às paredes dos tachos, durante a fabricação
go científico, que uma estrela bastante velha do açúcar. Atualmente, o posicionamento da
e fria (6.000 K), da constelação de Centauro, rapadura como “produto natural” ou “produ-
tem um núcleo quase totalmente cristalizado. to rural” é um valor agregado que a diferencia
Esse núcleo seria constituído principalmente do açúcar refinado, seu principal concorrente.
de carbono e a estrela estaria a caminho de se A produção da rapadura, a partir do caldo de
transformar em uma estrela de diamante, com cana, envolve as etapas apresentadas a seguir
a cristalização do carbono.
Caldo de cana
a. O pesquisador relata ter identificado
mais 42 estrelas com as mesmas carac- Concentração
terísticas e afirma: "Enquanto não termi-
na o processo de cristalização do núcleo, Moldagem e
as estrelas de diamante permanecem Cristalização secagem
com a temperatura constante". No que
diz respeito à temperatura, independen- Açúcar mascavo Rapadura Mel
temente de seu valor absoluto, ele com-
PV-13-14

plementa essa afirmação fazendo uma Comercialização


analogia entre o processo que ocorre na e distribuição
estrela e a solidificação da água na Terra.
Com base no conhecimento científico, Na concentração, o caldo de cana é aquecido até
você concorda com a analogia feita pelo transformar-se em um xarope denso e viscoso
pesquisador? Justifique. que borbulha no tacho. Quando atinge esse pon-
b. Ao final da reportagem, afirma-se que: to, o xarope é transferido para um tipo de tanque
"No diamante da estrela, apenas 0,01 Å redondo onde é moldada a rapadura.
separa os núcleos dos átomos do ele-
mento que o compõem". Considerando- A concentração do caldo de cana ocorre por-
se que o raio atômico do carbono no que a água está sendo eliminada por:
diamante da Terra é de 0,77 Å, quanto a. destilação. d. solidificação.
valeria a relação numérica entre os vo-
lumes atômicos do carbono (Terra/es- b. evaporação. e. condensação.
trela)? Mostre seu raciocínio. c. sublimação.

55
Química Química geral e estequiometria

20. ENEM Considerando o procedimento anterior, a água


Sob pressão normal (ao nível do mar), a água volta a ferver porque esse deslocamento:
entra em ebulição à temperatura de 100 °C. a. permite a entrada de calor do ambien-
Tendo por base essa informação, um garoto te externo para o interior da seringa.
residente em uma cidade litorânea fez a se-
guinte experiência: b. provoca, por atrito, um aquecimento
• Colocou uma caneca metálica contendo da água contida na seringa.
água no fogareiro do fogão de sua casa. c. produz um aumento de volume que au-
• Quando a água começou a ferver, en- menta o ponto de ebulição da água.
costou cuidadosamente a extremidade d. proporciona uma queda de pressão no
mais estreita de uma seringa de injeção, interior da seringa que diminui o ponto
desprovida de agulha, na superfície do
de ebulição da água.
líquido e, erguendo o êmbolo da serin-
ga, aspirou certa quantidade de água e. possibilita uma diminuição da densida-
para seu interior, tapando-a em seguida. de da água que facilita sua ebulição.
• Verificando após alguns instantes que a
água da seringa havia parado de ferver, ele
ergueu o êmbolo da seringa, constatando,
intrigado, que a água voltou a ferver após
um pequeno deslocamento do êmbolo.

PV-13-14

56
Química geral e estequiometria Química

Capítulo 02
21. PUC-RS
A substância simples constituída somente por moléculas é:
a. o metano.
b. o cloreto de sódio.
c. o alumínio.
d. a amônia.
e. o ozônio.
22. UFU-MG
Observe os dados apresentados na tabela a seguir.

Substância Molécula

Amônia NH3

Hélio He

Fósforo branco P4

Ácido sulfúrico H2SO4

É incorreto afirmar que:


a. o hélio é uma substância simples.
b. a molécula de fósforo branco possui 4 átomos de fósforo.
c. a proporção de átomos na molécula de amônia é 1 : 3.
d. a molécula de ácido sulfúrico possui 2 átomos de enxofre.
23. Fuvest-SP
Hidrogênio reage com nitrogênio formando amônia. A equação não balanceada que representa
essa transformação é:
H2(g) + N2(g) → NH3(g)
Outra maneira de escrever essa equação química, mas agora balanceando-a e representando as
moléculas dos três gases, é:
PV-13-14

a. +

b. +

c. +

d. +

e. +

Obeservação: e representam átomos.

57
Química Química geral e estequiometria

24. UESPI 25. UEG-GO


A descrição abaixo que pode ser considerada A curva de aquecimento representada abaixo:
como a descrição de uma substância simples é
Temperatura (°C)
a seguinte:
a. Um líquido azul que pode ser separado
em dois compostos pela cromatografia. T2 (IV)
b. Cristais cor de rosa que, pelo aqueci-
T1 (III)
mento, produzem vapor de água, per- (II)
manecendo uma parte sólida. (I)
c. Um sólido negro que pode queimar Tempo
completamente no oxigênio, resultan-
a. mostra que o calor do processo III é la-
do um único produto, que é um óxido.
tente.
d. Um líquido incolor que queima no oxigê-
nio formando dióxido de carbono e água. b. refere-se a uma mistura eutética.
e. Um líquido preto que produz frações c. refere-se a uma substância pura.
diferentes pela destilação. d. refere-se a uma mistura azeotrópica.

26.
Dois béqueres iguais, de capacidade calorífica desprezível, contendo quantidades diferentes
de água pura a 25 °C, foram aquecidos, sob pressão constante de 1 atm, em uma mesma
chama. A temperatura da água em cada béquer foi medida em função do tempo de aque-
cimento, durante 20 minutos. Após esse tempo, ambos os béqueres continham expressivas
quantidades de água. Os resultados encontrados estão registrados nos gráficos a seguir.
Temperatura Temperatura
Patamar Patamar
Béquer A Béquer B

TA TB

0,0 5,0 10,0 15,0 20,0 0,0 5,0 10,0 15,0 20,0
PV-13-14

Tempo/minuto Tempo/minuto

a. Indique o valor das temperaturas TA e TB .


Justifique sua resposta.
b. Indique o béquer que contém maior quantidade de água.
Justifique sua resposta.
c. Calcule a massa de água no béquer B, caso o béquer A contenha 200 g de água.
Indique seu cálculo.
d. Indique qual dos dois gráficos apresentaria um patamar maior se a temperatura dos bé-
queres continuasse a ser anotada até a vaporização total da água.
Justifique sua resposta.

58
Química geral e estequiometria Química

27. PUCCamp SP guir, que representa a variação da temperatu-


Marte é permanentemente encoberto por ca- ra em função do tempo.
lotas polares, em ambos os polos, compos-
tas, em sua maior parte, de dióxido de car- 120
bono. Durante o verão norte, o dióxido de 100
carbono sublima completamente, deixando 98
80
uma camada residual de gelo de água. Esse
processo que ocorre no verão norte para o 60
dióxido de carbono pode ser representado 40
graficamente por:
20

a. 0 5 10 15 20
Temperatura

Gás
Tempo (minutos)

Sólido De acordo com as informações sobre esse pro-


Tempo cesso e com os dados apresentados no gráfico,
é correto afirmar que:
a. a água muda do estado líquido para o
b. gasoso, a 100 °C.
Temperatura

Líquido
b. a amostra é constituída por um único
tipo de molécula.
Sólido
c. o material se apresenta totalmente ga-
Tempo soso, a 98 °C.
d. o experimento foi realizado ao nível
c. do mar.
Temperatura

Gás
29. UFAC
O gráfico abaixo mostra a curva de aquecimen-
Líquido to para o clorofórmio, usualmente utilizado
Tempo como solvente para lipídeos.

100 E
d.
Temperatura

Sólido D
60
Gás 20 C
–20
Tempo B
–60 A
PV-13-14

–100
e.
Temperatura

Líquido
Analisando a curva, observa-se que a tempe-
Gás ratura de fusão, a temperatura de ebulição e o
estado físico do clorofórmio nos segmentos A
Tempo e D são, respectivamente:
a. 60 °C, –60 °C, sólido e gás.
28. UEMG b. –60 °C, 60 °C, sólido e líquido.
Um estudante aqueceu uma amostra de água c. –60 °C, 60 °C, sólido e mudança de lí-
quido para gás.
líquida por, aproximadamente, 18 minutos.
Durante o processo, ele mediu e anotou a d. 60 °C, –60 °C, líquido e gás.
temperatura da amostra, a cada 30 segundos. e. –60 °C, 60 °C, líquido e mudança de lí-
Com os dados obtidos, ele fez o gráfico, a se- quido para gás.

59
Química Química geral e estequiometria

30. 32. Unicamp-SP


T (°C) Três frascos não rotulados encontram-se na prate-
leira de um laboratório. Um contém benzeno,
outro, tetracloreto de carbono, e o terceiro,
60 metanol. Sabe-se que as suas densidades são:
0,87 g/cm3 (benzeno); 1,59 g/cm3 (tetracloreto
40 de carbono) e 0,79 g/cm3 (metanol). Dos três
líquidos, apenas o metanol é solúvel na água,
20 cuja densidade é 1,00 g/cm3. Com base nessas
informações, explique como você faria para
reconhecer os três líquidos.
1 2 3 4 5 6 7 8 t (min)
Observação: os três líquidos são altamente tó-
Analisando o gráfico acima, referente ao aque- xicos e não devem ser cheirados.
cimento de uma substância sólida, podemos 33. UFPE
afirmar que: Para identificar três líquidos – de densidades
a. quando t = 2 minutos, tem-se um siste- 0,8, 1,0 e 1,2 – o analista dispõe de uma peque-
ma monofásico. na bola de densidade = 1,0. Conforme a posição
b. quando t = 4 minutos, coexistem subs- das bolas apresentadas no desenho abaixo, po-
tância sólida e substância líquida. demos afirmar que:
c. em t = 1 minuto, inicia-se a liquefação
da substância.
d. a substância tem ponto de fusão igual
a 40 °C.
e. no intervalo de 5 a 8 minutos, a subs-
tância encontra-se totalmente na for-
ma de vapor.
31. Unicap-PE 1 2 3

Julgue os itens abaixo. a. os líquidos contidos nas provetas 1, 2 e


0. Qualquer porção de qualquer mate- 3 apresentam densidades 0,8, 1,0 e 1,2.
rial possui massa e ocupa lugar no b. os líquidos contidos nas provetas 1, 2 e
espaço. 3 apresentam densidades 1,2, 0,8 e 1,0.
1. Quando afirmamos que a densidade do c. os líquidos contidos nas provetas 1, 2 e
alumínio é de 2,7 g/cm3, estamos afir- 3 apresentam densidades 1,0, 0,8 e 1,2.
mando que, se pesarmos um volume
PV-13-14

d. os líquidos contidos nas provetas 1, 2 e


de alumínio puro igual a 1 cm3, obtere- 3 apresentam densidades 1,2, 1,0 e 0,8.
mos uma massa de 2,7 g.
e. os líquidos contidos nas provetas 1, 2 e
2. Quando dois materiais possuem densi- 3 apresentam densidades 1,0, 1,2 e 0,8.
dades diferentes, sob a mesma pressão
34. Mackenzie-SP
e temperatura, podemos afirmar que
se trata de materiais diferentes. Num recipiente calibrado contendo 485 mL de
3. Quando temos volumes iguais de ma- água (d = 1,00 g/cm3), colocou-se um objeto
teriais diferentes, o material de maior (feito de um único material) de massa igual
a 117 g. Observou-se que o objeto imerge e
densidade apresenta maior massa.
que o nível da água no recipiente passa a ser
4. Quando temos massas iguais de ma- de 500 mL. Com esses dados e consultando a
teriais diferentes, o material de maior tabela a seguir, pode-se afirmar que o objeto
densidade apresenta o maior volume. pode ser feito de:

60
Química geral e estequiometria Química

Material Densidade (g/cm3)

Chumbo 11,3

Ferro 7,8

Osso 2,0
Rômbico Monoclínico
Cortiça 0,3
A respeito do enxofre rômbico e do enxofre
Pedra 5,0 monoclínico, é correto afirmar que eles se
constituem em:
a. chumbo. a. formas alotrópicas do elemento quími-
co enxofre, cuja fórmula é S8.
b. ferro.
b. átomos isótopos do elemento químico
c. osso.
enxofre, cujo símbolo é S.
d. cortiça.
c. átomos isótopos do elemento químico
e. pedra. enxofre, cuja fórmula é S8.
35. ITA-SP d. formas alotrópicas do elemento quími-
Considere que sejam feitas as seguintes afir- co enxofre, cujo símbolo é S.
mações a respeito das formas cristalinas do e. formas isobáricas da substância quími-
carbono: ca enxofre, cujo símbolo é S.
I. As formas polimórficas do carbono são: 37. UFOP-MG
diamante, grafite e fulerenos. Nas CNTP, o carbono é sólido e pode existir
II. O monocristal de grafite é bom condu- como grafite ou como diamante, que são suas
tor de corrente elétrica em uma dire- formas mais importantes. Essas duas formas
ção, mas não o é na direção perpendi- do carbono têm:
cular à mesma. a. propriedades diferentes e as mesmas
III. O diamante é uma forma polimórfica estruturas cristalinas.
metaestável do carbono nas condições b. propriedades diferentes e estruturas
normais de temperatura e pressão. cristalinas diferentes.
IV. No grafite, as ligações químicas entre c. as mesmas propriedades e as mesmas
os átomos de carbono são tetraédricas. estruturas cristalinas.
Então, das afirmações acima, está(ão) corre- d. as mesmas propriedades e estruturas
PV-13-14

ta(s): cristalinas diferentes.


a. apenas I, II e III. 38. UFF-RJ
b. apenas I e III. “Diamante” é uma palavra que vem do latim e
significa “inflexível”. Por isso se diz que os diaman-
c. apenas II e IV. tes são eternos. Eles são formados pelo carbono
d. apenas IV. submetido a calor e pressão extremos, a partir de
e. todas. rocha vulcânica. As principais formas alotrópicas
do carbono são o grafite e o diamante.
36. UFPel-RS
O enxofre é uma substância simples cujas mo-
léculas são formadas pela união de 8 átomos;
logo, sua fórmula molecular é S8. Quando no es-
tado sólido, ele forma cristais do sistema rômbi-
co ou monoclínico, conforme figura abaixo. Diamante Grafite

61
Química Química geral e estequiometria

Sobre essas formas alotrópicas, assinale a al-


ternativa correta.
a. O grafite que apresenta carbono com hi-
bridização sp3 não conduz eletricidade.
b. Os átomos de carbono possuem o mes-
mo tipo de hibridização no diamante e
no grafite.
c. Os átomos de carbono, no diamante,
estão separados por ângulos de 180.
d. Os átomos de carbono possuem hibridi- a. Explique por que grafite e diamante se
zação sp2 no diamante e sp3 no grafite. comportam de forma diferente em re-
e. O diamante possui átomos de carbono lação ao fenômeno de fusão.
com hibridização sp3. b. Dê a hibridização dos átomos de carbo-
39. UEG-GO no no grafite e no diamante e explique
o fato de um deles ser isolante e o ou-
O carbono é um elemento químico sólido à
tro, um bom condutor de eletricidade.
temperatura ambiente. Dependendo das con-
dições de formação, pode ser encontrado na 40. UFMG
natureza em diversas formas alotrópicas, en-
Dois tubos de ensaio contêm volumes iguais
tre as quais, grafite e diamante. Esse elemento
de líquidos. O tubo 1 contém água destilada e
é o pilar básico da química orgânica, fazendo
parte de todos os seres vivos. As principais o tubo 2, água com sal de cozinha completa-
propriedades dos dois alótropos mencionados mente dissolvido.
no texto estão organizadas na tabela abaixo. Ao se aquecerem simultaneamente esses tu-
Use-a, juntamente com a figura, para respon- bos, observa-se que a água do tubo 1 entra em
der aos itens subsequentes. ebulição antes da solução do tubo 2.

Grafite Diamante Considerando-se esse experimento, é correto


afirmar que a diferença de comportamento
Bom condutor de
Isolante dos dois líquidos se explica porque:
eletricidade
a. a temperatura de ebulição da solução
Lubrificante seco de alta Máxima dureza na é mais alta, para que o sal também se
eficiência escala de Mohs vaporize.
Densidade igual a b. a temperatura de ebulição da solução
Densidade igual a 2,2 g · cm–3
3,5 g · cm–3 é mais alta, pois as ligações iônicas do
sal, a serem quebradas, são fortes.
Cristais opacos Cristais translúcidos
PV-13-14

c. a água destilada, sendo uma substân-


Infusível, exceto cia simples, entra em ebulição antes da
Ponto de fusão: ≅ 4.600 K
em altas pressões mistura de água com sal.
d. a água destilada, sendo uma substância
pura, entra em ebulição a uma tempe-
ratura mais baixa.
41. UEMG
O modelo a seguir representa o estado inicial
de um sistema em que átomos de um mesmo
elemento químico são representados por esfe-
ras de mesma cor, e átomos de elementos quí-
micos distintos são representados por esferas
de cores diferentes.

62
Química geral e estequiometria Química

43. UEPG-PR
Estão representados abaixo quatro sistemas
diferentes, nos quais as figuras de mesma for-
ma e cor representam o mesmo elemento quí-
mico. Com base nessas informações, assinale
o que for correto.
Assinale a alternativa que corresponde ao mo-
delo correto para o sistema final, após uma rea-
ção química envolvendo as moléculas represen-
tadas no sistema inicial, acima descrito.
a.
(I)

b.

(II)

c.

(III)

d.

(IV)
PV-13-14

01. O sistema I contém somente substân-


42. UFAC cias simples.
Fazendo uma classificação entre substâncias 02. No sistema II ocorre alotropia.
puras e misturas, quais dos seguintes mate-
riais seriam classificados como substâncias 04. O sistema III contém substância pura.
puras: ar, gás carbônico, amônia, prata, aço 08. Os sistemas I e IV contêm misturas.
inoxidável, bronze. 44.
a. Gás carbônico, ar e aço inoxidável
b. Prata, aço inoxidável e amônia Como se pode mostrar graficamente o resfria-
mento da mistura azeotrópica: álcool (32%) e
c. Gás carbônico, aço inoxidável e prata
benzeno (68%)?
d. Bronze, ar e amônia
PE do azeótropo = 68,2°C
e. Gás carbônico, prata e amônia

63
Química Química geral e estequiometria

45. c. durante a determinação da massa e do


volume, ela se encontrava no estado
Uma mistura eutética muito importante na
líquido.
metalurgia é a mistura formada por zinco (20%)
e estanho (80%). Sabe-se que o zinco funde a d. a 80 °C, ela será um gás.
418 °C e o estanho, a 232 °C. No entanto, o e. é constituída do material C.
eutético formado por eles funde a 192 °C. Re-
presente em um gráfico θ (°C) x t (min) o aque- 47. UEL-PR
cimento dessa importante liga desde o estado Considere as fiiguras a seguir.
sólido até o estado gasoso.
Texto para as questões 46 e 47.
A massa e o volume dos minerais A, B e C foram
determinados a 30 °C; amostras sólidas dos três
materiais foram aquecidas, mantendo a tempe- Com base nas informações disponíveis sobre
ratura controlada a partir de 0 °C durante todo os materiais A, B e C, e sabendo-se que eles
esse processo de aquecimento. Os gráficos re- não interagem entre si, a figura que melhor
presentam os resultados obtidos. representa uma mistura dos três materiais, a
46. UEL-PR 30 °C, é a:
A massa e o volume da amostra de um dos a. I
três materiais foram determinados a 30 °C, b. II
encontrando-se os valores de 25 g e 50 mL, c. III
respectivamente. Com base nessa informa- d. IV
ção e nas figuras I e II, é incorreto afirmar,
com relação à amostra, que: e. V
48. Unesp
m/g A B Em um laboratório, foi encontrado um frasco,
4 sem identificação, contendo um pó branco
cristalino. Aquecendo este pó com taxa cons-
3 C tante de fornecimento de calor, foi obtida a
2 seguinte curva de aquecimento:
1 T (°C)
0
1 2 3 4 V/mL

T/°C B A
PV-13-14

100
C
80 Sólido Sólido + Líquido Líquido
60 Tempo (minutos)
40
Pode-se afirmar que o pó branco encontrado é:
20
a. uma substância simples.
0 2 4 6 8 10 t/min b. uma substância composta.
igura II c. uma mistura de cristais com tamanhos
diferentes.
a. o seu ponto de ebulição é de 60 °C. d. uma mistura de duas substâncias.
b. é constituída do material mais denso e. uma mistura de três substâncias.
entre os três.

64
Química geral e estequiometria Química

49. Fuvest-SP a. mistura alotrópica.


Aquece-se uniformemente uma solução aquosa b. mistura binária.
de cloreto de sódio, sob pressão de uma atmos-
fera. Qual dos gráficos abaixo melhor representa c. mistura osmótica.
a temperatura da solução em função do tempo? d. mistura azeotrópica.
e. mistura isotônica.
q
a. 100 °C 51. UEPG-PR
Considerando as substâncias e as suas mistu-
ras, assinale o que for correto.
t 01. Durante as mudanças de estado físico
das substâncias puras, a temperatura
q não permanece constante, diferente-
b. 100 °C mente das misturas.
02. Substâncias simples são formadas por
átomos de vários elementos.
t 04. Alótropos são substâncias simples com
diferentes estruturas formadas de áto-
q mos de um mesmo elemento.
c. 100 °C 08. O número de componentes numa mis-
tura não é necessariamente igual ao
número de fases do sistema.
t
16. Substâncias compostas podem ser se-
paradas por filtração.
52. Unifor-CE
q
O elemento estanho (Sn) apresenta as seguin-
d. 100 °C tes propriedades físicas (constantes físicas):
ponto de fusão = 232 °C, ponto de ebulição =
2.270 °C (sob pressão de 1 atm), entalpia molar
de fusão = 536 cal/mol.
t Considere o aquecimento, sob pressão de 1 atm,
em aparelhagem adequada, de um mol de esta-
q nho de 25 °C até 2.500 °C.
e. 100 °C Obs.: (patamar = região de temperatura
PV-13-14

constante)
O diagrama (gráfico) da temperatura versus
tempo de aquecimento deve ter:
t a. dois patamares, um deles na tempera-
tura de 232 °C, o outro na temperatura
50. Unioeste-PR de 2.270 °C. Ao primeiro patamar cor-
responde a absorção de 536 cal.
Na pressão de 1 atm, a água pura entra em
b. dois patamares, um deles na tempera-
ebulição a 100 °C e o etanol puro a 78,4 °C.
tura de 232 °C, o outro na temperatura
Uma mistura contendo 95% de etanol e 5% de 2.270 °C. Ao segundo patamar cor-
de água atinge a ebulição a 78 °C na mesma responde a absorção de 536 cal.
pressão de 1 atm, destilando como se fosse c. somente um patamar na temperatura
uma substancia pura. Esse tipo de mistura é de 232 °C . A este patamar corresponde
denominada: a liberação de 536 cal.

65
Química Química geral e estequiometria

d. somente um patamar na temperatura a. uma substância pura.


de 2.270 °C . A este patamar correspon- b. uma mistura heterogênea.
de a absorção de 536 cal.
c. uma mistura azeotrópica.
e. uma curva ascendente. A temperatu-
ra cresce continuamente de 25 °C até d. uma mistura eutética.
2.500 °C . Há absorção de 536 cal. e. uma mistura bifásica.
53. UEPG-PR 55. UESC-BA
O gráfico abaixo registra o aquecimento de um Propriedades, como ponto de fusão, ponto de
líquido transparente e incolor, até sua comple- ebulição e densidade, que são importantes para
ta evaporação. identificar as substâncias puras e diferenciá-las
T (°C)
de misturas, não dependem de quantidade de
120 substância, mas tão somente da sua natureza.
100 Temperatura (°C)
80 Vapor
60
40
20 Líquido
0
1 2 3 4 5 6 7 8 9 10 11 12 13 14 15 183
Tempo (min)
Sólido
A respeito desse líquido, assinale o que for 0 Tempo
correto.
01. É uma mistura azeotrópica. PERUZZO, Tito Maragaia; CANTO, Eduardo
Leite de. Química. São Paulo:
02. É uma mistura de duas substâncias. Moderna, v. 1, 2007, p. 47.
04. É uma substância pura.
08. É uma solução verdadeira. Uma análise dessas informações e do gráfico
que representa a curva de aquecimento da
16. É uma mistura homogênea. solda, material sólido utilizado em componen-
54. Fatec-SP tes e equipamentos eletrônicos, permite con-
O gráfico representa a curva de aquecimento, cluir corretamente:
temperatura em função do tempo, do álcool de 01. A solda é uma mistura porque apresen-
supermercado, à pressão de 1 atm. ta temperatura de fusão constante e
Temperatura (°C) temperatura de ebulição variável.
Vapor 02. A vaporização da solda evidencia o
PV-13-14

equilíbrio físico entre a fase líquida e a


fase gasosa em recipiente aberto.
03. A solda é uma substância pura porque
Tebulição = constante apresenta temperatura de ebulição
78
Líquido constante.
Faixa de 04. As propriedades, como ponto de fusão
temperatura e densidade, são denominadas proprie-
Sólido dades gerais da matéria.
0 Tempo 05. A solda de fios de componentes eletrô-
nicos só é possível quando a tempera-
Após a análise do gráfico, pode-se afirmar que tura desse material atinge 180 °C.
o álcool de supermercado é:

66
Química geral e estequiometria Química

56. Unimontes-MG
No processo de resfriamento de uma mistura de 40% de cádmio e 60% de bismuto, a cristalização
desses metais inicia-se a 270 °C e termina a 140 °C, quando a solução atinge a composição eutética. A
recristalização encontra-se corretamente representada através da curva:

a. T/°C c. T/°C

270 270

140 140

Tempo Tempo

b. T/°C d. T/°C
270 270
140 140

Tempo Tempo

57. Unesp
No campo da metalurgia é crescente o interesse nos processos de recuperação de metais, pois
é considerável a economia de energia entre os processos de produção e de reciclagem, além da
redução significativa do lixo metálico. E este é o caso de uma microempresa de reciclagem, na qual
se desejava desenvolver um método para separar os metais de uma sucata, composta de, aproxi-
madamente, 63% de estanho e 37% de chumbo, usando aquecimento. Entretanto, não se obteve
êxito nesse procedimento de separação. Para investigar o problema, foram comparadas as curvas
de aquecimento para cada um dos metais isoladamente com aquela da mistura, todas obtidas sob
as mesmas condições de trabalho.

T/ °C T/ °C T/ °C

Gás
1.749 Gás 2.602 Gás ∆

Líquido Líquido Líquido


PV-13-14

327 232 183


Sólido Sólido Sólido
t/h t/h Mistura t/h
Chumbo Estanho (37% chumbo;
63% estanho)

Considerando as informações das figuras, é correto afirmar que a sucata é constituída por uma:
a. mistura eutética, pois funde à temperatura constante.
b. mistura azeotrópica, pois funde à temperatura constante.
c. substância pura, pois funde à temperatura constante.
d. suspensão coloidal que se decompõe pelo aquecimento.
e. substância contendo impurezas e com temperatura de ebulição constante.

67
Química Química geral e estequiometria

58. Udesc 59. Unifor-CE


Analise as proposições em relação aos gráficos O gráfico a seguir representa a variação da
abaixo. temperatura em função do tempo para uma
determinada amostra líquida.
Temperatura (°C)

Temperatura
D

(d) (e)
B
C
(b) (c)
A
(a)
Tempo
Tempo
(X) Os resultados obtidos permitem afirmar que:
I. no trecho A-B, está ocorrendo a ebuli-
ção da mistura.
Temperatura (°C)

II. a partir do ponto C, só há substâncias


no estado gasoso.
III. o líquido é uma mistura.
(d)
(e) Está correto o que se afirma somente em:
(b) (c) a. I. c. III. e. II e III.
(a) b. II. d. I e II.
Tempo 60. ITA-SP
(Y)
Assinale a opção que contém a afirmação
I. O gráfico (X) representa uma substân- errada relativa à curva de resfriamento apre-
cia pura. sentada a seguir.
II. O gráfico (Y) representa uma substân-
cia pura.
Temperatura °C

III. No gráfico (X), o caminho representado


pela letra (b) corresponde à coexistên-
cia das fases sólida e líquida.
IV. No gráfico (Y), o caminho representa-
do pela letra (b) corresponde apenas à Tempo / min
existência da fase sólida.
PV-13-14

a. A curva pode representar o resfriamen-


V. O gráfico (X) representa uma mistura. to de uma mistura eutética.
Assinale a alternativa correta. b. A curva pode representar o resfriamen-
a. Somente a afirmativa I é verdadeira. to de uma substância sólida, que apre-
senta uma única forma cristalina.
b. Somente as afirmativas I, II e IV são ver- c. A curva pode representar o resfriamen-
dadeiras. to de uma mistura azeotrópica.
c. Somente as afirmativas I, III e V são ver- d. A curva pode representar o resfriamen-
dadeiras. to de um líquido constituído por uma
d. Somente as afirmativas I e III são ver- substância pura.
dadeiras. e. A curva pode representar o resfriamen-
e. Somente as afirmativas I, II, III e IV são to de uma mistura líquida de duas subs-
verdadeiras. tâncias que são completamente miscí-
veis no estado sólido.

68
Química geral e estequiometria Química

61. UEPG-PR 04. Soluções são misturas homogêneas de


Sobre a matéria, suas características e seu duas ou mais substâncias.
comportamento, assinale o que for correto. 08. Soluções homogêneas entre dois sóli-
01. Misturas homogêneas são monofásicas dos não são possíveis de se obter.
e chamadas de soluções. 64. Unimontes-MG
02. A água é classificada como substância O ouro denominado branco, usado em confec-
simples e tem seu ponto de ebulição ção de joias, contém dois elementos: ouro e
dependente da altitude. paládio. Duas amostras distintas de ouro bran-
04. Uma mistura de dois compostos, que co diferem em relação às quantidades de ouro
em condições ambiente são gases e en- e paládio que contêm.
contram-se na forma pura, será sempre Sabendo-se que ambas apresentam composi-
homogênea. ção uniforme, pode-se afirmar corretamente
08. Nas misturas heterogêneas sólido-líqui- que o ouro branco é:
do, as fases podem ser separadas por a. um material heterogêneo.
processos como decantação, centrifu-
b. uma solução sólida.
gação e filtração.
c. uma substância composta.
16. Oxigênio (O2) e Ozônio (O3) são gases clas-
sificados como substâncias compostas. d. uma mistura heterogênea.
62. UFRJ 65. UFAL
Uma festa de aniversário foi decorada com dois A maioria dos materiais não é nem elementos
tipos de balões. Diferentes componentes gaso- puros nem compostos puros; são misturas de
sos foram usados para encher cada tipo de ba- substâncias mais simples. Por exemplo, um
lão. As figuras observadas representam as subs- medicamento, tal como xarope expectorante,
tâncias presentes no interior de cada balão. é uma mistura de vários ingredientes formu-
lados para conseguir um efeito biológico. Um
sistema constituído por açúcar dissolvido em
água, limalha de ferro, vapor-d’água e nitrogê-
nio gasoso pode ser classificado como:
a. sistema heterogêneo com 4 fases e 3
componentes.
b. sistema homogêneo com 4 fases e 4
componentes.
c. sistema heterogêneo com 3 fases e 3
Balão I Balão II componentes.
a. Indique quantos elementos diferentes d. sistema homogêneo com 3 fases e 4
PV-13-14

e quantas substâncias simples diferen- componentes.


tes existem nos balões. e. sistema heterogêneo com 3 fases e 4
b. Classifique o tipo de sistema de cada componentes.
balão quanto à homogeneidade. 66. Unesp
63. UEPG-PR Uma amostra de água do rio Tietê, que apre-
sentava partículas em suspensão, foi subme-
Sobre as soluções e seus critérios de classifica- tida a processos de purificação obtendo-se,
ção, assinale o que for correto. ao final do tratamento, uma solução límpida
01. Numa solução formada entre um sólido e cristalina. Em relação às amostras de água
e um líquido, o sólido corresponde à fase antes e após o tratamento, podemos afirmar
dispersa e o líquido à fase dispersante. que correspondem, respectivamente, a:
02. Uma mistura de sal em água forma a. substâncias composta e simples.
uma solução sólido-líquido. b. substâncias simples e composta.

69
Química Química geral e estequiometria

c. misturas homogênea e heterogênea. a. 3, 2 e 4. d. 3, 2 e 5.


d. misturas heterogênea e homogênea. b. 3, 3 e 4. e. 3, 3 e 6.
e. mistura heterogênea e substância c. 2, 2 e 4.
simples.
70. PUC-RJ
67. Ufla-MG
Considere as seguintes afirmativas:
Considere os sistemas abaixo. I. Durante a mudança de estado de uma
substância pura, a temperatura se man-
Água Água tém constante.
+ Água + II. As misturas são sistemas polifásicos.
Etanol Gasolina III. Um sistema homogêneo pode apresen-
tar mais de um componente, mas não
I II III mais que uma fase.
Os sistemas I, II e III correspondem, respecti- Responda qual(is) é (são) sempre verdadeira(s).
vamente, a: a. Apenas a I. d. A I e a III.
a. mistura heterogênea, substância com- b. Apenas a II. e. A I e a II.
posta, mistura heterogênea. c. Apenas a III.
b. mistura homogênea, substância sim- 71.
ples, mistura heterogênea.
Em cinco recipientes encontramos misturas
c. mistura homogênea, substância sim-
distintas, a saber:
ples, mistura homogênea.
d. mistura homogênea, substância com- I II III
posta, mistura heterogênea.
68. PUCCamp-SP
“O sistema formado por etanol, água e três cubos água + álcool + nitrogênio +
de gelo é X e contém Y substâncias químicas.” sílica água hidrogênio

Completa-se corretamente a afirmação mencio-


IV V
nada substituindo-se X e Y, respectivamente, por:
a. bifásico – duas
água +
b. bifásico – três água + cloreto de
sódio (sem
c. trifásico – duas óleo corpo de
fundo)
d. tetrafásico – três
e. pentafásico – duas
Em quais recipientes encontramos uma mistu-
PV-13-14

69. UFES
ra homogênea?
Observe a representação dos sistemas I, II e III a. I, II, III e V d. II, III e V
e seus componentes. O número de fases em
cada um é, respectivamente: b. I e V e. II, III, IV e V
I II III
c. II, III e IV
72. FEI-SP
Assinale a alternativa onde encontramos uma
substância pura, uma mistura homogênea e um
sistema heterogêneo.
a. Açúcar, água doce, água do mar.
b. Leite, suco de laranja, feijoada.
Óleo, água e gelo Água gaseificada Óleo, gelo, água
e gelo salgada e granito c. Água destilada, água potável, água e gelo.

70
Química geral e estequiometria Química

d. Vinagre, vinho, álcool etílico. e. No processo de liofilização, a água pas-


e. Geleia, água potável, suco de frutas. sa por uma transformação física deno-
minada sublimação.
73. UFRGS-RS
75. ENEM
Analise os sistemas materiais abaixo, estando
ambos na temperatura ambiente. Produtos de limpeza indevidamente guarda-
dos ou manipulados estão entre as principais
Sistema I. Mistura de 10 g de sal de cozinha, 30
causas de acidentes domésticos. Leia o relato
g de areia fina, 20 mL de óleo e 100 mL de água.
de uma pessoa que perdeu o olfato por ter
Sistema II. Mistura de 2,0 L de CO2, 3,0 L de N2 e misturado água sanitária, amoníaco e sabão
1,5 L de O2. em pó para limpar um banheiro:
Sobre esses sistemas, é correto afirmar que: "A mistura ferveu e começou a sair uma fumaça
a. ambos são heterogêneos, pois apresen- asfixiante. Não conseguia respirar e meus olhos,
tam mais de uma fase. nariz e garganta começaram a arder de maneira
b. em I, o sistema é bifásico, após forte agi- insuportável. Saí correndo à procura de uma ja-
tação, e, em II, o sistema é monofásico. nela aberta para poder voltar a respirar".
c. em I, o sistema é trifásico, após forte agi- O trecho destacado no texto poderia ser reescri-
tação, e, em II, o sistema é monofásico. to, em linguagem científica, da seguinte forma:
d. ambos apresentam uma única fase, for- a. As substâncias químicas presentes nos
mando sistemas homogêneos. produtos de limpeza evaporaram.
e. em I, o sistema é trifásico, independente- b. Com a mistura química, houve produ-
mente da ordem de adição dos compo- ção de uma solução aquosa asfixiante.
nentes, e, em II, o sistema é bifásico. c. As substâncias sofreram transfomações
pelo contato com o oxigênio do ar.
74. UFJF-MG
d. Com a mistura, houve transformação
Atualmente, é comum encontrar, nas prateleiras química que produziu rapidamente
de supermercados, alimentos desidratados, isto gases tóxicos.
é, isentos de água em sua composição. O pro-
cesso utilizado na desidratação dos alimentos é e. Com a mistura, houve transformação
a liofilização. A liofilização consiste em congelar química, evidenciada pela dissolução
o alimento a uma temperatura de –197°C e, de- de um sólido.
pois, submeter o alimento congelado a pressões 76. Vunesp
muito baixas. Na temperatura de –197 °C, a água
contida no alimento encontra-se no estado sóli- A elevação da temperatura de um sistema pro-
do e, com o abaixamento de pressão, passa dire- duz, geralmente, alterações que podem ser in-
terpretadas como sendo devidas a processos
PV-13-14

tamente para o estado de vapor, sendo, então,


eliminada. Assinale a afirmação correta: físicos ou químicos.
Medicamentos, em especial na forma de so-
a. No processo de liofilização, a água pas-
luções, devem ser mantidos em recipientes
sa por uma transformação química,
fechados e protegidos do calor para que se
produzindo H2 e O2, que são gases.
evite:
b. No processo de liofilização, a água pas-
I. a evaporação de um ou mais de seus
sa por um processo físico conhecido
componentes;
como evaporação.
II. a decomposição e consequente dimi-
c. No processo de liofilização, o alimento
nuição da quantidade do composto
sofre decomposição, perdendo água.
que constitui o príncipio ativo;
d. No processo liofilização, a água sofre
III. a formação de compostos indesejáveis
decomposição.
ou potencialmente prejudiciais à saúde.

71
Química Química geral e estequiometria

A cada um desses – (I), (II) e (III) – corresponde d. Três fases, sendo uma de gasolina sem
um tipo de transformação classificada, respec- etanol, outra de gasolina com etanol e
tivamente, como: uma terceira de solução salina.
a. física, física e química. e. Uma fase, visto que o etanol vai evapo-
rar quando misturado à solução salina
b. física, química e química.
e gasolina.
c. química, física e física.
79. UEM-PR
d. química, física e química.
Assinale o que for correto.
e. química, química e física.
01. Uma mistura de heptano com areia é
77. UFPE um exemplo de mistura homogênea.
Em quais das passagens destacadas a seguir 02. O soro fisiológico (que é constituído
está ocorrendo transformação química? de uma solução não saturada de sal e
açúcar comuns) é um exemplo de um
1. “O reflexo da luz nas águas onduladas
sistema homogêneo.
pelos ventos lembrava-lhe os cabelos
de seu amado”. 04. Uma mistura de manteiga com água fria
é um exemplo de mistura heterogênea.
2. “A chama da vela confundia-se com o
08. Sublimação é a mudança de estado físi-
brilho nos seus olhos”.
co de sólido para gás como também de
3. “Desolado, observava o gelo derreten- gás para sólido.
do em seu copo e ironicamente compa- 16. O método de separação que se baseia
rava-o ao seu coração”. na diferença de densidade se chama
4. “Com o passar dos tempos começou a decantação.
sentir-se como a velha tesoura enferru- 80. UEG-GO
jando no fundo da gaveta”.
Em um laboratório de química, um estudante
Estão corretas apenas: separou em frascos semelhantes três solven-
a. 1 e 2. d. 2 e 4. tes que utilizaria em seu experimento. Entre-
b. 2 e 3. e. 1 e 3. tanto, esqueceu de rotular esses frascos no
momento da coleta e, posteriormente, não
c. 3 e 4. tinha certeza a respeito do componente de
78. UEPB cada um deles. Mas, conhecendo a densida-
Um dos procedimentos para adulteração da de de cada um dos líquidos, para sanar sua
dúvida, efetuou o seguinte experimento. Adi-
gasolina é a adição de uma quantidade maior
cionou 3 mL de cada solvente em tubos de
de etanol do que a permitida pela legislação.
ensaios separados e, posteriormente, adicio-
Os postos de distribuição de combustíveis são
nou 1 mL de água. A análise dos resultados
obrigados, quando pedido pelo cliente, a fazer
permitiu a identificação inequívoca dos com-
PV-13-14

o teste para determinação do teor de álcool na ponentes presentes em cada frasco. Os resul-
gasolina, que se baseia na maior miscibilidade do tados observados para cada tubo de ensaio e
etanol em solução salina do que em gasolina. Se a tabela com as respectivas densidades dos
forem adicionadas em uma proveta partes iguais líquidos estão mostrados a seguir:
de gasolina comercializada no Brasil e solução
salina, quantas e quais fases serão observadas? Líquido Densidade a 25 °C (g · mL–1)
a. Três fases, sendo uma de gasolina, ou-
H2O 1,0
tra de solução salina e outra de etanol.
b. Duas fases, sendo uma de gasolina com CH3CH2OH 0,8
etanol e outra de solução salina com
Gasolina 0,7
etanol.
c. Duas fases, sendo uma da gasolina e HCCl3 1,5
outra da solução salina com etanol.

72
Química geral e estequiometria Química

III. O caldo primário, resultante do esma-


gamento, é passado por filtros e sofre
tratamento para transformar-se em
açúcar refinado e etanol.
Com base nos destaques da observação dos
alunos, quais operações físicas de separação
de materiais foram realizadas nas etapas de
beneficiamento da cana-de-açúcar?
a. Separação mecânica, extração, de-
cantação
Tubo 1 Tubo 2 Tubo 3 b. Separação magnética, combustão, fil-
tração
A partir das informações acima, responda ao
que se pede. c. Separação magnética, extração, filtração
d. Imantação, combustão, peneiração
a. Determine as substâncias presentes em
cada um dos tubos, justificando, em se- e. Imantação, destilação, filtração
guida, o motivo de sua escolha. 83. UFSCar-SP
b. Comente a validade da seguinte afir-
Uma das fontes de poluição ambiental gera-
mativa: “Todos os líquidos indicados na
da pelas atividades de um posto de gasolina é
tabela acima são exemplos de substân-
o efluente resultante da lavagem de veículos.
cias puras”.
Este efluente é uma mistura que contém geral-
81. UFG-GO mente água, areia, óleo e sabão. Para minimi-
Considere a descrição da seguinte técnica: O zar a poluição ambiental, antes de ser lançado à
minério pulverizado é recoberto com óleo, rede de esgoto, esse efluente deve ser subme-
água e detergente; nessa mistura, é borbu- tido a tratamento, cujo processo inicial consiste
lhado ar. Essa descrição refere-se a um mé- na passagem por uma “caixa de separação”, es-
todo de separação de misturas muito utiliza- quematizada na figura que se segue.
do em indústrias metalúrgicas. Qual é essa Efluente Esgoto
técnica?
a. Decantação d. Destilação Espaço Água e
3 sabão
b. Flotação e. Sublimação
Espaço
c. Cristalização 2
82. ENEM Espaço
1
Em visita a uma usina sucroalcooleira, um
grupo de alunos pôde observar a série de pro- Esquema da caixa de separação
PV-13-14

cessos de beneficiamento da cana-de-açúcar,


entre os quais se destacam: Sabendo-se que água e sabão formam uma
única fase, e que os óleos empregados em ve-
I. A cana chega cortada da lavoura por ículos são menos densos e imiscíveis com esta
meio de caminhões e é despejada em fase (água + sabão), pede-se:
mesas alimentadoras que a conduzem
para as moendas. Antes de ser esma- a. Escreva os nomes dos componentes
gada para a retirada do caldo açuca- desse efluente que se acumulam nos
rado, toda a cana é transportada por espaços 1 e 2.
esteiras e passada por um eletroímã b. Escreva o nome do processo responsá-
para a retirada de materiais metálicos. vel pela separação dos componentes
II. Após se esmagar a cana, o bagaço se- do efluente nos espaços 1 e 2.
gue para as caldeiras, que geram vapor
e energia para toda a usina.

73
Química Química geral e estequiometria

84. UTFPR
O diagrama a seguir representa as etapas de separação de uma mistura heterogênea, em seus
componentes finais.
Mistura heterogênea
Etapa 1

Fase líquida colorida Fase sólida


Etapa 3 Etapa 2

Líquido Líquido Sólido 1 Sólido 2


colorido colorido PF = 50 °C PF = 64 °C
1 2 solúvel solúvel
PE = 82 °C PE = 86 °C em água em água
PE = Ponto de ebulição PF = Ponto de fusão

Com base nestas informações, é correto afirmar que:


a. a fase líquida colorida forma uma mistura azeotrópica.
b. a etapa 1 só pode ser realizada por meio de uma desti lação simples.
c. a etapa 2 pode ser realizada uti lizando-se um funil de separação.
d. o líquido 2 ferve antes do líquido 1.
e. a etapa 2 pode ser realizada por meio de uma cristalização fracionada.
85. UECE
Em um laboratório de química, o
professor solicita aos alunos que Mistura
façam a relação das vidrarias e
Funil com papel de filtro
materiais necessários para a reali-
Resíduo
zação de uma experiência de dis-
solução fracionada, com o objeti-
vo de separar sal e areia de uma
mistura heterogênea, para usar o Filtrado
sal em outra experiência. Assinale
a alternativa que contém a relação
correta. Filtração comum para obtenção de areia. Vaporização para obtenção do sal.
PV-13-14

a. Água, 2 béqueres, bastão de vidro, funil, papel de filtro, argola de ferro, suporte metálico,
bico de bunsen, botijão de gás butano, tripé de ferro, fósforo, mufla e pinça metálica.
b. Água, 2 béqueres, bastão de vidro, funil, papel de filtro, argola de ferro, suporte metálico,
bico de bunsen, botijão de gás butano, tripé de ferro, fósforo e tela de amianto.
c. Água, 2 frascos de erlenmeyer, bastão de vidro, funil, argola de ferro, suporte metálico,
chapa aquecedora, tripé de ferro, tela de amianto, garras metálicas e espátula.
d. Água, 2 béqueres, proveta, espátula, funil, papel de filtro, trompa de vácuo, argola de
ferro, suporte metálico, bico de bunsen, botijão de gás butano, tripé de ferro, fósforo e
tela de amianto.

74
Química geral e estequiometria Química

86. Fuvest-SP a. Para cada uma das filtrações, indique o


A obtenção de água doce de boa qualidade que é retido no filtro e o que constitui
o filtrado.
está se tornando cada vez mais difícil devido
ao adensamento populacional, às mudanças b. Escreva a equação da reação química
climáticas, à expansão da atividade industrial que ocorre quando oxalato de cálcio
e à poluição. A água, uma vez captada, precisa monoidratado é aquecido na mufla.
ser purificada, o que é feito nas estações de c. Admitindo que todo o cálcio presen-
tratamento. Um esquema do processo de pu- te nas conchas esteja sob a forma de
rificação é: CaCO3 e que os outros componentes
dessas conchas não interferem nos
A B C D E F procedimentos da análise, calcule a
porcentagem em massa de carbonato
em que as etapas B, D e F são: de cálcio na amostra de conchas anali-
B – adição de sulfato de alumínio e óxido de sada. Mostre os cálculos.
cálcio; 88. UESC-BA
D – filtração em areia; As manchas de óleo que atingiram os
mares e as praias do sul do Estado estão se
F – fluoretação. tornando menores. O óleo afetou as praias
Assim sendo, as etapas A, C e E devem ser, res- de Guaibim, Ilhéus e Itacaré. Em Itacaré,
pectivamente: a mancha negra ainda incomoda os mora-
dores, prejudicando o meio ambiente e o
a. filtração grosseira, decantação e cloração. turismo da região.
b. decantação, cloração e filtração gros- ARAÚJO, Denise. Manchas de óleo estão
seira. sendo retiradas no sul. A Tarde. Salvador,
c. cloração, neutralização e filtração gros- 31 out. 2008. Caderno Bahia p.10.
seira.
Dentre as propriedades do sistema formado
d. filtração grosseira, neutralização e de- pela água do mar e pelo petróleo, é correto
cantação.
afirmar:
e. neutralização, cloração e decantação.
01. O petróleo é separado da água do mar
87. UFABC-SP por destilação fracionada.
O teor de cálcio em uma amostra de conchas 02. A viscosidade da água do mar é menor
de massa igual a 5,0 g foi determinado da se- que a do petróleo.
guinte maneira:
03. O petróleo e a água do mar são imiscí-
– trituração das conchas;
veis porque possuem densidades dife-
– aquecimento do material triturado com rentes.
HCl;
04. As interações entre os íons encon-
PV-13-14

– filtração;
trados na água do mar e os alcanos
– precipitação de íons Ca2+ sob forma existentes no petróleo são de natu-
de oxalato de cálcio monoidratado,
CaC2O4 · H2O, utilizando-se oxalato de reza dipolo permanente-dipolo per-
amônio, (NH4)2C2O4, como reagente; manente.
– filtração e secagem do oxalato de cálcio 05. As manchas de óleo são removidas da
monoidratado; superfície da água do mar pela utiliza-
– calcinação em mufla, produzindo-se 2,0 g ção de solventes apolares, a exemplo
de CaO como único resíduo sólido. do querosene e da gasolina.

75
Química Química geral e estequiometria

89. Unicid-SP
Em 1849, Usiglio identificou e quantificou as substâncias obtidas pela evaporação da água do
mar. A tabela a seguir mostra os resultados de seu trabalho.
Sais depositados durante a concentração da água do mar (gramas)*

Volume
Fe2O3 CaCO3 CaSO4 · 2 H2O NaCl MgSO4 MgCl2 NaBr KCl
(litros)

1,000 – – – – – – – –

0,533 0,0030 0,0642 – – – – – –

0,316 – – – – – – – –

0,245 – – – – – – – –

0,190 – 0,0530 0,5600 – – – – –

0,1445 – – 0,5620 – – – – –

0,131 – – 0,1840 – – – – –

0,112 – – 0,1600 – – – – –

0,095 – – 0,0508 3,2614 0,0040 0,0078 – –

0,064 – – 0,1476 9,6500 0,0130 0,0356 – –

0,039 – – 0,0700 7,8960 0,0262 0,0434 0,0728 –

0,0302 – – 0,0144 2,6240 0,0174 0,0150 0,0358 –

0,023 – – – 2,2720 0,0254 0,0240 0,0518 –

0,0162 – – – 1,4040 0,5382 0,0274 0,0620 –

0,0000 – – – 2,5885 1,8545 3,1640 0,3300 0,5339

Total 0,0030 0,1172 1,7488 29,6959 2,4787 3,3172 0,5524 0,5339


*Segundo Usiglio, 1849.
SKINNER, B. J. e TUREKIAN, K. K.; O homem e o oceano, 1977.
PV-13-14

O trabalho de Usiglio exemplifica um processo de separação de misturas conhecido como:


a. filtração.
b. centrifugação.
c. cristalização fracionada.
d. decantação.
e. destilação fracionada.
90. UEPB
Uma das alternativas viáveis ao Brasil para o uso de fontes renováveis de energia e com menor impac-
to ambiental é o biodiesel. No Brasil foi instituída a Lei 11.097, de 13 de janeiro de 2005, que obriga, a
partir de 2008, em todo o território nacional, o uso de uma mistura em volume de 2% de biodiesel e
98% de diesel de petróleo, denominada de B2. Em janeiro de 2013, essa obrigatoriedade passará para
5% (B5). Este biocombustível é substituto do óleo diesel, que é um combustível fóssil, pois é obtido da

76
Química geral e estequiometria Química

destilação fracionada do petróleo. O procedimento normalmente utilizado para obtenção do biocom-


bustível é através da transesterificação catalítica entre um óleo vegetal com álcool de cadeia curta,
sendo obtidos ésteres graxos, como pode ser representado pela equação química abaixo:
O
H2C O C C17H35 H2C OH
O O
HC O C C17H35 + CH3OH HC OH + C17H35 C OCH3
O
H2C O C C17H35 H2C OH

Como pode ser observado na equação química do texto, a glicerina é um dos subprodutos do pro-
cesso de obtenção do biosiesel. Ela é um subproduto, pois não é um composto de interesse para
essa reação. Sabendo que a glicerina possui uma densidade bem mais elevada que o biodiesel,
qual processo que pode ser conduzido para separação da mistura glicerina/biodiesel?
a. Destilação fracionada
b. Decantação
c. Catação
d. Condensação
e. Eletrodeposição

91. UFPB livre dessas impurezas, os procedimentos cor-


Nas salinas, a água do mar é evaporada retos são:
pela ação do vento e do calor, obtendo-se a. catação, dissolução em água e decan-
o sal grosso. Em seguida, por processos de tação.
separação, esse sal é purificado, resultan- b. separação magnética, destilação e dis-
do no cloreto de sódio cristalizado, que é solução em água.
utilizado na indústria como matéria-prima c. sublimação, dissolução em água e pe-
para produção de diversos produtos quími- neiração.
cos, conforme exemplificado no esquema
abaixo. d. dissolução em água, filtração simples e
evaporação.
NaCl H2O e. dissolução em água, decantação e su-
blimação.
Na2CO3 92. PUCCamp-SP
PV-13-14

(IV) Energia
elétrica O leite de caixinha e a saúde pública
O escândalo do leite ganhou as man-
Processo H2
NaOH(I)
eletrolítico
chetes dos jornais por conta das fraudes
praticadas na produção do leite longa
vida. Para se ter uma ideia, a adultera-
ção envolve a adição de 8% em massa de
NaClO(II) Cl2 HCl(III) compostos diversos, como água oxigena-
da, soda cáustica, ácido cítrico, citrato de
Disponível em: <http://www.caii.com.br/ctudo-produtos- sódio, sal e açúcar. A seguir, estão algu-
processo.html>. Acesso em: 10 jul. 2009. (Adaptado) mas dessas práticas:
O sal grosso obtido nas salinas contém impu- • Soro de queijo – é um subproduto da
rezas insolúveis em água. Para se obter o sal fabricação de diferentes tipos de queijo,

77
Química Química geral e estequiometria

obtido após a coagulação e precipitação


Sal H2O fria H2O quente
da caseína. Nos países desenvolvidos,
esse subproduto é desidratado e comer- AgCl Insolúvel Insolúvel
cializado como soro em pó. No Brasil, é
comercializado na forma líquida, sendo NaCl Solúvel Solúvel
utilizado para fraudar o leite.
PbCl2 Insolúvel Solúvel
• Soda cáustica – o leite apresenta uma
acidez de 1,5 g/L a 1,8 g/L, expressa em
ácido lático. Um leite ácido é impróprio Baseando-se nesses dados de solubilidade, es-
para o tratamento térmico. Assim, num quematize uma separação desses três sais que
leite ácido é adicionado NaOH, soda constituem a mistura.
cáustica, para regular a acidez. 94. UFPE
• Coliformes fecais – a determinação Considere uma mistura de parafina (hidrocarbo-
da população de coliformes fecais é neto de cadeia longa) finamente dividida e açú-
utilizada como indicativo do grau de car (sacarose – C12H22O11) refinado. Selecione os
higiene do sistema de produção de processos de separação, na sequência indicada,
produtos alimentícios. No caso do lei- mais adequados para essa mistura.
te, esses microrganismos produziriam
rapidamente ácidos orgânicos e gás. a. Dissolução em água, filtração, evaporação
Como consequência, seriam observa- b. Filtração, evaporação, combustão
dos uma queda brusca de pH e estufa- c. Dissolução em água, floculação, de-
mento precoce da embalagem. cantação
Ismael de Mancilha. Jornal da USP. 3 d. Destilação fracionada a 50 °C
a 9/12/2007. p.2. Adaptado.
e. Combustão, destilação
A desidratação do soro de queijo é um proces-
95. Fuvest-SP
so utilizado em separações de misturas:
Uma certa amostra de cloreto de sódio con-
a. líquido-gás.
tém areia. Descreva resumidamente um mé-
b. líquido-líquido. todo que permita purificar o cloreto de sódio,
c. sólido-líquido. tal que se tenha no final o sal sólido.
d. sólido-sólido. 96. Unicamp-SP
e. sólido-gás. Deseja-se fazer a separação dos componentes
93. Fuvest-SP da pólvora negra, que é constituída de nitrato
de sódio, carvão e enxofre. Sabe-se que o ni-
Uma mistura sólida é constituída de cloreto de trato de sódio é solúvel em água, o enxofre é
prata (AgCl), cloreto de sódio (NaCl) e cloreto solúvel em dissulfeto de carbono, enquanto o
de chumbo (PbCl2). A solubilidade desses sais carvão é insolúvel nesses solventes. Proponha
PV-13-14

em água está resumida na tabela a seguir: um procedimento para realizar essa separação.

97. UFR-RJ
Um dos critérios utilizados pelos químicos para classificar as substâncias leva em consideração, prin-
cipalmente, o tipo de elemento e o número de átomos desse elemento. Muitas propriedades são
decorrentes dessas combinações. A tabela a seguir contém propriedades de algumas substâncias.
Substâncias PF (°C) PE (°C) d(g/ml) Solubilidade em H2O

Glicerina 20 290 1,26 Muito solúvel

Eugenol –7,5 253 1,07 Insolúvel

Etanodiol –144 35 0,84 Pouco solúvel

78
Química geral e estequiometria Química

a. Em que estado físico se encontra a gli- com água e pressionados contra uma folha
cerina num dia muito frio, com a tem- de papel especial, de modo a deixar amos-
peratura próxima a 0 °C? tras dos corantes em pontos igualmente
b. Uma mistura de eugenol e glicerina espaçados, sempre a 2 cm da base da fo-
pode ser separada por adição de água? lha. A seguir, a folha foi colocada em um
Justifique. recipiente com água, de forma a mergulhar
somente a base da folha de papel na água,
98. UFPA sem que o líquido tocasse os pontos colori-
Dentre as alternativas, escolha o processo dos. Após algum tempo, quando a água ha-
adequado para a separação de água e tetra- via atingido o topo da folha, observou-se a
cloreto de carbono, ambos não miscíveis. formação de manchas de diferentes cores,
a. Decantação aqui simbolizadas por diferentes formas e
tamanhos:
b. Filtração
c. Separação magnética
d. Dissolução fracionada
e. Liquefação
99. USF-SP
Considerando-se as aparelhagens esquema-
tizadas:

Laranja
Vermelha

Verde
Amarela
Cor do

marrom
Azul

2 cm
confeito

x indica o ponto de aplicação de cada amostra.

Os confeitos em cuja fabricação é empregado


um corante amarelo são os de cor:
I II a. vermelha, amarela e marrom.
b. amarela, verde e laranja.
a afirmação correta é:
c. verde, azul e marrom.
a. A aparelhagem I pode ser utilizada para
separar água e gasolina. d. vermelha, amarela e verde.
b. A aparelhagem I pode ser utilizada para e. vermelha, laranja e marrom.
separar solução de água e sal. 101. UFMS
PV-13-14

c. A aparelhagem I pode ser utilizada para


Quando chega às refinarias, o petróleo pas-
separar solução de água e carvão.
sa por processo que resulta na separação de
d. A aparelhagem II pode ser utilizada seus diversos hidrocarbonetos, como gasolina,
para separar água e óleo. querosene e óleo diesel. Assinale a alternativa
e. A aparelhagem II pode ser utilizada que apresenta o nome do processo utilizado
para separar solução de água e álcool. nas refinarias.
100. Fuvest-SP a. Flotação
Os confeitos de chocolate de determina- b. Filtração
da marca são apresentados em seis cores. c. Destilação fracionada
Com eles, foi feito o seguinte experimento, d. Extração por solvente
destinado a separar os corantes utilizados
em sua fabricação: confeitos de cada uma e. Extração com água
das seis diferentes cores foram umedecidos

79
Química Química geral e estequiometria

102. Uespi a. azeótropo.


Numa destilação fracionada, produziram-se 95% b. azoto.
de etanol e 5% de água. Essa mistura é conheci- c. vinhoto.
da como: d. racemato.
e. cachaça.

103. UFG-GO
Um alambique é uma forma artesanal de realizar uma separação de misturas. O mesmo procedi-
mento pode ser realizado com vidrarias e equipamentos de laboratórios químicos.

Considerando as vidrarias e os equipamentos representados, responda:


PV-13-14

a. Qual o nome da técnica de separação de misturas, que representa o processo que ocorre
no alambique?
b. Utilizando as vidrarias e os equipamentos representados acima, esquematize um aparelho
de laboratório para realizar o mesmo processo que ocorre no alambique.
104. UFF-RJ
Em 11 de novembro de 1999, trinta e três pessoas morreram em Salvador em razão da ingestão
de cachaça de fabricação clandestina. Segundo os médicos, os sintomas eram dor de cabeça,
hipertensão e vertigem. O que levou a crer que a cachaça tenha sido contaminada com metanol.
Um laboratório confirmou tal hipótese. Durante muito tempo, a única maneira de produzir me-
tanol era destilando a madeira a seco e na ausência de ar, daí o nome “álcool da madeira” que
alguns comerciantes inescrupulosos vendiam como sendo álcool etílico. Atualmente, é produzido
em escala industrial a partir do carvão e água, sendo monitorado pelos órgãos do governo a fim
de que não seja utilizado incorretamente.

80
Química geral e estequiometria Química

Com base nas informações, assinale a opção b. dessalinização … destilação … adicionan-


correta. do … em proporções
a. A destilação é um processo físico de se- c. dessalinização … destilação … remo-
paração. vendo … por técnicas
b. O metanol, assim obtido, constitui uma d. desinfecção … cloração … adicionando
mistura heterogênea. … em proporções
c. A cachaça é uma substância pura. e. clarificação … decantação … adicionan-
d. Toda substância pura é constituída por do … em proporções
apenas dois tipos de elementos químicos. 107. UFOP-MG
e. O percentual de carbono no metanol é Um aluno encontrou em um laboratório três
maior do que no etanol. frascos contendo três misturas binárias, con-
105. UPE forme descrito a seguir.
A mistura álcool + água (95% álcool, 5% de água) 1ª mistura: heterogênea, formada por dois
é denominada de mistura azeotrópica. Em rela- sólidos
ção a essa mistura, é correto afirmar que: 2ª mistura: heterogênea, formada por dois
a. a separação de seus componentes é obti- líquidos
da adicionando-se óxido de cálcio à mis- 3ª mistura: homogênea, formada por dois líqui-
tura e, em seguida, realizando-se uma dos cujos pontos de ebulição diferem em 20 °C
filtração com papel de filtro adequado.
b. a separação dos componentes da mistura Marque a alternativa que indica os processos
é obtida submetendo-se a mistura a uma de separação mais adequados para recuperar
destilação fracionada, seguida de uma fil- as substâncias originais na 1a, 2a e 3a misturas,
tração à temperatura constante. respectivamente.
c. não é possível separar a água do álcool, a. filtração, decantação e destilação simples.
pois o álcool e a água são infinitamente b. evaporação, destilação simples e de-
miscíveis em quaisquer proporções, sob cantação.
quaisquer condições físicas ou químicas. c. decantação, destilação simples e destila-
d. a separação dos componentes da mis- ção fracionada.
tura é possível, apenas, pela adição de d. sublimação, decantação e destilação fra-
anidrido sulfúrico, pois esse óxido, ao re- cionada.
agir com a água, origina o ácido sulfúrico,
que, por decantação, separa-se do álcool. 108. UFMS
e. a separação dos componentes da mistura A figura abaixo consiste numa representação
é facilmente obtida adicionando-se sódio esquemática de um sistema de destilação
metálico, pois toda água é transformada simples. Essa técnica se aplica à separação de
em hidrogênio gasoso que se desprende misturas homogêneas de sólidos em líquidos.
do sistema.
PV-13-14

106. Unesp
Em nosso planeta, a maior parte da água encon-
tra-se nos oceanos (água salgada) e é imprópria
para consumo humano.
Um processo para tornar a água do mar potável
seria: “Promover a ____________ por ____________ Considere uma mistura constituída por água
ou osmose reversa e, em seguida, retificá-la, em sua forma líquida e cloreto de sódio dissol-
____________ sais ____________ adequadas”. vido. Ao final do processo de destilação sim-
Assinale a alternativa que permite preencher, ples dessa mistura, verifica(m)-se:
na sequência, as lacunas de forma correta. 01. presença de água no Erlenmeyer.
a. purificação … destilação … removendo 02. presença de cloreto de sódio no balão
… em proporções de destilação.

81
Química Química geral e estequiometria

04. presença de água + cloreto de sódio no b.


Erlenmeyer.
08. presença de água + ácido clorídrico no
balão.
16. circulação de água no condensador.
109. UESC-BA
Os constituintes da mistura de NO2, SO2 e CO
podem ser separados usando-se a técnica:
a. cristalização fracionada.
b. destilação fracionada.
c. flotação.
d. liquefação fracionada.
e. sublimação. c.
110. UFCG-PB
Um dos processos para a produção de biodiesel,
que é uma fonte renovável de energia, chama-se
transesterificação. Nesse processo, o biodiesel é
produzido pela reação de óleo vegetal com um
excesso de álcool de cadeia curta (metanol ou
etanol) na presença de um catalisador (KOH).
A reação é dada por:
CH2OOCR1 CH2OH

CHOOCR1 + 3 ROH 3 ROOCR1 + CHOH


d.
CH2OOCR1 CH2OH

Onde R: CH3 ou C2H5, R1: grupo alquila.


O excesso de metanol utilizado no proces-
so de produção de biodiesel pode constituir
um problema ambiental, por isso precisa
ser purificado para ser reutilizado no pro-
cesso. Identifique, dentre as figuras, a mais
adequada para o processo de purificação do
PV-13-14

metanol.
e.

a.

82
Química geral e estequiometria Química

111. UFPR Sobre a temática apresentada, é incorreto


O processo de destilação é importante para a afirmar que:
separação de misturas. Assinale a alternativa a. o mercúrio é um metal líquido a 25 °C e
correta sobre o processo de destilação da água. 1 atm de pressão.
a. Na passagem do líquido, ocorre a que- b. a diferença nas temperaturas de ebuli-
bra das ligações covalentes entre os ção é aproveitada para separar o mer-
átomos de hidrogênio e de oxigênio. cúrio do ouro, já que estes não formam
b. A temperatura de ebulição varia duran- mistura eutética.
te a destilação da água. c. mercúrio e ouro formam um sistema
c. A fase vapor é constituída por uma mis- heterogêneo.
tura dos gases hidrogênio e oxigênio. d. a separação do sistema amálgama-la-
d. A temperatura de ebulição depende da ma constitui um processo físico.
pressão atmosférica local. e. os vapores de mercúrio eliminados
e. A temperatura de ebulição depende do durante a última etapa da extração
tipo de equipamento utilizado no pro- podem contaminar os garimpeiros e
cesso. também os ecossistemas em torno do
garimpo.
112. UFCG-PB
114. UEM-PR
Na separação de misturas, podem ser em-
pregados vários processos que são classifi- Assinale o que for correto.
cados como mecânicos ou físicos. Dentre os 01. A formação da neve e a secagem de
processos de separação (filtração, decan- roupa no varal são exemplos de fenô-
tação, destilação, cristalização, tamização menos físicos, chamados solidificação e
e ventilação), indique quantos podem ser evaporação da água.
classificados como físicos e quantos podem 02. Um líquido homogêneo que apresenta
ser classificados como mecânicos e assinale ponto de ebulição constante é, necessa-
a alternativa correta. riamente, uma substância pura.
a. 2 físicos e 4 mecânicos. 04. Em um sistema, constituído por álcool
b. 4 físicos e 2 mecânicos. etílico, água e óleo de cozinha, o número
de fases é igual a três.
c. 1 físicos e 5 mecânicos.
08. Um dos processos frequentemente usa-
d. 3 físicos e 3 mecânicos. dos para separar o sal da água do mar é a
e. 5 físicos e 1 mecânicos. filtração.
113. PUC-RJ 16. Sublimação, fusão e condensação são
processos endotérmicos de mudança de
PV-13-14

A despeito dos sérios problemas ambien- estado físico.


tais, o mercúrio é ainda muito utilizado nos
115. UFRN
garimpos devido à sua singular capacidade
de dissolver o ouro, formando com ele um Atualmente, o Brasil é o maior produtor mun-
amálgama. Em muitos garimpos, o ouro se dial de etanol (CH3CH2OH) obtido a partir da
encontra na forma de partículas dispersas cana-de-açúcar. Usado como combustível, em
na lama, ou terra, dificultando, assim, a sua automóveis, o etanol é menos poluente que
os combustíveis fósseis. Os monossacarídeos,
extração. Nestes casos, adiciona-se mercú- provenientes da sacarose (cana-de-açúcar),
rio à bateia, forma-se o amálgama (que não produzem, em presença de um microrganismo
se mistura com a lama) e, em seguida, é feita vivo específico, uma solução que apresenta
a sua separação. Após separado da lama, o em torno de 8% de etanol.
amálgama é aquecido com um maçarico até O sistema abaixo é utilizado, no laboratório de
a completa evaporação do mercúrio, restan- análise de uma indústria, para a purificação do
do, assim, apenas o ouro. etanol.

83
Química Química geral e estequiometria

a. Nomeie o processo de transformação da sacarose em etanol e o de purificação do etanol


(mostrado na figura).
b. Qual a função da vidraria (V1) indicada nessa figura? Com base nas interações intermolecula-
res, explique por que, na mistura gasosa, a quantidade do etanol será maior que a da água.
116. UFPB
Gigantes reservas de petróleo foram encontradas recentemente no Brasil. Essas reservas situam-se
em regiões de grandes profundidades em águas oceânicas e abaixo de uma camada de sal, por isso,
denominadas de pré-sal. Com a exploração dessas reservas, o Brasil aumentará significativamente
a produção de petróleo. Após a extração, o petróleo é transportado até as refinarias, onde passa-
rá por uma série de processos de purificação denominada de refino, em que o petróleo entra na
fornalha, é aquecido e segue para a torre de destilação, onde serão separadas as diversas frações.
Ao abastecer o veículo em um posto, o consumidor adquire a gasolina “C”, uma mistura de gaso-
lina “A” (pura) com álcool anidro. A proporção em volume de álcool anidro nessa mistura é de-
terminada por resoluções federais, podendo variar entre 20% e 25%. O teor de álcool na gasolina
deve ser analisado, a fim de aferir a qualidade desse combustível, como ilustrado a seguir:

PV-13-14

Disponível em: <http://www.labvirtq.fe.usp.br/simulacoes/quimica/sim_qui_


gasolinaadulterada.htm>. Acesso em: 29 set. 2009. Adaptado.

84
Química geral e estequiometria Química

A partir dos dados da ilustração, é correto afir- Ciclo do nitrogênio


mar que, em volume, o teor de álcool presente
nessa gasolina é de: Luz
CO2
Alimento
a. 32%
b. 68%
Plantas
c. 66% e algas
d. 20% Peixes

e. 24% O2

117. PUCCamp-SP Nitrato NO3–


Em aquários estabilizados, ou seja, Matéria
orgânica
com equilíbrio biológico, a ocorrência de Bactérias
Nitrobacter Amônia
doenças e a morte prematura de peixes di- NO2–
Bactérias
Nitrito NH3
ficilmente ocorrerão. Bactérias e fungos
NH+4
Alguns dos cuidados fundamentais Nitrosomonas
com a qualidade da água estão relacio-
nados ao ciclo do nitrogênio e sistema de A filtragem ou filtração é um método de separa-
filtragem, além de outros parâmetros que ção de misturas do tipo sólido-líquido e sólido-
devem ser controlados, como a dureza -gás. Outras técnicas que permitem separar
misturas desses tipos são, respectivamente:
total, a dureza em carbonatos, o oxigênio
dissolvido e o pH. a. destilação simples e sedimentação.
Disponívem em: <http://www.labcon.com.br/ b. destilação fracionada e centrifugação.
livreto/aquario/qualidadedaagua.htm>.
c. centrifugação e decantação.
A imagem ao lado ilustra o ciclo do nitrogênio d. cristalização e cromatografia.
no aquário. e. cromatografia e destilação simples.

118. PUCCamp-SP
A obtenção do álcool etílico hidratado, a partir da cana-de-açúcar, pode ser representada pelo
esquema seguinte.
I II
Cana-de- Moagem e Aquecimento
Garapa Melaço
-açúcar separação para concentrar
do bagaço o açúcar
PV-13-14

III
Mosto Fermentação –
Transformação do açúcar
em álcool

IV
Separação dos
componentes
mais voláteis

Vinhoto Álcool
hidratado

85
Química Química geral e estequiometria

Em I e IV, que envolvem processos de fracionamento, são realizadas, respectivamente:


a. filtração e destilação.
b. destilação e decantação.
c. filtração e decantação.
d. destilação e filtração.
e. decantação e decantação.
119. UFBA
Com base no diagrama abaixo, é correto afirmar:

Sistema
trifásico

Processo mecânico
de separ ação X

SistemaB
SólidoA (mistura líquida heterogênea)

Aquecimento Processo de separaçãoY


(com base na diferença de densidade)

Si stemaC SistemaD
CO2(g) CaO(s) (monofásico)
PF = 16,3 °C (monofásico)

Processo de
separação Z PV-13-14

Água CaCO3(s)

01. O processo X é a filtração.


02. O sólido A é o carbonato de cálcio, CaCO3 .
04. O processo Y é a decantação.
08. O sistema C é uma mistura homogênea.
16. O sistema D é uma substância pura.
32. O processo Z é uma destilação simples.
64. A água destilada é uma mistura.

86
Química geral e estequiometria Química

120. Unifesp
A fenolftaleína apresenta propriedades catárticas e por isso era usada, em mistura com a-lactose
monoidratada, na proporção de 1 : 4 em peso, na formulação de um certo laxante. Algumas das
propriedades dessas substâncias são dadas na tabela.
Solubilidade (g/100 mL)

Substância Ponto de fusão (° C) Água Etanol

Fenolftaleína 260 – 265 Praticamente insolúvel 6,7 a 25 °C

25 a 25 °C
α-lactose · H2O 201 – 202 Praticamente solúvel
95 a 80 °C

Água
100 mL
Etanol 80 °C
350 mL
25 °C Lactose

Sólido Filtração
Agitar e
resfriar a Filtrado
Mistura Filtração 25 °C
(100 g) Agitar
F aleína
Evaporar até
Filtrado Filtração
começar a Agitar
cristalizar Filtrado

Água
100 mL
25 °C

Deseja-se separar e purificar essas duas substâncias, em uma amostra de 100 g da mistura. Com
base nas informações da tabela, foi proposto o procedimento representado no fluxograma acima.
a. Supondo que não ocorram perdas nas etapas, calcule a massa de lactose que deve se cris-
talizar no procedimento adotado.
b. Com relação à separação / purificação da fenolftaleína, explique se o volume de etanol
proposto é suficiente para dissolver toda a fenolftaleína contida na mistura. Usando seus
PV-13-14

conhecimentos sobre solubilidade do etanol em água, explique por que a adição de água
à solução alcoólica provoca a cristalização da fenolftaleína.

87
Química Química geral e estequiometria

Capítulo 03
121. UFPE mica média do neônio? (Massa atômica média
As massas atômicas são essenciais para os cál- do neônio na escala atual = 20,18 u)
culos da química. Se uma nova escala de massas a. 20,18/12 u d. 20,18 · 12 u
atômicas médias fosse definida, baseada na su- b. 12 u e. 12/20,18 u
posição da massa de um átomo de carbono-12
(12C) ser exatamente 1 u, qual seria a massa atô- c. 20,18 u

122. UFTM-MG
Uma amostra de cromo foi analisada com espectrômetro de massa, que determina a composição
isotópica de um elemento químico. O gráfico obtido mostra a constituição aproximada, em por-
centagem de átomos, dos 4 isótopos naturais desse elemento.
Isótopos estáveis de cromo
100
90 84
Abundância (%)

80
70
60
50
40
30
20
10 4 10
2
0
48 49 50 51 52 53 54 55 56
Massa atômica (u)

Como mostra o gráfico, dos isótopos de números 50 a 54, apenas o isótopo 51 não ocorre na na-
tureza. O cromo-51 é artificial, sendo produzido em reatores e cíclotrons e utilizado em medicina
nuclear e na marcação radioativa de células. Uma forma de produzi-lo é irradiar com prótons um
alvo metálico de certo elemento X, ocorrendo a reação nuclear representada por:
X + p → 51Cr + n
De acordo com os resultados do espectro de massa, o valor que mais se aproxima da massa atô-
mica do cromo é:
a. 51,7 u. c. 52,5 u. e. 53,5 u.
PV-13-14

b. 52,1 u. d. 52,9 u.

123. E.E.Mauá-SP a. no núcleo do átomo de cloro, devem


Uma vez que as massas atômicas do oxigênio e existir outras partículas além dos pró-
do sódio são, respectivamente, 16 e 23, então tons e nêutrons.
a massa de 23 átomos de oxigênio é a mesma b. o cloro apresenta-se na natureza como
que a de 16 átomos de sódio. Essa afirmativa é uma mistura de isótopos.
verdadeira ou falsa? Justifique. c. há um erro experimental na determina-
124. Fuvest-SP ção das massas atômicas.
A massa atômica do cloro é 35,457. O fato de d. a constante de Avogadro não é um nú-
esse número não ser inteiro indica que: mero inteiro.
e. a massa atômica leva em conta a massa
dos elétrons.

88
Química geral e estequiometria Química

125. UFPE Com base nesses dados, a massa atômica do


O cobre consiste em dois isótopos com massa boro, expressa em u, é igual a:
62,96 u e 64,96 u e abundância isotópica de a. 10
70,5% e 29,5%, respectivamente. A massa atô- b. 10,5
mica do cobre é: c. 10,8
a. 63,96 u d. 62,55 u d. 11
b. 63,00 u e. 63,55 u e. 11,5
c. 63,80 u 130. UEM-PR
126. O composto (CH3)2SiCl2 é um precursor im-
portante na produção do polímero silicona.
A massa atômica de um elemento químico é Considere que, na natureza, o cloro é cons-
dada pela média ponderada dos isótopos. Por tituído de 75% de 35Cl e 25% de 37Cl; o silício
exemplo, a massa do oxigênio que aparece na é constituído de 92% de 28Si, 5% de 29Si e 3%
tabela é 15,99, isto porque na natureza en- de 30Si; o carbono é constituído de 99% de
contramos 16O – 99,76%, 17O – 0,04% e 18O – 12C e 1% de 13C; considere, ainda, que todo
0,20%. Sabendo-se que na natureza existe 10B hidrogênio seja 1H.
– 20%, 11B – 80%, podemos dizer que a massa
do boro que aparece na Tabela Periódica é: Sobre o exposto, assinale o que for correto.
a. 10,5 u d. 11 u 01. A probabilidade de se encontrarem 2
átomos de 35Cl em uma molécula do
b. 10 u e. 10,2 u
composto é 45%.
c. 10,8 u
02. A massa de uma molécula do composto
pode variar entre 128 u e 136 u.
127. FGV-SP
04. A massa atômica média do Si é 28,11 g/mol.
O cloro é encontrado na natureza em duas for-
mas isotópicas de 35 e 37 unidades de massa 08. A probabilidade de se encontrar 1 áto-
atômica. Dado que a massa atômica média do mo de 37Cl ligado a um átomo de 28Si
cloro é de 35,45 u, qual a percentagem dos em uma molécula do composto é maior
dois isótopos na natureza? que 30%.
a. 86,7% 35Cl + 13,3% 37Cl 16. A molécula (13CH3)230Si35Cl2 possui o nú-
b. 66,7% 35Cl + 33,3% 37Cl mero total de nêutrons igual ao núme-
ro total de prótons.
c. 80,0% 35Cl + 20,0% 37Cl
d. 72,2% 35Cl + 27,8% 37Cl 131. Unimontes-MG
e. 77,5% 35Cl + 22,5% 37Cl O átomo do elemento químico cálcio (Ca), de
número atômico 20, é encontrado na natureza
PV-13-14

128. Vunesp como uma mistura de 6 isótopos, nas seguin-


O elemento cloro tem o número atômico 17 e tes abundâncias relativas (%):
a massa atômica 35,45. Na natureza, há ape-
nas dois isótopos desse elemento: 35Cl = 34,97 Isótopos Abundâncias relativas (%)
e 37Cl = 36,97
40 96,96
a. Indique o número de prótons, elétrons
e nêutrons do 37Cl. 42 0,64
b. Calcule a composição percentual de 43 0,145
cada isótopo.
44 2,07
129. Vunesp
Na natureza, de cada 5 átomos de boro, 1 tem 46 0,0033
massa atômica igual a 10 u. (unidade de massa 48 0,185
atômica) e 4 têm massa atômica igual a 11 µ.

89
Química Química geral e estequiometria

De acordo com a tabela mostrada e as proprie- 02. Para íons de mesma carga, quanto
dades dos isótopos, é incorreto afirmar que: maior quantidade de movimento de
a. a abundância relativa de amostras na- um íon analisado por espectroscopia
turais diferentes é a mesma. de massa, tanto maior será o raio de
curvatura da trajetória deste íon na re-
b. o número de nêutrons de todos os isó- gião de detecção do equipamento.
topos do Ca é igual a 22.
04. Íons de isótopos são distinguíveis por
c. o isótopo 48 do átomo de cálcio possui espectroscopia de massa.
o maior número de massa.
08. Íons de isóbaros são distinguíveis por
d. o núcleo do isótopo 40 é o que apre- espectroscopia de massa.
senta a maior estabilidade.
16. Íons de isótonos são distinguíveis por
132. UEPB modificado espectroscopia de massa.
O Brasil, querendo assumir uma projeção 134. Cesesp-PE
no cenário diplomático internacional, jun- Existem dois isótopos do rubídio que ocor-
tamente com a Turquia, fez um acordo com rem na natureza: 85Rb, que tem massa igual a
o Irã sobre o enriquecimento de urânio. De 84,91, e 87Rb, cuja massa é 86,92. A massa atô-
fato, o processo de enriquecimento de urâ- mica do rubídio é 85,47. Qual é a porcentagem
nio significa aumentar o teor do urânio-235, do 87Rb?
utilizado em fissão nuclear. Sabendo que as
a. 72,1%
proporções dos isótopos naturais do urâ-
nio são: 99,27% de urânio-238, 0,72% de b. 20,1%
urânio-235 e 0,0055% de urânio-234, qual c. 56,0%
a massa atômica do urânio enriquecido se d. 27,9%
as quantidades forem 70% de urânio-238 e e. 86,9%
30% de urânio-235?
135. UFSC
a. 237,1 g/mol
b. 238,03 g/mol A massa de um determinado elemento é 5/6
c. 237,1 u da massa de isótopo 12 do carbono. Qual sua
d. 238,03 u massa atômica?
e. 236,5 g/mol 136.
133. UEM-PR Sabendo que a massa atômica da prata é igual
O espectrômetro de massa é um equipa- a 108 u, podemos afirmar que um átomo de
mento capaz de determinar massas atômi- prata pesa:
cas e moleculares de íons, através da análise I. 108 g.
do movimento dessas partículas sob a ação
PV-13-14

II. 108 u.
de campos magnéticos uniformes e ortogo-
nais à direção de propagação desses íons. III. 108 vezes mais que o átomo de 12C.
Considere um espectrômetro de massa no IV. 108 vezes mais que 1/12 do átomo de
12 C.
qual a velocidade dos íons injetados na re-
gião do campo magnético é sempre a mes- V. 9 vezes mais que um átomo de 12C.
ma. Analise as alternativas abaixo e assinale
Está(ão) correta(s) a(s) afirmação(ões):
o que for correto.
01. Para íons de mesma massa, quanto a. I, III e V, apenas.
maior a carga do íon analisado por es- b. II, III e V, apenas.
pectroscopia de massa, tanto maior c. II, IV e V, apenas.
será o raio de curvatura da trajetória
deste íon na região de detecção do d. II e IV, apenas.
equipamento. e. I, apenas.

90
Química geral e estequiometria Química

137.
Massas
Isótopo Ocorrência (%)
O alumínio tem número atômico igual a 13 e atômicas
é constituído por um único isótopo, contendo
A 60 80 u
14 nêutrons.
Com base nessa informação, podemos afirmar B 20 84 u
que: C 20 88 u
I. A massa atômica do alumínio é 27 u.
II. O átomo de alumínio pesa 27 vezes
mais que 1/12 do átomo de 12C. 140. Fuvest-SP
III. O átomo de alumínio pesa 2,25 vezes A definição atual de massa atômica de um ele-
mais que o átomo de 12C. mento corresponde a:
IV. 12 átomos de alumínio pesam tanto a. 1 x (massa do átomo desse elemento:
quanto 27 átomos de 12C. massa do átomo C “doze”).
b. 12 x (massa do átomo desse elemento:
Estão corretas as afirmações: massa do átomo C “doze”).
a. I e IV, apenas. c. 1/12 x (massa do átomo desse elemen-
b. II, III e IV, apenas. to: massa do átomo C “doze”).
c. I, II e IV, apenas. d. 12/16 x (massa do átomo desse ele-
d. I, II e III, apenas. mento: massa do átomo C “doze”).
e. I, II, III e IV. e. 16/12 x (massa do átomo desse ele-
mento: massa do átomo C “doze”).
138.
141.
O cloro é formado dos isótopos 35Cl (75%) e 37Cl
Usando a tabela de massas atômicas, aproxi-
(25%).
mando, porém, os valores para os números
Com base nessa informação, podemos afirmar inteiros mais próximos, calcule as massas mo-
que: leculares das seguintes substâncias:
I. Um átomo de cloro pesa 35,5 u. a. C2H6
II. Um átomo de cloro pesa, em média, b. SO3
35,5 u. c. NH3
III. Não existe átomo de cloro com massa d. S8
35,5 u.
e. H2SO4
IV. Um átomo de cloro tem massa aproxi-
f. CaCO3
madamente igual a 35 u ou 37 u.
PV-13-14

g. NaHSO4
Estão corretas somente as afirmações:
h. Al2(SO4)3
a. I, III e IV.
i. (NH4)3PO4
b. II, III e IV.
j. Cu(NO3)2
c. II e IV.
k. Fe4[Fe(CN)6]3
d. I e IV. l. Na2SO4 · 10 H2O
e. II e III. m. H4P2O7
139. n. CuSO4 · 5 H2O
Calcule a massa atômica de um elemento X,
constituído dos isótopos A, B e C, cuja ocor-
rência e respectivas massas atômicas estão
indicadas na tabela a seguir:

91
Química Química geral e estequiometria

142. A substância, chamada de abiraterona,


Um mol de ácido clorídrico (HCl) contém: possui a propriedade de inibir a forma-
ção de testosterona, sendo capaz de re-
(Dado: constante de Avogadro = 6 · 1023) verter a forma mais agressiva do câncer.
a. 6,0 · 1023 átomos de hidrogênio. Cerca de 70% dos pacientes que usaram
b. 1 mol de átomos. a droga apresentaram uma melhora sig-
nificativa. O medicamento bloqueia os
c. 6,0 · 1023 átomos. hormônios que nutrem as células cance-
d. 2 mol de cloro. rígenas.
e. 24 · 1023 moléculas. Band News, julho de 2008.
143.
Fórmula estrutural da abiraterona
Qual (quais) das afirmações seguintes está(ão)
correta(s)? N
I. Massa molecular é a massa da molécu-
la expressa em u.
II. A massa molecular é numericamente
igual à soma das massas atômicas de
todos os átomos da molécula.
III. A massa molecular indica quantas ve- H

zes a molécula pesa mais que 1/12 da H H

massa do átomo de 12C. HO


144. UERJ
Uma molécula de água, isolada, não apresenta A massa de uma única molécula de abiraterona é:
certas propriedades físicas – como ponto de
fusão e de ebulição – que dependem de inte- Dado: constante de Avogadro = 6,0 · 1023 mol–1
rações entre moléculas. a. 5,8 · 10–22 g
Em 1998, um grupo de pesquisadores deter- b. 6,0 · 10–23 g
minou que, para exibir todas as propriedades c. 1,2 · 10–24 kg
físicas, é necessário um grupamento de, no d. 350 g
mínimo, 6 moléculas de água.
e. 350 · (6 · 1023) g
O número desses grupamentos mínimos que
estão contidos em um mol de moléculas de 146. Uniube-MG
água corresponde a: A quantidade de átomos em um mol de ácido
Dado: constante de Avogadro: 6 · 1023 mol–1 sulfúrico é:
PV-13-14

a. 1,0 · 1023 a. 3 · 6,02 · 1023 átomos/mol


b. 3,0 · 1023 b. 4 · 6,02 · 1023 átomos/mol
c. 6,0 · 1023 c. 5 · 6,02 · 1023 átomos/mol
d. 9,0 · 1023 d. 6 · 6,02 · 1023 átomos/mol
145. UFABC-SP e. 7 · 6,02 · 1023 átomos/mol
147.
Cientistas desenvolvem droga contra
câncer de próstata Consultando a Tabela Periódica, determine a
massa molecular de:
Um grupo de cientistas britânicos
desenvolveu um medicamento contra o a. ureia: CO(NH2)2
câncer de próstata, que é considerado a b. ácido tiossulfúrico: H2S2O3
descoberta mais importante em 60 anos. c. bórax: Na2B4O7 · 10 H2O

92
Química geral e estequiometria Química

148. 152. PUC-RS


Considere a adição de água em meio litro de vi- Atualmente, o termo “mol” é definido como:
nagre contendo 0,3 mol de ácido acético, até um a. a quantidade de matéria de um sistema
volume final de 2,0 litros. Qual o número de mo- que contém tantas entidades elemen-
léculas de ácido acético na amostra de vinagre? tares quantos são os átomos contidos
(Dado: constante de Avogadro = 6 · 1023) em 0,012 kg de carbono 12.
a. 0,3 · 1023 b. a massa atômica ou molecular expressa
b. 1,8 · 1023 em gramas.
c. 2,4 · 1023 c. um sinônimo de molécula-grama, áto-
mo-grama ou íon-grama.
d. 3,0 · 1023
d. a massa de qualquer substância que
e. 3,6 · 1023
encerra 6,02 ⋅ 1023 moléculas.
149. e. o número de átomos que estão pre-
Considere a mistura de 5 mol de butano (C4H10) sentes em 1,0 g de qualquer substân-
e 3,5 mol de etano (C2H6), contidos num reci- cia elementar, em repouso e no estado
piente de 10 litros a 127 °C. fundamental.
O número total de átomos de carbono no sis- 153.
tema é:
A quantidade de átomos em 1 mol de NaOH é:
(Dado: constante de Avogadro = 6 · 1023)
(Dado: constante de Avogadro = 6,02 · 1023)
a. 8,5
a. 1 · 6,02 · 1023 átomos/mol
b. 8,5 · 1023
b. 2 · 6,02 · 1023 átomos/mol
c. 6 · 1023
c. 3 · 6,02 · 1023 átomos/mol
d. 1,62 · 1025
d. 4 · 6,02 · 1023 átomos/mol
e. 1,2 · 1025
e. 5 · 6,02 · 1023 átomos/mol
150.
154. UnB-DF
No ar poluído de uma cidade, detectou-se
uma concentração de NO2 correspondente a Os microprocessadores atuais são muito pe-
1,0 · 10–8 mol/L. Supondo que uma pessoa ina- quenos e substituíram enormes placas con-
le 3 litros de ar, o número de moléculas de NO2 tendo inúmeras válvulas. Eles são organiza-
por ela inaladas é: dos de forma que apresentem determinadas
respostas ao serem percorridos por um im-
(Dado: constante de Avogadro = 6,0 · 1023) pulso elétrico. Só é possível a construção de
a. 1,0 · 108 dispositivos tão pequenos devido ao diminu-
b. 6,0 · 1015 to tamanho dos átomos.
PV-13-14

c. 1,8 · 1016 Sendo estes muito pequenos, é impossível


d. 2,7 · 1022 contá-los. A constante de Avogadro – e não
o número de Avogadro – permite que se cal-
e. 6,0 · 1023
cule o número de entidades – átomos, molé-
151. culas, fórmulas unitárias etc. – presentes em
A quantidade de mols existentes em 1,5 · 1024 uma dada amostra de substância. O valor
moléculas de ácido fosfórico (H3PO4) é igual a: dessa constante, medido experimentalmen-
te, é igual a 6,02 · 1023 mol–1. Com relação ao
a. 0,5
assunto, julgue os seguintes itens.
b. 1,0
0. A constante de Avogadro é uma gran-
c. 1,5 deza, sendo, portanto, um número
d. 2,0 (6,02 · 1023) multiplicado por uma uni-
e. 2,5 dade de medida (mol–1).

93
Química Química geral e estequiometria

1. A constante de Avogadro, por ser uma Ca3(PO4)2 e 6,5 · 10–3 mol de Mg3 (PO4)2 está
grandeza determinada experimental- ingerindo:
mente, pode ter seu valor alterado em a. a dose correta de Mg e excesso de Ca e P.
função do avanço tecnológico. b. a dose correta de Ca e excesso de Mg e P.
2. Massas iguais de diferentes elementos c. excesso de Mg, Ca e P.
químicos contêm o mesmo número de
átomos. d. excesso de Mg e escassez de Ca e P.
e. a dose correta de P e Ca e excesso de Mg.
3. Entre os elementos químicos, o único
que, em princípio, não está sujeito a 156.
uma variação de massa atômica é o isó- Em 3,0 mol de H2SO4 e 5,0 mol de Br2, existem,
topo do carbono de massa 12,00 u. respectivamente:
155. (Dado: constante de Avogadro: 6,02 · 1023)
Dose diária recomendada para um adulto: a. 1,8 · 1024 moléculas e 3,01 · 1024 mo-
léculas.
Mg ........ 1,20 · 10–2 mol
b. 3,0 · 1023 moléculas e 5,0 · 1023 moléculas.
Ca ......... 1,95 · 10–2 mol
c. 1,8 · 1024 moléculas e 3,01 · 1024 átomos.
P ............ 2,60 · 10–2 mol d. 1,8 · 1024 átomos e 3,01 · 1024 moléculas.
Um indivíduo que toma diariamente um su- e. 6,02 · 1023 moléculas e 12,04 · 1023 mo-
plemento alimentar com 6,5 · 10–3 mol de léculas.

157. UnB-DF
Linus Pauling desenvolveu o conhecimento relativo a princípios fundamentais relacionados à na-
tureza das ligações químicas e à estrutura das moléculas, propiciando explicações em torno das
propriedades da matéria. A partir de 1936, juntamente com assistentes e colegas, dedicou-se
ao estudo das propriedades de sistemas vivos. Em 1960, introduziu a Medicina Ortomolecular,
termo utilizado por Pauling para denominar uma nova área do conhecimento, que consiste no
estudo de nutrientes, que inclui a administração de megadoses de minerais e vitaminas. Pauling
assegurou, em 1972, que a vitamina C poderia aliviar, prevenir e, em certos casos, curar o câncer,
o que gerou uma polêmica que dura até hoje. Tanto as vitaminas quanto os sais minerais agem
nos diferentes ciclos metabólicos do organismo, ajudando na produção de trifosfato de adenosi-
na (ATP), fonte mais comum de energia nos sistemas biológicos.
Trabalhos pioneiros, relacionados a enzimas que participam da conversão do ATP, cuja fórmula es-
trutural é apresentada abaixo, foram realizados por três cientistas laureados com o Prêmio Nobel
de Química, em 1997 – Boyer (EUA), Walker (Inglaterra) e Skou (Dinarmarca).
PV-13-14

94
Química geral e estequiometria Química

Esse Prêmio Nobel, pela natureza das pesqui- a. 8,0 kg. d. 16,0 kg.
sas envolvidas, ressalta a interdisciplinaridade b. 4,0 kg. e. 10,0 kg.
nos processos de produção de conhecimento,
c. 12,0 kg.
pois está inter-relacionado com bioquímica,
biologia etc. O conhecimento da linguagem 160. Fuvest-SP
química e da simbologia, em seus enunciados
fundamentais, é imprescindível para a com- A região metropolitana de São Paulo tem cerca
preensão das explicações propostas para os de 8.000 km2. Um automóvel emite diariamente
fenômenos estudados. A esse respeito, julgue cerca de 20 mol de CO. Supondo que esse gás se
os seguintes itens. distribua uniformemente por toda a área metro-
politana até uma altura de 10 km, quantas molé-
1. Na fórmula estrutural do ATP, é possível culas de CO emitidas por esse automóvel serão
identificar a presença de três anéis. encontradas em 1 m3 do ar metropolitano?
2. A partir da fórmula apresentada, con-
(Dados: número de Avogadro: 6 · 1023 molé-
clui-se que o ATP é uma substância
culas/mol)
composta.
3. Em cinco mol de moléculas de ATP, exis- 161. UFPR
tirão vinte e cinco mol de átomos de Este ano ocorreu um terrível acidente ambien-
nitrogênio e quinze mol de átomos de tal com o vazamento de petróleo no Golfo do
fósforo. México. O vazamento, que durou meses na
4. Em sua atividade no laboratório, um plataforma Deepwater Horizon, da British Pe-
químico pode medir diretamente, por troleum, pode ter derramado 4,5 milhões de
meio de balanças ou frascos volumétri- barris de petróleo no mar. Considerando que
cos, massas ou volumes, mas não exis- um barril corresponde a 159 litros e que a den-
te maneira de se medir diretamente a sidade do petróleo é de 0,88 kg . L–1, qual é a
quantidade de matéria de uma amostra quantidade de matéria (em mols) aproximada
de ATP ou de qualquer outra substância de carbono presente no petróleo derramado?
estudada. Assuma que a composição do petróleo é de
alcanos de fórmula geral CnH2n+2. (M (g.mol–1):
158. Vunesp H = 1,008; C = 12,01)
Estudos apontam que a amônia presente na Assinale a alternativa correta.
fumaça do cigarro aumenta os níveis de absor-
ção de nicotina pelo organismo. Nos cigarros a. 4,4 · 1010.
canadenses, por exemplo, os níveis de amônia b. 4,5 · 102.
(NH3) são por volta de 8,5 mg por cigarro. c. 6,02 · 1023.
O número aproximado de moléculas NH3 na d. 1,0 · 10200.
fumaça emitida pela queima de um cigarro ca- e. 4,6 · 106.
nadense é:
PV-13-14

162. FGV-SP
a. 1,2 · 1026 .
O rótulo de um pacote de batata frita indi-
b. 3,0 · 1026 . ca que o produto possui 5% do valor diário
c. 3,0 · 1023 . de referência (VD) de NaCl. Dadas as massas
d. 3,0 · 1020. molares em g · mol–1, Na+ = 23; Cl– = 35,5 e a
constante de Avogadro, 6,02 · 1023 mol–1, e
e. 1,2 · 1020.
sabendo-se que o VD definido pela Organiza-
159. UFR-RJ ção Mundial da Saúde para o NaCl é de 2,4 g,
Um balão de oxigênio contendo 3 · 1026 átomos quantos íons Na+ serão ingeridos se o conteú-
foi completamente utilizado por uma equipe do total desse pacote for consumido?
médica durante uma cirurgia. Admitindo-se a. 0,012. d. 31 · 1020.
que havia apenas gás oxigênio neste balão, a
b. 0,020. e. 20 · 1020.
massa utilizada do referido gás foi equivalente
a: (O = 16) c. 12 · 1020.

95
Química Química geral e estequiometria

163. PUC-RJ 166. FMJ-SP


A massa de um átomo pode ser calculada a Suplementos alimentares, muito utilizados
partir do número de massa do átomo e da atualmente, podem trazer grandes benefí-
constante de Avogadro (6,02 · 1023 mol−1). As- cios à saúde, mas se utilizados sem limites
sinale a alternativa que indica a massa aproxi- e sem a orientação de um especialista, po-
mada de um átomo de 56Fe, em gramas. dem ser prejudiciais. “Ferro” é o nome de
a. 1,8 · 10−19 d. 5,5 · 10−22 um suplemento mineral comercializado em
frascos de 50 cápsulas com 14 mg de ferro
b. 2,2 · 10−20 e. 9,3 · 10−23
cada. Dada a constante de Avogadro 6,0 1023
c. 3,2 · 10−21 mol–1, a quantidade total de átomos de fer-
164. UESPI ro, massa molar 56 g/mol, presente nas cáp-
A butadiona (C4H6O2) é uma substância orgâ- sulas de um frasco do suplemento alimentar
nica líquida, amarela, volátil, com cheiro de “Ferro” é igual a
queijo e utilizada na fabricação da manteiga. a. 7,5 · 1024
Quando se usa 4,3 g dessa substância na pre- b. 7,5 · 1021
paração da manteiga, quantos átomos de car-
bono estão sendo adicionados? c. 2,4 · 1024
Dados: Massas molares em g · mol−1: C = 12; d. 1,5 · 1023
H = 1; O = 16. Constante de Avogadro = e. 1,5 · 1020
6 · 1023 mol−1. 167. UESPI
a. 1,2 · 1023átomos
O silício (Si) é utilizado para a produção de li-
b. 12 átomos gas metálicas, na preparação de silicones, na
c. 12 · 1023 átomos indústria cerâmica, e como material básico
d. 6 · 1022 átomos para a produção de transistores para chips,
e. 6 · 1023 átomos células solares e em diversas variedades de
circuitos eletrônicos, tendo sido preparado
165. UFJF-MG
pela primeira vez por Jöns Jacob Berzelius,
Garrafas PET são muito usadas para o engar- em 1823. Um chip de silício, usado em um
rafamento de água e refrigerantes. Estima-se circuito integrado de computador pesando
que a produção de 1 kg de garrafas PET utilize 5,68 mg, apresenta:
17,5 L de água e libere 2,3 kg de gás carbônico.
Outro problema relacionado à sua utilização é Dados: massa atômica do silício = 28,09.
que muitas dessas garrafas plásticas vão entu- a. 0,21 · 1020 átomos.
pir as redes de esgoto e parar nos leitos dos b. 1,02 · 1020 átomos.
rios. Assinale a opção incorreta. Dado: núme-
ro de Avogadro = 6,02 · 1023 mol–1. c. 1,21 · 1020 átomos.
a. A produção de 1 kg de garrafas PET libe- d. 2,01 · 1020 átomos.
PV-13-14

ra, aproximadamente, 52,3 mols de CO2. e. 2,21 · 1020 átomos.


b. A produção de 0,5 kg de garrafas PET li- 168. Unicid
bera, aproximadamente, 1,57 · 1025 mo-
léculas de CO2. No inverno, com o clima seco e excesso de
c. O volume ocupado nas CNTP pelo CO2 poluentes nas cidades metropolitanas, es-
liberado na produção de 1 kg de garra- pecialmente São Paulo, aumenta o número
fas PET é, aproximadamente, 1171 L. de casos de doenças respiratórias. Algumas
d. A produção de 1(uma) tonelada de gar- pessoas utilizam suplementos vitamínicos e
rafas PET utiliza 1,75 · 104 mL de água. minerais para ajudar no combate a resfria-
dos. Um deles é um medicamento apresen-
e. As garrafas PET constituem um grave
problema ambiental porque, além de tado em comprimidos efervescentes, cada
poluírem os rios, na sua produção está qual com 1.000 mg de vitamina C (massa
envolvida a liberação de um gás que é molar 176 g/mol), 10 mg de zinco e, como
responsável pelo aquecimento global. adoçante, o aspartame.

96
Química geral e estequiometria Química

OH
O O
O OH
O
O
O
NH
OH
NH2 OH HO
Vitamina C

Aspartame

Cada comprimido efervescente do suplemento alimentar contém, aproximadamente, ____ áto-


mos de zinco e ____ mg de carbono contidos na vitamina C.
Dado: Constante de Avogadro = 6,0 · 1023 mol–1
As lacunas podem ser preenchidas, correta e respectivamente, por:
a. 1,5 · 10–4 … 409 d. 1,5 · 10–4 … 386
b. 9,2 · 1019 … 409 e. 9,2 · 1019 … 176
c. 2,0 · 1020 … 386
169. UERJ
Algumas doenças infecciosas, como a dengue, são causadas por um arbovírus da família
Flaviridae.
São conhecidos quatro tipos de vírus da dengue, denominados DEN 1, DEN 2, DEN 3 e DEN 4;
os três primeiros já produziram epidemias no Brasil.
A doença, transmitida ao homem pela picada da fêmea infectada do mosquito Aedes aegypti, não tem
tratamento específico, mas os medicamentos frequentemente usados contra febre e dor devem ser
prescritos com cautela. Na tabela a seguir, são apresentadas informações sobre dois medicamentos:
Massa
Medicamento Fórmula estrutural molar
(g · mol– 1)

H
PV-13-14

N
Paracetamol 151

O
HO

O OH

Ácido O 180
acetilsalicílico

97
Química Química geral e estequiometria

O número de átomos existentes em uma amos- 172. Fuvest-SP


tra de 1 g de ácido acetilsalicílico é igual a: A embalagem de um sal de cozinha comer-
a. 3,3 · 1021 cial com reduzido teor de sódio, o chamado
b. 7,0 · 1022 “sal light”, traz a seguinte informação: “Cada
c. 6,0 · 1023 100 g contém 20 g de sódio...”. Isso significa
que a porcentagem (em massa) de NaCl nes-
d. 1,3 · 1025
se sal é, aproximadamente, igual a:
170. PUCCamp-SP
Massas molares (g/mol): Na = 23; NaCl = 58
A queima incompleta da biomassa produz a. 20
a fuligem e o monóxido de carbono, CO. Em
níveis próximos de 1.000 mg/L, esse poluen- b. 40
te ocasiona a inconsciência, podendo levar o c. 50
indivíduo à morte. Nesses casos, o número de d. 60
moléculas de CO inaladas para cada litro de ar e. 80
é, aproximadamente:
173. Unicamp-SP
Dados:
O sabão, apesar de sua indiscutível utilida-
Massas molares (g/mol) de, apresenta o inconveniente de precipitar
C = 12 o respectivo sal orgânico insolúvel em água
O = 16 que contenha íons de cálcio dissolvidos. Em
época recente, foram desenvolvidos os de-
Constante de Avogadro = 6 · 1023 mol–1 tergentes, conhecidos genericamente como
a. 6,0 · 1023 alquilsulfônicos, solúveis em água e que não
b. 1,2 · 1023 precipitam na presença de íons de cálcio.
c. 2,1 · 1022 a. Dê o símbolo e o nome do elemento
d. 9,0 · 1020 químico que aparece na fórmula de um
detergente alquilsulfônico e que não
e. 1,2 · 1020 aparece na fórmula de um sabão
171. Fuvest-SP b. Considerando que a fórmula de certo
A dose diária recomendada do elemento cál- detergente alquilsulfônico é C12H25O4XNa,
cio para um adulto é de 800 mg. Suponha cuja massa molar é 288 g/mol, calcule a
certo suplemento nutricional, à base de casca massa molar do elemento X.
de ostras, que seja 100% CaCO3. Se um adulto Dados:
tomar diariamente dois tabletes desse suple- massas molares em g/mol: H = 1; C = 12; O = 16;
mento de 500 mg cada, qual porcentagem de Na = 23
PV-13-14

cálcio da quantidade recomendada essa pes-


soa estará ingerindo? 174. Unesp
Uma amostra do hormônio feminino estra-
Massas molares (g/mol) diol, de fórmula molecular C18H24O2, contém
Ca .............. 40 3,0 · 1020 átomos de hidrogênio. O número
O .............. 16
C .............. 12 de átomos de carbono existentes na mesma
massa de estradiol é:
a. 25% a. 1,8 · 1020
b. 40% b. 2,25 · 1020
c. 50% c. 3,0 · 1020
d. 80% d. 2,4 · 1023
e. 125% e. 6,0 · 1018

98
Química geral e estequiometria Química

175. Unicamp-SP modificado 178. FEI-SP


Dois frascos idênticos estão esquematizados a Determine o número de átomos de hidrogênio
seguir. Um deles contém certa massa de água contidos em 100,0 g de álcool etílico (C2H6O).
(H2O) e o outro, a mesma massa de álcool etí- Dados: H = 1; C = 12; O = 16 e constante de
lico (CH3CH2OH). Avogadro = 6 · 1023
179. PUCCampinas-SP
A B
Uma das metas do Conselho Nacional do Meio
Ambiente é que os carros novos, em 1997,
emitam 2,0 g de monóxido de carbono por
quilômetro. Nestas condições, quantas molé-
culas do gás serão emitidas, aproximadamen-
Usando-se uma bolinha de densidade adequa- te, por um carro ao percorrer 15 km?
da, fez-se o seguinte experimento: Dados: Massas molares
C = 12,0 g/mol
O = 16,0 g/mol
Constante de Avogadro: 6,0 · 1023
a. 2,0
Água Álcool b. 3,0
c. 3,2 · 1023
Qual dos frascos contém maior número de d. 6,4 ·1023
átomos? e. 9,0 · 1023
Massas molares: H2O = 18 g/mol; C2H6O = 46 g/mol 180. Unifesp
176. UFMT
O rótulo de um frasco contendo um suplemen-
Propano (C3H8) é um dos componentes do gás to vitamínico informa que cada comprimido
de cozinha. O número de mols de propano contém 6,0 · 10–6 gramas de vitamina B12 (cia-
contidos em 3,01 · 1022 moléculas dessa subs- nocobalamina). Essa vitamina apresenta 1 mol
tância é igual a: de cobalto por mol de vitamina e sua por-
Dado: constante de Avogadro = 6,02 · 1023 centagem em peso é de, aproximadamente,
a. 5 · 10–2 4%. Considerando a constante de Avogadro
b. 2 · 10–2 6,0 · 10 23 mol –1 e a massa molar de cobal-
to 60 g/mol, qual o número aproximado de
c. 5 · 10–1
átomos de cobalto que um indivíduo ingere
d. 2 · 10–1 quando toma 2 comprimidos?
PV-13-14

e. 10 · 10–1 a. 4,8 · 1015


177. Unimep-SP b. 2,4 · 1015
O número de átomos de carbono presente c. 4,8 · 1012
em 8 gramas de etanol (C2 H6 O) é aproxima-
damente igual a: d. 2,4 · 1012
e. 4,8 · 107
Dados: M.A: H = 1; C = 12; O = 16 e constante de
Avogadro = 6,0 · 1023 181. FEPECS-DF
a. 3,4 · 1022 Para tentar explicar a presença de
b. 1,1 · 1025 clorometano e diclorometano em amos-
tras recolhidas em Marte pela sonda Vi-
c. 3,0 · 1023 king, cientistas aqueceram uma mistura
d. 2,1 · 1023 de percloratos com o solo do deserto do
e. 4,0 · 1027 Atacama, no Chile, possivelmente similar

99
Química Química geral e estequiometria

ao solo marciano. As reações químicas da 184. UFMS


experiência destruíram os compostos or- Contribuindo para o aumento dos índices de
gânicos do solo, liberando traços de cloro- poluição atmosférica, os motores do ciclo die-
metano e diclorometano como os encon- sel lançam no ar que respiramos diversos ga-
trados pela sonda. ses tóxicos, entre eles o dióxido de enxofre e o
O Globo, 7/9/2010. Adaptado. monóxido de carbono. A análise de uma amos-
Considerando que em uma amostra foram tra dos gases emitidos por um motor a diesel
encontradas 6 · 1015 moléculas de clorome- mostrou que ela continha 0,5 mol de dióxido
tano e 1,2 · 1017 moléculas de diclorome- de enxofre e 3,0 · 1023 moléculas de monóxido
tano, os números de mols aproximados de de carbono. A massa total, em gramas, refe-
clorometano e de diclorometano são, res- rente aos gases citados é igual a:
pectivamente: Dados: Massas atômicas em g/mol: C = 12;
Dado: NA = 6 · 10 23 O = 16; S = 32
a. 1 · 10–9 e 2 · 10–7 a. 12,8 d. 40,4
b. 1 · 10–8 e 2 · 10–7 b. 14,4 e. 46,0
c. 1 · 10–6 e 5 · 10–6 c. 24,4
d. 1 · 106 e 2 · 108 185. UFMS
e. 1 · 108 e 5 · 106
Sabendo-se que o número de Avogadro é igual
182. UFAL a 6,02 · 1023 e dadas as massas molares dos
Quando bebemos 250 g de água (aproxima- elementos químicos em g/mol: C = 12, O = 16,
damente 250 mL), admitindo ser desprezível H = 1, N = 14, assinale a(s) proposição(ões)
a presença de impurezas, podemos considerar correta(s).
correto dizer que estamos ingerindo, aproxi- 01. A cafeína é um alcaloide estimulante
madamente: do sistema nervoso central, encontrado
nos grãos de café, nas folhas de certos ti-
Dado: Constante de Avogadro = 6,0 · 1023 mol−1
pos de chá e em refrigerantes à base de
a. 2,0 · 1024 átomos de oxigênio. cola. Sabendo-se que 60 mL de café de
b. 4,0 · 102 átomos de hidrogênio. coador possui 44,4 mg de cafeína cuja
c. 2,0 · 1023 moléculas de água. fórmula molecular é C8H10N4O2, a quan-
tidade de mols de moléculas de cafeína
d. 25 mols de átomos. presentes em uma xícara de 100 mL de
e. 42 mols de átomos. café comum será de 3,8 · 10–4 mol.
183. FEPECS-DF 02. A creatina, de fórmula molecular
Um herbicida muito tóxico produz efeitos con- C4H9N3O2, é uma substância comu-
gênitos, câncer, danos ao fígado, supressão mente ingerida pelos frequentadores
PV-13-14

de academias de musculação, pois é


do sistema imunológico e até a morte. A dose
derivada de um aminoácido presen-
letal desse composto foi determinada em co-
te nas células musculares e serve
baias e é igual a 6,0 · 10–7g/kg.
para repor energia quando há fadiga
A dose letal para um indivíduo de 60 kg é al- muscular. Em 1 kg de creatina, ha-
cançada quando o número de mols do herbi- verá, aproximadamente, 7,63 mols e
cida presente no seu organismo corresponde, 4,60 · 1024 moléculas.
aproximadamente, a: 04. Considerando-se que a dose diária reco-
a. 6,7 · 1016 mendada de vitamina C (C6H8O6) é de,
b. 5,4 · 108 aproximadamente, 70 mg, quando uma
pessoa ingere essa massa de vitamina C
c. 8,9 · 106
significa que o número de átomos de car-
d. 6,0 · 10–7 bono ingeridos foi de, aproximadamente,
e. 1,1 · 10–7 2,39 · 1023 átomos de carbono.

100
Química geral e estequiometria Química

08. Feromônios são hormônios sexuais se- 188. Ufla-MG


cretados pelas fêmeas de muitos inse- Segundo orientações nutricionais, a dose di-
tos. Normalmente, a quantidade secre- ária recomendada de vitamina C (C6H8O6) a
tada é de, aproximadamente, 1 · 10–12 g. ser ingerida por uma pessoa adulta é 62 mg.
O número de moléculas existentes nes- Um determinado cientista, grande defensor
sa massa de feromônio de fórmula mo- das propriedades terapêuticas dessa vitami-
lecular C19H38O é de cerca de 4,3 · 1015. na, consumia, diariamente, 7,05 · 10–3 mol da
16. Num dado maço de cigarros, consta a in- mesma. A dose ingerida pelo cientista é quan-
formação de que o produto contém mi- tas vezes maior que a recomendada?
lhares de substâncias tóxicas, entre elas a. 200,0
a nicotina (C10H14N2), com 0,65 mg dessa
substância por unidade. Sabe-se que a de- b. 1,2
pendência do cigarro se deve à presença c. 2,0
da nicotina, e o teor dessa substância refe- d. 20,0
re-se à fumaça gerada pela queima de um
189. UFPel-RS
cigarro. A quantidade em mol de molécu-
las de nicotina presentes na fumaça de um Sabe-se que o mol é um termo que designa
cigarro desse maço é cerca de 4 · 10–6 mol. o número de Avogadro. Do mesmo modo, é
sabido que, nas CNTP, o volume molar é 22,4
186. FEPECS-DF
litros e contém 1 mol de moléculas de qual-
Os sulfitos são conservantes usados
quer substância no estado gasoso que nele se
em sucos concentrados, frutas secas, bebi-
encontre.
das alcoólicas e outros produtos, e podem
provocar reações alérgicas se ingeridos Obs.: para facilitar possíveis cálculos, conside-
acima da IDA (ingestão diária aceitável), re o número de Avogadro igual a 6 · 1023.
que é de 0,7 mg/kg de peso corpóreo. Entre os sistemas apresentados pelas alternati-
Ciência Hoje, vol 43, p. 54, 2008. Adaptado. vas, consideradas as CNTP, assinale aquele que
contém o maior número de átomos de oxigênio.
O número de milimols de sulfito ingerido por
uma criança de 40 kg que alcançou a IDA cor- a. 1,5 litro de ozônio (O3)
responde a: b. 3 litros de oxigênio (O2)
a. 0,35 c. 2 litros de gás carbônico (CO2)
b. 0,70 d. 500 gramas de formol (solução aquosa
c. 1,40 a 40% em massa de metanal)
d. 2,10 e. 1 quilograma de solução aquosa de clo-
reto de sódio (NaCl) a 10% em massa
e. 2,85
de soluto
187. Unicid-SP
PV-13-14

190.
Macroelementos essenciais, tais como Ca, P,
Na, Mg e K, são aqueles que o organismo ne- Gasolina nacional gera mais ozônio,
cessita em quantidades iguais ou superiores a diz estudo da USP
100 mg por dia. O ozônio troposférico não é elimi-
A quantidade mínima em mol de átomos de cál- nado diretamente pelos escapamentos dos
cio e de fósforo recomendada para um indivíduo carros. Ele resulta de uma reação química
consumir é, aproximada e respectivamente entre compostos orgânicos voláteis pre-
a. 4,0 e 3,1 sentes nos combustíveis, óxido nítrico
(NO), oxigênio do ar (O2) e a luz solar.
b. 2,5 e 3,2 Uma gasolina “suja”, como a paulista,
c. 0,40 e 0,31 possui 45% em massa de aromáticos, 30%
d. 2,5 · 10–3 e 3,2 · 10–3 em massa de olefinas e 1. 000 ppm (m/v)
e. 2,5 · 10–3 e 2,6 · 10–3 de enxofre (S), enquanto que a gasoli-

101
Química Química geral e estequiometria

na “limpa”, como a californiana, possui porção de 10 % em massa de ródio, calcule a


22% em massa de aromáticos, 4% em massa e o número de átomos de platina.
massa de olefinas e 15 ppm (m/v) de en-
Massas atômicas: Rh = 103 e Pt = 195; cons-
xofre. Essas quantidades fazem com que
a concentração de ozônio em São Paulo tante de Avogadro = 6,0 · 1023 .
ultrapasse os limites permitidos pela le- 193. UEL-PR
gislação, causando vários problemas de
saúde na população, como, por exemplo, SAÚDE
prejudicando a respiração. Uma notícia divulgada pela Inter-
Folha de S. Paulo. Ciência. 31/8/2008. A26. Adaptado. net está deixando os consumidores de
aspartame preocupados. O adoçante pode
Comparada com a gasolina da Califórnia, nos Es-
tados Unidos, a gasolina de São Paulo tem maior causar danos à saúde. Alguns especialistas
quantidade de aromáticos. Considerando que apontam que doses superiores a 5 gramas di-
os aromáticos fossem apenas tolueno, C6H5CH3, árias para pessoa pesando 70 kg podem cau-
a quantidade de matéria em mol que diminuiria sar intoxicações e distúrbios neurológicos.
O Estado de S. Paulo, 31 ago. 1999
dessa substância, para cada 1,0 kg de combustí-
vel paulista que fosse trocado pelo californiano,
corresponderia a, aproximadamente: O OCH3
O C
Dados:
CH2 C CH
Massas molares (g/mol): HOOC CH N CH2
H=1 NH2 H
C = 12
O = 16 (Em água, cada mol de aspartame é cerca de
170 vezes mais adoçante do que 1 mol de sa-
a. 1,3 d. 4,8 carose, C12H22O11.)
b. 2,5 e. 6,0
Supondo que o poder adoçante seja propor-
c. 3,5 cional à quantidade ingerida, em mol, aproxi-
191. Unesp madamente quantos gramas de sacarose cor-
O ferro (massa atômica 56) é essencial à vida do respondem a 5 g de aspartame, para adoçar
homem porque está presente, na forma iônica, igualmente um alimento?
no glóbulo vermelho do sangue e transporta oxi- Massas molares:
gênio para os tecidos. No sangue de um adulto
há 2,9 g de ferro, que estão contidos em cerca H = 1,0; C = 12,0; N = 14,0; O = 16,0
de 2,6 · 1013 glóbulos vermelhos. O número de a. 1 · 104 g
átomos de ferro em cada glóbulo vermelho é: b. 1 · 103 g
PV-13-14

a. 6,0 · 1023 c. 1 · 102 g


b. 1,2 · 109
d. 1 · 101 g
c. 3,1 · 1022
e. 1 g
d. 0,051
e. 2,9 · 6,0 · 1023 194. Unicamp-SP

192. Unesp Em uma pessoa adulta com massa de 70,0 kg,


há 1,6 kg de cálcio. Qual seria a massa dessa
Ligas constituídas de platina e de ródio, com pessoa, em kg, se a natureza houvesse, ao lon-
diferentes composições, são utilizadas como go do processo evolutivo, escolhido o bário
sensores de temperatura. Para 1,00 g de uma em lugar do cálcio?
liga contendo apenas platina e ródio, na pro-
Dados: massas atômicas relativas:
Ca = 40, Ba = 137

102
Química geral e estequiometria Química

195. PUCCamp-SP
Na batalha contra o Mycobacterium tuberculosis, a principal bactéria causadora da tubercu-
lose, um grupo de pesquisadores brasileiros espera, em breve, fornecer um novo arsenal de
armas químicas: drogas sintéticas desenhadas para explorar os pontos fracos do patógeno. O
objetivo é matá-lo de forma mais rápida e menos tóxica ao homem.
Uma molécula com aparente potencial para atingir esse objetivo foi batizada informalmente de
IQG 607, um ferrocianeto associado à molécula da isoniazida (INH), droga que há 50 anos é o
carro-chefe no tratamento contra tuberculose.
A isoniazida corresponde a um derivado do ácido nicotínico, denominado também de isonicoti-
nil-hidrazina. Sua fórmula estrutural é:
N

C
O N NH2

Pesquisa FAPESP 97. Março de 2004. p. 32. Adaptado.

Sabendo-se que:
Elemento Hidrogênio Carbono Nitrogênio Oxigênio

Massa molar
1 12 14 16
g · mol–1

pode-se calcular a massa molar da isoniazida. Seu valor, em g · mol–1, é:


a. 137 c. 121 e. 107
b. 125 d. 120

196. Fuvest-SP
PV-13-14

O isocianato de metila, H 3 C – N = C = O, é um líquido volátil e tóxico. Tolera-se, no máximo,


5 . 10-5 g do seu vapor por metro cúbico de ar.
Qual é o número aproximado de moléculas de H3CNCO por m3 de ar na condição de tolerância
máxima?
Dados: • massa molar do H3CNCO = 57 g/mol
• constante de Avogadro = 6,0 . 1023
197. Fuvest-SP
Em uma amostra de 1,15 g de sódio, o número de átomos existentes será igual a:
Dados: Na = 23; constante de Avogadro = 6,0 · 1023
a. 6 · 1022 c. 6 · 1023 e. 1023
b. 3 · 1023 d. 3 · 1022

103
Química Química geral e estequiometria

198. UFTM-MG a. H2SO4 > NH3 > NaOH


A urina apresenta 95% de água e 5% de subs- b. H2SO4 > NaOH > NH3
tâncias orgânicas dissolvidas. Em um litro de c. NH3 > H2SO4 > NaOH
urina existem, aproximadamente, 2,5 · 1023 mo- d. NH3 > NaOH > H2SO4
léculas de ureia CO(NH2)2 e o restante corres-
ponde a sais, creatinina, ácido úrico e amônia. e. NaOH > NH3 > H2SO4
A massa aproximada de ureia, em gramas, 200. Unesp
existente em 1 L de urina é: Peixes machos de uma certa espécie são ca-
Dados: massa molar ureia = 60 g/mol; cons- pazes de detectar a massa de 3,66 · 10–8 g de
tante de Avogadro: 6 · 1023 2-fenil-etanol, substância produzida pelas fê-
meas, que está dissolvida em um milhão de li-
a. 205 d. 2,5 tros de água. Supondo-se diluição uniforme na
b. 60 e. 0,25 água, indique o número mínimo de moléculas
de 2-fenil-etanol por litro de água, detectado
c. 25
pelo peixe macho.
199. Fuvest-SP Dados: Massa molar do 2-fenil-etanol = 122 g/mol
Massas molares Constante de Avogadro = 6,0 · 1023
H2SO4 98 g/mol moléculas/mol
a. 3 · 10–16
NaOH 40 g/mol
b. 3,66 · 10–8
NH3 17 g/mol
c. 1,8 · 108
O Brasil produz, por ano, aproximadamente, d. 1,8 · 1022
5,0 · 106 t de ácido sulfúrico, 1,2 · 106 t de amô- e. 6,0 · 1023
nia e 1,0 · 106 t de soda cáustica. Transforman-
do-se toneladas em mols, a ordem decrescen-
te de produção dessas substâncias será:

PV-13-14

104
Química geral e estequiometria Química

Capítulo 04
201. tório, é comprovada uma importante lei dos
Transforme em atm as seguintes pressões: gases, atribuída a:
a. 19 cmHg a. Charles.
b. 1.140 torr b. Gay-Lussac.
c. 304 mmHg c. Avogadro.
d. 380 cmHg d. Boyle.
e. 60,8 torr 206. PUC-RS
f. 152 mmHg Observe a análise das informações a seguir,
202. sobre o ar inspirado e o ar expirado por uma
Transforme em litros os volumes: pessoa em repouso.
a. 0,15 m3 Composição do ar inspirado e expirado
(aproximada)
b. 280 cm3
c. 40 dm3 Ar inspirado
Ar expirado
d. 25 mL (8 L/min)
e. 5 · 10–3 m3 Oxigênio 21,0% 16,4%
f. 2 · 103 cm3
Dióxido de
203. 0,03% 4,0%
carbono
Transforme em Kelvin (K) as temperaturas:
Variável
a. 273 °C Vapor de
(raramente Saturado
b. 0 °C água
saturado
c. 727 °C
Aproximadamente
d. – 23 °C Temperatura Variável
a do corpo
e. – 273 °C
Variáveis,
f. –100 °C Partícula de mas Poucas ou
204. UEG-GO poeira geralmente nenhuma
presentes
Um gás ideal:
a. pode sofrer transição de fase do estado Disponível em: <Fonte: http://library.thinkquest.
líquido para o estado gasoso. org/19347/frameless_respiratorysys.htm>.
b. pode sofrer transição de fase do estado Pela análise das informações, conclui-se que:
gasoso para o líquido.
PV-13-14

a. o processo respiratório humano é um


c. pode sofrer qualquer tipo de transição
dos principais fatores que contribuem
de fase.
para o efeito estufa e para o aqueci-
d. não pode sofrer nenhuma transição de mento global.
fase.
b. uma pessoa normal inspira cerca de
205. UECE 22L de oxigênio por hora.
Quando da nossa respiração, na etapa da ina- c. aproximadamente 1/5 do oxigênio pre-
lação, o diafragma se expande deixando maior sente no ar inspirado participa, princi-
o volume do pulmão. Neste caso, a pressão palmente, do processo de combustão
interna do pulmão fica menor que a pressão no interior das células.
atmosférica, permitindo a entrada de ar no
pulmão até uniformizar as pressões. Quando d. o ar expirado pelo corpo humano é seco.
a pressão aumenta, o volume do pulmão di- e. o nitrogênio é totalmente absorvido pelo
minui e o ar é expelido. No processo respira- corpo humano no processo respiratório.

105
Química Química geral e estequiometria

207. UFRN
O material orgânico do lixo gera grande quantidade de metano (CH4), muito mais danoso à ca-
mada de ozônio do que o CO2. A solução ideal, que já começa a ser adotada em algumas cidades
do Brasil, é o aproveitamento do CH4 na geração de energia. A compressão do metano tem papel
importante, desde a produção até o consumo, para desenvolver as atividades de transporte, ar-
mazenagem ou alimentação de equipamentos.
Em um determinado processo de compressão, os valores de pressão e temperatura usados, para
uma massa fixa de CH4, são mostrados nas figuras (I), (II) e (III) abaixo:
1 atm 2 atm 4 atm

25°C 25°C 25°C

V1 V2 V3
(I) (II) (III)

a. Sabendo que o volume inicial do gás é V1 = 12L e considerando o seu comportamento


ideal, calcule os valores de V2 e V3 e construa na figura abaixo uma curva de pressão versus
volume.
Pressão (atm)
5
4
3
2
1

0 1 2 3 4 5 6 7 8 9 10 11 12 Volume (L)

b. Considerando o volume do gás na figura (I) e o comportamento ideal do CH4 (PV = nRT),
calcule o número de mols de CH4 contidos nesse recipiente.
208. UEMS
Na cozinha, as panelas de pressão são bastante utilizadas por facilitarem o processo de cozimento
PV-13-14

dos alimentos. É correto afirmar que o cozimento em uma panela de pressão é mais rápido porque:
a. o material da panela de pressão tem alta condutividade térmica, o que ocasiona uma
transferência de calor mais eficiente.
b. na panela de pressão, a água entra em ebulição em uma temperatura menor do que em
uma panela aberta.
c. na panela de pressão, a pressão interna pode atingir valores maiores que a pressão at-
mosférica.
d. como a panela de pressão é um sistema adiabático perfeito, ocorre menos perda de calor
para o ambiente.
e. a válvula da panela de pressão impede que haja perda de calor para o ambiente, permitin-
do a obtenção de temperaturas maiores no seu interior.

106
Química geral e estequiometria Química

209. UEG-GO 210. FMJ-SP


A maioria dos gases são compostos mole- A figura apresentada a seguir descreve o pro-
culares, com exceção dos gases nobres, que cesso da respiração.
são formados por átomos isolados. Sobre
esse assunto, julgue a validade das afirma-
tivas a seguir.
Pressão (atm)

Volume Disponível em: <http: //www.qmc.ufsc.br>.

Na respiração, a lei de Boyle pode ser obser-


I. O gás hélio utilizado para encher ba- vada. À temperatura constante, na etapa da:
lões apresenta energia de ionização
menor que o gás hidrogênio utilizado a. inalação, o diafragma se expande dei-
como combustível para foguete. xando o volume do pulmão maior.
Como o produto PV deve ser constante,
II. O gráfico acima mostra uma curva a pressão interna do pulmão diminui.
isoterma, pois as variações de pres-
b. inalação, o diafragma se expande dei-
são e volume ocorrem na mesma
xando o volume do pulmão menor.
temperatura.
Como o produto PV deve ser constante,
III. Determinada massa de um gás ocu- a pressão interna do pulmão aumenta.
pa um volume de 30 litros a 27 °C e
c. inalação, o diafragma retrai deixando o
exerce uma pressão de 4,0 atm. Caso
volume do pulmão menor. Como o pro-
o volume se altere isotermicamente
duto PV deve ser constante, a pressão
para 62 litros, a nova pressão estará
interna do pulmão aumenta.
entre 1,5 atm e 2,5 atm.
d. exalação, o diafragma se expande dei-
IV. É comum liquefazerem-se os gases,
xando o volume do pulmão maior.
pois eles ocupam grandes volumes.
Como o produto PV deve ser constante,
Exemplo disto é o que acontece com
a pressão interna do pulmão diminui.
PV-13-14

o botijão de gás, daí o nome GLP (gás


liquefeito de petróleo). e. exalação, o diafragma se retrai deixan-
do o volume do pulmão maior. Como o
Assinale a alternativa correta: produto PV deve ser constante, a pres-
a. Apenas a afirmativa I é verdadeira. são interna do pulmão aumenta.
b. Apenas as afirmativas I, II e III são ver- 211. UFRN
dadeiras.
Reduzindo-se à metade a pressão exercida so-
c. Apenas as afirmativas II e IV são verda- bre 150 cm3 de ar, o novo volume, à tempera-
deiras.
tura constante, será, em cm3:
d. Apenas as afirmativas II, III e IV são ver-
dadeiras. a. 75 d. 750
b. 150 e. 1.500
c. 300

107
Química Química geral e estequiometria

212. 217. UEBA


Uma amostra gasosa se encontra em um reci- Um balão-propaganda cheio de gás hélio, ao
piente de 9 L a 25 °C e 1,6 atm. Se todo esse gás nível do mar, ocupa um volume de 250 L. Seu
for transferido para outro recipiente de 12 L, qual volume, após lançamento, numa altitude de
será a nova pressão, também a 25 °C? 3.000 m será (obs.: admita que a temperatura
tenha se mantido constante):
a. 0,9 atm d. 2,4 atm
a. menor, pois a pressão externa aumenta
b. 1,2 atm e. 3,2 atm com a altitude.
c. 2,1 atm b. maior, pois a pressão externa diminui
213. FAAP-SP com a altitude.
Um recipiente, munido de êmbolo móvel, con- c. permanecerá constante, pois a pressão
tém 10 L de O2 à pressão de 1 atm. Mantendo- não varia com a altitude.
-se constante a temperatura por movimenta- d. permanecerá constante, pois a tempe-
ção do êmbolo, pressiona-se o gás até que seu ratura se manteve constante.
volume seja reduzido para 2 L. Pede-se:
e. maior, pois a pressão externa aumenta
a. a lei que rege a transformação indicada. com a altitude.
b. a pressão de O2 contido no recipiente,
218.
ao final.
Se, numa transformação isotérmica, uma mas-
214. PUC-RS
sa tem seu volume aumentado de 2/3, o que
De acordo com a Lei de Robert Boyle (1660), acontecerá com a pressão?
para proporcionar um aumento na pressão de
219. Fuvest-SP
uma determinada amostra gasosa numa trans-
formação isotérmica, é necessário: Se um certo gás, contido em um comparti-
a. aumentar o seu volume. mento e exercendo pressão de 10 cmHg, for
comprimido de maneira a ocupar um vigésimo
b. diminuir a sua massa. do seu volume inicial, qual será a pressão fi-
c. aumentar a sua temperatura. nal? (temperatura constante)
d. diminuir o seu volume. a. 20 cmHg
e. aumentar a sua massa. b. 50 cmHg
215. c. 100 cmHg
Uma certa massa de gás oxigênio (O2) ocupa um d. 200 cmHg
volume de 5 mL a uma pressão de 2 atm. Qual e. 400 cmHg
deverá ser o novo volume dessa massa gasosa 220. Unirio-RJ
se ela sofrer uma transformação isotérmica até
que a pressão passe a valer 760 mmHg? Você brincou de encher, com ar, um balão de
PV-13-14

gás, na beira da praia, até um volume de 1L e


a. 1 mL d. 10 mL o fechou. Em seguida, subiu uma encosta pró-
b. 2 mL e. 50 mL xima carregando o balão, até uma altitude de
c. 7,5 mL 900 m, onde a pressão atmosférica é 10% me-
nor do que a pressão ao nível do mar.
216. Considerando que a temperatura na praia e na
À temperatura T e pressão de 1 atm, um gás encosta seja a mesma, o volume de ar no ba-
ocupa o volume V. Reduzindo-se a pressão lão, em L, após a subida, será de:
para 0,5 atm e mantendo-se a temperatura a. 0,8
constante, o volume ocupado pelo gás será: b. 0,9
a. 1/2 d. 2V c. 1,0
b. 2 e. 4V d. 1,1
c. V/2 e. 1,2

108
Química geral e estequiometria Química

221. a. 21,1 atm


Uma amostra de gás encontra-se num recipiente b. 4,4 atm
fechado e indeformável, a –73 °C e 600 mmHg. c. 0,9 atm
Se a temperatura for elevada até 77 °C, qual será d. 760 mmHg
a nova pressão?
e. 2,2 atm
222. UnB-DF
225.
Um balão contendo gás oxigênio (O2), mantido
Numa transformação isobárica sofrida por
à pressão constante, tem volume igual a 10 L,
a 27 °C. Se o volume for dobrado, poderemos uma amostra de gás, o volume é diretamente
afirmar que: proporcional:
a. a temperatura em °C dobrará. a. à pressão, em qualquer unidade.
b. a temperatura em K dobrará. b. à pressão, em atmosferas.
c. a temperatura em K diminuirá à metade. c. à temperatura, em qualquer unidade.
d. a temperatura em °C diminuirá à metade. d. à temperatura, na escala Celsius.
e. a temperatura em K aumentará de 273 K. e. à temperatura, na escala Kelvin.
223. Unesp 226.
Segundo a lei de Charles Gay-Lussac, manten- Uma massa de nitrogênio gasoso encontra-se
do-se a pressão constante, o volume ocupa- a 27 °C e 1,0 atm. Se essa amostra sofrer uma
do por um gás aumenta proporcionalmente transformação isocórica até chegar a 177 °C,
ao aumento da temperatura. Considerando qual será sua pressão final?
a teoria cinética dos gases e tomando como 227. FIAM-SP
exemplo o gás hidrogênio (H2), é correto afir-
Uma amostra de nitrogênio gasoso ocupa
mar que este comportamento está relaciona-
do ao aumento: um volume de 20 mL, a 27 °C e à pressão de
800 mmHg. Que volume ocuparia a amostra
a. do tamanho médio de cada átomo de a 0 °C e 800 mm de Hg?
hidrogênio (H), devido à expansão de
suas camadas eletrônicas. a. 20,2 mL
b. do tamanho médio das moléculas de b. 19,5 mL
hidrogênio (H2), pois aumentam as dis- c. 18,2 mL
tâncias de ligação. d. 12,5 mL
c. do tamanho médio das moléculas de e. 10,2 mL
hidrogênio (H2), pois aumentam as in-
terações entre elas. 228. UFC-CE
PV-13-14

d. do número médio de partículas, devido O gráfico abaixo representa um processo cícli-


à quebra das ligações entre os átomos co (ciclo) a que é submetido um gás ideal.
de hidrogênio (H2 → 2 H).
V (L)
e. das distâncias médias entre as molécu-
las de hidrogênio (H2) e das suas veloci- 3
dades médias.
224. PUC-RJ
1 2
Um pneu de bicicleta é calibrado a uma pres-
são de 4 atm em um dia frio, à temperatura de T (K)
7 °C. Supondo que o volume e a quantidade de
gás injetada são os mesmos, qual será a pressão Analise-o. A opção em que aparece a correspon-
de calibração nos dias em que a temperatura dência das etapas numeradas (1 → 2, 2 → 3 e
atinge 37 °C? 3 → 1), com suas respectivas denominações, é:

109
Química Química geral e estequiometria

a. Isobárica, adiabática e isotérmica. β


b. Isovolumétrica, isobárica e isotérmica. α1
2
c. Isovolumétrica, isotérmica e isobárica. β2
d. Isotérmica, isobárica e isovolumétrica. 1
β1
e. Isovolumétrica, isobárica e adiabática.
229. UFF-RJ
Num recipiente com 12,5 mL de capacidade, δ1 δ2 δ
está contida certa amostra gasosa cuja massa Gráfico II
exercia uma pressão de 685,0 mmHg, à tem-
peratura de 22 °C. Dentre as alternativas abaixo, assinale aquela
que se ajusta aos gráficos acima.
Quando esse recipiente foi transportado com
as mãos, sua temperatura elevou-se para 37 °C a. α é o volume, β é a temperatura, δ é a
e a pressão exercida pela massa gasosa passou pressão e o processo é uma expansão a
a ser, aproximadamente: temperatura constante.
b. δ é a temperatura, β é a pressão, α é o
a. 0,24 atm d. 1,50 atm
volume e o processo é uma compressão.
b. 0,48 atm e. 2,00 atm
c. α é o volume, β é a pressão, δ é a tem-
c. 0,95 atm peratura e o processo é um resfriamen-
to isobárico.
230. Fuvest-SP
d. α é o volume, β é a temperatura, δ é a
Uma amostra de gás foi aquecida sob pressão pressão e o processo é uma compres-
constante. Nessa transformação, ocorreu: são isotérmica.
a. diminuição do volume do gás e da ener- e. α é a pressão, β é o volume, δ é a tem-
gia cinética média das moléculas. peratura e o processo é um aquecimen-
b. aumento do volume do gás e da ener- to isobárico.
gia cinética média das moléculas. 232. Unifor-CE
c. aumento do volume do gás e diminuição
da energia cinética média das moléculas. A respiração é um processo no qual ocorre a
troca de gases no corpo humano. Na inspira-
d. diminuição do volume do gás, porém a ção, o diafragma é contraído, ocasionando um
energia cinética média das moléculas aumento de volume da cavidade torácica. Esta
manteve-se constante. mudança em volume faz com que a pressão in-
e. aumento de volume do gás, porém a terna diminua em relação à pressão atmosféri-
energia cinética média das moléculas ca (pressão externa). Como a pressão externa
manteve-se constante. está maior que a pressão interna (cavidade
PV-13-14

231. IME-RJ torácica), o ar é transportado aos pulmões. Na


Um gás ideal sofre uma mudança de estado expiração, ocorre o processo inverso.
ilustrada pelos gráficos I e II a seguir. De acordo com o texto, podemos afirmar que:
a. a relação envolvida representa a lei de
α Charles que afirma que o volume de
uma quantidade fixa de gás mantido
1 2 α1 em uma temperatura constante varia
δ2 inversamente com a pressão aplicada.
b. a relação envolvida representa a lei
δ1 de Boyle que afirma que o volume de
uma quantidade fixa de gás mantido
β1 β2 β
em uma temperatura constante varia
Gráfico I inversamente com a pressão aplicada.

110
Química geral e estequiometria Química

c. a relação envolvida representa a lei de 235.


Charles que afirma que o volume de Certa massa de gás ocupa um volume de 1m3
uma quantidade fixa de gás mantido
a 323 °C, exercendo uma pressão de 1 atm no
em uma temperatura constante varia
recipiente que a contém. Reduzindo-se a tem-
diretamente com a pressão aplicada.
peratura para 25 °C e o volume ocupado
d. a relação envolvida representa a lei pelo gás para 25 litros, qual será a pressão
de Boyle que afirma que o volume de
no sistema, em atm?
uma quantidade fixa de gás mantido
em uma temperatura constante varia 236. Unioeste-PR
diretamente com a pressão aplicada. A região Oeste do Paraná possui um grande
e. a relação envolvida representa a lei rebanho de suínos, cujos dejetos são os princi-
de Gay-Lussac que afirma que o vo- pais poluidores das águas. Uma forma de mi-
lume de uma quantidade fixa de gás nimizar o impacto ambiental desta atividade
mantido em uma temperatura cons- é a utilização de biodigestores anaeróbicos. O
tante varia diretamente com a pres- biogás gerado, constituído principalmente por
são aplicada.
metano e dióxido de carbono, muitas vezes é
233. IME-RJ armazenado em balões de borracha. Um balão
Um sistema fechado e sem fronteiras móveis de 100.000 L, completamente cheio e subme-
contém uma determinada massa gasosa tido à temperatura de 27 °C, foi deixado ao sol,
inerte. Sabe-se que, após aquecimento, o sofrendo um aquecimento até a temperatura
sistema registra um aumento de 5% na pres- de 77 °C.
são e de 15 °C na temperatura (considere A pressão constante, o volume final aproxima-
que o gás se comporta idealmente). A res- do do balão e a porcentagem aproximada do
peito do valor da temperatura inicial, pode- aumento de volume são:
-se dizer que:
a. 116.600L e 16,6%.
a. é igual ou inferior a 30 °C.
b. 116.600L e 116,6%.
b. é superior a 30 °C e inferior a 300 °C.
c. é igual ou superior a 300 °C. c. 150.600L e 50,6%.
d. somente pode ser calculado conhecen- d. 150.600L e 150,6%.
do-se o volume e a massa de gás. e. 100.600L e 10%.
e. somente pode ser calculado conhecen- 237. UFJF-MG
do-se o volume, a massa e a pressão
inicial do gás. A calibração dos pneus de um automóvel deve
ser feita periodicamente. Sabe-se que o pneu
234. UESPI deve ser calibrado a uma pressão de 30 lb/pol2
PV-13-14

Um balão contendo gás metano, com volume em um dia quente, a uma temperatura de 27 °C.
igual a 4,9 L, foi retirado de um sistema de Supondo que o volume e o número de mol inje-
aquecimento, cuja temperatura era de 86 °C, tados sejam os mesmos, qual será a pressão de
e resfriado até a temperatura de 5 °C. Após calibração (em atm) nos dias mais frios, em que
este procedimento, realizado a uma pressão a temperatura atinge 12 °C?
constante, o volume do referido balão pas-
sou a ser de: Dado: considere 1 atm ≅ 15 lb/pol2
a. 0,28 L a. 1,90 atm
b. 1,22 L b. 2,11 atm
c. 2,45 L c. 4,50 atm
d. 3,79 L d. 0,89 atm
e. 4,90 L e. 14,3 atm

111
Química Química geral e estequiometria

238. UFOP-MG 01. Na situação II, seria possível calcular


Considere o seguinte ciclo, envolvendo um a massa M do corpo, se soubéssemos
mol de gás ideal, inicialmente a 273 °C. também a pressão interna na bexiga e a
pressão atmosférica (ambiente).
1 Etapa 1 2
1,0 02. Ao aumentar-se a temperatura do sis-
Pressão (atm)

Isoterma tema na situação I para 51 °C, a bexiga


reversível irá explodir.
04. Ao colocar-se o corpo de massa M so-
bre a bexiga, mantendo-se o sistema a
0,5 3 25 °C, sua pressão interior deverá au-
mentar em virtude do aumento do vo-
lume do gás.
22,4 Volume (L) 44,8 08. Na situação II, a pressão exercida pelo
sistema corpo+bexiga sobre o piso é
Ao final da etapa I, a temperatura será igual a: dependente da pressão atmosférica no
a. 273 K local do experimento.
b. 546 K 240. UESC-BA
c. 1.092 K A pressão exercida pelo propano, C3H8(g) um
d. 273 °C propelente, no interior de uma embalagem de
239. UEM-PR 200,0mL de tinta spray, é 1,5 atm, a 27 °C.
Para as situações (I) e (II) expressas abaixo, à Admitindo-se que o propano se comporta
mesma altitude, e o dado fornecido a seguir, como gás ideal, ocupa 50% do volume da
considerando uma bexiga de borracha de- embalagem e que a quantidade de vapor
formável e de massa desprezível, hermetica- produzido por qualquer outra substância,
mente fechada, contendo 2,0 g de gás hélio no interior da embalagem, é desprezível, é
(supondo que seja um gás ideal), inicialmente correto afirmar:
a 25 °C, que pode explodir quando atingido o
a. A pressão no interior da embalagem é
dobro de sua capacidade volumétrica inicial,
assinale o que for correto. igual a 2,0 atm quando a temperatura
aumenta para 127 °C.
Dado: constante dos gases ideais = 0,082 atm · L/
mol · K b. A efusão do propano causa o aqueci-
mento da válvula que controla a saída
Situações: de tinta do spray.
I. A bexiga permanece em repouso so- c. A tinta que se espalha no ar, durante a
PV-13-14

bre um piso plano e horizontal, cuja pintura de um objeto, forma uma solu-
área de contato entre a bexiga e o
ção gasosa.
piso é 1,0 cm2 e a pressão no interior
da bexiga é de 2,0 atm. d. O número de moléculas de propano no
II. Com a situação descrita em (I), é colo- interior da embalagem é igual a 3,0 · 1021.
cado sobre a bexiga um corpo de mas- e. A massa de propelente existente no in-
sa M. A área de contato entre a bexiga terior da embalagem é 0,2 g.
e o piso se torna igual a 10 cm2 e é exa- 241. UFSC
tamente igual à área de contato entre
o corpo e a bexiga. Considere que a Suponha que 57 litros de um gás ideal a 27 °C e
face do corpo de massa M que toca a 1,00 atmosfera sejam simultaneamente aqueci-
bexiga é plana e possui área sempre dos e comprimidos até que a temperatura seja
maior do que a área de contato entre 127 °C e a pressão, 2,00 atmosferas. Qual o vo-
o corpo e a bexiga. lume final, em litros?

112
Química geral e estequiometria Química

242. 248. PUC-RJ modificado


Certa massa de Ne(g) ocupa o volume de Uma amostra de gás, a 327 °C e 120 atm de
500 cm3 nas CNTP. A que pressão, em atm, pressão, ocupa um recipiente de 10 L. Qual a
essa massa de Ne(g) ocupará um volume de variação de temperatura que se deve efetu-
1,00 m3 à temperatura de 1.727 °C? ar para que a pressão seja de 20 atm quando
243. Unesa-RJ se transferir este gás para um recipiente de
40 L?
Um volume de 10 L de um gás perfeito teve
sua pressão aumentada de 1 para 2 atm e 249. E.E.Mauá-SP
sua temperatura aumentada de –73 °C para Uma determinada massa gasosa, confinada em
+ 127 °C. O volume final, em litros, alcança- um recipiente de volume igual a 6,0 L, está
do pelo gás foi de: submetida a uma pressão de 2,5 atm e tem-
peratura de 27 °C. Quando a pressão é elevada
a. 50 d. 20 em 0,5 atm, nota-se uma contração no volume
b. 40 e. 10 de 1,0 L.
c. 30 a. Qual a temperatura em que o gás se
244. Unicamp-SP encontra?
A partir de dados enviados de Vênus por son- b. Que tipo de transformação ocorreu?
das espaciais norte-americanas e soviéticas,
pode-se considerar que, em certos pontos da 250. FASP
superfície desse planeta, a temperatura é de Uma amostra de um gás ideal ocupa um volu-
327 °C e a pressão atmosférica é de 100 atm. me de 60 mL, a 0,8 atm e –73 °C. Que volume
Sabendo-se que, na superfície, da Terra o vo- ocupará esse mesmo gás a 0,4 atm e 127 °C?
lume molar de um gás ideal é 24,6 L a 27 °C e
251. UFU-MG
1,00 atm, qual seria o valor desse volume nes-
ses pontos de Vênus? A atmosfera é composta por uma camada de
245. UnB-DF gases que se situam sobre a superfície da Terra.
Imediatamente acima do solo, ocorre uma região
Um mol de gás hélio ocupa um volume de 22,4 L da atmosfera conhecida como troposfera, na qual
na praia de Ipanema, no Rio de Janeiro, a 0 °C. Em ocorrem as nuvens, os ventos e a chuva. Ela tem
Brasília, pressão atmosférica a 700 mm de Hg, o uma altura aproximada de 10 km, a temperatura
volume ocupado a 27 °C será: no seu topo é cerca de –50 °C e sua pressão é de
a. 24,3 L 0,25 atm. Se um balão resistente a altas pressões,
b. 24,6 L cheio com gás hélio até um volume de 10,0 L, a
c. 26,7 L 1,00 atm e 27,0 °C, é solto, o seu volume, quando
chegar ao topo da troposfera, será de:
d. 27,6 L
e. mesmo valor que no Rio de Janeiro. Dado: 0 Kelvin = –273 °C
PV-13-14

a. 40,0 L
246. UFPE
b. 74,1 L
Uma certa quantidade de gás ideal ocupa 30 li-
c. 36,3 L
tros à pressão de 2 atm e à temperatura de 300 K.
Que volume passará a ocupar se a temperatura e d. 29,7 L
a pressão tiverem seus valores dobrados? e. 52,5 L
247. FRB-BA 252.
Um balão meteorológico com 50 L de gás hé- Certa quantidade de H2(g) ocupa um volume V
lio, a 20 °C e ao nível do mar, é lançado na at- a uma dada pressão e temperatura. Qual é o
mosfera. Ao atingir a estratosfera, a pressão volume ocupado pela mesma quantidade de
desse gás torna-se 0,4 atm e a temperatura, H2(g), em função de V, quando a pressão dimi-
–50 °C. Determine, em L, a capacidade que o nuir em 3/8 da inicial e a temperatura absoluta
balão deve ter antes do lançamento. se tornar igual a 5/8 da inicial?

113
Química Química geral e estequiometria

253. 257.
Reduza às condições normais de pressão e Temos 40 L de uma certa massa de gás sub-
temperatura 38 L de cloro que foram medidos metidos a 1,5 atm e 27 °C. Qual deve ser a
a 127 °C e à pressão 720 mm de mercúrio, e temperatura dessa massa de gás para que o
calcule o novo volume ocupado pelo gás. seu volume se reduza à metade e a pressão
passe para 4 atm?
254. FUR-RN
258. UFG-GO
No alto de uma montanha, o termômetro mar-
ca 15 °C e o barômetro, 600 mmHg. Ao pé da O motor de Stirling é um sistema que regenera
montanha, a temperatura é de 25 °C, e a pres- o ar quente em um ciclo fechado. As transfor-
são é 760 mmHg. A relação entre os volumes mações que ocorrem nesse motor podem ser
ocupados pela mesma massa de gás no alto da representadas, idealmente, pelas seguintes
montanha e no pé da montanha é: etapas:
1. O gás é aquecido a volume constante;
a. 2,1 d. 21
2. O gás se expande a uma temperatura
b. 1,5 e. 1,2 constante;
c. 12 3. O gás é resfriado a volume constante;
4. O gás se contrai a uma temperatura
255. constante.
O que acontece com a pressão de uma deter- Faça o diagrama pressão × volume para essas
minada massa de gás quando o seu volume etapas do motor de Stirling.
aumenta de 1/4 e a sua temperatura absoluta
se reduz de 1/4? 259. UFCG-PB
256. UFV-MG O volume de uma garrafa de ar usada para o
mergulho submarino é igual a 15 L. A pressão
Considere uma amostra de gás contida num do ar que ela contém é igual a 200 bars. O vo-
cilindro com pistão, nas condições normais de lume dos pulmões é suposto invariável. Consi-
temperatura e pressão (0 °C ou 273 K e 1 atm), dera-se que, durante o mergulho, um homem
conforme figura a seguir. Suponha que a pres- inala 1 L de ar a cada inspiração, à razão de 15
são sobre o gás seja dobrada (2 atm) e que a inspirações por minuto.
temperatura seja aumentada para 273 °C. Se o
gás se comporta como gás ideal, nessas novas A pressão do ar nos pulmões é de 2 bars numa
condições, a figura que melhor representa a profundidade de 10 m e de 4 bars numa pro-
amostra gasosa no cilindro com pistão é: fundidade de 30 m.
A garrafa é equipada com uma válvula de pres-
são, que permite abaixar a pressão do ar no in-
terior da garrafa até a pressão dos pulmões. O ar
verifica a lei de Boyle-Mariotte nestas condições.
PV-13-14

Analise essa situação e assinale a afirmativa


a. correta.
d.
a. A autonomia na profundidade de 10 m
é superior a 1,5 hora.
b. A autonomia na profundidade de 30 m
b. é inferior a 30 minutos.
e. c. A autonomia na profundidade de 10 m
é exatamente 3 vezes a autonomia na
profundidade de 30 m.
d. A autonomia na profundidade de 10 m
c. é exatamente a metade da autonomia
na profundidade de 30 m.
e. A autonomia é diretamente proporcio-
nal à profundidade do mergulho.

114
Química geral e estequiometria Química

260. Unesp-SP 263.


Alterações na composição química da atmos- Qual é o volume, em m3, ocupado por 8,0 kg
fera são fortes indícios de problemas am- de O2(g) nas CNTP?
bientais, tais como o efeito estufa. Frequen-
temente, pesquisadores lançam balões que Dado: massa molar do O2 = 32 g · mol–1
enviam informações de grandes altitudes. 264. PUC-SP
Suponha que um desses balões, com volume Têm-se dois balões, A e B, de mesmo volume.
de 10 L de H2, tenha sido lançado ao nível do
O balão A contém cloro (Cl2) e o balão B, ozone
mar (P = 760 mmHg e T = 27 °C). Enquanto
o balão sobe, a redução da pressão atmosfé- (O3), à mesma temperatura e pressão. Pode-se
rica irá favorecer o aumento de seu volume. afirmar que o que há de comum entre os dois
Porém, a temperatura também é reduzida na balões é:
medida em que o balão sobe, o que favorece a. a mesma massa.
a diminuição de seu volume. b. a mesma densidade.
Para saber se o balão irá continuar subindo, cal- c. o mesmo número de moléculas.
cule seu volume quando esse atingir a altitude d. o mesmo número de átomos.
de 7.000 m, onde T = –33 °C e P = 300 mmHg.
e. a mesma coloração.
261. PUCCamp-SP
265.
Um recipiente de 100 litros contém nitrogênio
à pressão normal e temperatura de 30 °C. A Qual é o número de moléculas contidas em 56 mL
massa do gás, em gramas, é igual a: de metano, CH4(g), nas CNTP?
Dado: Volume molar dos gases a 1,0 atm e 266. UCSal-BA
30 °C = 25,0 L/mol Que volume ocupam 100 mol de oxigênio nas
a. 112 condições ambientes de temperatura e pressão?
b. 56,0 Volume molar de gás nas condições ambientes
c. 42,0 de temperatura e pressão = 25 L/mol
d. 28,0 a. 0,25 L
e. 14,0 b. 2,5 L
c. 2,5 · 102 L
262. UFES d. 2,5 · 103 L
Três balões contêm H2, N2 e O2, conforme ilus- e. 2,5 · 104 L
trado a seguir. 267.
Dois cilindros metálicos iguais contêm gases
H2 N2 O2 comprimidos em grau de elevada pureza, sen-
do que um deles contém 8 m3 de gás nitrogê-
PV-13-14

nio, e o outro, 8 m3 de gás hidrogênio. Conside-


rando que os dois cilindros estão armazenados
V = 10 mL V = 10 mL V = 10 mL nas mesmas condições ambientais, podemos
afirmar que:
Considerando-se que os gases estão sob pres-
Dados: massas atômicas: H = 1,0 e N = 14,0
são de 1 atm e à mesma temperatura, assinale
a alternativa com o número possível de molé- a. a massa de gás armazenado é a mesma.
culas de H2, N2 e O2 contidas nos balões. b. a pressão do cilindro contendo nitrogê-
nio é maior.
a. 1 · 1023, 7 · 1023 e 8 · 1023
c. o número de moléculas é o mesmo.
b. 1 · 1023, 14 · 1023 e 16 · 1023
d. a velocidade média das moléculas dos
c. 2 · 1023, 2 · 1023 e 2 · 1023 dois gases é igual.
d. 2 · 1023, 28 · 1023 e 32 · 1023 e. a temperatura interna dos cilindros é
e. 2 · 1023, 32 · 1023 e 32 · 1023 menor que a temperatura ambiente.

115
Química Química geral e estequiometria

268. Unisa-SP 272. FEI-SP


O número de moléculas existentes em 110 li- Uma residência consumiu no ano 2000, entre
tros de gás carbônico, nas CNTP, é igual a: os meses de janeiro e março, 1,6 kg de gás na-
a. 6 · 1023 tural. O volume consumido, em metros cúbi-
cos (m3), medido nas CNTP, considerado o gás
b. 2,94 · 1024
natural como metano (CH4) puro, é:
c. 8,8 · 1024
(H = 1, C = 12, volume molar nas CNTP = 22,4 L/mol)
d. 1,47 · 1024
e. 8,82 · 1023 a. 2,24 d. 4,48
269. UFTM-MG b. 22,4  e. 2,48
Os recipientes I, II, III e IV contêm substâncias c. 44,8
gasosas nas mesmas condições de temperatu-
273.
ra e pressão.
O gás de cozinha, também chamado de gás li-
quefeito de petróleo (GLP), é formado por 50%
de propano e 50% de butano. Sabendo-se que
CO CO2 C2H4 H2 uma família domiciliada em São Paulo gastou
2,46 L de GLP no mês de janeiro, o número de
50 L 50 L 25 L 25 L
mols de gás butano queimado nesse mês foi:
Dados: volume molar do butano a 27°C e 1 atm
I II III IV
= 24,6 litros; Constante de Avogadro = 6,0 · 1023
O princípio de Avogadro permite-nos afirmar a. 0,5 d. 0,025
que o número: b. 0,05 e. 0,0025
a. de átomos de oxigênio é maior em I. c. 0,005
b. de átomos de hidrogênio é igual em III
e IV. 274. PUC-MG
c. de átomos de carbono é maior em I. O número de átomos existentes em 44,8 litros
de nitrogênio (N2), nas CNTP, é igual a:
d. total de átomos é igual em III e IV.
a. 1,2 · 1022
e. total de átomos é igual em II e III.
b. 1,2 · 1023
270. c. 2,4 · 1024
Qual é o volume ocupado por 17,6 g de CO2(g) d. 6,0 · 1023
nas condições ambientes (1 atm e 25 °C)? e. 6,0 · 1024
Dados: massa molar do CO2 = 44 g · mol–1, vo- 275. PUCCamp-SP
PV-13-14

lume molar a 25 °C e 1 atm = 25 L · mol–1


O quociente entre os números de átomos exis-
271. Mackenzie-SP tentes em volumes iguais de oxigênio (O2) e
As 355 g de um certo gás Xn ocupam 112,0 L propano (C3H8), medidos nas mesmas condi-
medidos nas CNTP. Se a massa atômica de X ções de pressão e temperatura, é:
é 35,5 u, então o gás tem fórmula molecular:
Dado: Volume molar nas CNTP = 22,4 L/mol 2 1
a. d.
8 8
a. X10
b. X8 2 2
b. e.
c. X4 5 11
d. X3 1
c.
e. X2 11

116
Química geral e estequiometria Química

276. E. E. Mauá-SP Assinale:


Massas iguais dos gases sulfidreto (H2S) e Dados: massas atômicas: O = 16, S = 32;
fosfina, nas mesmas condições de pressão e constante de Avogadro: 6,02 · 1023
temperatura, ocupam o mesmo volume. Qual
a massa molecular da fosfina, sabendo que as a. São todas corretas.
massas atômicas do hidrogênio e do enxofre b. Somente a I é correta.
são iguais a 1 e 32, respectivamente? c. Somente a II é correta.
277. Acafe-SC d. Somente a III é correta.
Nas condições normais de temperatura e pres- e. Somente I e III são corretas.
são (CNTP), o volume ocupado por 48 g de me- 281. Unisa-SP
tano (CH4) é:
O gás obtido pela completa decomposição
Massas atômicas: C = 12, H = 1 térmica de uma amostra de carbonato de cál-
a. 224 L d. 67,2 L cio é recolhido em um recipiente de 600 mL
b. 22,4 L e. 7,96 L a 37 °C.
c. 72 L Considerando que a pressão dentro do re-
cipiente é de 5 bar, pode-se afirmar que a
278. massa da amostra utilizada é, aproximada-
mente, em gramas:
Supondo um comportamento de gás ideal, as-
sinale a opção que indica, aproximadamente, Dados: 1atm = 1,013 bar
a massa em gramas de 10 L de CO2 nas CNTP: Constante Universal dos Gases (R) = 0,082 atm
Dados: massas molares em g/mol: C = 12; O = 16 · L · mol–1 · K–1
a. 0,2
a. 22,4 g d. 20 g
b. 5,2
b. 30,0 g e. 50 g
c. 12,0
c. 2,0 g
d. 29,8
279. e. 43,5
Os principais constituintes do “gás de lixo” e 282. UFMT
do “gás liquefeito de petróleo” são, respecti-
vamente, o metano e o butano. Comparando No processo de biodegradação, os micror-
volumes iguais dos dois gases, nas mesmas ganismos se alimentam da matéria orgânica
condições de pressão e temperatura, qual de- do lixo transformando-o em compostos mais
les fornecerá maior quantidade de energia da simples que são devolvidos ao ambiente. Es-
combustão? Justifique sua resposta a partir da tudantes de uma cidade litorânea puderam
observar a decomposição anaeróbica do lixo
PV-13-14

hipótese de Avogadro para os gases.


doméstico e coletaram uma amostra de gás
Massas molares: Calores de combustão (–DH) formado nesse processo. 0,34 g dessa amos-
metano = 16 g/mol metano = 208 kcal/mol tra, na pressão ambiente, ocupavam 246 mL
butano = 58 g/mol butano = 689 kcal/mol quando a temperatura local era 27 °C. Assi-
nale a alternativa que apresenta o nome e a
280. Fameca-SP fórmula desse gás.
Para que certo volume de SO2 nas CNTP conte- Considere : R = 0,082 L atm · K–1 · mol–1
nha o mesmo número de moléculas que 88 g
de CO2, é necessário que: a. Metano; CH4(g)
I. contenha 12,04 · 1023 moléculas; b. Dióxido de carbono; CO2(g)
II. esse volume tenha a massa de 64 g; c. Oxigênio; O2(g)
III. esse volume corresponda a 2 vezes o d. Dióxido de nitrogênio; NO2(g)
volume molar. e. Sulfeto de hidrogênio; H2S(g)

117
Química Química geral e estequiometria

283. Mackenzie-SP Então, na temperatura de 25 °C e pressão de


Um recipiente, de capacidade 500 cm , con- 3 1 atm, o volume de gás hidrogênio, consi-
tém 340 mg de uma substância no estado ga- derado gás ideal, para inflar o balão foi de,
soso, a 0,4 atm e 43 °C. De acordo com essas aproximadamente:
informações, a substância contida no recipien- Dados:
te possui a fórmula molecular: Massas molares (g · mol–1): H = 1,0; Cl = 35,5 e
Dados: massa molar (g/mol) H = 1, C = 12, Fe = 56,0;
N = 14, O = 16 e R = 0,082 L · atm · mol–1 · K–1
R = 0,082 L · atm · K–1 · mol–1 (Constante Uni-
a. NH3 d. CO2
versal dos Gases);
b. CO e. C2H2
T (K) = T (°C) + 273
c. CH4
a. 5.000 L
284. UERJ b. 15.000 L
A bola utilizada em uma partida de futebol é uma c. 35.000 L
esfera de diâmetro interno igual a 20 cm. Quan- d. 70.000 L
do cheia, a bola apresenta, em seu interior, ar sob e. 3.000 L
pressão de 1,0 atm e temperatura de 27 °C.
286. UFRJ
Considere π = 3, R = 0,080 atm · L · mol–1 · K–1 e,
para o ar, comportamento de gás ideal e massa Na busca por combustíveis mais "limpos", o hi-
molar igual a 30 g · mol–1. drogênio tem-se mostrado uma alternativa mui-
to promissora, pois sua utilização não gera emis-
No interior da bola cheia, a massa de ar, em sões poluentes. O esquema a seguir mostra a
gramas, corresponde a: utilização do hidrogênio em uma pilha eletroquí-
a. 2,5 c. 7,5 mica, fornecendo energia elétrica a um motor.
b. 5,0 d. 10,0
285. UFAC
Em novembro de 1783, na presença do rei
Luís XVI e da rainha Maria Antonieta, foi re-
alizado em Paris o primeiro voo tripulado
em um balão inflado com ar quente. Poucos
dias depois, em dezembro do mesmo ano,
o químico francês Jacques Alexandre Cesar
Charles (1746-1823), famoso por seus expe-
rimentos com balões, foi responsável pelo
PV-13-14

segundo voo usando um gás muito mais leve


que o ar – o hidrogênio, recém descoberto
à época. Mesmo apresentando grande risco
de explosão, porque esse gás é altamente Um protótipo de carro movido a hidrogênio foi
inflamável, a aventura teve sucesso e o hi- submetido a um teste em uma pista de provas.
drogênio ainda foi usado por mais de 150 Sabe-se que o protótipo tem um tanque de com-
anos. Imaginando-se que, para geração do bustível (H2) com capacidade igual a 164 litros e
hidrogênio, Charles usou cerca de 240,0 li- percorre 22 metros para cada mol de H2 consu-
tros de solução aquosa de ácido clorídrico mido. No início do teste, a pressão no tanque era
de concentração 12 mol · L–1 e 80,64 kg de de 600 atm e a temperatura, igual a 300 K.
ferro metálico puro, através da reação quí- Sabendo que, no final do teste, a pressão no
mica representada pela equação: tanque era de 150 atm e a temperatura, igual
Fe(s) + 2 HCl(aq) → FeCl2(aq) + H2(g) a 300 K, calcule a distância, em km, percorrida
pelo protótipo.

118
Química geral e estequiometria Química

287. UEG-GO pal circuito profissional foi conquistado justa-


Uma bola de futebol de volume constante de mente em um torneio da exclusivíssima série
2,73 L é cheia com oxigênio gasoso até alcan- Grand Slam, o de Roland Garros, em 1997.
çar a pressão interna de 4,1 atm a 0 °C. De Então um ilustre desconhecido no cenário in-
acordo com estas informações, determine: ternacional, Guga, aos 20 anos, mostrou, nas
quadras de terra mais famosas do mundo,
Dados: Massa molar do O2 = 32 g · mol–1
as credenciais que fariam dele, mais tarde, o
R = 0,082 atm · L · mol–1 · K–1 “Rei do Saibro”.
a. o número de mols do gás, na bola; A desenvoltura nos saques desses profissio-
b. a massa do gás, na bola. nais tem íntima relação com tecnologias em-
288. Unesp pregadas nas atuais bolas de tênis, que são
normalmente cheias com ar ou gás N2, com
As populações de comunidades, cujas mora-
pressão acima da pressão atmosférica, para
dias foram construídas clandestinamente sobre
aterros sanitários desativados, encontram-se aumentar seus ‘quiques’.
em situação de risco, pois podem ocorrer des- Se uma bola de tênis em particular tem volu-
moronamentos ou mesmo explosões. Esses lo- me de 144 cm3 e contém 0,33 g de gás N2, qual
cais são propícios ao acúmulo de água durante é a pressão dentro da bola a 24 °C?
os períodos de chuva e, sobretudo, ao acúmulo
de gás no subsolo. A análise de uma amostra Dado: R = 0,082 atm · L · mol–1 · K–1
de um gás proveniente de determinado aterro a. 2,0 atm
sanitário indicou que o mesmo é constituído b. 4,0 atm
apenas por átomos de carbono (massa molar =
12,0 g · mol–1) e de hidrogênio (massa molar = c. 1,2 atm
1,0 g · mol–1) e que sua densidade, a 300 K e 1 d. 0,16 atm
atmosfera de pressão, é 0,65 g · L–1. Calcule a e. 2,0 · 10–3 atm
massa molar do gás analisado e faça a represen-
tação da estrutura de Lewis de sua molécula. 291. UESPI
Dado: R = 0,082 L · atm · K–1 · mol–1 Um extintor de incêndio contém 3 kg de CO2.
O volume máximo de gás que é liberado na at-
289. FGV-SP mosfera, a 27 °C e 1 atm, é:
O gás hélio é utilizado para encher balões e be- Dados:
xigas utilizados em eventos comemorativos e
em festas infantis. Esse gás pode ser comercia- Constante dos gases: 0,082 atm · L · K−1 · mol−1.
lizado em cilindros cujo conteúdo apresenta Massa molar em g · mol−1: C = 12 e O = 16
pressão de 150 bar a 300 K. Considerando-se a. 732 L
que 1 atm = 1 bar, e que a massa de gás He b. 954 L
no cilindro é 170 g, então, o valor que mais se
aproxima do volume de gás hélio contido na- c. 1.677 L
PV-13-14

quele cilindro a 300 K é: d. 2.405 L


Dado: R = 0,082 atm.L . K–1 . mol–1 e. 3.170 L
a. 14 L d. 500 mL 292. UFT-TO
b. 7,0 L e. 140 mL Os trajes espaciais, usados pelos astronau-
c. 1,0 L tas durante os reparos na estação espacial,
possuem cilindros de oxigênio armazena-
290. UFT-TO dos para reposição. Apesar disso, o CO 2
Gustavo Kuerten impôs respeito entre os tra- precisa ser retirado. Sabendo-se que o CO 2
dicionais e melhores tenistas do mundo. Seu é um óxido ácido, esses trajes possuem um
saque, por exemplo, é considerado entre os sistema de purificação do ar contendo basi-
mais velozes da história do tênis. Ele entrou camente hidróxido de lítio, substância que,
para o clube dos grandes nomes do tênis pela em contato com o CO 2, produz carbonato
porta da frente. Seu primeiro título no princi- de lítio e água.

119
Química Química geral e estequiometria

Considerando que o traje espacial possui uma 295. Unifesp


capacidade máxima de volume para expan- A oxigenoterapia, tratamento terapêutico
dir, além do corpo da pessoa, de 5,2 Litros, a com gás oxigênio, é indicada para pacientes
temperatura e pressão constantes de 29 °C e que apresentam falta de oxigênio no san-
760 mmHg, respectivamente. Qual a massa de gue, tais como portadores de doenças pul-
hidróxido de lítio necessário para retirar todo monares. O gás oxigênio usado nesse trata-
o CO2 produzido quando o traje atingir seu vo- mento pode ser comercializado em cilindros
lume máximo? a elevada pressão, nas condições mostradas
Dado: R = 0,082 atm · L · mol–1 · K–1 na figura.
a. 24,0 g
b. 5,0 g
c. 10,0 g 150 atm
d. 44,0 g 20 L
e. 20,0 g
293. UFScar-SP
Diversos gases formam a atmosfera da Terra, sen- No cilindro, está indicado que o conteúdo cor-
do que a quantidade de alguns deles vem aumen- responde a um volume de 3 m3 de oxigênio
tando por ação antropogênica, o que pode causar nas condições ambiente de pressão e tem-
problemas. O oxigênio, em suas diferentes formas peratura, que podem ser consideradas como
alotrópicas, tem funções distintas e essenciais sendo 1 atm e 300 K, respectivamente.
para a manutenção da vida no planeta. Dado R = 0,082 atm · L · K–1 · mol–1, a massa de
a. Escreva a fórmula química das duas for- oxigênio, em kg, armazenada no cilindro de gás
mas alotrópicas mais comuns do oxigê- representado na figura é, aproximadamente:
nio, apontando a função de cada uma a. 0,98
delas relacionada com a manutenção
b. 1,56
da vida na Terra.
c. 1,95
b. Considerando que cerca de 20% em
volume da atmosfera é constituída de d. 2,92
oxigênio em sua forma alotrópica mais e. 3,90
abundante, calcule a massa desse gás 296. UFC-CE
contido num reservatório de 24,6 m3
cheio de ar a 27 °C e 1 atm de pressão. O principal componente da cal, importante
produto industrial fabricado no Ceará, é o
Dados: P · V = n · R · T; R = 0,082 atm · L · mol–1 · K–1.
óxido de cálcio (CaO). A produção de CaO se
294. UEG-GO modificado processa de acordo com a seguinte reação
PV-13-14

O cloro é utilizado para purificar águas forneci- química:


das pelos municípios e para tratar águas de pisci- CaCo3(s) ∆
→ CaO(s) + CO2(g)
nas. Admitindo-se que uma amostra de Cl2 está
contida em um cilindro de 10 L, a uma pressão
Considerando o comportamento ideal, assina-
de 2 atm e temperatura de 27 °C, conclui-se que:
le a alternativa que expressa corretamente o
Dado: R = 0,082 atm · L · mol–1 · K–1 volume (em L) de CO2 gerado na produção de
a. nessas condições, o Cl2 é um gás ideal. 561 kg de CaO a 300 K e 1 atm.
b. a amostra contém menos de 50 g de Cl2. Dado: R = 0,082 atm · L/mol · K
c. o Cl2 pode ser obtido pela decompos-
ção do NaClO. a. 22,4 d. 24.600
d. a 177 ° C, a pressão no cilindro passa a b. 224 e. 246.000
ser de 3 atm. c. 2.460

120
Química geral e estequiometria Química

297. Unesp a. 10,0 atm


Nos frascos de spray, usavam-se como pro- b. 8,00 atm
pelentes compostos orgânicos conhecidos c. 5,80 atm
como clorofluorcarbonos. As substâncias d. 4,20 atm
mais empregadas eram CClF3 (fréon 12) e
C2Cl3F3 (fréon 113). Num depósito abandona- e. 2,46 atm
do, foi encontrado um cilindro supostamente 300. FGV-SP
contendo um destes gases. Identifique qual é Muitas frutas são colhidas ainda verdes, para
o gás, sabendo que o cilindro tinha um volu- que não sejam danificadas durante o seu trans-
me de 10,0 L, a massa do gás era de 85 g e a porte. São deixadas em armazéns refrigerados
pressão era de 2,00 atm a 27 °C. até o momento de sua comercialização, quando
R = 0,082 atm · L · mol–1 · K–1 são colocadas em um local com gás eteno por
Massas molares em g · mol–1: H = 1, C = 12, F = 19, determinado período, para que o seu amadure-
Cl = 35,5 cimento ocorra mais rapidamente.
298. UFPE As reações I e II representam dois métodos di-
ferentes na produção de eteno.
As propriedades físicas de um gás ideal são
I. CH3 − CH3 Catal
 → CH2 = CH2 + H2
descritas por quatro parâmetros (quantida-
de de matéria, n; temperatura, T; pressão, P;
volume, V). Esses quatro parâmetros não são II. CH3 − CH2OH H
SO , 270 ° C
2 4
→ CH2 = CH2 + H2O
independentes, e as relações entre eles estão Dado: R = 0,082 atm · L · K–1 · mol–1
explicitadas na equação de estado do gás ide- A massa aproximada de eteno, equivalente a
al, P · V = n · R · T. Qual das afirmações a seguir, 50,0 L desse gás contido num cilindro a 300 K e
relacionadas à equação citada, é incorreta? 2,00 atm, é igual a:
a. Um gás ideal é definido como aquele
a. 4.000 g d. 224 g
que obedeceria rigorosamente à equa-
ção de estado P · V = n · R · T. b. 2.050 g e. 112 g
b. Em certas circunstâncias, gases reais c. 816 g
comportam-se, aproximadamente, se- 301. Unesp
gundo o modelo de um gás ideal. Magnésio metálico reagiu com HCl suficiente
c. O valor numérico da constante R de- para produzir 8,2 litros de gás hidrogênio, me-
pende das unidades de P, V, n e T. didos à temperatura de 27 °C. A pressão do gás
d. O parâmetro P, na equação P · V = n · R · T, foi de 1,5 atmosfera. Quantos mols de hidro-
é definido necessariamente pela pressão gênio foram produzidos?
externa exercida sobre o sistema. Dados: constante geral dos gases =
e. A pressão osmótica de uma solução di- 0,082 atm · L/mol · K
PV-13-14

luída-ideal, π , é calculada com o uso de a. 0,5


uma equação análoga a P · V = n · R · T. b. 2,73
299. Unicid-SP c. 5,56
O gás butano é utilizado como propelente em d. 380
desodorantes e em cremes de barbear. O rótu- e. 0,37
lo de um creme de barbear indica a composi-
ção de 4% em massa de butano numa embala- 302. Fatec-SP
gem de 145 g de produto. Qual a massa de CO2 existente em 8,2 L desse
gás, submetido à temperatura de 27 °C e pres-
A pressão a 300 K, em um recipiente de 1,00 L,
são de 3 atm?
exercida pela igual quantidade de butano conti-
da na embalagem de creme de barbear é: Dados: R= 0,082 atm . L/mol1 . K1: massa molar do
CO2 = 44 g . mol–1
Dado: R = 0,082 atm · L · K–1 · mol–1

121
Química Química geral e estequiometria

303. Fatec-SP 307. Unesp


Qual é o número de moléculas de um gás qual- O valor considerado normal para a quantidade
quer, existente em 8,2 L do mesmo, à temperatura de ozônio na atmosfera terrestre é de, aproxi-
de 127 °C e à pressão de 6 atm? madamente, 336 UD (Unidade Dobson), o que
Dado: R = 0,082 atm · L · mol–1 · K–1 equivale a 3,36 L de ozônio por metro quadra-
do de superfície ao nível do mar e à tempera-
304. UFPI tura de 0 °C.
Compressores são máquinas que, na pres- a. Calcule a quantidade O3, em número de
são atmosférica local, captam o ar, compri- mols por m2, nessas condições (336 UD
mindo-o até atingir a pressão de trabalho no nível do mar e a 0 °C).
desejada. Ao nível do mar, a pressão atmos-
férica normal é 1,0 bar. Em equipamentos b. Sabendo que um átomo de cloro (Cl)
pneumáticos, a 25 °C, a pressão mais utiliza- pode reagir com 100.000 moléculas
da é a de 6,0 bar. Nessas condições, e consi- de ozônio (um dos processos respon-
derando o oxigênio (O2) como um gás ideal, sáveis pela destruição da camada de
calcule a massa aproximada de O2, em gra- ozônio), qual a massa de cloro, em gra-
mas, contida em um compressor de volume mas por metro quadrado, suficiente
igual a 5,0 litros. (A constante R vale 0,082 para reagir com dois terços do ozônio
bar · L · mol–1 · K–1; MO2 = 32 g/mol nessas condições?

a. 8,0 Dados: Massa molar do cloro (Cl): 35,5 g/mol;


b. 16 Constante de Avogadro: 6,0 · 1023
c. 39 308. UEL-PR
d. 47 Um balão de vidro de 1 litro, com torneira, aber-
e. 55 to ao ar, foi ligado a uma “bomba de vácuo” du-
rante algum tempo. Considerando-se que essa
305. UFRJ
bomba é eficiente para baixar, a 25 °C, a pressão
Necessita-se armazenar certa quantidade de até 10–4 mmHg, após fechar a torneira, quantos
oxigênio gasoso. A massa do gás é de 19,2 g, à mols de oxigênio (O2) foram retirados do balão?
temperatura de 277 °C e à pressão de 1,5 atm. (O que resta de ar no balão é desprezível).
O único recipiente capaz de armazená-lo terá, Dados:
aproximadamente, o volume de:
Volume molar dos gases a 1 atm e 25 °C = 25 L/mol
Dados: massa molar de O2= 32 g · mol–1; R = 0,082 Composição aproximada do ar = 80% de N2
atm · L · mol–1 · K–1 e 20% de O2 (% em mols); pressão atmosfé-
a. 4,50 L rica = 760 mmHg
b. 9,00 L
PV-13-14

a. 1 · 10–2 mol
c. 18,0 L b. 2 · 10–2 mol
d. 20,5 L c. 4 · 10–2 mol
e. 36,0 L d. 8 · 10–3 mol
306. Fuvest-SP e. 8 · 10–4 mol

Indique os cálculos necessários para a de- 309. Unicamp-SP


terminação da massa molecular de um gás, 1,0 litro de nitrogênio líquido, N2( ), foi colocado
sabendo que 0,800 g desse gás ocupa o vo- num recipiente de 30,0 litros, que foi imediata-
lume de 1,12 L a 273 °C e 2,00 atm. Qual mente fechado. Após a vaporização do nitrogê-
valor se encontra para a massa molecular nio líquido, a temperatura do sistema era 27 °C.
desse gás? (Densidade do N 2( ) a –196 °C = 0,81 g/cm 3;
Dado: 0,082 atm · L · mol–1 · K–1 massa molar do N2 = 28 g/mol;

122
Química geral e estequiometria Química

R = 0,082 atm . L/K . mol) d. 727 °C


a. Qual foi a massa de nitrogênio colocada e. 125 °C
no recipiente? 314. UFRGS-RS
b. Qual será a pressão final dentro do reci-
Dois recipientes A e B, de paredes rígidas, apre-
piente? Considere que a pressão do ar,
sentam iguais volumes. O recipiente A contém
originalmente presente no recipiente, é
uma massa de hélio (He) igual à massa de me-
de 1,0 atm.
tano (CH4) contida no recipiente B. Inicialmente
310. UFRGS-RS os dois gases estão a 100 K. Elevando-se a tem-
Resfria-se um recipiente contendo 1.530 litros peratura do metano para 400 K, sua pressão em
de vapor d’água a 100 °C e 1 atm até liquefa- relação ao hélio será, aproximadamente: (H = 1;
zer todo o vapor. Considerando os valores para He = 4; C = 12)
R = 0,082 atm . L . K–1 . mol–1 e massa específica a. quatro vezes menor.
da H2O(  ) = 1g · ml–1, o volume de água líquida b. a metade.
obtida, a 20 °C, será de, aproximadamente:
c. a mesma.
(H = 1; O = 16) d. o dobro.
a. 0,9 L e. quatro vezes maior.
b. 20 L
315. Unesp-SP
c. 50 L
Durante o transporte do etano gasoso (C2H6)
d. 186 L em um caminhão-tanque com capacidade de
e. 900 L 12,3 m3, à temperatura de –23 °C, houve um
311. acidente e verificou-se uma queda de pres-
são de 0,6 atm. Admitindo-se a temperatura
Um balão A contém 4 g de O2 a uma dada tem- constante, calcule a massa de etano perdida no
peratura e pressão; um balão B, cuja capacidade ambiente.
é igual a 3/4 da do primeiro, contém N2 à mesma
temperatura que o O2. A pressão do N2 é 4/5 da Dados: masas atômicas: C = 12,0; H = 1,01;
do O2. Qual é a massa de N2 no balão B? O = 16,0; constante dos gases:
R = 0,0821 atm · L · mol–1 · K–1
312. FAAP-SP
316. PUC-SP
Em um recipiente A de 30 L de capacidade,
está contido H2(g) a 0,82 atm e 27 °C. Determi- Uma amostra de gás oxigênio (O2) a 327 °C
ne a massa de O2(g) necessária a ser colocada está armazenada em um recipiente fechado
em outro recipiente B, de mesmo volume que de vidro de volume igual a 6 L, exercendo uma
A, de modo a se ter para o O2 as mesmas con- pressão de 1,64 atm (situação I). Esse recipien-
te é resfriado para 27 °C (situação II). Após o
PV-13-14

dições de P e T apresentadas pelo H2.


sistema atingir o equilíbrio térmico, é correto
Dados: R = 0,082 atm · L · mol–1 · K–1; H = 1; afirmar que:
O = 16
a. o recipiente contém 0,2 mol de molé-
313. UFBA culas de gás oxigênio.
A temperatura a que deve ser aquecido um b. o volume ocupado pelo gás na situação
gás contido em um recipiente aberto, inicial- II é de 3 L.
mente a 25 °C, de tal modo que nele perma- c. a pressão na situação II é de 0,135 atm.
neça 1/5 das moléculas nele inicialmente
contidas, é: d. a pressão na situação II é de 4,92 atm.
a. 1.217 °C e. o recipiente contém 3,2 g de gás oxigê-
nio. (O = 16)
b. 944 °C
c. 454 °C

123
Química Química geral e estequiometria

317. FUVEST-SP 320. Unesp


Dados referentes aos planetas Vênus e Terra: Se todo o ozone (O3) da camada de 20 km
de espessura, situada bem acima da super-
Vênus Terra fície da Terra (estratosfera), fosse coletado e
submetido à pressão de 1 atm, ele ocuparia
% (em volume) de N2 na uma camada na superfície da Terra de 3 mm
4,0 80
atmosfera
de espessura. A temperatura média na su-
Temperatura na superfície (K) 750 300 perfície da Terra é de 20 °C, na estratosfera
é de – 40 °C e a superfície da Terra tem uma
Pressão na superfície (atm) 100 1,0 área de 150 · 106 km2.
Dados: Massa atômica: O = 16;
A relação entre o número de moléculas de N2 constante universal dos gases =
em volumes iguais das atmosferas de Vênus e da 0,082 atm · L · mol–1 · K–1; 1 m3 = 103 litros
Terra é: a. Por que a camada rarefeita de ozo-
a. 0,10 ne é mais espessa do que o mesmo
b. 0,28 gás na superfície da Terra? Escrever a
equação matemática que representa
c. 2,0
essa relação.
d. 5,7
b. Calcular a massa aproximada de ozone
e. 40 ao redor da Terra.
318. Cesgranrio-RJ 321.
Num tanque de gás, havia 8,2 m3 de oxigênio Calcule a densidade do:
a –23 °C e 2 atm de pressão. Tendo ocorrido
um vazamento, verificou-se que a pressão di- a. CO nas condições normais.
minuiu em 0,5 atm. Que massa de oxigênio foi b. CO2 a 27 °C e 600 mmHg.
perdida, sabendo-se que a temperatura per- c. CH4 em relação ao H2.
maneceu constante? d. Ar (argônio) em relação ao ar.
(O = 16; R = 0,082 atm · L/mol · K) (MA: C = 12; O = 16; H = 1; Ar = 40)
a. 0,6 kg
322.
b. 6,4 kg
A densidade de um gás A nas CNTP é igual a
c. 19,2 kg
2,5 g/L.
d. 25,6 kg
Calcule:
e. 32,0 kg
a. a massa molecular desse gás;
319.
PV-13-14

b. a densidade desse gás a 273 °C e 2,00 atm;


Têm-se dois balões, A e B, ligados por um tubo c. a densidade desse gás em relação ao He;
de comunicação munido de torneira. O balão
A contém 1,6 g de CH4(g), e o balão B está vazio d. a densidade desse gás em relação ao ar.
(vácuo). Abrindo-se a torneira de comunica- (MA: He = 4; ar ≅ 29)
ção, depois de um tempo prolongado, a pre- 323.
são do CH4(g) nos balões interligados é igual a
0,20 atm. Sabendo-se que o volume do balão A densidade de um gás X a 127 °C e 0,82 atm é
A é igual a 8,2 L, calcule o volume do balão B. igual a 1,45 g/L.
A temperatura foi mantida em 27 °C durante Calcule:
toda a experiência. Considere desprezível o
volume do tubo de comunicação. a. a massa molecular desse gás;
b. sua densidade em relação ao H2;
(Massa molar do CH4 = 16 g/mol.)
c. sua densidade em relação ao ar.

124
Química geral e estequiometria Química

324. Unicamp-SP-modificado a 15% em volume quando em ar am-


Os gêiseres são um tipo de atividade vulcânica biente com 21% de oxigênio) e existe
que impressiona pela beleza e imponência do uma faísca ou iniciador, a explosão irá
espetáculo. A expulsão intermitente de água ocorrer”. Partindo-se do ar atmosférico
em jatos na forma de chafariz é provocada pela e de metano gasoso, seria possível ob-
súbita expansão de água profunda, superaque- ter a mistura com a composição acima
cida, à pressão de colunas de água que chegam mencionada, pela simples mistura des-
até à superfície. Quando a pressão da água ses gases? Justifique.
profunda supera a da coluna de água, há uma 327. UEM-PR
súbita expansão, formando-se o chafariz até a
exaustão completa, quando o ciclo recomeça. Recentemente, um astrônomo amador con-
seguiu realizar fotos a cerca de 30 km da su-
Se a água profunda estiver a 300°C e sua perfície da terra, utilizando um pequeno balão
densidade for 0,78 g/cm3, qual será a pres- que carregava uma máquina fotográfica pro-
são (em atmosferas) de equilíbrio dessa gramada para coletar fotos automaticamen-
água supondo-se comportamento de gás te. A respeito dessa afirmação, assinale a(s)
ideal? R = 82 atm ·cm3 · mol–1 · K–1. alternativa(s) correta(s).
325. UFC-CE 01. O astrônomo amador poderia utilizar
O monóxido de carbono é um dos poluentes gases, como o hidrogênio, o hélio ou o
do ar presente, especialmente, em zona ur- nitrogênio, para encher e fazer voar o
bana. A pressão do monóxido de carbono de seu balão.
0,004 atm, no ar, resulta em morte, em pou- 02. Os dirigíveis muito utilizados antigamen-
co tempo. Um carro ligado, porém parado, te em transporte aéreo eram considera-
pode produzir, entre outros gases, 0,60 mol dos bombas aéreas, pois continham gás
de monóxido de carbono por minuto. Se uma hélio, que é extremamente inflamável.
garagem a 27 °C tem volume de 4,1 · 104 L, em 04. Sabendo-se que o astrônomo amador
quanto tempo, na garagem fechada, atingir- preencheu seu balão com uma quanti-
-se-á a concentração letal de CO? dade de x gramas de gás hidrogênio, se
Obs.: Considere que a pressão, na garagem, o mesmo utilizasse 2x gramas desse gás
permanece constante e que nela não há ini- no mesmo balão, este alcançaria uma
cialmente monóxido de carbono presente altura muito maior antes de estourar.
(constante dos gases = 0,082 L . atm/mol . K). 08. Se o astrônomo utilizasse uma mistura
326. Unicamp-SP de gases em seu balão, essa mistura po-
deria ser considerada como homogênea.
Recentemente a Prefeitura de São Paulo ame-
açava fechar as portas de um centro comercial 16. Os balões utilizados na prática de ba-
por causa do excesso de gás metano em seu lonismo esportivo alçam voo devido
subsolo. O empreendimento foi construído à queima de gás butano, que causa o
PV-13-14

nos anos 1980 sobre um lixão e, segundo a CE- aquecimento dos gases do interior do
TESB, o gás metano poderia subir à superfície balão, fazendo com que esses gases se
e, eventualmente, causar explosões. tornem menos densos que o ar.
a. Uma propriedade que garante a ascen- 328. UFG-GO
são do metano na atmosfera é a sua Balões voam por causa da diferença de den-
densidade. Considerando que os gases se sidade entre o ar interno e o externo ao ba-
comportam como ideais, e que a massa lão. Considere um planeta com atmosfera de
molar média do ar atmosférico é de 28,8 nitrogênio e um balão cheio com esse gás.
g · mol–1, justifique esse comportamento Demonstre, e explique, se esse balão vai flu-
do metano em relação ao ar atmosférico. tuar quando o ar interno estiver a 100 °C e o
b. Na época do acontecimento, veiculou- externo, a 25 °C. Admita o comportamento
-se na imprensa que, “numa mistura ideal dos gases, pressão de 1 atm e descon-
com o ar, se o metano se encontra den- sidere a massa do balão.
tro de um determinado percentual (5% Dado: R = 0,082 atm · L/K · mol

125
Química Química geral e estequiometria

329. UESC-BA 331. UFRR modificado


Na década de 1880, Lord Rayleigh, físico Um mol de gás Ideal X, sob pressão de
inglês, ficou surpreso com a diferença en- 760mmHg e temperatura de 27 °C, é aqueci-
contrada em duas medições experimentais do até que a pressão e o volume dupliquem.
de densidade absoluta do nitrogênio. Na Pode-se afirmar que a temperatura final e a
primeira, o valor encontrado, após elimi- densidade desse gás são, respectivamente:
nação do oxigênio, do dióxido de carbono Dados: massa molar do gás X = 12 g/mol e R =
e da água do ar, foi de 1,2561 g · L–1 e, na 0,082 atm · L/mol · K
segunda, a densidade do nitrogênio pro- a. 1.000 °C e 0,22 g/L
veniente da combustão do gás amônia, se-
gundo a equação química 4 NH3(g) + a 3 O2(g) b. 2.400 °C e 0,29 g/L
→ 2 N2(g) + 6H2O(), foi de 1,2498 g · L–1, nas c. 1.200 K e 0,24 g/L
mesmas condições de pressão e de tempe- d. 600 K e 0,48 g/L
ratura. Rayleigh e Willian Ramsay fizeram o
e. 525 K e 0,14 g/L
nitrogênio, obtido a partir do ar atmosféri-
co, reagir com magnésio, de acordo com a 332. UEG-GO
equação química N2(g) + 3Mg(s) → Mg3N2(s), e Alguns compostos orgânicos formam mis-
concluíram que, após a reação, restou uma turas explosivas com o ar quando se evapo-
pequena quantidade de gás denso, ao qual ram. Suponha que um aluno distraidamente
deram o nome de argônio. tenha esquecido um frasco de éter etílico
A partir dessas informações sobre a descober- aberto no laboratório, após as atividades
ta do argônio, é correto concluir: de pesquisa do dia. Admita que o risco de
01. A massa em gramas de argônio existen- explosão inicia-se quando a pressão parcial
te em 1,0 L da mistura de nitrogênio do vapor atinge valor igual a pi, que µ, M, V,
com argônio, de densidade absoluta T e R sejam, respectivamente, a densidade
1,2561 g · L–1, é 6,3 · 10–3 g. do líquido, a massa molar da substância, o
02. O número de átomos de argônio e o volume aproximado do laboratório, a tem-
de nitrogênio encontrados na mistura peratura no interior do laboratório e a cons-
são iguais. tante universal dos gases. Com base nessas
informações, o volume mínimo de líquido
03. A densidade absoluta do nitrogênio que deve evaporar para se atingir o risco de
puro varia com o método de obtenção explosão é igual a:
dessa substância.
04. A massa em gramas de nitreto de mag- pi VM
a. .
nésio, formado após a reação de 1,0L RTµ
de nitrogênio com magnésio nas CNTP,
é 1,2137g. pi VM
PV-13-14

b. .
05. A densidade absoluta do argônio, nas RT2 µ
CNTP, é de 1,7857gL–1.
330. UFPB µT2
c. .
Rpi V
75 mg de etano gasoso (C2H6) ocupam um vo-
lume de 90mL sob pressão de 1,0 atm a uma
3µVM
dada temperatura. d. .
2 RT
(H=1; C=12)
a. Calcule a temperatura do sistema.
b. Que massa do gás ocupará 600 mL se
a temperatura for 27 °C e a pressão,
1 atm?

126
Química geral e estequiometria Química

333. UEG-GO 335. Mackenzie-SP


Analise as figuras I e II e responda ao que se pede. Quatro balões idênticos foram enchidos com
um mol de gás e colocados em uma caixa fe-
chada, conforme a figura abaixo. Todos os ga-
ses encontram-se a P = 1 atm e T = 25 °C.

O2 NH3 He CO2

I II III IV

Dados: massa molar (g/mol): H =


1; He = 4; C = 12; N = 14; O = 16
Massa aparente do ar = 28,96 g/mol
Figura I – Balão a gás
Se abrirmos a caixa, os balões que vão subir são:
a. I e III, apenas.
b. II e III, apenas.
c. I e IV, apenas.
d. II e IV, apenas.
e. I, II e III, apenas.
336. UEPB
Armas químicas
Em várias épocas da história, algumas substân-
cias químicas reforçaram o arsenal das armas
físicas de impacto para fins militares. O uso
Figura I – Bexiga e dirigíveis dessas substâncias de guerra se concretizou
de fato na 1ª Guerra Mundial (1914-1918),
a. No caso da figura I, explique por que o determinando a morte de cerca de 100.000
balão sobe. pessoas, entre civis e militares. O uso mais re-
b. Suponha que as bexigas da figura II con- cente de armas químicas foi comprovado na
tenham gás hélio. Caso a criança solte Guerra Irã-Iraque (22/09/1980-20/08/1988).
o balão, este subirá indefinidamente? Após séculos de aplicação, somente em 1989
Justifique. deu-se início a tratados internacionais de bani-
PV-13-14

334. Unimontes-MG mento das armas químicas.


O volume molar, 22,4 L, nas condições normais Uma forma moderna de aplicação dessas ar-
de temperatura e pressão, é útil para se deter- mas consiste nas chamadas armas binárias;
minar a massa molar ou mesmo a densidade em que duas substâncias, não tóxicas, pre-
de um gás ou de substâncias que podem ser cursoras do produto final entram em conta-
facilmente vaporizadas. to e reagem formando o composto tóxico.
Entre estes produtos destacam-se o “sarin”
Se 2,0 L de um gás, medidos nas condições ci- e o “soman”.
tadas acima, apresentam massa igual a 3,23 g,
pode-se afirmar que a densidade desse gás (g/L) Sabe-se que a dose letal de uma substância
é, aproximadamente: (DL50) provoca a morte de 50 % dos animais
testados e que a volatilidade é uma medida da
a. 1,6 c. 0,1 quantidade do material que pode ser reduzido
b. 3,6 d. 3,2 a gás ou vapor.

127
Química Química geral e estequiometria

Tabela 1 – Algumas propriedades de substâncias utilizadas como armas químicas


Volatilidade
Ponto de Ponto de
Substância Fórmula (20 °C mg · DL50 (mg · min · m–3)
fusão (°C) ebulição (°C)
m = 3)

Irritantes
pulmonares

Difosgênio ClCOOCCl3 –57 127 54.300 3.200

Cloropicrina CCl3NO2 –69 170.000 20.000

Gase do vômito

DM – Adamsita Ph2NAsClH 195 <1 30.000

PD PhAsCl2 –16 404 2.600

Gases
lacrimogêneos

CN PhCOCH2Cl 55 105 11.000

CS PhCHC(CN)2 95 310 10 2.500

Gases
vesicantes

HD (gás
Cl (CH2)2S(CH2)2Cl 14 215 610 1.500
mostarda)

Lewisita ClCHCHAsCl2 –18 2.300 1.300

Gases
neurotóxicos

GA – Tabun C5H11N2PO2 –50 240 400 400

GB – Sarin C4H10PO2F –56 12.100 100

GD – Soman C7H16PO2F 167 3.000 70

VX C11H26PO2SN < –50 10 36


PV-13-14

Analise os itens a seguir, relativos à ação dos gases tóxicos em campo de batalha.
I. A ocorrência de ventos fortes, ao mesmo tempo em que espalha o produto por uma área
maior, também o dilui em concentração; neste caso, os gases mais efetivos são os mais volá-
teis, pois uma quantidade menor de gás é suficiente para atingir concentrações letais.
II. Em épocas ou regiões muito quentes, as quantidades desses materiais necessárias para se
obter as concentrações letais são bem maiores do que as de regiões normais ou frias.
III. Para que uma substância seja utilizada como um gás de guerra é necessário que ela se
mostre eficaz em baixa dose; que ela seja estável, isto é, que não se decomponha durante
o transporte; e que a proteção à sua ação seja difícil.
a. Todas as afirmações estão corretas. d. Apenas I está correta.
b. Apenas II e III estão corretas. e. Apenas II está correta.
c. Apenas I e III estão corretas.

128
Química geral e estequiometria Química

337. UFJF-MG surgiu, então, uma chama quase invisível e


Existem vários tipos de extintores de incên- o ovo explodiu. Todos aplaudiam, enquanto
dio, um deles é chamado de "extintor de Dina explicava que, no interior do ovo (na
espuma". No seu interior, encontram-se, em verdade era só a casca dele), ela havia colo-
dois compartimentos separados, carbonato cado gás hidrogênio e que o que eles tinham
de sódio sólido e ácido sulfúrico aquoso. acabado de ver era uma reação química.
Quando esse tipo de extintor é colocado de Aplausos novamente.
cabeça para baixo, um tampão que separa a. Se o gás que ali estava presente era o
os compartimentos se rompe e permite que hidrogênio, a que reação química Dina
ocorra a seguinte reação: fez referência? Responda com a equa-
∆ Na SO + H O + CO ção química correspondente.
Na CO + H SO →
2 3(s) 2 4(aq) 2 4(aq) 2 () 2(g)
b. Se a quantidade (em mol) dos gases
a. O CO2 formado produz uma pressão
reagentes foi maior que a do produto
dentro do recipiente e é eliminado.
gasoso, então o ovo deveria implodir, e
Sendo mais denso do que o ar, o CO2
não explodir. Como se pode, então, ex-
evita o contato do material que está
plicar essa explosão?
queimando com o oxigênio. Por que o
CO2 é mais denso que o ar ? Dado: A 339. UEL-PR
massa molar média do ar é 28,9 g/mol. As figuras abaixo representam recipientes
b. Outros extintores são chamados de contendo gases considerados ideais. Os vo-
extintores de pó químico e utilizam lumes de A, B e C são iguais; o volume de D é
substâncias sólidas, por exemplo, a metade do volume de C; todos os recipien-
bicarbonato de sódio que, ao entrar tes contêm o mesmo número de moléculas.
em contato com o material que está Informações adicionais são dadas sob cada
queimando, decompõe-se pelo calor figura.
e libera CO2 de acordo com a reação A B C
abaixo:
2NaHCO3(s) → Na2CO3(aq) + H2O() + CO2(g)
Quantos litros de CO2 seriam gerados a partir D
de 1 kg de bicarbonato, considerando as CNTP?
c. Explique a seguinte frase: "O oxigênio do
ar propaga as chamas de um incêndio".
338. Unicamp-SP
2He 2He 18Ar 18Ar
Eles estão de volta: Omar Mitta, vulgo Rango, tA = 10 °C tB = 50 °C tC = 10 °C tD = 10 °C
e sua esposa Dina Mitta, vulgo Estrondosa, a
dupla explosiva que já resolveu muitos misté- Com relação à densidade e à pressão dos ga-
PV-13-14

rios utilizando o conhecimento químico. Hoje ses contidos nos recipientes, é incorreto afir-
estão se preparando para celebrar uma data mar que:
muito especial. a. a pressão do gás contido em C é maior
Rango, logo depois de servir o bolo, levou os do que a do gás contido em A.
convidados de volta ao bar. Lá, para entreter b. o argônio contido em D é o gás de
os convidados, Dina acomodou um ovo so- maior densidade.
bre um suporte plástico. Esse ovo tinha fitas c. o gás contido em C é mais denso do que
de vedação nas duas extremidades, tapando o contido em A.
pequenos furos. Dina retirou as vedações, d. a densidade do gás contido em A é
apoiou o ovo novamente no suporte plástico igual à do gás contido em B.
e levou um palito de fósforo aceso próximo a e. a pressão do gás contido em D é o do-
um dos furos: de imediato, ouviu-se um pe- bro da pressão do gás contido em A.
queno barulho, parecido a um fino assovio;

129
Química Química geral e estequiometria

340. ITA-SP 344.


Considere as afirmações abaixo relativas ao aque- Dois balões (A e B) foram conectados por um
cimento de um mol de gás N2 contido em um cilin- tubo contendo uma válvula fechada. O balão A
dro provido de um pistão móvel sem atrito: continha 3 L de O2(g) a 2 atm e o balão B conti-
I. A massa específica do gás permanece nha 2 L de He(g). Abrindo-se a torneira, manten-
constante. do-se a temperatura depois de estabelecido o
equilíbrio no sistema, a pressão total da mistura
II. A energia cinética média das moléculas
gasosa passou a ser 3,2 atm. Qual a pressão do
aumenta.
recipiente que continha hélio inicialmente?
III. A massa do gás permanece a mesma.
a. 4 atm
IV. O produto pressão x volume permane-
b. 2 atm
ce constante.
c. 3,2 atm
Das afirmações feitas, estão corretas: d. 6 atm
a. apenas I, II e III. e. 5 atm
b. apenas I e IV. 345.
c. apenas II e III. Dois balões (I e II) contêm, respectivamente,
d. apenas II, III e IV. 5 L de CO2(g) a 2 atm e 10 L de He(g) a 4 atm.
e. todas. Se os dois gases forem reunidos no balão II,
mantendo-se a temperatura, qual será a nova
341. UFPA pressão dessa mistura?
Em um recipiente cuja capacidade é de 5,0 li- 346. PUC-RJ
tros, misturam-se 2,8 g de nitrogênio e 1,6 g de
oxigênio. A pressão total da mistura a 27 °C é: O gás natural, embora também seja um com-
bustível fóssil, é considerado mais limpo do que
Dados: R = 0,082 atm · L/mol · K; N = 14 u; a gasolina, por permitir uma combustão mais
O = 16 u completa e maior eficiência do motor. Assim,
a. 0,05 atm d. 0,54 atm um número crescente de táxis rodam na cidade
b. 0,25 atm e. 0,74 atm movidos por este combustível. Estes veículos
c. 0,49 atm podem ser reconhecidos por terem parte de
seu porta-malas ocupada pelo cilindro de aço
342. que contém o gás. Um cilindro destes, com vo-
Temos um recipiente com 4 L de H2(g) puro lume de 82 litros, foi carregado em um posto,
a 2 atm conectado a outro (com válvula fe- numa temperatura de 27oC, até uma pressão de
chada) contendo 3 L de CO2(g) puro a 6 atm. 6 atm. Qual a massa de gás natural nele con-
Abrindo a torneira que separa os gases e tido, considerando o gás natural formado (em
mantida a temperatura, calcule a pressão in- mols) por 50% de metano (CH4) e 50% de etano
PV-13-14

terna na qual o sistema se estabiliza. (C2H6)?


343. UFRGS-RS Dado: R = 0,082 atm · L/K · mol
347. Fuvest-SP
Dois balões indeformáveis (I e II), à mesma
temperatura, contêm, respectivamente, 10 L Dois frascos, A e B, mantidos à temperatura am-
de N2 a 1 atm e 20 L de CO a 2 atm. Se os dois biente, contêm, respectivamente, 1 litro de ni-
gases forem reunidos no balão I, a pressão to- trogênio a 2 atmosferas de pressão e 3 litros de
tal da mistura será: dióxido de carbono a 3 atmosferas de pressão.
a. 1 atm a. Qual é a razão entre o número de molé-
culas nos frascos A e B?
b. 2 atm
b. Se os gases forem transferidos para um
c. 3 atm
frasco de 10 litros, à mesma tempera-
d. 4 atm tura ambiente, qual será a pressão da
e. 5 atm mistura gasosa resultante?

130
Química geral e estequiometria Química

348. PUCCamp-SP a. 2,0


Um sistema é formado por dois recipientes de b. 1,5
volumes diferentes, interligados por tubulação c. 1,0
com registro. De início, estando o registro fe- d. 0,50
chado, cada recipiente contém um gás perfei-
to diferente, na pressão de uma atmosfera. A e. 0,25
seguir, o registro é aberto. Considerando que 352. UFRGS-RS
a temperatura se manteve constante durante Se o sistema representado abaixo for manti-
todo o processo, podemos afirmar que a pres- do a uma temperatura constante e se os três
são final no sistema: balões possuírem o mesmo volume, após se
a. será de 1/2 atm. abrirem as válvulas A e B, a pressão total nos
b. será de 1 atm. três balões será:
c. será de 2 atm.
A B
d. dependerá dos volumes iniciais.
H2 He
e. dependerá dos volumes iniciais e da 3 atm Vácuo
9 atm
natureza dos dois gases.
349.
Em um recipiente com capacidade para 8 li- a. 3 atm
tros, misturam-se 1,6 g de metano e 5,6 g de b. 4 atm
nitrogênio. Determine a pressão total da mis-
c. 6 atm
tura a 27 °C.
d. 9 atm
Dados: massas molares: CH4(g) = 16 g/mol; N2(g)
= 28 g/mol; R = 0,082 atm · L · mol–1 · K–1 e. 12 atm
350. Cesgranrio-RJ 353. UFRJ
O gás de cozinha, também chamado de gás Dois gramas de hélio e x gramas de hidrogênio
liquefeito de petróleo (GLP), é formado por estão contidos num frasco de volume igual a
50% de propano e 50% de butano. Sabendo-se 22,4 litros, nas CNTP. (H = 1; He = 4; R = 0,082
que uma família domiciliada no Rio de Janeiro atm · L · mol–1 · K–1)
gastou 49,2 m3 de GLP no mês de setembro, o a. Determine o valor de x.
número de moléculas de gás butano queima- b. Qual será a pressão se essa mistura for
do nesse mês foi: transferida para um vaso de volume
Dados: volume molar do butano a 27 oC e 1 igual a 5,6 litros a 0 °C?
atm = 24,6 litros; 354. ITA-SP
Constante de Avogadro = 6,0 · 10 23
Temos um recipiente com N2 puro e outro com
PV-13-14

a. 6,0 · 1027 O2 puro.


b. 6,0 · 1026 Volumes e pressões iniciais estão assinalados
c. 6,0 · 1025 no esquema seguinte.
d. 3,0 · 1026 Mesma temperatura
e. 3,0 · 1025
351. PUCCamp-SP N2 O2
Um balão de vidro de 60,0 L contém uma mis- 1,00 atm 5,00 atm
tura gasosa exercendo a pressão de 0,82 atm a 3,00 litros 2,00 litros
300 K. O número total de mols dos gases con-
tidos no recipiente é igual a:
Dado: R = 0,082 atm ⋅ L ⋅ mol–1 ⋅ K–1

131
Química Química geral e estequiometria

Abrindo a torneira que separa os dois gases e 357. UFPR


mantendo a temperatura, a pressão interna se Considere os seguintes dados:
estabiliza no valor de:
a. 6,00 atm – O ar atmosférico é uma mistura gasosa.
Cem litros (100 L) desta mistura contêm,
b. 3,00 atm aproximadamente: 78,084% de N2; 20,948%
c. 2,60 atm de O2; 0,934% de Ar; 0,032% de CO2 e 0,002%
d. 2,50 atm de outros gases.
e. 2,17 atm – Devido aos efeitos da poluição, outros cons-
tituintes podem ser encontrados, tais como
355. ITA-SP
poeira, fumaça e dióxido de enxofre.
Dois compartimentos, 1 e 2, têm volumes iguais
e estão separados por uma membrana de palá- – Para a separação de gases de uma mistura,
dio, permeável apenas à passagem de hidrogê- utiliza-se o processo de liquefação, seguido de
nio. (Inicialmente, o compartimento 1 contém uma destilação fracionada. Este procedimento
hidrogênio puro (gasoso) na pressão P(H2,puro) é empregado, por exemplo, na obtenção de O2
= 1 atm, enquanto o compartimento 2 contém utilizado nos hospitais.
uma mistura de hidrogênio e nitrogênio, ambos – Massas atômicas: O = 16; C = 12.
no estado gasoso, com pressão total P(mist) =
(P(H2)+ P(N2)) = 1 atm. Após o equilíbrio termo- Com base nesses dados, é correto afirmar que:
dinâmico entre os dois compartimentos ter sido 01. A liquefação é um processo físico e
atingido, é correto afirmar que: pode ser obtida com o aumento de
a. P(H2,puro) = 0. pressão do sistema.
b. P(H2,puro) = P(N2,mist). 02. Considerando-se um balão contendo
c. P(H2,puro) = P(mist). 1L de ar atmosférico a temperatura am-
biente, a pressão parcial do N2 é menor
d. P(H2,puro) = P(H2,mist). que a pressão parcial do O2.
e. P(compartimento 2) = 2 atm. 04. Na mesma temperatura e pressão, vo-
356. UFAL lumes iguais de N2 e O2 irão conter o
As proposições a seguir referem-se ao estudo mesmo número de moléculas.
geral dos gases. Julgue-as (V ou F). 08. A 0 °C e 1 atm (CNTP), o volume molar
(  ) Sob pressão constante, o volume de de 44g de CO2 é 44,8L.
uma amostra de gás é diretamente pro- 16. A presença de poluentes sólidos faz
porcional à sua temperatura com que a mistura homogênea se
(  ) A volume constante, para determinada transforme em heterogênea.
massa de gás, a pressão é função linear
Some os números dos itens corretos.
PV-13-14

da temperatura. Tal afirmação refere-


-se à lei de Boyle. 358. Unifenas-MG
(  ) No ar atmosférico comum existem Qual a pressão parcial do oxigênio que chega
aproximadamente, 78%, em volume, aos pulmões de um indivíduo, quando o ar ins-
de nitrogênio. Logo, nessa mistura ga- pirado está sob pressão de 740 mm Hg? Admita
sosa a fração em mol do nitrogênio é que o ar contém 20% de oxigênio (O2), 78% de
igual a 78. nitrogênio (N2) e 12% de argônio (Ar) em mols.
(  ) Para gases, a P e T constantes, V = k a. 7,4 mm Hg
· (N), em que V = volume, N = núme-
ro de moléculas e k = uma constante. b. 148,0 mm Hg
Esta é a expressão matemática da lei c. 462,5 mm Hg
de Avogadro. d. 577,0 mm Hg
(  ) Na equação geral dos gases, P · V = n · R · T, e. 740,0 mm Hg
R é denominado constante de Avogadro.

132
Química geral e estequiometria Química

359. UCS-RS b.
Considerando que o ar atmosférico apresenta

Pressão
cerca de 22,4% de oxigênio em mol, conclui-se
que o número de mols de oxigênio gasoso em
1,0 L de ar, nas CNTP, é:
a. 1,0 · 10–2 Tempo de adição
b. 1,0 · 10–1 c.
c. 1,0 · 100

Pressão
d. 1,0 · 102
e. 1,0 · 103
360. Fuvest-SP
Tempo de adição
Na respiração humana, o ar inspirado e o ar
expirado têm composições diferentes. A ta-
bela a seguir apresenta as pressões parciais, d.
em mmHg, dos gases da respiração em de-

Pressão
terminado local.

Gás Ar inspirado Ar expirado


Tempo de adição
Oxigênio 157,9 115,0
e.
Dióxido de
0,2 x
carbono
Pressão

Nitrogênio 590,2 560,1

Argônio 7,0 6,6


Tempo de adição
Vapor d’água 4,7 46,6

Legenda
Qual é o valor de x, em mmHg?
Pressão total
a. 12,4 d. 56,5 Pressão de CO2
b. 31,7 e. 71,3
Pressão de NO2
c. 48,2
Pressão de N2
361. UFPE
PV-13-14

Em um recipiente fechado de volume constan- 362. PUC-SP


te, contendo 0,5 mol de CO2 e 0,2 mol de NO2, Uma mistura gasosa, constituída por 14 g de ni-
adiciona-se N2 até completar 0,3 mol. Identifi- trogênio (N2) e 9 g de hidrogênio (H2), está num
que, dentre os gráficos a seguir, o que melhor recipiente fechado, exercendo uma pressão de
representa o que acontece com as pressões total 1,0 bar. As pressões parciais de nitrogênio e de
e parciais no interior do recipiente durante a adi- hidrogênio exercidas nas paredes desse reci-
ção do nitrogênio. piente são, respectivamente: (H = 1; N = 14)
a. a. 0,5 bar e 0,5 bar.
b. 1,0 bar e 1,0 bar.
Pressão

c. 0,1 bar e 0,9 bar.


d. 0,61 bar e 0,39 bar.
Tempo de adição e. 0,75 bar e 0,25 bar.

133
Química Química geral e estequiometria

363. UFPE b. A pressão total do biogás é igual a 30 atm.


O ideal é que a pressão parcial do oxigênio no c. As pressões parciais de CH4 e CO2, no
pulmão seja de 0,20 atm. Um mergulhador, biogás, são iguais a 9,84 atm.
que está sujeito a altas pressões, deve dosar o d. O volume parcial do H2S, no biogás, é
gás que respira para manter a pressão parcial igual a 40 litros.
do oxigênio neste valor. Se ele estiver mergu-
lhando a uma profundidade onde a pressão 367. FCMSC-SP
seja de 2,5 atm, qual deve ser a fração molar Admitindo que o ar inspirado por um indivíduo
de oxigênio numa mistura oxigênio/nitrogênio contenha 78% de nitrogênio, 21% de oxigênio e
para que ele possa respirar sem dificuldades? 1% de argônio (% em mols), qual a pressão par-
364. EFEI-MG cial de oxigênio que atinge os pulmões quando
o ar inspirado está sob pressão de 1,0 atm?
Um mergulhador, numa profundidade de 76.000
metros, está sob uma pressão de 8,380 atmos- a. 1,0 atm d. 0,21 atm
feras. Qual deve ser a porcentagem em mol de b. 0,50 atm e. 0,10 atm
oxigênio em um cilindro de mergulho, para que c. 0,42 atm
na profundidade mencionada a pressão parcial
na mistura seja 0,210 atm, a mesma que no ar a 368. UFPR
1,000 atm? Admita que o cilindro contenha so- Uma mistura gasosa formada por 44 g de CO2 e
mente nitrogênio e oxigênio (N2 e O2). 6 g de H2 está contida num recipiente, a 27 °C e
Dado: lei das pressões parciais de Dalton: pressão a 8,2 atm. Qual é o volume parcial de
pi = xi · P(total) (xi = fração molar do i-ésimo H2 nessa mistura?
componente) (H = 1; C = 12; O = 16)
365. FEI-SP 369. UEL-PR
Relativamente a 100 g de uma mistura gasosa Um cilindro com volume constante igual a 1 L
que contém 64% O2 e 36% H2 em massa, a 27°C e a 25°C contém, inicialmente, no seu interior,
e 1 atm, assinale a alternativa correta: 0,2 mol de argônio e 0,8 mol de nitrogênio ga-
Dados: massa atômica (uma) soso (mistura 1). Em um determinado momen-
O = 16; H = 1 to, foi adicionado, no interior do cilindro, a cada
1 minuto até completar 3 minutos, 0,2 mol de
R = 0,082 atm · L/mol · K
acetileno, originando as misturas 1.1, 1.2 e
a. a mistura ocupa um volume de 72,35 L. 1.3, respectivamente.
b. a mistura apresenta composição molar Dados: Constante dos gases (R): 0,082 atm · L/
10% O2 e 90% H2. mol · K; Equação geral dos gases: P · V = n · R · T
c. a massa molecular média da mistura é
Com base no texto e nos conhecimentos sobre
34.
gases, considere as afirmativas a seguir.
PV-13-14

d. a pressão parcial do O2 na mistura é


I. A pressão parcial do argônio no cilindro
0,64 atm.
da mistura 1 é maior que a sua pressão
e. o número de mols da mistura é 2,94. parcial na mistura 1.1.
366. UESC modificado II. A pressão parcial do gás nitrogênio no
O biogás, mistura gasosa de CH4, CO2 e H2S, cilindro da mistura 1.1 é menor que a
fonte alternativa de energia, resulta da de- sua pressão parcial na mistura 1.3.
composição anaeróbica de restos de vegetais III. A pressão parcial do gás acetileno no ci-
e de dejetos de animais. lindro na mistura 1.3 é três vezes maior
Considerando-se que 100 litros de biogás con- que na mistura 1.1.
têm 40 mols de CH4, 40 mols de CO2 e 20 mols IV. A pressão total no interior do cilindro
de H2S a 27 °C, pode-se afirmar: após os três minutos da primeira adi-
a. A fração, em mols, de H2S no biogás é ção do gás acetileno, é aproximada-
igual a 25%. mente, 39,1 atm.

134
Química geral e estequiometria Química

Assinale a alternativa correta. em CNTP. As pressões parciais de H 2 e O 2,


a. Somente as afirmativas I e II são corretas. em atm, são, respectivamente:
b. Somente as afirmativas I e IV são corretas. Dado: R = 0,082 atm · L · mol–1 · K–1
c. Somente as afirmativas III e IV são a. 1,0 e 1,0.
corretas. b. 3,0 e 4,5.
d. Somente as afirmativas I, II e III são c. 0,8 e 1,2.
corretas. d. 1,0 e 1,5.
e. Somente as afirmativas II, III e IV são e. 2,0 e 3,0.
corretas.
373. Mackenzie-SP
370. FEI-SP
Uma mistura de 1,5 mol de gás carbônico, 8 g
A mistura gasosa ciclopropano-oxigênio pode de metano e 12 · 1023 moléculas de monóxido
ser usada como anestésico. Sabendo-se que de carbono está contida em um balão de 30 li-
as pressões parciais do ciclopropano C3H6 e tros a 27 °C. Podemos afirmar que:
do oxigênio O2 na mistura são, respectiva-
Dado: R = 0,082 atm · L · mol–1 · K–1
mente, iguais a 160 mmHg e 525 mmHg, a re-
lação entre suas correspondentes massas é: a. a pressão parcial do CO é o dobro da
do CH4.
a. 160/525
b. a pressão parcial do CH4 é o triplo da
b. 42/32
do CO2.
c. 2/5 c. a pressão parcial do CO2 é 1/4 da do CO.
d. 160/685 d. a pressão parcial do CO é o quádruplo
e. 2/7 da do CH4.
Massas molares: e. a pressão total é igual a 4 atm.
C3H6 = 42 g/mol 374. UEL-PR
O2 = 32 g/mol Considere a mistura de 0,5 mol de CH4 e 1,5
mol de C2H6, contidos num recipiente de 30,0
371. Uespi
litros, a 300 K.
Uma criança com severa infecção nos brôn-
A pressão parcial do CH4, em atmosfera, é
quios apresenta problemas respiratórios, e o igual a:
médico administra “heliox”, uma mistura de
oxigênio e hélio com 90,0% em massa de O2. a. 1,0
Se a pressão atmosférica é igual a 1 atm, cal- b. 0,82
cule a pressão parcial de oxigênio que foi ad- c. 0,50
ministrada à criança.
PV-13-14

d. 0,41
Dados: massas molares em g · mol−1: He = 4; e. 0,10
O = 16
375. UEL-PR
a. 0,53 atm
Considere a mistura de 0,5 mol de CH4 e 1,5
b. 0,60 atm mol de C2H6, contidos num recipiente de 30,0
c. 0,69 atm litros, a 300 K.
d. 0,75 atm O número total de moléculas no sistema é:
e. 0,82 atm a. 2,0
372. FEI-SP b. 2,0 · 1023
Num recipiente de 44,8 litros, mantido a c. 6,0 · 1023
273 K, foram misturados 4 mols do gás hi- d. 9,0 · 1023
drogênio (H 2) e 6 mols do gás oxigênio (O2) e. 1,2 · 1024

135
Química Química geral e estequiometria

376. PUC-SP 378. ITA-SP


Um recipiente de 82 L de capacidade contém Em um recipiente está contida uma mistura de
6 g de hidrogênio e 44 g de gás carbônico, à 5,6 g de N2(g) com 6,4 de O2(g). A pressão total
temperatura de 127 °C. As pressões parciais da mistura é de 2,5 atm. Nessas condições, a
do hidrogênio e do gás carbônico, em atm, pressão parcial do N2 na mistura é:
são, respectivamente: (Dado: N = 14; O = 16)
Dados: H = 1; C = 12; O = 16; R = 0,082 atm · L · a. 0,2/0,4 · 2,5 atm
mol–1 · K–1 b. 0,4/0,2 · 2,5 atm
a. 0,2 e 1,4. c. 0,2 · 2,5 atm
b. 1,2 e 0,4. d. 0,4 · 2,5 atm
c. 0,5 e 1,5. e. (0,2 + 0,4) · 2,5 atm
d. 3,0 e 1,0.
379. Fuvest-SP
e. 0,8 e 0,8.
Os humanos estão acostumados a respirar ar
377. Unesp com pressão parcial de O2 próxima de 2,1 · 104
Dois maçaricos, 1 e 2, operando sob as mes- Pa, que corresponde, no ar, a uma porcentagem
mas condições de fluxo dos gases, com as (em volume) desse gás igual a 21%. No entanto,
pressões mostradas na tabela a seguir, são uti- podem se adaptar a uma pressão parcial de O2 na faixa
lizados para a produção de calor na execução de (1 a 6) · 104 Pa, mas não conseguem sobreviver
de corte e solda em peças metálicas. se forçados a respirar O2 fora desses limites.
a. Um piloto de uma aeronave, em uma cabi-
Pressão parcial ne não pressurizada, voando a uma altitu-
Maçarico Gases na mistura relativa do gás de de 12 km, onde a pressão atmosférica é
na mistura de 2,2 · 104 Pa, poderá sobreviver se a ca-
acetileno (C2H2) 1/4 P bine for alimentada por O2 puro? Explique.
1
oxigênio (O2) 3/4 P
b. Um mergulhador no mar, a uma pro-
acetileno (C2H2)
2 ar (20% de O2 e
1/4 P fundidade de 40 m, está sujeito a uma
80% de N2)
3/4 P pressão cinco vezes maior do que na
superfície. Para que possa sobreviver,
ele deve respirar uma mistura de gás
Nestas condições de operação, observa-se He com O2, em proporção adequada.
que a temperatura da chama do maçarico 1 é Qual deve ser a porcentagem de O2,
maior do que a do maçarico 2. Essa diferença nessa mistura, para que o mergulhador
nas temperaturas das chamas dos dois maça- respire um “ar” com a mesma pressão
ricos ocorre porque: parcial de O2 existente no ar da superfí-
a. o N2 presente na mistura gasosa do ma- cie, ou seja, 2,1 · 104 Pa? Justifique.
PV-13-14

çarico 2 reage preferencialmente com Obs.: O He substitui com vantagem o N2.


o acetileno, liberando menos calor do 380. Unesp
que a reação deste com o O2.
No modelo cinético dos gases ideais, a pressão
b. o N2 presente na mistura gasosa do ma-
é o resultado da força exercida nas paredes do
çarico 2 reage preferencialmente com o
recipiente pelo choque das moléculas. As mo-
oxigênio, liberando menos calor do que
léculas são consideradas como pontos infinite-
a reação deste com o C2H2.
simalmente pequenos.
c. a entalpia de combustão do acetileno é a. Explique a lei de Dalton das pressões
menor na ausência de N2. parciais em termos do modelo cinético
d. a entalpia de combustão do acetileno é dos gases.
maior na ausência de N2. b. Usando o modelo cinético, explique por
e. a pressão parcial do oxigênio no maça- que a pressão de um gás é diretamente
rico 1 é maior do que no maçarico 2. proporcional à temperatura.

136
Química geral e estequiometria Química

381. UESC-BA 383. UEM-PR


O Conama (Conselho Nacional do Assinale o que for correto.
Meio Ambiente) resolveu definir os limi-
tes máximos para a emissão de poluentes 01. A velocidade de efusão do H2(g) é qua-
atmosféricos, como óxidos de nitrogênio, tro vezes maior que a do O2(g).
óxidos de enxofre, monóxido de carbono 02. Sabendo-se que a velocidade de difusão
e material particulado. Aprovada a resolu- de uma gás qualquer G é menor do que
ção, serão limitadas também as emissões a de um gás qualquer E, pode-se con-
geradas nos processos de combustão ex- cluir que a densidade do gás G é maior.
terna de óleo combustível, de gás natural, 04. Nas CNTP, a densidade absoluta de um
de bagaço de cana-de-açúcar e de deriva- m
gás ideal, em g/L, equivale a , em
dos da madeira, a partir da fabricação da que M = massa molar. 22 ,4
celulose, da fusão secundária de chumbo, 08. Nas CNTP, uma massa de 8,0 g de gás
da indústria de alumínio primário, da pro- He ocupará um volume de 89,6L .
dução de fertilizantes, de ácido fosfórico,
de ácido sulfúrico e de ácido nítrico, e por 16. A pressão de um gás é provocada pelas
usinas de pelotização de minério de ferro. colisões das moléculas desse gás com
Disponível em:<http//noticias.terra.com.br/
as paredes de seu recipiente.
ciencia/interna>. Acesso: 3 jan. de 2007. 384. UEG-GO
A velocidade com a qual cada gás presente na As moléculas de ozônio (O3) são um filtro natu-
mistura de NO, SO3, CH4 e CO2 difundirá no am- ral que diminui a incidência de raios ultraviole-
biente depende diretamente de: ta no planeta.
a. volume molar. Nesse contexto:
b. densidade. Dado: número de Avogadro = 6 · 1023
c. concentração molar. a. a energia cinética de uma molécula de
d. ponto de ebulição. ozônio, com velocidade v, é aproxima-
e. pressão atmosférica. damente 8,0 · 10–23 v2.
382. UPE b. a molécula de ozônio apresenta uma
ordem de ligação igual a dois.
Em relação à teoria cinética molecular dos ga-
c. têm-se 8 elétrons de valência na estru-
ses, é correto afirmar que:
tura de Lewis do ozônio.
a. a energia cinética média de um conjun-
d. a radiação UV apresenta maior frequên-
to de moléculas de um gás depende,
apenas e exclusivamente, das massas cia do que a luz visível.
das moléculas desse gás. 385. UEL-PR
b. quando quadruplicamos a temperatura Os gases reais obedecem à equação P·V = n·R·T:
PV-13-14

absoluta de um conjunto de moléculas


a. para qualquer valor de T.
de um gás, suas moléculas terão veloci-
dade média quadruplicada. b. para qualquer valor de V.
c. quanto maiores as interações entre as c. para qualquer valor de P.
moléculas de um gás, mais rigorosamen- d. somente a altas temperaturas e baixas
te ele se comportará como um gás ideal. pressões.
d. numa mesma temperatura, indepen- e. somente a baixas temperaturas e altas
dentemente das massas molares de pressões.
cada gás, as moléculas têm energias
cinéticas médias iguais. 386. Fuvest-SP
e. as colisões entre moléculas de um gás A velocidade com que um gás atravessa uma
perfeito com as paredes do recipien- membrana é inversamente proporcional à raiz
te que as contém são inelásticas para quadrada de sua massa molar. Três bexigas
qualquer tipo de gás ideal. idênticas, feitas com membrana permeável

137
Química Química geral e estequiometria

a gases, expostas ao ar e inicialmente vazias, 388. UEM-PR


foram preenchidas, cada uma, com um gás di- Balões vendidos em parques e festas sobem
ferente. Os gases utilizados foram hélio, hidro- porque são preenchidos com hélio ou hidro-
gênio e metano, não necessariamente nesta
gênio. Após algumas horas, esses balões ten-
ordem. As bexigas foram amarradas, com cor-
dem a murchar, pois o gás escapa pela bor-
dões idênticos, a um suporte.
racha do balão. A esse respeito, assinale a(s)
alternativa(s) correta(s).
01. Hidrogênio e hélio escapam do balão
através de um processo chamado difu-
são de gases.
A B C 02. Se um balão fosse preenchido com hi-
Suporte drogênio e hélio, esta mistura de gases
seria homogênea.
Decorrido algum tempo, observou-se que as 04. A velocidade de efusão de gases de-
bexigas estavam como na figura. Conclui-se pende somente do meio pelo qual es-
que as bexigas A, B e C foram preenchidas, res- ses gases efundem.
pectivamente, com:
08. A densidade absoluta de um gás pode
Dados: massas molares (g/mol): ser expressa como sendo a razão entre
H = 1,0; He = 4,0; C = 12 a sua massa molar em gramas e 22,4 li-
tros, nas CNTP.
Massa molar média do ar = 29 g/mol
16. Gás sulfídrico, um gás tóxico, por ser mais
a. hidrogênio, hélio e metano. denso que o ar, acumula-se junto ao solo
b. hélio, metano e hidrogênio. quando escapa de seu recipiente.
c. metano, hidrogênio e hélio. 389. UEL-PR
d. hélio, hidrogênio e metano. Utilizando-se uma “bomba de vácuo”, pode-
e. metano, hélio e hidrogênio. -se reduzir a pressão dentro de um recipien-
387. UEL-PR te fechado até valores próximos de zero. Um
recipiente de capacidade igual a 30 litros,
Três extintores de fogo (cilindros metálicos) evacuado, foi aberto ao ar contendo SO2(g)
de volumes, respectivamente, 20, 40 e 80 e CO(g), à temperatura de 27 °C e pressão
litros contêm dióxido de carbono (CO2), à de 9,2 · 104Pa, a fim de retirar uma amostra
mesma temperatura. Quando fechados, o para análise. Com esse procedimento, quan-
manômetro desses cilindros acusa pressões, tos mols desse ar poluído serão coletados?
respectivamente, de 20, 10 e 5 atm. Sendo (1 atm = 105Pa)
assim, pode-se afirmar que:
PV-13-14

a. 1,1 mol
a. os três cilindros contêm massas iguais
b. 2,2 mol
de CO2.
c. 3,3 mol
b. os três cilindros contêm massas dife-
rentes de CO2. d. 4,0 –mol
c. o cilindro de menor volume contém e. 5,0 mol
menor massa de CO2. 390. UEL-PR
d. as massas de CO2 nos cilindros são in- Comparando-se os sistemas representados
versamente proporcionais às pressões. a seguir, de mesmo volume e à mesma tem-
e. as massas de CO 2 nos cilindros são peratura, em que P = pressão, conclui-se que
inversamente proporcionais aos vo- aquele que contém maior massa de gás é:
lumes. (H = 1; He = 4; N = 14; O = 16)

138
Química geral e estequiometria Química

(Massas molares em g/mol: H = 1; N = 14, C =


3P
a. 12; O = 16; Cl = 35,5)
Recomendação 1: dirigir-se para o lugar mais
alto possível.
He Recomendação 2: situar-se em depressões ou
lugar mais baixo possível.
Recomendação 1 Recomendação 2
a. Cloro Amônia
2P
b. Metil isocianato Dióxido de carbono
b. c. Amônia Metano
d. Propano Metil isocianato
O2 e. Butano Propano
392. PUC-SP modificado
Três balões de borracha idênticos, designados
por A, B e C, foram inflados com gases dife-
P rentes, obtendo-se balões de mesmo volume,
c.
com a mesma pressão interna e sob a mesma
temperatura.
O gás utilizado para encher o balão A foi o
O3 hélio (He), para o balão B foi o argônio (Ar), e
para o C foi o dióxido de carbono (CO2).
Dados: He = 4; Ar = 36; C = 12; O = 16; Mar = 29.
1P Com relação aos balões, é incorreto afirmar que:
d. 2 a. apenas o balão A ascenderá no ar, enquan-
to que os balões B e C ficarão no chão.
b. logo após encher os balões, a quantida-
de de moléculas no estado gasoso no
H2 balão A será maior do que a encontrada
nos balões B e C.
c. logo após encher os balões, a massa do
balão A será menor do que a de B que
1P será menor do que a de C.
e. 3
d. os gases utilizados para encher os ba-
PV-13-14

lões A e B são classificados como gases


nobres devido à sua inércia química.
NH3 393. UFAL
A equação de estado de um gás ideal, PV = nRT,
pode ser usada para descrever o comporta-
391. UFPI mento aproximado de um gás real submetido a:
A maioria dos gases industriais, exceto hidro- a. alta pressão e baixa temperatura.
gênio, amônia e metano, são mais densos que b. baixa pressão e alta temperatura.
o ar. Considerando essa informação, em caso c. alta pressão e alta temperatura.
de acidente com liberação de gases na estra-
da, assinale a opção que associa a orientação d. baixa pressão e baixa temperatura.
correta para as pessoas próximas ao local do e. qualquer condição de pressão e tem-
acidente com a natureza do gás envolvido. peratura.

139
Química Química geral e estequiometria

394. UEG-GO Esse fato deve-se à:


Considere a figura abaixo e responda ao que a. massa molecular dos gases envolvidos.
se pede. b. massa dos recipientes e dos gases.
Vácuo
c. energia cinética média das moléculas.
NH3(g) d. quantidade de moléculas dos gases.
397. UFG-GO
O processo de enriquecimento de urânio pas-
sa pela separação de hexafluoretos de urânio,
CO2(g) UF6, que são constituídos por diferentes isóto-
pos de urânio. As velocidades de efusão desses
Pequeno orifício hexafluoretos são muito próximas, sendo que
a razão entre a velocidade de efusão do hexa-
a. Calcule a diferença de velocidade com fluoreto que contém o isótopo de urânio mais
que as moléculas de gás irão atravessar leve em relação ao que contém o mais pesado
o orifício. é de 1,0043. De acordo com a lei de efusão de
Graham, essa razão é igual à raiz quadrada da
b. Comente acerca do caráter ácido e/ou
relação inversa de suas massas molares.
básico de soluções aquosas obtidas pelo
borbulhamento dos gases acima em re- Sendo a massa molar da substância que contém
cipientes contendo água deionizada. o isótopo de urânio mais leve igual a 349 g/mol,
calcule a massa atômica do isótopo mais pesado.
395. UEM-PR
398. FASP
A respeito dos gases e das leis que regem o seu
comportamento, assinale o que for correto. Qual é o gás mais leve, depois do hidrogênio,
que pode ser, por exemplo, usado em balões
01. A pressão parcial de um gás em uma mis- por ser não inflamável ou utilizado para substi-
tura é medida por meio de sua fração em tuir o nitrogênio quando o ar precisa ser respi-
quantidade de matéria multiplicada pela rado a alta pressão?
pressão total.
a. Argônio
02. Em um recipiente fechado contendo um
gás, a energia pode ser transferida entre b. Lítio
as moléculas do gás durante as colisões, c. Hélio
mas a energia cinética média do gás não d. Oxigênio
varia, considerando-se a temperatura e. Cloro
constante.
399. ITA-SP
04. A lei de difusão de gases de Graham con-
sidera que, se existirem dois gases sob Considere dois cilindros idênticos (C1 e C2), de
condições idênticas, o de maior massa paredes rígidas e indeformáveis, inicialmente
PV-13-14

evacuados. Os cilindros C1 e C2 são preen-


molar funde-se mais rapidamente.
chidos, respectivamente, com O2(g) e Ne(g) até
08. Os gases reais tendem a se comportar atingirem a pressão de 0,5 atm e temperatura
como gases ideais a altas pressões. de 50 °C. Supondo comportamento ideal dos
16. Qualquer mistura de gases pode ser con- gases, são feitas as seguintes afirmações:
siderada como uma mistura homogênea. I. O cilindro C1 contém maior quantidade
396. Unimontes-MG de matéria que o cilindro C2 .
Dois frascos idênticos encontram-se cheios, cada II. A velocidade média das moléculas no
um deles com gás a 0 °C. Um frasco contém 1,0 cilindro C1 é maior que no cilindro C2.
mol de dióxido de carbono, CO2, o outro, 1,0 mol III. A densidade do gás no cilindro C1 é maior
de neônio, Ne. Comparando o comportamento que a densidade do gás no cilindro C2.
ideal dos gases, observou-se que as moléculas IV. A distribuição de velocidades das mo-
de neônio apresentaram um maior número de léculas contidas no cilindro C1 é maior
colisões por unidade de tempo. que a das contidas no cilindro C2.

140
Química geral e estequiometria Química

Assinale a opção que apresenta a(s) afirmação(ões) correta(s).


a. Apenas I e III.
b. Apenas I e IV.
c. Apenas II.
d. Apenas II e IV.
e. Apenas III.
400. Fuvest-SP

Uma estudante de Química realizou um experimento para investigar as velocidades de difusão


dos gases HCl e NH3 .
Para tanto, colocou, simultaneamente, dois chumaços de algodão nas extremidades de um tubo
de vidro, como mostrado na figura acima. Um dos chumaços estava embebido de solução aquosa
de HCl(g), e o outro, de solução aquosa de NH3(g). Cada um desses chumaços liberou o respectivo
gás. No ponto de encontro dos gases, dentro do tubo, formou-se, após 10 s, um anel de sólido
branco (NH4Cl), distante 6,0 cm do chumaço que liberava HCl(g).
a. Qual dos dois gases, desse experimento, tem maior velocidade de difusão? Explique.
b. Quando o experimento foi repetido a uma temperatura mais alta, o anel de NH4Cl(s) se
formou na mesma posição. O tempo necessário para a formação do anel, a essa nova
temperatura, foi igual a, maior ou menor do que 10 s? Justifique.
c. Com os dados do experimento descrito, e sabendo-se a massa molar de um dos dois ga-
ses, pode-se determinar a massa molar do outro. Para isso, utiliza-se a expressão:
velocidade dedifusão do NH3(g) massa molar do HCl
=
velocidade dedifusão do HCl(g) massa molar do NH3

Considere que se queira determinar a massa molar do HCl. Caso o algodão embebido de solução
aquosa de NH3(g) seja colocado no tubo um pouco antes do algodão que libera HCl(g) (e não simul-
PV-13-14

taneamente), como isso afetará o valor obtido para a massa molar do HCl? Explique.

141
Química Química geral e estequiometria

CAPÍTULO 05
401. UESPI Nos pratos da esquerda de cada balança são
A Teoria Atômica de Dalton pode ser expressa adicionados pesos de modo que os pratos fi-
em quatro postulados: cam em equilíbrio (mesmo peso) antes da
queima. Considere que as balanças estão ex-
1. Cada elemento é composto por partí- postas às condições ambiente e que nenhum
culas extremamente pequenas, deno- produto sólido escapa dos pratos após a
minadas átomos. queima. Assuma, ainda, que o papel seja um
2. Os átomos de um mesmo elemento são polímero cuja unidade polimérica é um mo-
idênticos entre si; os átomos de ele- nômero de fórmula mínima (C6H10O5)n e que
mentos diferentes têm propriedades a esponja de aço utilizada seja constituída uni-
diferentes. camente de átomos de ferro.
3. Nas reações químicas, os átomos de um Assinale a alternativa que apresenta o compor-
elemento não se transformam em outros tamento qualitativo mais provável da balança
tipos de átomos; nestas reações, não há após a queima de cada material:
nem criação nem destruição de átomos.
4. Os compostos se formam quando áto-
a.
mos de dois ou mais elementos se com-
binam; um certo composto tem sempre Restos
a mesma espécie de átomos e o mesmo Restos
da queima
número relativo de átomos. da esponja
da queima
do papel
A partir destes postulados, Dalton deduziu a
chamada:
a. Lei da Conservação da Massa.
b. Lei da Composição Constante.
c. Lei das Proporções Múltiplas. b.
Restos Restos
da queima da queima
d. Lei das Decomposições Radioativas. do papel da esponja
e. Lei da Quantização da Energia.
402. UEPB
Combustão
Um professor realizou um experi- c.
mento em um laboratório no qual ele quei-
mou papel amassado e esponja de aço,
PV-13-14

Restos Restos
respectivamente, em balanças artesanais, da queima da queima
conforme figura. do papel da esponja

Papel Esponja
de aço
d. Restos
da queima
do papel Restos
da queima
da esponja

Balanças contendo papel amassado


e esponja de aço, respectivamente,
e. Nenhuma das alternativas, pois a es-
antes da queima desses materiais.
ponja de aço não queima.

142
Química geral e estequiometria Química

403. UEL-PR
Com papel Com palha de aço
Provoca-se reação da mistura formada por 10,0
g de hidrogênio e 500 g de cloro. Após a rea- a. A e B no mesmo nível A e B no mesmo nível
ção, constata-se a presença de 145 g de cloro b. A abaixo de B A abaixo de B
remanescente, junto com o produto obtido. A
massa, em gramas, da substância formada é: c. A acima de B A acima de B
a. 155 d. 490 d. A acima de B A abaixo de B
b. 290 e. 510
e. A abaixo de B A e B no mesmo nível
c. 365
404. UEL-PR 408. Unesp
46,0 g de sódio reagem com 32,0 g de oxigênio Quando um objeto de ferro enferruja ao ar, sua
formando peróxido de sódio. Quantos gramas massa aumenta. Quando um palito de fósforo
de sódio são necessários para se obter 156 g é aceso, sua massa diminui. Essas observações
de peróxido de sódio? violam a lei da conservação das massas? Justi-
a. 23,0 d. 78,0 fique sua resposta.
b. 32,0 e. 92,0 409. Fatec-SP
c. 69,0 A queima de uma amostra de palha de aço
produz um composto pulverulento de massa:
405. Unesp
a. menor que a massa original de palha
Aquecendo-se 21 g de ferro com 15 g de enxo- de aço.
fre obtém-se 33 g de sulfeto ferroso, restando
b. igual à massa original da palha de aço.
3 g de enxofre. Aquecendo-se 30 g de ferro
com 16 g de enxofre obtém-se 44 g de sulfeto c. maior que a massa original da palha de
ferroso, restando 2 g de ferro. aço.
Demonstre que esses dados obedecem às d. igual à massa de oxigênio do ar que
leis de Lavoisier (conservação da massa) e de participa da reação.
Proust (proporções definidas). e. menor que a massa de oxigênio do ar
que participa da reação.
406. FGV-SP
Ao dissolver-se um comprimido efervescen- 410. Mackenzie-SP
te em uma dada massa de água, ao término A tabela a seguir, com dados relativos à equa-
do processo observa-se uma diminuição da ção citada, refere-se a duas experiências reali-
massa do conjunto. A referida observação zadas. Então, podemos afirmar que:
contraria a lei de Lavoisier? Justifique a sua
PV-13-14

resposta. C + O2 → CO2

407. Fuvest-SP 1a experiência 12 g 32 g Xg


Os pratos A e B de uma balança foram equili- 2a experiência 36 g Yg 132 g
brados com um pedaço de papel em cada pra-
to e efetuou-se a combustão apenas do mate- a. X é menor que a soma dos valores das
rial contido no prato A. Esse procedimento foi massas dos reagentes da 1a experiência.
repetido com palha de aço em lugar de papel.
Após cada combustão, observou-se: b. X = Y
c. Y é igual ao dobro do valor da massa de
A B carbono que reage na 2a experiência.
d. 32/Y = X/132
e. Y = 168

143
Química Química geral e estequiometria

411. UFTM-MG 1. Lei de Lavoisier: A soma das massas dos


Considere as três afirmações: reagentes é igual à soma das massas
dos produtos.
I. O íon de sódio tem número de carga
igual a +1. 2. Lei de Proust: A proporção das massas
que reagem permanece constante.
II. Sais de estrôncio são empregados em
rojões sinalizadores que emitem luz 3. Lei de Dalton: Mudando-se a reação,
vermelha. se a massa de um reagente permanece
constante, a massa do outro reagente só
III. Há conservação de massa em uma pode variar segundo valores múltiplos.
transformação química.
Considerando a reação N2 + O2 → NxOy:
A teoria atômica de Dalton permite explicar o
que se afirma apenas em: a. demonstre a lei de Lavoisier para a for-
mação de 46 g do produto;
a. I.
b. demonstre a lei de Proust, consideran-
b. II. do duas reações químicas, em que a
c. III. massa de O2, que reagiu completamen-
d. I e II. te, mudou de 64 para 128 g;
e. II e III. c. preencha o quadro de modo a demons-
trar a lei de Dalton.
412. UECE 1/2 N2 + O2 → NO2
A análise de 3 amos- termos aproximados, ------------------ 32 g ----------------
tras de um líquido os dados contidos na ----------------- + ---------------- → N2O2
incolor revelou, em tabela a seguir.
----------------- ----------------- -----------------

----------------- + ----------------- → ----------------


Massa de Massa de 42 g
Amostra ---------------- ----------------
carbono hidrogênio
414. Cefet-PR
1 8,62 g 1,64 g A partir do final do século XVII,
com o desenvolvimento de balanças de
2 13,35 g 2,54 g maior precisão, é que os químicos pude-
3 8,52 g 1,62 g ram testar velhas teorias sobre a compo-
sição da matéria. O principal teste veio
com a derrubada da ideia de que a maté-
Com base na tabela, considere a seguin- ria poderia ser destruída, ou criada, uma
te afirmação: “De acordo com a Lei de vez que quando se queima um pedaço de
________________, o material analisado é madeira o que se observa após a reação
PV-13-14

_______________”. é apenas uma massa menor de cinzas, ou


quando se aquece um pedaço de ferro,
Os termos que preenchem de maneira ade- a massa resultante é maior do que an-
quada as lacunas da afirmação anterior são, tes. Esta crença foi derrubada principal-
respectivamente: mente pelo trabalho do francês Antoine
a. Lavoisier e substância. L. de Lavoisier, que após meticulosos
b. Proust e substância. trabalhos utilizando a balança de maior
precisão existente na época, conclusiva-
c. Dalton e mistura. mente, provou que um tipo de matéria
d. Richter e mistura. se transformava em outro. Ou seja, par-
413. UFG-GO te da madeira era transformada em outro
tipo de matéria que não se podia medir
As leis ponderais referem-se às massas de em recipiente aberto, mas se o recipien-
substâncias e elementos. Essas leis são: te fosse mantido fechado, a massa dessa
matéria poderia ser medida.

144
Química geral e estequiometria Química

Outras leis foram elaboradas basea- 40,0 g de Cálcio, são produzidas 56 g


das no uso intensivo de balanças precisas. de Óxido de Cálcio.
Dentre estas, pode-se citar a lei de Proust, III. a Lei das Proporções Definidas, se
Lei de Dalton e Lei de Richte Wenzel. 1,0 g de Ferro reage com 0,29 g de
Destas, a mais conhecida é a lei de Proust. Oxigênio para formar o composto
A tabela a seguir indica as massas que reagiram Óxido Ferroso, 2,0 g de Ferro reagirão
e foram medidas com bastante precisão e re- com 0,87 g de Oxigênio, produzindo o
ferem-se a uma reação entre duas substâncias, mesmo composto.
A e B, que geraram uma terceira substância, C. IV. a Lei das Proporções Múltiplas, dois
mol de Ferro reagem com dois mol de
Massa de A (g) Massa de B (g) Massa de C (g) Oxigênio para formar Óxido Ferroso;
logo, dois mol de Ferro reagirão com
0,117 0,321 0,438 três mol de Oxigênio para formar Óxi-
0,234 0,642 0,876
do Férrico.
Assinale a opção correta.
0,0167 0,0459 0,4757
a. As afirmativas I e II estão corretas.
b. A afirmativa II está correta.
Pode-se afirmar que ao reagir 1 grama de A: c. As afirmativas II e III estão corretas.
a. consomem-se 3,21 gramas de B. d. As afirmativas II e IV estão corretas.
b. formam-se 1,876 gramas de C. e. A afirmativa III está correta.
c. consomem-se 4,59 gramas de B. 416. Fuvest-SP
d. formam-se 2,74 gramas de C. Maçaricos são queimadores de gás utilizados
para produzir chamas de elevadas tempera-
e. consomem-se 2,74 gramas de B.
turas, como as requeridas para soldar metais.
415. UFF-RJ Um gás combustível, muito utilizado em ma-
çaricos, é o acetileno, C2H2, sendo que a sua
Desde a Antiguidade, diversos povos obtive- combustão pode ser promovida com ar atmos-
ram metais, vidro, tecidos, bebidas alcoóli- férico ou com oxigênio puro.
cas, sabões, perfumes, ligas metálicas, desco-
briram elementos e sintetizaram substâncias a. Escreva a equação química balanceada
que passaram a ser usadas como medicamen- da combustão completa do acetileno
tos. No século XVIII, a Química, a exemplo da com oxigênio puro.
Física, torna-se uma ciência exata. Lavoisier b. Em uma oficina de solda, existem
iniciou na Química o método científico, estu- dois cilindros idênticos, um deles
dando os porquês e as causas dos fenôme- contendo oxigênio puro (cilindro A)
nos. Assim, descobriu que as transformações e o outro, ar atmosférico (cilindro
PV-13-14

químicas e físicas ocorrem com a conserva- B). Sabendo que, no interior dos dois
ção da matéria. Outras leis químicas também cilindros, as condições de pressão e
foram propostas e, dentre elas, as ponderais, temperatura são as mesmas, qual dos
ainda válidas. dois cilindros contém a maior massa
Com base nas leis ponderais, pode-se afirmar gasosa? Explique.
que, segundo: c. A temperatura da chama do maçarico
é maior quando se utiliza a mistura de
I. a Lei da Conservação da Massa (Lavoi- oxigênio e acetileno do que quando
sier), 1,0 g de Ferro, ao ser oxidado pelo se usa a mistura de ar atmosférico
Oxigênio, produz 1,0 g de Óxido Férrico. e acetileno, mesmo estando os re-
II. a Lei da Conservação da Massa, ao agentes em proporção estequiomé-
se usar 16,0 g de Oxigênio molecu- trica nos dois casos. Considerando
lar para reagir completamente com as substâncias gasosas que recebem
o calor liberado na combustão, em

145
Química Química geral e estequiometria

cada caso, explique essa diferença de utilizada, é menor do que o consumido pela
temperatura. gramínea durante todo o seu crescimento.
VARGAS, 2010, p. 112.
Massa molar g · mol–1
A partir das informações do texto sobre o
O2 32 aproveitamento do capim-elefante, Pennise-
N2 28 tom purpureum, uma gramínea, na geração de
energia elétrica, é correto afirmar:
417. UFPE 01. O CO2(g), resultante da queima da bio-
massa de capim-elefante, é completa-
Massas iguais de metano, CH4, e hexa-fluoreto
de enxofre, SF6, foram introduzidas em reci- mente absorvido durante o crescimen-
pientes separados, de iguais volumes, à mes- to dessa gramínea.
ma temperatura. A massa molar do hexa-fluo- 02. A quantidade de CO2(g) produzida
reto de enxofre é maior do que a massa molar durante a combustão completa de
do metano. 1,0 kg de capim-elefante, em um forno
Na tentativa de descrever corretamente a rela- fechado, é igual à absorvida durante
ção de comportamento dos dois gases armaze- o crescimento de 1,0 kg dessa gramí-
nados nos respectivos recipientes, admitindo-se nea, em ambiente fechado, com base
comportamento ideal, podemos afirmar que: na Lei de Lavoisier.
00. ambos os recipientes contêm o mesmo 03. A energia calorífica resultante da com-
número de moléculas. bustão do capim-elefante e transporta-
01. as pressões exercidas pelos gases nos da pelo vapor de água é completamen-
dois recipientes são diferentes. te transformada em energia elétrica.
02. as quantidades de matéria dos dois ga- 04. O capim-elefante não necessita de
ses nos recipientes são diferentes. água para crescer durante o processo
03. as massas molares dos dois gases, a de fotossíntese.
uma dada temperatura e pressão, são 05. A coleta e a moagem são processos físicos
iguais. de separação, cujo objetivo é diminuir a
04. os volumes molares dos dois gases, a uma velocidade do processo endotérmico de
dada temperatura e pressão, são iguais. combustão do capim-elefante.
418. Uneb-BA 419. PUC-SP
O capim, do tipo elefante, foi impor-
tado da África há 100 anos para alimentar Querendo verificar a lei da Conservação das
o gado em períodos de estiagem. Resis- Massas (lei de Lavoisier), um estudante realizou
tente à seca e capaz de se desenvolver a experiência esquematizada abaixo:
mesmo em solos pobres, ele foi usado du- Solução de
PV-13-14

rante décadas por pecuaristas de regiões HNO3(aq) K2CO3(s)


Solução
final
Erlenmeyer
vazio
inóspitas do país.
O capim-elefante não precisa ne- (1) (2)
(1)
(1) (2)

cessariamente ser irrigado e é triturado


pela mesma máquina que o colhe. Em
seguida, o farelo é jogado sem nenhum (2)

tratamento prévio diretamente no forno 1.000 g 995 g

para esse fim. Queimado, produz vapor Balança Balança


que movimenta um gerador. A energia
resultante é transferida para uma subes- Terminada a reação, o estudante verificou que a
tação conectada à rede nacional de dis- massa final era menor que a massa inicial. Assi-
tribuição elétrica. nale a alternativa que explica o ocorrido.
A conversão de capim-elefante em a. A lei de Lavoisier só é válida nas condi-
energia não polui. Mesmo o gás carbônico, ções normais de temperatura e pressão.
CO2, emitido durante a queima da biomassa

146
Química geral e estequiometria Química

b. A lei de Lavoisier não é válida para reações Durante um experimento, realizado em re-
em solução aquosa. cipiente fechado, foi colocado para reagir
c. De acordo com a lei de Lavoisier, a massa 1,00 g do referido metal, obtendo-se 1,40 g
dos produtos é igual à massa dos reagen- do seu óxido. Considerando-se que todo o
tes, quando estes se encontram na mes- oxigênio presente no frasco foi consumido,
ma fase de agregação. pode-se determinar que a massa de oxigê-
d. Para que se verifique a lei de Lavoisier, é nio presente no sistema inicial é x.
necessário que o sistema seja fechado, o Em outro recipiente fechado, foi colocado
que não ocorreu na experiência realizada. 1,50 g do referido metal em contato com
e. Houve excesso de um dos reagentes, o 1,20 g de oxigênio. Considerando que a re-
que invalida a lei de Lavoisier. ação ocorreu até o consumo total de pelo
420. PUC-SP menos um dos reagentes, pode-se afirmar
que a massa de óxido gerado é y.
Um determinado metal queima ao ar para
formar o respectivo óxido, um sólido de alta Sabendo que o metal em questão forma ape-
temperatura de fusão. A relação entre a massa nas um cátion estável e considerando que em
do metal oxidado e a massa de óxido formado todas as reações o rendimento foi de 100%,
está representada no gráfico a seguir. os valores de x e y são, respectivamente:
a. 0,40 g e 2,70 g
200
180 b. 0,40 g e 2,50 g
160 c. 0,56 g e 2,50 g
Massa do óxido (g)

140 d. 0,56 g e 3,00 g


120
e. 0,67 g e 2,70 g
100
80
60
40
20
0
0 24 48 72 96 120
Massa do metal (g)
PV-13-14

147
Química Química geral e estequiometria

CAPÍTULO 06
421. Uncisal É correto o que se afirma somente em:
De uma amostra de 100 g de caldo de cana a. I.
submetida à secagem até massa constante, b. II.
restaram 28,0 g de matéria seca. c. III.
A quantidade de água dessa amostra de caldo d. I e II.
de cana é:
e. II e III.
a. 7,2 g
425. Uninove-SP
b. 2,8%
Considere as seguintes informações sobre o
c. 28,0%
ácido acetilsalicílico, conhecido como Aspiri-
d. 72,0% na® ou AAS®.
e. 720 mg Fórmula estrutural:
422. Unesp COOH
A porcentagem em massa de carbono no clo-
rofórmio, CHCl3, é: O CH3

(Massas molares, em g/mol: H = 1; C = 12;


Cl = 35,5) O
a. 1%
b. 10% Fórmula bruta:
c. 12% C9H8O4
d. 24% Massa molar:
e. 50%
180 g · mol–1
423. Fuvest-SP
A porcentagem, em massa, de carbono na as-
A respeito do glutamato de sódio (NaC2H8O4N) pirina é:
e cloreto de sódio (NaCl), usados em alimentos, a. 20% d. 80%
responda: b. 30% e. 90%
Dados: massas molares (g/mol): H = 1,0; C = 12,0; c. 60%
N = 14,0; O = 16,0; Na = 23,0; Cl = 35,5: 426. UFPB
a. Quantos elementos químicos diferen- O cloreto de magnésio é usado como suple-
tes constituem o sal orgânico? mento alimentar para fortalecimento dos os-
PV-13-14

b. Qual a porcentagem, em massa, de só- sos, pois o magnésio é necessário para o me-
dio contido no sal inorgânico? tabolismo de absorção do cálcio.
424. UFTM-MG A respeito do cloreto de magnésio, julgue as
Uma pessoa preparou uma mistura fertilizante afirmativas:
sólida constituída por 100 g de nitrato de po- I. A formação de 0,5 mol desse composto
tássio e 100 g de nitrato de amônio. Sobre essa requer 0,25 mol de Cl.
mistura, afirma-se que: II. A formação de 0,5 mol desse composto
I. confere cor violeta à chama do bico de requer 0,5 mol de Mg.
Bunsen. III. A massa molar desse composto é 95 g/mol.
II. origina solução aquosa fortemente bá- IV. Uma massa de 47,5 g desse composto
sica ao ser dissolvida em água. contém 12 g de Mg.
III. apresenta um teor de 90% em massa V. A composição percentual, em massa,
de nitrogênio. desse composto é 40,3% Mg e 59,7% Cl.

148
Química geral e estequiometria Química

É correto o que se afirma em: 430. Unesp


a. I, II e III. Em um experimento de combustão, 3,69 g de
b. II, III e IV. um hidrocarboneto formaram 11,7 g de dióxido
c. I, II e IV. de carbono e 4,50 g de água. Considerando as
massas molares (g · mol–1), H = 1, C = 12 e O = 16,
d. III, IV e V.
podemos afirmar que a fórmula mínima e a classi-
e. I, II e V. ficação do hidrocarboneto são, respectivamente:
427. UFMS a. CH e alcano.
A análise elementar de um hidrocarboneto, b. CH2 e alceno.
de massa molar igual a 78 g/mol, revelou c. CH3 e alcano.
a presença de 92,31% de carbono em sua
composição. Calcule o número de átomos d. C3H4 e alcino.
de carbono, presente numa molécula desse e. C3H4 e cicloalcano.
hidrocarboneto. 431. Unifesp
(Massa molar, em g/mol: C = 12; H = 1) Estanho e iodo reagem quantitativamente
428. UFOP-MG formando um produto, cuja fórmula pode
Qualquer amostra de oxigênio e hidrogênio ser determinada reagindo-se quantidades
que contenha o mesmo número de átomos conhecidas de iodo (dissolvido em um sol-
desses elementos apresenta uma proporção vente) com excesso de estanho e determi-
em massa de: nando-se a massa do metal remanescente
após a reação. Os resultados de uma expe-
a. 1:2
riência foram:
b. 2:1
massa de iodo utilizado: 5,08 g
c. 1:8
massa inicial de estanho: 4,18 g
d. 16:1
429. UEG-GO massa final de estanho: 3,00 g
A tabela abaixo mostra os porcentuais em Dadas as massas molares, em g/mol, Sn = 118
massa, obtidos da análise elementar do ácido e I = 127, pode-se concluir que a fórmula míni-
ascórbico. ma do composto obtido é:
a. SnI
Elemento químico Porcentagem (%)
b. SnI2
Carbono 40,91 c. SnI3
Hidrogênio 4,58 d. SnI4
e. SnI5
PV-13-14

Oxigênio 54,51
432. Unicamp-SP
Dado: M (ácido ascórbico) = 176,12 g · mol –1 Um balão contém 1,31 g de oxigênio gasoso,
Logo, a fórmula mínima desse composto é: O2, e outro balão, de mesmo volume, contém
1,72 g de hidrocarboneto gasoso, ambos à
a. CHO
mesma temperatura.
b. C2H2O2
Dados: H = 1, C = 12 e O = 16
c. C3H4O3
a. Qual a massa molecular do hidrocar-
d. C6H8O6 boneto?
b. Sabendo-se que a fórmula mínima do
hidrocarboneto é CH2, calcule sua fór-
mula molecular.

149
Química Química geral e estequiometria

433. A fórmula molecular do cianogênio é:


Um óxido de nitrogênio contém 56 g de nitro- a. CN
gênio e 3,6 . 1024 átomos de oxigênio. A fórmu- b. CN2
la mínima do composto é: c. C2N
Dados: massa molar (g/mol): N = 14, O = 16; d. C2N2
número de Avogadro = 6,0 · 1023 e. C3N2
a. NO 437. ITA-SP
b. NO2 Certo composto AxBy contém 9,1% em massa
c. N2O5 de A e o resto sendo de B. Se a massa atômica
d. N2O3 do elemento A for 30 e a de B for 100, pode-
e. N2O mos concluir que:
x
a. = 2
434. y
Lindano, usado como um inseticida, tem com- x 1
posição percentual em massa de 24,78% de b. =
y 2
carbono, 2,08% de hidrogênio e 73,14% de
cloro, e massa molar igual a 290,85 g · mol-1. x
c. =3
Dadas as massas atômicas dos elementos: y
C = 12, H = 1 e Cl = 35,5, a fórmula molecular
do lindano é: x 1
d. =
y 3
a. C4H5Cl2
e. X2Y5
b. C5H7Cl6
438. FCMSC-SP
c. C6H5Cl6
A fórmula mínima do sulfato duplo de alumínio
d. C6H6Cl2
e potássio hidratado é AlKS2H24O20. Quantas
e. C6H6Cl6 moléculas de água participam dessa fórmula?
435. a. 12
Um composto orgânico destilado da madeira b. 14
possui massa molar de 32,4 g · mol–1 e compo-
c. 16
sição: 37,5% de carbono, 12,6% de hidrogênio
e 49,9% de oxigênio. Dadas as massas atômi- d. 18
cas: C = 12,0 u, H = 1,01 u, O = 16,0 u e os nú- e. 20
meros atômicos: C = 6, O = 8 e H = 1: 439. Uncisal
a. determine a fórmula molecular do A cana-de-açúcar é uma plan-
composto orgânico e deduza o grupo ta composta, em média, de 65 a 75%
PV-13-14

funcional; de água, mas seu principal compo-


b. escreva a estrutura de pontos (estrutu- nente é a sacarose, que corresponde
ra de Lewis) do composto e dê o nome de 70% a 91% das substâncias sóli-
da figura geométrica em torno do áto- das solúveis. O caldo de cana con-
mo de carbono. serva todos os nutrientes da cana-
436. UFSCar-SP -de-açúcar, entre eles minerais como
ferro, cálcio, potássio, sódio, fósforo,
Cianogênio, um gás tóxico, é composto de magnésio e cloro, além de vitaminas de
46,2% de C e 53,8% de N, em massa. A 27 °C complexo B e C. A planta contém ainda
e 750 torr, a massa de 1,04 g de cianogênio glicose (de 2% a 4%), frutose (de 2% a
ocupa um volume de 0,496 L. 4%), proteínas (de 0,5% a 0,6%), ami-
Dados: massas molares em g/mol: C = 12,0 e do (de 0,001% a 0,05%), ceras e ácidos
N = 14,0; P · V = n · R · T; R ≈ 62 L · torr · mol–1 · K–1; graxos (de 0,05% a 0,015%) e corantes,
0 °C = 273 K entre 3% a 5%.

150
Química geral e estequiometria Química

Um professor solicitou aos seus alunos que es- d. A massa molecular desse composto é
crevessem a fórmula molecular da sacarose e igual a 161 g · mol–1.
forneceu os seguintes dados: e. O percentual, em massa, de carbono nes-
Análise elementar da sacarose: se composto é, aproximadamente, 73%.
C = 42,1%; H = 6,4%; O = 51,6% 441. Mackenzie-SP
O bromato de potássio, ao ser aquecido, de-
Massas molares (g.mol–1):
compõe-se em brometo de potássio e gás oxi-
Sacarose = 342; C = 12; H = 1; O = 16. gênio. A equação dessa decomposição, corre-
tamente balanceada, é:
Um estudante apresentou a seguinte fórmula
molecular da sacarose: C144H22O176. a. KBrO3 → KBr + O3
b. 2 KBrO3 → 2 KBr + 3 O2
Essa resposta é:
c. 3 KBrO3 → KBr + 3 O2
a. correta, porque indica que na molécula
de sacarose o número de átomos de hi- d. KBrO3 → KBr + O2
drogênio é 8 vezes menor que o núme- e. 2 KBrO3 → KBr + 3 O2
ro de átomos de oxigênio. 442.
b. correta, porque indica que na sacarose Acerte os coeficientes, se necessário, das
a soma das massas de hidrogênio, car- equações a seguir.
bono e oxigênio é igual à massa molar
a. ...H2CO3 + ...Ca(OH)2 → ...CaCO3 + ...H2O
da sacarose.
b. ...H3PO4 + ...Fe(OH)2 → ...Fe3(PO4)2 + ...H2O
c. incorreta, porque a fórmula molecular
da sacarose tem que descrever as mas- c. ...H2SO4 + ...Al(OH)3 → ...Al2(SO4)3 + ...H2O
sas de C, H e O presentes em 100 g de d. ...Ag2SO3 + ...HCl → ...AgCl + ...H2O + ...SO2
sacarose. 443.
d. incorreta, porque a fórmula molecular Derramaram-se algumas gotas de ácido clo-
da sacarose tem que descrever a quan- rídrico em uma pia de mármore e observou-
tidade de átomos de C, H e O presentes -se uma leve efervescência. Esse fenômeno
em uma molécula de sacarose. pode ser representado pela equação não ba-
e. incorreta, porque a fórmula molecu- lanceada:
lar da sacarose tem que descrever a CaCO3 + HCl → CaCl2 + H2O + CO2
proporção mínima entre os átomos
de C, H e O presentes em um mol de Acertando-se os coeficientes da equação com
sacarose. os menores valores inteiros, a soma será:
a. 2
440. Unir-RO
b. 3
PV-13-14

Um mol de um composto contém 72 g de c. 4


carbono, 12 mol de átomos de hidrogênio e
12 · 1023 átomos de oxigênio. A partir dessas d. 5
informações, assinale a afirmativa correta. e. 6
a. A fórmula molecular desse composto é 444. Ufla-MG
igual à sua fórmula mínima. Considere a equação química não balanceada:
b. O número total de átomos nesse com- SO2 + O2 → SO3
posto é 12 · 1024.
Efetuando-se o balanceamento, a soma dos
c. Para se produzir um mol desse com- coeficientes estequiométricos é igual a:
posto, a partir de seus constituintes, é
necessário reagir 6 mol de átomos do a. 4 d. 5
elemento carbono, 12 mol do gás hi- b. 3 e. zero
drogênio e 6 mol do gás oxigênio. c. 7

151
Química Química geral e estequiometria

445. UFMG a. 1/2


A equação Ca(OH)2 + H3PO4 → Ca3(PO4)2 + H2O não b. 2/3
está balanceada. Balanceando-a com os menores nú- c. 1
meros possíveis, a soma dos coeficientes estequiomé- d. 3/2
tricos será: e. 2
a. 4 c. 10 e. 12 451.
b. 7 d. 11 Uma das maneiras de impedir que o
SO2, um dos responsáveis pela chuva
446. UERJ ácida, seja liberado para a atmosfera
Faça o balanceamento das equações: é tratá-lo previamente com óxido de
a. CaO + P2O5 → Ca3 (PO4)2 magnésio em presença de ar. Analisan-
do a equação dada, e balanceando-a,
b. Al(OH)3 + H2SO4 → Al2(SO4)3 + H2O o menor coeficiente inteiro do oxigê-
447. UFF-RJ nio é:
Faça o balanceamento das equações: Dado: SO2 + MgO + O2 → MgSO4
K PO + Ca(NO3 )2(aq) → KNO3(aq) + Ca3 (PO4 )2 a. 1/2
a. 3 4(aq) b. 1
b. BaCl2 + Na2 SO4 → NaCl + BaSO4 c. 1,5
448. Mackenzie-SP d. 2
Na equação x Fe2O3+ 3 CO → y CO2 + 2Fe, a soma dos e. 2,5
coeficientes x e y que tornam a equação corretamente 452. UFPE
balanceada é:
a. 5 c. 7 e. 4 Considere as reações químicas abaixo.
b. 3 d. 2 1. 2 K(s) + Cl2(g) → KCl(s)
449. Mackenzie-SP 2. 2 Mg(s) + O2(g) → 2 MgO(s)
A água oxigenada, usada para limpar ferimentos, é 3. PbSO4(aq) + Na2S(aq) → PbS(s) +
uma solução aquosa de peróxido de hidrogênio que, Na2SO4(s)
na presença de luz, decompõe-se em água e gás oxi- 4. CH4(g) + 2 O2(g) → CO2(g) + 2 H2O(  )
gênio. 5. SO2(g) + H2O(  ) → H2SO4(aq)
A alternativa que possui essa reação corretamente Podemos afirmar que:
equacionada e balanceada é:
a. todas estão balanceadas.
a. H2O2 luz→ H2 + O2
(aq) (g) (g) b. 2, 3 e 4 estão balanceadas.
c. somente 2 e 4 estão balancea-
PV-13-14

b. H2O2 luz
 → H2O() + O2 (g)
(aq) das.
d. somente 1 não está balancea-
c. H2O2 luz
 → 2 H2 O() + O2 (g)
(aq) da.
e. nenhuma está corretamente
d. 2 H2 O2
(aq)
luz
 → 2 H2 O() + O2 (g)
balanceada, porque os estados
físicos dos reagentes e produ-
luz
e. 2 H2O2 (aq) 
→ 2 H2 O() + H2 (g) tos são diferentes.
453. UFSM-RS
450. UEL-PR
Considere as equações:
Na equação química que representa a transformação
I. Al2O3 + HCl → AlCl3 + H2O
de oxigênio diatômico em ozônio, quando o coeficien-
te estequiométrico do oxigênio diatômico é 1, o do II. SO2 + NaOH → Na2SO3 + H2O
ozônio é:

152
Química geral e estequiometria Química

III. BaO4 + HNO3 → Ba(NO3)2 + H2O2 + O2 457. Unicamp-SP


A Terra é um sistema em equilíbrio altamente
A sequência correta dos coeficientes dos rea-
complexo, possuindo muitos mecanismos au-
gentes e produtos necessários para o balance-
torregulados de proteção. Esse sistema admi-
amento estequiométrico dessas equações é:
rável se formou ao longo de um extenso pro-
I II III cesso evolutivo de 4.550 milhões de anos.
a. 6, 3, 3, 2 1, 2, 1, 1 2, 1, 2, 2, 2 A atmosfera terrestre é parte integrante des-
b. 1, 6, 2, 3 2, 1, 1, 1 1, 1, 2, 1, 1 se intrincado sistema. A sua existência, den-
c. 1, 3, 3, 2 2, 1, 2, 2 1, 2, 1, 1, 1 tro de estreitos limites de composição, é es-
d. 6, 1, 2, 3 2, 1, 2, 2 2, 1, 2, 2, 2 sencial para a preservação da vida. No gráfico
abaixo, pode-se ver a abundância relativa de
e. 1, 6, 2, 3 1, 2, 1, 1 1, 2, 1, 1 alguns de seus constituintes em função da
altitude. Um outro constituinte, embora mi-
454. noritário, que não se encontra na figura, é o
ozônio, que age como filtro protetor da vida
Faça o balanceamento das equações: na alta atmosfera. Na baixa atmosfera, a sua
I. Fe2O3(s) + H2SO4(aq) → Fe2(SO4)3(aq) + H2O() presença é danosa à vida, mesmo em concen-
II. Al(OH)3(s) + H2SO4(aq) → Al2(SO4)3(aq) + trações relativamente baixas.
H2O() Oxigênio
atômico
III. CaO(s) + H3PO4(aq) → Ca3(PO4)2(aq) + H2O() 80
Abundância relativa

IV. MgCl2(aq) + Na2CO3(aq) → MgCO3(s) NaCl(aq) 60


455. PUC-RJ
40
O óxido de alumínio (Al2O3) é utilizado como
antiácido. A reação que ocorre no estômago é: 20 Nitrogênio
molecular
x Al2O3 + y HCl → z AlCl3 + w H2O 0 Oxigênio
0 100 200 300 400 molecular
Os coeficientes x, y, z e w são, respectivamente: Altitude / km
a. 1, 2, 3 e 6. d. 2, 4, 4 e 3.
a. Considerando que o ozônio seja forma-
b. 1, 6, 2 e 3. e. 4, 2, 1 e 6. do a partir da combinação de oxigênio
c. 2, 3, 1 e 6. molecular com oxigênio atômico, e que
este seja formado a partir da decom-
456. UFPI posição do oxigênio molecular, escreva
A reação de X com Y é representada a seguir. uma sequência de equações químicas
Indique qual das equações melhor representa que mostre a formação do ozônio.
a equação química balanceada. b. Tomando como base apenas o gráfico
PV-13-14

e as reações químicas citadas no item


a, estime em que altitude a formação
de ozônio é mais favorecida do ponto
de vista estequiométrico. Justifique.

Átomo X 458. PUC-PR


Átomo Y A oxidação enérgica do metanol (com dicro-
mato de potássio em meio ácido sulfúrico),
a. 2X + Y2 → 2XY produzindo aldeído fórmico, está representa-
b. 6X + 8Y → 6XY + 2Y da a seguir:
c. 3X + Y2 → 3XY + Y H3C – OH + Cr2O72– + H+ → H2CO + Cr3+ + H2O
d. X + Y → XY Após o seu balanceamento, tem-se como so-
e. 3X + 2Y2 → 3XY + Y2 matória dos coeficientes um valor igual a:

153
Química Química geral e estequiometria

a. 25 460. Fuvest-SP
b. 22 Ácido nítrico é produzido pela oxidação de
c. 23 amônia com excesso de oxigênio, sobre um
d. 24 catalisador de platina, em uma sequência de
reações exotérmicas. Um esquema simplifi-
e. 21 cado desse processo é:
459. Fuvest-SP
NO
A sequência de reações: NH3 Ar

x KHCO3 → M + CO2 + H2O H2 O


CO2+Ba(OH)2 → N + H2O
ficará correta se x, M e N forem substituídos, Pt
respectivamente, por:
a. 1,K2CO3 e Ba2CO3 A
HNO3
b. 1,K2O2 e Ba2C NO NO2
c. 1,K2O e BaHCO3 Reator Torre de Torre de
oxidação absorção
d. 2,K2CO3 e Ba2HCO3
e. 2,K2CO3 e BaCO3 Escreva as equações químicas balanceadas
das reações que ocorrem no reator, na torre
de oxidação e na torre de absorção.
Note que, desta última, sai NO(g), nela gerado.
A maior parte desse gás é aproveitada na pró-
pria torre, onde há oxigênio em excesso. Duas
reações principais ocorrem nessa torre.

PV-13-14

154
Química geral e estequiometria Química

CAPÍTULO 07
461. Unesp 465. Fesp
O óxido nitroso, N2O, é conhecido como “gás O método mais usado em laboratório para a
hilariante” e foi um dos primeiros anestésicos obtenção do cloro é através da oxidação do
a serem descobertos. Esse gás pode ser obti- ácido clorídrico com permanganato de potás-
do pelo aquecimento cuidadoso de nitrato de sio. A equação abaixo representa a reação que
amônio sólido. ocorre nesse método.
a. Escreva a equação da decomposição 2 KMnO4 + 16 HCl → 5 Cl2 + 2 KCl + 2 MnCl2 + 8 H2O
por aquecimento do nitrato de amônio
Para se obterem 10 mols de cloro são necessários:
em óxido nitroso e água.
a. 5 mols de KMnO4 e 5 mols de HCl.
b. Calcule a massa de nitrato de amônio
necessária para se obterem 880 g de b. 1 mol de KMnO4 e 16 mols de HCl.
óxido nitroso. c. 8 mols de KMnO4 e 28 mols de HCl.
Dados: H = 1 ; N = 14 ; O = 16. d. 2 mols de KMnO4 e 30 mols de HCl.
462. UFBA e. 4 mols de KMnO4 e 32 mols de HCl.
Hidreto de sódio reage com água, dando hi- 466. UFF-RJ
drogênio, segundo a reação: O fósforo elementar é, industrialmente, obti-
NaH + H2O → NaOH + H2 do pelo aquecimento de rochas fosfáticas com
Para obter 10 mols de H2, são necessários: coque, na presença de sílica.
a. 40 mols de água. Considere a reação:
b. 20 mols de água. 2 Ca3(PO4)2 + 6 SiO2 + 10 C → P4 + 6 CaSiO3 + 10 CO
c. 10 mols de água. e determine quantos gramas de fósforo ele-
d. 15 mols de água. mentar são produzidos a partir de 31,0 g de
e. 2 mols de água. fosfato de cálcio.
463. UMC-SP Dados: massas molares (g/mol) → P = 31,0;
Ca3 (PO4)2 = 310,0
Dada a equação: 3 PbCl2 + Al2(SO4)3 → 3 PbSO4
+ 2 AlCl3, o número de mols de PbSO4 que serão a. 3,10 g
formados a partir de 3 mols de Al2(SO4)3 é igual a: b. 6,20 g
a. 1 c. 12,40 g
b. 2 d. 32,00 g
PV-13-14

c. 3 e. 62,00 g
d. 6
467. Unimep-SP
e. 9
Quantos mols de ácido sulfúrico devem reagir
464. FMU-SP com cloreto de bário, a fim de formar 0,50 mol
Na reação: 3 Fe + 4 H2O → Fe3O4 + 4 H2, o nú- do correspondente hidrogenossal?
mero de mols de hidrogênio produzidos pela
reação de 4,76 mols de ferro é: 2 H2SO4 + BaCl2 → Ba(HSO4)2 + 2 HCl
a. 6,35 mols a. 0,5
b. 63,5 mols b. 0,25
c. 12,7 mols c. 1,0
d. 1,27 mol d. 2,0
e. 3,17 mols e. 2,5

155
Química Química geral e estequiometria

468. Massas molares em g/mol: Xe = 131; Pt = 195;


O sulfato de sódio (Na2SO4) é uma substância F = 19
utilizada para fabricar papel e vidros. Para ob- a. 131.
tê-lo, faz-se reagir ácido sulfúrico (H2SO4) com b. 195.
cloreto de sódio (NaCl) segundo a equação: c. 220.
H2SO4 + 2 NaCl → Na2SO4 + 2 HCl d. 326.
Partindo-se de 7,0 mols de NaCl, calcule as e. 440.
quantidades, em mols, de Na2SO4 e HCl que
472. PUCCamp-SP
podem ser obtidas.
469. Corais
A alternativa que indica a massa, em gramas, Recifes de corais artificiais estão sendo
de Al2O3, obtida pela reação de 13,5 g de alu- usados para acelerar o processo de restau-
mínio com o oxigênio, é: ração dos recifes naturais. Para isso, a Bio-
4 Al + 3 O2 → 2 Al2O3 rock Inc. utiliza armações de aço que são
a. 156,5 energizadas por uma corrente elétrica de
baixa voltagem. Isto faz com que os mi-
b. 102,0
nerais da água do mar nelas se prendam,
c. 54,0 formando uma fina camada de calcário.
d. 25,5 Desse modo, pode-se prender pequenos
e. 23,0 pedaços de coral nas armações, que ficam
seguras devido ao calcário acumulado.
Dados: Al = 27 ; O = 16.
BBC Knowledge, outubro de 2009, p. 9.
470.
O calcário pode se formar por reações quími-
Quando se coleta sangue para análises labora- cas, como na equação representada a seguir:
toriais, utiliza-se como agente anticoagulante
o citrato de sódio (Na3C6H5O7). Para obtê-lo, (aq) + 2 HCO3(aq) →
Ca2+ ← CaCO3(s) + H2O() + CO2(g)
faz-se a reação entre ácido cítrico (C6H8O7) e Assim, para cada 1,0 g de CaCO3 que se forma,
hidróxido de sódio (NaOH). deve reagir uma quantidade, em mol, de íons
1 C6H8O7 + 3 NaOH → 1 Na3C6H5O7 + 3 H2O bicarbonato, HCO–3 , correspondente a:
Calcule a massa de ácido cítrico consumida Dados:
para se obterem 8 mols de citrato de sódio. Massas molares (g · mol−1)
Dados: H = 1; C = 12; O = 16. HCO3– = 61
471. Fatec-SP CaCO3 = 100
PV-13-14

Durante muito tempo acreditou-se que os ga- a. 0,01


ses nobres eram muito estáveis, inertes e, por- b. 0,02
tanto, não poderiam existir compostos desses c. 0,03
elementos químicos. Entretanto, essa concep-
d. 0,04
ção mudou quando, em 1962, na Universidade
de British Columbia, no Canadá, o primeiro e. 0,05
composto de gás nobre foi obtido pela reação 473. UFPB
entre xenônio e hexafluoreto de platina, re- Gigantes reservas de petróleo foram
presentada por: encontradas recentemente no Brasil. Essas
Xe + PtF6 → Xe+(PtF6)– reservas situam-se em regiões de grandes
profundidades em águas oceânicas e abai-
Para cada mol de xenônio que reage comple- xo de uma camada de sal, por isso, são de-
tamente, a massa do produto formado na rea- nominadas de pré-sal. Com a exploração
ção é, em gramas: dessas reservas, o Brasil aumentará sig-

156
Química geral e estequiometria Química

nificativamente a produção de petróleo. A massa de oxigênio, em gramas, presente no to-


Após a extração, o petróleo é transportado tal de CO2(g) gerado quando se aluga um filme é:
até as refinarias, onde passará por uma sé-
Dados:
rie de processos de purificação denomina-
da de refino, em que o petróleo entra na Massas molares (g · mol–1)
fornalha, é aquecido e segue para a torre C = 12;
de destilação, onde serão separadas as di-
versas frações. O = 16.
O gás natural, usado como combustível para a. 2,5 · 102
automóveis, é a fração mais leve resultante do b. 7,2 · 102
refino do petróleo. Esse gás é constituído prin- c. 1,7 · 103
cipalmente de metano, CH4, cuja combustão
d. 3,4 · 103
completa ocorre conforme a equação:
e. 5,1 · 103
CH4(g) + 2O2(g) → CO2(g) + 2H2O(v)
475. Fuvest-SP
De acordo com essa equação, se 8 g de me-
tano reagirem com oxigênio, a quantidade Sob condições adequadas, selênio (Se) e es-
de gás carbônico, CO2, liberada para a at- tanho (Sn) podem reagir, como representado
mosfera será de: pela equação:
a. 44 g 2 Se + Sn → SnSe2
b. 11 g Em um experimento, deseja-se que haja rea-
c. 22 g ção completa, isto é, que os dois reagentes se-
d. 8 g jam totalmente consumidos. Sabendo-se que
a massa molar do selênio (Se) é 2/3 da massa
e. 32 g
molar do estanho (Sn), a razão entre a massa
474. PUCCamp-SP de selênio e a massa de estanho (mSe : mSn), na
reação, deve ser de:
Baixe filmes, salve o planeta
a. 2 : 1
Existem muitas maneiras de dimi-
nuir o impacto humano ao planeta. Re- b. 3 : 2
ciclar o lixo, plantar muda de espécies c. 4 : 3
ameaçadas, ir ao trabalho de bicicleta, d. 2 : 3
assistir a filmes em streaming em vez de
ir ao cinema... Como? Isso mesmo. Usar e. 1 : 2
computador e Internet para fazer com- 476. Mackenzie-SP
pras, alugar filmes e ler livros, em vez O ferro é um metal essencial para a vida,
de se deslocar para realizar essas ativi- responsável pela formação da hemoglo-
PV-13-14

dades, reduz a emissão de carbono.


bina, da mioglobina e de certas enzimas.
Veja o esquema a seguir: Apenas 8% do ferro ingerido são absorvidos
Alugar um filme e entram na corrente sanguínea. A dose di-
ária recomendada é de cerca de 15mg para
2,3 kg de CO2 adultos e de 30mg para gestantes. Café ou
chá em grandes quantidades inibem a ab-
sorção de ferro. O ferro ajuda no cresci-
83% das emissões são 17% são emitidos na
causadas pelo deslocamento produção do DVD.
mento, promove a resistência às doenças,
de carro até a locadora. evita a fadiga, a anemia e garante uma boa
tonalidade à pele. Supondo que uma colher
0 kg de CO2 de sopa de feijão possua cerca de 4,4 ·10 –5
quantidade de energia gasta mol de ferro, uma gestante, para obter a
quantidade diária de ferro recomendada,
Revista Galileu, out. 2009, p. 17. deverá ingerir:

157
Química Química geral e estequiometria

Dado: Fe = 56 479. FEPECS-DF


a. 4 colheres de sopa de feijão. Paracelsus já havia percebido, no século dezes-
b. 6 colheres de sopa de feijão. seis, que “todas as substâncias são venenos; o
c. 8 colheres de sopa de feijão. que diferencia um medicamento de um vene-
no é a dose”. O ferro é um metal importante
d. 10 colheres de sopa de feijão. para o funcionamento do organismo humano,
e. 12 colheres de sopa de feijão. no entanto, em excesso, pode depositar-se
477. nos tecidos provocando uma doença denomi-
nada hemocromatose. Os principais locais de
Um comprimido efervescente contém bicar- depósito são o fígado, o pâncreas, o coração
bonato de sódio (NaHCO3) e um ácido orgâni- e a hipófise, que podem perder, progressiva-
co. Em contato com a água, ocorre a reação: mente, suas funções.
HCO3– + H+ → H2O + CO2 A dose letal de ferro em crianças pequenas
Dados: Massas atômicas: H = 1; C = 12; O = 16; está em torno de 0,05 mol. A massa de ferro,
Na = 23; volume molar nas CNTP = 22,4 L em gramas, que corresponde, aproximada-
Sabendo-se que em cada comprimido existe mente, a essa dose letal é:
0,84 g de NaHCO3, qual o número de compri- a. 56
midos necessários para produção de 22,4 L b. 30
de gás carbônico nas condições normais de c. 14
temperatura e pressão?
d. 2,8
a. 10 d. 100
e. 1,4
b. 20 e. 1.000
c. 50 480. UEPB

478. UFPE O Brasil possui uma política de substituição


do petróleo como fonte energética desde
Ácido sulfúrico (H2SO4) é um importante insu- os anos 1960, como com a criação do Pró-
mo industrial, obtido como subproduto do re- -Álcool, um programa governamental de es-
fino de cobre. A matéria prima deste processo, tímulo à produção de etanol combustível a
sulfeto de cobre (CuS), é decomposta termica- partir da cana-de-açúcar e de confecção de
mente, na presença de oxigênio, produzindo automóveis que utilizem esta fonte energé-
cobre metálico e SO2. Por ser um gás tóxico, tica. Em 2009 completaram-se três décadas
o SO2 não pode ser liberado no ambiente, e, da implementação dos primeiros postos de
portanto, é oxidado a SO3, que em seguida re- distribuição de combustíveis que comercia-
age com água para formar ácido sulfúrico. Ao lizaram o etanol, atualmente o mais impor-
iniciarmos o processo com 19,1 toneladas de tante biocombustível da matriz energética.
sulfeto de cobre puro, e assumindo um rendi- Sua produção é baseada no melaço da cana-
PV-13-14

mento de 100% em todas as etapas, podemos de-açúcar como matéria-prima. O processo


afirmar que serão: utiliza a fermentação da sacarose, presente
Dadas as massas atômicas: Cu, 63,5 g/mol; S, no melaço, pela proteína invertase, origi-
32 g/mol; O = 16 g/mol e H = 1 g/mol. nando glicose (C6H12O6) e frutose (C6H12O6),
00. consumidos 300.000 mols de oxigênio que, sob influência de outra proteína, a zi-
molecular. mase, e na presença de água, produzem o
etanol e gás carbônico.
01. consumidos 200.000 mols de água.
Considerando uma produção de etanol
02. produzidos e posteriormente consumi- combustível de 11,5 bilhões de litros e uma
dos 80.000 mols de SO3. densidade desse líquido de 0,8 g/cm 3, qual
03. produzidas 196 toneladas de ácido a quantidade de matéria de glicose/fruto-
sulfúrico. se requerida para a produção de todo esse
04. produzidas 1,31 toneladas de cobre álcool? Considere a reação com 100 % de
metálico. rendimento.

158
Química geral e estequiometria Química

a. 1.1011 mol de glicose/frutose. O volume de CO2(g), nas condições ambientais


b. 1.108 mol de glicose/frutose. de temperatura e pressão, CATP, que é emitido
na produção do DVD corresponde a, aproxi-
c. 1.1012 mol de glicose/frutose.
madamente:
d. 1.1011 g de glicose/frutose.
Dados:
e. 1.1010 g de glicose/frutose.
Massa molar do CO2 = 44 g · mol–1
481. UFMS
Volume molar dos gases nas CATP = 25 L mol–1
Os carros que disputam o campeonato brasi-
leiro de Stock Car ganharam, na temporada de a. 25
2007, um acessório que permite um aumento b. 135
considerável de potência por alguns instantes c. 171
durante a corrida. Trata-se de um cilindro con- d. 222
tendo nitro, ou mais precisamente, 200 g do
gás óxido nitroso (N2O). e. 578
Considerando o comportamento de gás ideal, 483. UFTM-MG
calcule o volume de N2O, em litros, contido no O composto comumente chamado de tri-iode-
cilindro, nas CNTP. to de nitrogênio é na realidade o composto de
Dados: Massas molares (g/mol): N = 14; fórmula NI3 · NH3 (uma molécula de tri-iodeto
O = 16; Volume molar = 22,4 L/mol; R = 0,082 de nitrogênio ligada a uma molécula de amô-
Latm/Kmol nia). Trata-se de um sólido preto, muito ins-
tável, que se decompõe explosivamente por
a. 101,8 d. 200,0 um simples atrito, produzindo forte estalido
b. 4480,0 e. 22,4 e uma “nuvem” de cor violeta. Os produtos
c. 149,3 da decomposição são nitrogênio (N2), amônia
(NH3) e iodo (I2) gasosos. O NI3 · NH3 é obtido
482. PUCCamp-SP pela reação entre amônia (NH3) aquosa e iodo
(I2) sólido. O outro produto da reação é o iode-
Baixe filmes, salve o planeta to de amônio (NH4I) em solução aquosa.
Existem muitas maneiras de dimi- Conhecido o volume molar de um gás nas
nuir o impacto humano ao planeta. Re- CATP (Condições Ambientais de Temperatura
ciclar o lixo, plantar muda de espécies e pressão) como sendo 25 L/mol, o volume de
ameaçadas, ir ao trabalho de bicicleta, nitrogênio gasoso, medido nas CATP, produzi-
assistir a filmes em streaming em vez de do na decomposição de 8,24 g de NI3 · NH3 é,
ir ao cinema... Como? Isso mesmo. Usar em litros, aproximadamente igual a:
computador e Internet para fazer com-
pras, alugar filmes e ler livros, em vez a. 0,25
de se deslocar para realizar essas ativi- b. 0,50
PV-13-14

dades, reduz a emissão de carbono. c. 1,25


Revista Galileu d. 2,5
Veja o esquema a seguir: e. 5,0
Alugar um filme 484. Fatec-SP
O “cheiro forte” da urina humana deve-se prin-
2,3 kg de CO2
cipalmente à amônia, formada pela reação quí-
mica que ocorre entre ureia, CO(NH2)2, e água:
83% das emissões são 17% são emitidos na
causadas pelo deslocamento produção do DVD.
CO(NH2)2(aq) + H2O() → CO2(g) + 2 NH3(g)
de carro até a locadora. O volume de amônia, medido nas CATP (Con-
dições Ambientais de Temperatura e Pressão),
0 kg de CO2
formado quando 6,0 g de ureia reagem com-
quantidade de energia gasta pletamente com água é, em litros:

159
Química Química geral e estequiometria

Dado: 487. UFC-CE


VM = 25 L · mol –1 A indústria automobilística tem utilizado um
Massas molares, em g · mol : C = 12; H = 1;
–1 novo dispositivo de segurança, instalado nos
O = 16; N = 14 carros, que é constituído por um balão inflável
(air bag), o qual, após impacto do veículo, infla
a. 0,5
em quarenta milissegundos pela injeção de ni-
b. 1,0 trogênio gasoso. Esse gás é originado do com-
c. 1,5 posto NaN3(s), armazenado no balão, o qual se
d. 2,0 decompõe através da reação:
e. 5,0 2NaN3(s) → 2Na(s) + 3N2(g)
485. Unesp Calcule a quantidade de NaN3(s) necessária
Explosivos são eficientes quando produzem para gerar um volume de 50 L à temperatura
um grande número de moléculas gasosas na de 27 °C e pressão de 2 atm.
explosão. A nitroglicerina, por exemplo, deto- Dados: constante universal dos gases:
na de acordo com a equação química, parcial-
R = 0,082 · L · atm/mol · K; Na = 23; N = 14.
mente balanceada:
488. UFRGS-RS
2 C3H5N3O9 → x CO2(g) + y N2(g) + 5 H2O(g) + z O2(g)
nitroglicerina A seguir são feitas três afirmações a respeito
da combustão completa de 5,80 g de butano
Massa molar: nitroglicerina = 227,0 g/mol conforme a seguinte equação:
C4H10(g) + 13/2 O2(g) → 4 CO2(g) + 5 H2O( )
a. Calcule a soma do número de mols dos
produtos gasosos que se formam para I. Ocorre o consumo de 0,650 mol de oxi-
a explosão de 4,54 g de nitroglicerina. gênio.
b. Calcule o volume total, em litros, dos II. Ocorre a formação de 90,0 g de água.
produtos gasosos, quando a pressão III. Ocorre a produção de 8,96 litros de gás
gasosa é de 1 atm e a temperatura é carbônico nas CNTP.
de 500 °C para a explosão da massa de Dados: C = 12; H = 1; O = 16; Vmolar = 22,4 L (CNTP).
4,54 g de nitroglicerina.
Quais estão corretas?
486. UFRGS-RS
a. Apenas I.
Um vazamento de gás de cozinha pode provocar
sérios acidentes. O gás de cozinha, quando pre- b. Apenas II.
sente no ar em concentração adequada, pode c. Apenas III.
ter sua combustão provocada por uma simples d. Apenas I e III.
faísca proveniente de um interruptor de luz ou
PV-13-14

e. I, II e III.
de um motor de geladeira. Essas explosões são,
muitas vezes, divulgadas erroneamente como 489. Unicamp-SP
explosões do botijão de gás. A reação de com-
bustão completa de um dos componentes do A Apolo 13, uma nave tripulada, não pôde
gás de cozinha é apresentada a seguir: concluir sua missão de pousar no solo lunar
devido a uma explosão num tanque de oxigê-
C3H8 + 5 O2 → 3 CO2 + 4 H2O nio líquido. Esse fato desencadeou uma série
A partir da equação acima, qual a massa de de problemas que necessitaram de soluções
oxigênio necessária para produzir a combus- rápidas e criativas. Um desses problemas foi
tão completa de 224 litros de propano nas o acúmulo de gás carbônico no módulo espa-
CNTP? cial. Para reduzir o teor desse gás na cabine da
a. 32 g d. 1.600 g nave, foi improvisado um filtro com hidróxido
b. 160 g e. 3.200 g de lítio que, por reação química, removia o gás
c. 320 g carbônico formado.

160
Química geral e estequiometria Química

a. Escreva a equação química que justifica 492. Unesp


o uso do hidróxido de lítio como absor- Considere o etanol anidro e o n-octano, dois
vedor desse gás. combustíveis que podem ser empregados em
b. Qual seria a massa de hidróxido de lítio motores de combustão interna. Sobre estes
necessária para remover totalmente o dois combustíveis, estão disponíveis os dados
gás carbônico presente, considerando- fornecidos a seguir.
-o a uma pressão parcial igual a 2% da
pressão ambiente total de 1,0 atm, es-
Etanol n-octano
tando a cabine à temperatura de 20 °C
e supondo-se um volume interno de Fórmula molecular C2H5OH C8H18
60m3?
Massa molar (g/mol) 46 114
Dados: R = 0,082 ⋅ atm ⋅ L ⋅ mol–1 ⋅ K–1; Li = 7;
O = 16; H = 1 Número de mols/ litro 17,2 6,15

490. Mackenzie-SP
Sabendo que Suponha dois motores idênticos em funcio-
2 C4H10 + 13O2 → 8 CO2 + 10 H2O, então o vo- namento, cada um deles movido pela queima
lume de ar, medido a 27 °C e 1 atm, necessário completa de um dos combustíveis, com igual
para a combustão de 23,2 g de gás butano, é: aproveitamento de energia gerada.
Dados: a. Escreva as equações químicas que re-
presentam a combustão completa de
1. Considerar a composição do ar (em vo-
cada um dos combustíveis.
lume):
b. Sabe-se que, para realizar o mesmo tra-
80% de N2 e 20% de O2
balho gerado pela queima de 10 litros
2. Constante universal dos gases = 0,082 de n-octano, são necessários 14 litros
atm · litro / mol · K de etanol. Nestas condições, compare,
3. Massa molar do butano = 58 g/mol através de cálculos, a poluição atmosfé-
a. 319,8 litros. rica por gás carbônico produzida pelos
dois combustíveis.
b. 116,4 litros.
c. 302,8 litros. 493. UFRJ
d. 127,9 litros. Recentemente, identificou-se um aumento
da concentração de metanal (formaldeído) no
e. 80,0 litros. ar da cidade do Rio de Janeiro, possivelmen-
491. Unicamp-SP te ocasionado por combustão incompleta em
A obtenção de etanol, a partir de sacarose motores de automóveis adaptados para uso
(açúcar) por fermentação, pode ser apresenta- de gás natural. Admita que o gás natural seja
PV-13-14

da pela seguinte equação: constituído exclusivamente por metano e que,


durante o processo de combustão, 1% dessa
C12H22O11 + H2O → 4 C2H5OH + 4 CO2 substância se converte apenas em formaldeí-
Admitindo-se que o processo tenha rendimen- do e água.
to de 100% e que o etanol seja anidro (puro), Determine a massa, em gramas, de meta-
calcule a massa (em kg) de açúcar necessária nal formado quando todo o metano, origi-
para produzir um volume de 50 litros de eta- nalmente contido em um tanque de 82 L, à
nol, suficiente para encher um tanque de um temperatura de 300 K e pressão de 150 atm,
automóvel. sofre combustão. Admita que o metano ar-
Dados: Densidade do etanol = 0,8 g/cm3 mazenado no tanque se comporte como um
gás ideal.
Massa molar da sacarose = 342 g/mol
Dados: R = 0,082 atm · L · mol–1 · K–1; H = 1;
Massa molar do etanol = 46 g/mol
C = 12; O = 16.

161
Química Química geral e estequiometria

494. UFES De acordo com o texto, calculando a área da


A uma velocidade constante de 50 km/h, um au- superfície ocupada por um fulereno, tem-se
tomóvel faz cerca de 10 km por litro de etanol que esta medida é, em metros quadrados,
(C2H5OH). Considerando a queima total do com- aproximadamente igual a:
bustível, qual deve ser o volume de gás carbôni- 1. Área da superfície esférica: A = 4πr2
co (CO2) em metros cúbicos, lançado na atmosfe- 2. Adote π = 3
ra pelo automóvel, após 2 horas de viagem? a. 4,2 · 10–81
Considere: b. 5,6 · 10–36
Densidade do álcool = 0,8 kg/L c. 3,5 · 10–27
Massa molar do etanol = 46 g/mol d. 1,47 · 10–18
Volume molar do CO2 = 25 L/mol e. 2,18 · 10–12
1 dm3 = 1L 496. UFRN
495. Fatec-SP Um air bag é uma bolsa que infla rapida-
mente e que, num acidente de carro, ajuda a
A nanotecnologia refere-se à tecnologia utili-
prevenir lesões graves, como mostra a figura
zada para manipular estruturas muito peque-
abaixo.
nas, tornando possível a criação de estruturas
funcionais que seriam inconcebíveis utilizan-
do-se tecnologia convencional.
Na formação da palavra Nanotecnologia, o
termo “tecnologia” refere-se ao desenvolvi-
mento e produção de novos materiais, já o
prefixo “nano” está relacionado a uma escala
de medida em que um nanômetro (nm) é um
bilionésimo do metro.
Estruturas de átomos de carbono
na escala nano

Quando se produz a desaceleração repenti-


na do carro, é conectado automaticamente
um interruptor, que inicia uma reação quí-
mica, liberando o gás nitrogênio em quan-
tidade suficiente, conforme a equação a
Dentre as estruturas apresentadas,
seguir:
tem-se o grafite, um semimetal que con-
duz eletricidade, e o diamante, um exce- 6 NaN3() + Fe2O3(s) → 3 Na2O(s) + 2 Fe(s) + 9 N2(g)
PV-13-14

lente isolante.
Considere que o volume de um mol de gás,
Os fulerenos são conhecidos como
moléculas semelhantes à bola de futebol. nas CNTP, corresponda a 22,4 litros. Nessas
Um fulereno é feito de 60 átomos de car- condições, de acordo com a equação química,
bono unidos de tal maneira que criam uma quando reagem 3 moles de NaN3, o volume de
esfera oca de 0,7 nm de diâmetro. nitrogênio gasoso que se obtém é, aproxima-
Outra estrutura de carbono é o nano- damente, de:
tubo, no qual os átomos de carbono estão li- a. 101 litros.
gados em forma de tubos, ocos como fulere-
nos, com diâmetros de uma a várias dezenas b. 202 litros.
de nanômetros. c. 56 litros.
Disponível em: <http: //www.cienciaviva. d. 45 litros.
org.br – Acesso em 13.3.2010>.

162
Química geral e estequiometria Química

497. UNIR-RO d. 0,04


Os metais alcalinos e alcalinos terrosos são e. 0,05
metais de baixa densidade, coloridos e ma- 500. UECE
cios. Reagem com água formando hidróxidos
fortemente básicos e liberando gás hidrogênio O fósforo branco, usado como arma química,
de forma exotérmica. A massa de água, em apresenta alta reatividade, queima com facilidade
gramas, necessária para reagir com magnésio na presença do ar atmosférico e é obtido pela re-
suficiente para formar 112 L de gás hidrogênio, ação representada pela equação não balanceada:
medidos nas CNTP, é, aproximadamente: Ca3(PO4)2(s)+ SO2(g) + C(s) → CaSO3(s) + CO(g) + P4(s)
a. 36 Em relação ao fósforo, suas variedades, seu
b. 560 processo de obtenção e suas propriedades,
c. 180 assinale o correto.
d. 390 a. Fósforo branco e fósforo vermelho são
denominações diferentes para isótopos
e. 72
do fósforo.
498. UECE b. A soma dos coeficientes da equação
O sulfato de alumínio, usado para a decan- acima, quando balanceada, é 34.
tação de sujeira na água das pscinas, para c. Na reação indicada acima, cada átomo
fixar as cores nos têxteis e como agente an- de fósforo sofre oxidação, perdendo
titranspirante nos desodorantes, é obtido cinco elétrons.
pela reação de deslocamento entre o alumí-
d. De acordo com a reação acima, quan-
nio e o ácido sulfúrico que produz, também,
do balanceada, o volume de dióxido de
hidrogênio gasoso.
enxofre, medido nas CNTP, consumido
Sobre essa reação, podemos afirmar corre- na produção de 0,75 mol de fósforo é,
tamente, que a massa de alumínio necessá- aproximadamente, 100 litros.
ria para produzir 89,6 L de hidrogênio, nas
501. UFC-CE
CNTP, é:
a. 18 g O ferro metálico pode ser produzido a partir
da reação do Fe2O3 com CO de acordo com a
b. 36 g seguinte equação química não balanceada:
c. 72 g Fe2O3(s) + y CO(g) → w Fe(s) + z CO2(g)
d. 90 g Considere a reação completa entre 1,60 g de
499. Unifor-CE Fe2O3 e 3,00 g de CO e assinale a alternativa
correta.
Componentes do sangue catalisam a decom-
posição da água oxigenada, segundo a equa- a. O reagente limitante dessa reação é o
ção: monóxido de carbono.
PV-13-14

b. A quantidade máxima de ferro metálico


H2O2(aq) → H2O() + 1/2 O2(g)
produzida será de, aproximadamente,
Considerando uma água oxigenada a 3% (m/V), a 1,12 g.
reação de 1,0 mL dessa solução utilizada para lim- c. Após a reação se completar, restará
par um ferimento, pode gerar um volume máximo 0,58 g de monóxido de carbono no
de O2(g), em L, nas condições normais de pressão e meio reacional.
temperatura, corresponde a: d. A quantidade máxima de dióxido de
Dado: Volume molar nas CNTP = 22,4 L carbono produzida será de, aproxima-
a. 0,01 damente, 4,60 g.
e. Se o rendimento for de 80%, serão pro-
b. 0,02
duzidos, aproximadamente, 2,50 g de
c. 0,03 ferro metálico.

163
Química Química geral e estequiometria

502. 505. Unesp


Considere uma reação hipotética que ocorre Considere a reação em fase gasosa:
em fase gasosa e envolve os reagentes X e Y N2 + 3 H2 → 2 NH3
e o produto Z. Fazendo-se 4 litros de N2 reagirem com 9 litros
Num experimento, foram misturados, em um de H2 em condições de pressão e temperatura
recipiente, 5 mol de X com 5 mol de Y. Após 1 constantes, pode-se afirmar que:
minuto, nesse recipiente, havia 4 mol de X, 3 a. os reagentes estão em quantidades es-
mol de Y e 1 mol de Z, como registrado neste tequiométricas.
quadro: b. o N2 está em excesso.
X Y Z
c. após o término da reação, os reagen-
tes serão totalmente convertidos em
Início 5 mols 5 mols 0 amônia.
d. a reação se processa com o aumento
Após 1 min 4 mols 3 mols 1 mol
do volume total.
Suponha que essa reação prossegue até o con- e. após o término da reação, serão forma-
dos 8 litros de NH3 .
sumo total do reagente limitante.
506. UEG-GO
Considerando-se a quantidade inicial de X e Y,
é correto afirmar que a quantidade máxima de A glicose (C6H12O6) sofre combustão comple-
Z a ser obtida nessa reação é de: ta na presença de oxigênio, produzindo CO2 e
H2O. Sabendo-se que nessa reação o sistema
a. 2,5 mols. c. 4 mols.
era formado por 216 g de cada um dos reagen-
b. 3,5 mols. d. 5 mols. tes, determine:
503. IME-RJ a. o reagente limitante na reação.
Certa massa de sódio reagiu com água, produ- b. a massa residual, em gramas, do rea-
zindo o composto A, o qual com ácido clorídri- gente em excesso.
co forneceu a substância B. Quando se tratou c. a pressão, em atm, exercida pelo gás
B com excesso de nitrato de prata, obteve-se carbônico, caso seja coletado em um
um precipitado que, depois de lavado e seco, recipiente de 200 mL, a 27 °C.
apresentou uma massa de 14,35 g. Qual a
Dado: R = 0,082 atm · L · mol–1 · K–1
massa de sódio usada?
507. UEM-PR
504. UFF-RJ
A dissolução de um medicamento antiácido que
Amônia gasosa pode ser preparada pela se- contém 1,92 g de bicarbonato de sódio (NaHCO3)
guinte reação balanceada: e 1,92 g de ácido cítrico (C6H8O7) provoca efer-
CaO(s) + 2 NH4Cl(s) → 2 NH3(g) + H2O(g) + CaCl2(s) vescência, conforme a seguinte reação:
PV-13-14

Se 112,0 g de óxido de cálcio e 224,0 g de clo- 3NaHCO3 + C6H8O7 → 3CO2 + 3H2O


reto de amônia forem misturados, então a + Na3C6H5O7
quantidade máxima, em gramas, de amônia Sobre esse processo, é correto afirmar que:
produzida será, aproximadamente:
01. o bicarbonato de sódio é o reagente li-
Dados: massas molares → CaO = 56 g/mol ; mitante da reação.
NH4Cl = 53 g/mol ; NH3 = 17 g /mol 02. será formado 0,03 mol de CO2.
a. 68,0 04. cerca de 0,46 g de ácido cítrico não
b. 34,0 reagirá.
c. 71,0 08. a efervescência ocorre devido à visuali-
d. 36,0 zação da formação de água na reação.
e. 32,0 16. será formado 0,01 mol de citrato de
sódio.

164
Química geral e estequiometria Química

508. UEPG-PR Dados: massas molares g/mol: CO: 28, H2: 2


A reação entre alumínio pulverizado e óxido e CH3OH: 32
de ferro (III), a. CO, 384 d. H2, 384
2Al + Fe2O3 → Al2O3 + 2Fe b. CO, 396 e. H2, 480
libera tanto calor que produz ferro derretido. c. CO, 480
Trata-se de um processo que é aproveitado na 511. Mackenzie-SP modificado
construção de ferrovias, para soldar trilhos de aço.
Conhecidos a reação de obtenção da amônia,
A respeito dessa reação, genericamente, e, abaixo equacionada, e ovolume de gás nitro-
em particular, considerando uma experiência gênio (5,0 litros),o volume de gás hidrogênio
em que são utilizados 4,20 mols de alumínio necessário para a obtenção de 6,0 litros de
e 1,8 mol de óxido de ferro, assinale o que for NH3 é igual a:
correto.
3 H2 (g) + N2 (g) → 2 NH3 (g) (P, T constantes)
Dados: Fe = 56 g/mol; Al = 27 g/mol;
O = 16,0 g/mol a. 6,0 litros
01. O alumínio, que é mais reativo, oxida, b. 12,0 litros
deslocando o ferro do óxido de ferro. c. 9,0 litros
02. Na experiência, o reagente limitante da d. 3,0 litros
reação é o óxido de ferro.
e. 1,0 litro
04. Da experiência resultam 214,2 g de óxi-
do de alumínio. 512. Mackenzie-SP
08. Da experiência resultam 201,60 g de Na reação equacionada:
ferro. x + y → xy, a razão entre as massas de x e y é
509. UFC-CE de 0,5. Ao se adicionarem 30,0 g de x a 90,0 g
O ácido fosfórico, H3PO4, pode ser produzi- de y, pode-se dizer que:
do a partir da reação entre a fluoroapatita, a. há excesso de 15,0 g de x.
Ca5(PO4)3F, e o ácido sulfúrico, H2SO4, de acor- b. reagiram 20,0 g de x e 70,0 g de y.
do com a seguinte equação química:
c. há excesso de 30,0 g de y.
Ca5(PO4)3 F(s) + 5H2SO4() → d. a lei de Lavoisier não foi obedecida.
→ 3 H3PO4() + 5 CaSO4(s) + HF(g) e. a lei de Proust não foi obedecida.
Considere a reação completa entre 50,45 g de 513. UFSCar-SP
fluoroapatita com 98,12 g de ácido sulfúrico.
Quando 56 g de ferro (massa atômica = 56)
a. Qual é o reagente limitante da reação? são colocados para reagir com 40 g de enxofre
b. Determine a quantidade máxima de
PV-13-14

(massa atômica = 32), de acordo com a reação


ácido fosfórico produzida. Fe + S → FeS, formam-se:
510. Fatec-SP a. 96 g de sulfeto de ferro.
Metanol é um excelente combustível que pode b. 40 g de sulfeto de ferro e sobram 16 g
ser preparado pela reação entre monóxido de car- de ferro.
bono e hidrogênio, conforme a equação química c. 56 g de sulfeto de ferro e sobram 8 g
CO(g) + 2 H2(g) → CH3OH( ) de enxofre.
Supondo rendimento de 100% para a reação, d. 88 g de sulfeto de ferro e sobram 8 g
quando se adicionam 336 g de monóxido de de enxofre.
carbono a 60 g de hidrogênio, devemos afir- e. 40 g de sulfeto de ferro e sobram 8 g
mar que o reagente em excesso e a massa de enxofre.
máxima, em gramas, de metanol formada
são, respectivamente:

165
Química Química geral e estequiometria

514. Cesgranrio-RJ
Tubo número 1 2 3 4 5
Os gases dióxido de enxofre e oxigênio, em
condições apropriadas, reagem para formar AgNO3
Quantidade de
trióxido de enxofre. Usando volumes iguais de matéria adicionada
4 6 8 12 14
reagentes, haverá excesso de um dos gases. = (10–3mol)
Indique a porcentagem, em volume, desse
Na3PO4
excesso em relação ao volume inicial dos re- Quantidade de
agentes: matéria adicionada 12 10 8 4 2
2 SO2 + 1 O2 → 2 SO3 = (10–3mol)

a. 25% O2 d. 75% O2
b. 25% SO2 e. 80% O2
Com base nessa tabela, é possível prever que
c. 50% O2
o tubo em que se formará a maior quantidade
515. Fuvest-SP de Ag3PO4 é o:
Qual a quantidade máxima de carbonato de a. tubo 1.
cálcio que pode ser preparada a partir da mis-
tura de 2 mols de carbonato de sódio e 3 mols b. tubo 2.
de cloreto de cálcio? c. tubo 3.
Dado: massa de um mol de carbonato de cál- d. tubo 4.
cio = 100 g e. tubo 5.
a. 100 g d. 400 g 518. UFPB
b. 200 g e. 500 g Gigantes reservas de petróleo fo-
c. 300 g ram encontradas recentemente no Brasil.
Essas reservas situam-se em regiões de
516. Mackenzie-SP
grandes profundidades em águas oceâni-
Conforme a equação abaixo equacionada: cas e abaixo de uma camada de sal, por
NaCl + AgNO3 → AgCl + NaNO3 isso, são denominadas de pré-sal. Com a
exploração dessas reservas, o Brasil au-
misturam-se 11,7g de cloreto de sódio e 34 g mentará significativamente a produção
de nitrato de prata, resultando em 1 litro, após de petróleo. Após a extração, o petróleo é
adição de água. A massa que se obtém do pre- transportado até as refinarias, onde pas-
cipitado branco vale: sará por uma série de processos de pu-
Dados: Ag = 108 ; Na = 23 ; Cl = 35,5 ; O = 16; rificação denominada de refino, em que
N = 14. o petróleo entra na fornalha, é aquecido
a. 2,87 g d. 45,7 g e segue para a torre de destilação, onde
PV-13-14

serão separadas as diversas frações.


b. 28,7 g e. 34,0 g
Considerando a combustão completa do me-
c. 17,0 g tano em um motor de carro, é correto afirmar:
517. PUC-SP
a. O oxigênio é o reagente limitante.
Ao adicionar uma solução aquosa de nitrato
b. O metano é o reagente limitante.
de prata (AgNO3) a uma solução aquosa de fos-
fato de sódio (Na3PO4), forma-se um sal branco c. O dióxido de carbono é o reagente li-
e insolúvel, o fosfato de prata (Ag3PO4). mitante.
Essa reação foi realizada utilizando-se quanti- d. O metano é o reagente em excesso.
dades variadas dos reagentes, segundo a tabe- e. O oxigênio determina o rendimento da
la abaixo: reação.

166
Química geral e estequiometria Química

519. A redução percentual do íon cloro no sal light


O fosfato de cálcio é a substância principal que em relação ao sal tradicional é igual a:
forma a estrutura dos ossos. Esse sal pode ser a. 10%
preparado, por exemplo, ao se juntar calcá- b. 20%
rio contendo 300 g de carbonato de cálcio
c. 40%
com 1,0 L de ácido fosfórico comercial (que
é uma solução aquosa de densidade igual a d. 50%
1,68 g mL–1 e que contém 87,5% em massa de 521. UFG-GO
H3PO4). Sobre essa reação, que ainda produz
água e gás carbônico, responda: Fertilizantes comumente empregados na agri-
cultura são comercializados em termos das por-
Dados: M(CaCO3) = 100 g · mol–1 e M(H3PO4) centagens em massa de nitrogênio, pentóxido
= 98 g · mol–1 de fósforo e óxido de potássio. Por exemplo,
a. Escreva a equação da reação que ocor- um fertilizante com 5% de P2O5 é representado
re entre o ácido fosfórico e o carbonato como 00-05-00. Com base nestas informações,
de cálcio. qual é a massa de fósforo em 1,0 ton de um fer-
b. Calcule o número de mols de H3PO4 na tilizante 00-26-00, com 15% de pureza?
quantidade de ácido fosfórico comer- 522. PUC-RJ
cial utilizado na reação. O elemento fósforo não ocorre livre na na-
c. Indique o reagente limitante da reação. tureza, sendo encontrado comumente como
d. Calcule a quantidade, em mol, de fosfa- rocha fosfática. Esta rocha é constituída prin-
to de cálcio que seria produzida consi- cipalmente por Ca3(PO4)2. O fósforo puro (P4)
derando a reação completa. pode ser obtido a partir desta matéria-prima
por redução com carbono, em forno elétrico, a
520. UERJ
1.500 °C (representação abaixo).
Para evitar a ingestão de quantidades exces-
sivas de sódio, foi desenvolvido o sal light, no 2Ca3(PO4)2(s) + 6SiO2(s) + 10C(s) → P4(g) + 6CaSiO3(s)
qual parte do cloreto de sódio é substituído + 10CO(g); ∆H = 3,1 · 103 kJ
por cloreto de potássio. Calcule a energia, na forma de calor (kJ), ne-
Os quadros a seguir comparam as informações cessária para o processamento de 500 kg de
nutricionais para porções iguais de dois tipos matéria-prima contendo 80% de Ca3(PO4)2.
de sal: a. 2,0 · 106
Sal tradicional
b. 5,0 · 106
c. 1,0 · 106
Quantidade por
Constituinte d. 5,0 · 103
porção
e. 2,0 · 103
Sódio 368,0 mg
PV-13-14

523. Unimontes-MG
Potássio ––– As concentrações a seguir são relativas às
massas (mg) de alguns metais ou minerais
Sal light em 100 g de plantas medicinais estudadas:
aroeira e poejo.
Quantidade por
Constituinte Minerais
porção

Sódio 184,0 mg Plantas Ca Cu Fe Mg Mn Zn

Potássio 249,6 mg Aroeira 2.020 1,11 6,79 97,9 3,04 2,26

Poejo 890 6,25 7,91 287 4,74 13,77


Além desses cloretos, não há outros compos-
tos de cloro, sódio ou potássio nos sais.

167
Química Química geral e estequiometria

Em relação aos dados fornecidos, é incorreto com rendimento total, que massa desse metal
afirmar que: é obtida a partir de 100 kg desse minério?
a. a planta poejo apresenta maior teor de Massas atômicas (g/mol): zinco = 63,5; enxo-
minerais alcalinos. fre = 32,0
b. a planta poejo apresenta 5,14% a mais a. 95,5 kg d. 40,0 kg
de teor de cobre. b. 63,5 kg e. 32,0 kg
c. a planta aroeira tem maior teor de ma- c. 52,5 kg
téria (mol) de Ca.
527. Unesp-SP
d. a planta aroeira apresenta maior quan-
tidade de minerais. Uma moeda de prata de massa 5,40 g é dis-
solvida completamente em ácido nítrico con-
524. UECE centrado. À solução aquosa resultante adicio-
O gás cloro, descoberto em 1774 pelo sueco na-se solução aquosa de cloreto de sódio de
Carl Wilhelm Scheele, pode ser obtido através modo que toda a prata é precipitada como
de eletrólise da solução aquosa de cloreto só- AgCl. A massa obtida de AgCl é de 5,74 g. São
dio cuja reação global ocorre de acordo com a dadas as massas molares, em g/mol: Cl = 35,5
equação: e Ag = 108. A porcentagem em massa de pra-
ta na moeda é igual a:
2NaCl(aq) + 2H2O() → 2NaOH(aq) + H2(g) + Cl2(g)
a. 65%
Considerando que a solução de sal apresenta b. 70%
45% em massa de NaCl, a partir de cada 100 kg
da mencionada solução, as massas de hidróxi- c. 75%
do de sódio e cloro obtidas serão, aproxima- d. 80%
damente: e. 85%
a. 36,00 kg e 31,95 kg. 528. Fuvest-SP
b. 36,00 kg e 63,00 kg. O nitrogênio pode ser obtido pela decomposi-
c. 30,77 kg e 27,30 kg. ção térmica do nitrito de amônio.
d. 30,77 kg e 54,60 kg. Calcule o volume de nitrogênio obtido, nas
525. Fuvest-SP CNTP, pela decomposição de 12,8 g de nitrito
de amônio, supondo que o rendimento da re-
Em um acidente, um caminhão carregado ação seja de 80% (em massa).
de solução aquosa de ácido fosfórico tom-
bou, derramando cerca de 24,5 toneladas Dados:
dessa solução no asfalto. Quantas toneladas • massas atômicas: H = 1,0; O = 16; N = 14.
de óxido de cálcio seriam necessárias para • NH4NO2 → N2 + 2 H2O
reagir totalmente com essa quantidade de 529. UEL-PR
PV-13-14

ácido?
O rendimento do processo de obtenção do
Porcentagem em massa do H3PO4 na solução = formaldeído (contituinte da solução aquosa co-
80%; massas molares (g/mol): H3PO4 = 98, CaO = 56 nhecida como formol) a partir do metanol, por
a. 7,5 d. 21,0 reação com O2 em presença de prata como ca-
b. 11,2 e. 22,9 talisador, é da ordem de 90%, em massa. Sendo
assim, a massa do aldeído obtida pela oxidação
c. 16,8 de 3,2 kg de metanol é: (H = 1; C = 12; O = 16)
526. UEL-PR a. 0,90 kg d. 2,7 kg
A oxidação, pela ação do oxigênio do ar, de mi- b. 1,2 kg e. 3,2 kg
nério de zinco contendo 95,5% de ZnS produz c. 2,4 kg
óxido de zinco. A redução deste óxido, pelo
carvão, produz o metal livre. Dessa maneira, Dado: (CH3OH + 1/2 O2 → CH2O + H2O)
admitindo um processo de obtenção de zinco

168
Química geral e estequiometria Química

530. Cesgranrio-RJ 533. Unisa-SP


Uma indústria adquire hidróxido de sódio 12,25 g de ácido fosfórico com 80% de pure-
impuro como matéria-prima para o seu pro- za são totalmente neutralizados por hidró-
cesso. Segundo as normas da empresa, de- xido de sódio, numa reação que apresenta
vem ser recusadas as remessas com teor de rendimento de 90%. A massa de sal obtida
NaOH inferior a 80%. Três amostras designa- nesta reação é de:
das por I, II e III, contendo cada uma 5 g do
produto, são analisadas com H2 SO4 , sendo as Dados: massas atômicas → H = 1; O = 16; Na
massas de ácido consumidas na neutraliza- = 23; P = 31.
ção, respectivamente: a. 14,76 g
b. 16,40 g
Amostra H2SO4(g) c. 164,00 g
I 4,98 d. 9,80 g
II 4,63 e. 10,80 g
III 4,52 534. Unimontes-MG
O octano, constituinte da gasolina, queima na
Do resultado da análise acima depreende-se presença de oxigênio, segundo a equação quí-
que a(s) amostra(s) aprovada(s) foi(foram): mica não balanceada:
Dados: Na = 23; O = 16; H = 1; S = 32. C8H18() + O2(g) → CO2(g) + H2O()
a. apenas a I. Nos motores de automóveis, o ar é a fonte de
oxigênio (O2) para que ocorra a combustão do
b. apenas a III.
octano. A quantidade de O2 presente no ar é
c. apenas a II e a III. de 21% v/v. Supondo que 5,0 g de octano se-
d. apenas a II. jam consumidos, a quantidade de ar (L), medi-
e. apenas a I e a II. da a 1,0 atm e 24 °C (297 K), necessária para
a combustão desse hidrocarboneto é, aproxi-
531. PUC-SP
madamente:
O clorato de potássio (KClO3) pode ser decom-
posto por aquecimento, segundo a equação: a. 13,4 c. 127,6
2 KClO3(s) → 2 KCl(s) + 3 O2(g) b. 63,8 d. 26,8
A decomposição de 2,45 g de uma amostra 535. Unifesp
contendo KClO3 produziu 0,72 g de O2. Consi- O CaCO3 é um dos constituintes do calcário,
derando que a reação foi completa e que so- importante matéria-prima utilizada na fabri-
mente a KClO3 reagiu sob o aquecimento, essa cação do cimento. Uma amostra de 7,50 g de
PV-13-14

amostra contém: carbonato de cálcio impuro foi colocada em um


a. 100% de KClO3. cadinho de porcelana de massa 38,40 g e calci-
b. 90% de KClO3. nada a 900 °C, obtendo-se como resíduo sólido
c. 75% de KClO3. somente o óxido de cálcio. Sabendo-se que a
massa do cadinho com o resíduo foi de 41,97 g,
d. 60% de KClO3. a amostra analisada apresenta um teor percen-
e. 30% de KClO3. tual de CaCO3 igual a:
532. UFPR a. 70%
Na reação de 5 g de sódio com água, houve b. 75%
desprendimento de 2,415 L de hidrogênio nas c. 80%
CNTP. Qual é o grau de pureza do sódio? d. 85%
Dados: massas atômicas → Na = 23 ; O = 16 ; H e. 90%
= 1; volume molar nas CNTP = 22,4 L/mol

169
Química Química geral e estequiometria

Para responder à questão de número 536, uti- a reserva mínima estimada de petróleo da cama-
lize o texto a seguir. da de pré-sal de Tupi, é, aproximadamente:
O debate sobre a reserva de petróleo da ca- a. 3,52 · 108 toneladas.
mada pré-sal é um dos temas polêmicos nes- b. 3,52 · 1010 toneladas.
te segundo semestre de 2008, já que envolve c. 1,14 · 1011 toneladas.
política e economia. No início de setembro, foi
feita a coleta simbólica do óleo dessa camada, d. 4,40 · 1012 toneladas.
no campo de Jubarte, Espírito Santo. e. 4,40 · 1013 toneladas.
537. FGV-SP
O clorato de potássio, KClO3, é uma substância
bastante utilizada nos laboratórios didáticos
para obtenção de gás oxigênio, a partir da sua
Camada pós-sal 2.000 m decomposição térmica, gerando, ainda, como
resíduo sólido, o cloreto de potássio. Uma
3.000 m
amostra de 12,26 g de uma mistura de sais
Camada de sal de clorato e cloreto de potássio foi aquecida
4.000 m
obtendo-se 9,86 g de resíduo sólido (KCl).
Camada pré-sal 5.000 m Considerando-se que todo o clorato de potás-
sio contido na amostra de mistura de sais foi
6.000 m decomposto, então a porcentagem em massa
de KClO3 na amostra era inicialmente igual a:
7.000 m a. 20% d. 60%
b. 40% e. 80%
A estimativa da Petrobras é que as reservas de
Tupi, Bacia de Santos, variem entre 5 bilhões c. 50%
de boe (barris de óleo equivalente; 1 boe = 538. PUC-SP
159 litros) e 8 bilhões de boe. O petróleo des- Dados: Volume molar nas condições ambiente
sas reservas é considerado de excelente quali- = 25,0L
dade, pois apresenta 28° API.
Massa molar do CaCO3 = 100 g · mol–1
O grau API, escala higrométrica idealizada para
medir a densidade relativa de líquidos, é calcu- Massa molar do CO2 = 44 g · mol–1
lado pela expressão: Massa molar do HCl = 36,5 g · mol–1
0 API =  141, 5  O calcário é um minério de grande interesse
 p  − 131, 5 industrial para a produção de cimentos e vi-
dros. O principal componente desse mineral
é o carbonato de cálcio, cuja reação em meio
PV-13-14

onde p é a densidade relativa a 15,6 °C.


ácido é representada por
Classificação do petróleo:
CaCO3(s) + 2 H+(aq) → CO2(g) + H2O() + Ca2+(aq)
API > 30: Petróleo de base parafínica
0
Uma amostra de 2,00 g de calcário foi tratada
22 ≤ 0API ≤ 30: Petróleo de base naftênica
0
com 250 mL de solução aquosa de ácido clorí-
API < 22: Petróleo de base aromática drico (HCl) de concentração 0,20 mol 0183 · L–1.
Todo o gás carbônico produzido foi recolhido,
536. FGV-SP obtendo-se 450 mL desse gás armazenado nas
Considerando que 20% do volume do petróleo condições ambiente.
pode ser completamente convertido em gasoli- Considerando-se que as impurezas presentes
na (C8H18 massa molar 114 g/mol), então a quan- no minério não produzem dióxido de carbono
tidade máxima de gás carbônico (CO2 massa em meio ácido, o teor de carbonato de cálcio
molar 44 g/mol) emitido na queima da gasolina no calcário é de:
(densidade 0,72 kg/L), produzida a partir de toda

170
Química geral e estequiometria Química

a. 25% d. 90% (AgI), conforme representado pelas seguin-


b. 45% e. 125% tes equações:
c. 70% KIO3(aq) + 3 H2SO3(aq) → KI(aq) + 3 H2SO4(aq)
KI(aq) + AgNO3(aq) → Agl(s) + KNO3(aq)
539. PUC-SP
Sabendo que a massa de iodeto de prata obtida
O anidrido sulfuroso (SO2) reage com oxigênio
foi de 4,70 · 10–5 g e considerando que M(KIO3)
(O2) e água (H2O) para formar H2SO4. Admita-
= 214 g/mol e M(AgI) = 235 g/mol, calcule, em
mos que usemos 6,4 toneladas de SO2 por dia,
gramas, a massa de iodato de potássio presente
com uma eficiência de conversão de 70%. Qual
em uma tonelada de sal. Despreze a parte fracio-
a produção de H2SO4 ao cabo de 10 dias?
nária de seu resultado, caso exista.
Dados: H = 1 ; O = 16 ; S = 32
543. PUC-SP
540. UFMG O elemento enxofre é um dos contaminantes
Em um tubo, 16,8 g de bicarbonato de sódio comuns encontrados no carvão mineral. A
são decompostos, pela ação do calor, em car- queima de compostos contendo enxofre pro-
bonato de sódio sólido, gás carbônico e água duz o dióxido de enxofre (SO2), um poluente
vapor. O volume de gás carbônico, em litros, atmosférico que causa irritação na mucosa e é
obtido nas CNTP, supondo o rendimento da precursor da chuva ácida.
reação igual a 90%, é igual a: Para se evitar a dispersão desse poluente na
Dados: NaHCO3 = 84 g/mol atmosfera, muitas fábricas utilizam em suas
a. 2,02 d. 4,03 chaminés filtros contendo carbonato de cál-
b. 2,48 e. 8,96 cio (CaCO3). Esse componente absorve o SO2,
formando o sulfato de cálcio (CaSO4), segun-
c. 4,48 do a reação:
541. UFTM-MG
2SO2(g) + 2CaCO3(s) + O2(g) → 2CaSO4(s) + 2CO2(g)
O carbonato de sódio, importante matéria-
-prima na fabricação de vidros, pode ser pro- Para absorver o SO2 liberado na queima de
duzido a partir da reação do cloreto de sódio, 320 kg de carvão, contendo 2% em massa de
amônia e gás carbônico, processo químico co- enxofre, são necessários, de CaCO3:
nhecido como processo Solvay. São apresenta- a. 6,4kg d. 20,0kg
das duas etapas deste processo. b. 10,0kg e. 100,0KG
Etapa I: NaCl + CO2 + NH3 + H2O → NaHCO3 + c. 12,8kg
NH4Cl
544.
Etapa II: 2 NaHCO3 → Na2CO3 + CO2 + H2O
Que massa de dióxido de manganês (MnO2)
Considerando que o rendimento da etapa I é deve reagir com HCl, a fim de que o gás des-
PV-13-14

75% e o da etapa II é 100%, a massa de carbo- prendido (Cl2), atravessando uma solução de
nato de sódio, em kg, que pode ser produzida NaOH concentrada e a quente, produza 53,2
a partir de 234 kg de cloreto de sódio é: gramas de NaClO3?
a. 159 d. 318 4 HCl + MnO2 → MnCl2 + 2 H2O + Cl2
b. 212 e. 424 3 Cl2 + 6 NaOH → 5 NaCl + NaClO3 + 3 H2O
c. 283 a. 21,72 g
542. UnB-DF b. 130,5 g
Um aluno decidiu realizar um projeto de c. 213,12 g
Química para sua escola, investigando o teor
d. 420 g
de iodato de potássio em uma marca de sal.
Uma amostra de massa igual a 1,0 g do sal e. 522 g
de cozinha foi dissolvida em água e o iodo Dados: O = 16; Na = 23; Cl = 35,5; Mn = 55
foi precipitado na forma de iodeto de prata

171
Química Química geral e estequiometria

545. UFPB 548. Cesgranrio-RJ


Com base nas equações: A cebola, ao ser cortada, desprende SO2 que,
em contato com o ar, transforma-se em SO3.
ZnS + 3/2 O2 → ZnO + SO2 Este gás, em contato com a água dos olhos,
ZnO + CO → Zn + CO2 transforma-se em ácido sulfúrico, causando
grande ardor e, consequentemente, as lágri-
calcule a massa de blenda (ZnS) necessária mas. Estas reações estão representadas abaixo:
para produzir uma tonelada de Zn.
SO2 + 1/2 O2 → SO3
Massas molares em g/mol: O = 16; S = 32; Zn = 65
SO3 + H2 O → H2 SO4
546. Fuvest-SP
Pela sequência de reações: Supondo que a cebola possua 0,1 mol de SO2
C + O2 → CO2 e o sistema esteja nas CNTP, determine o volu-
me de ácido sulfúrico produzido.
CO2 + NaOH → NaHCO3
a. 2,24 L d. 22,4 L
qual é a massa de hidrogenocarbonato de
sódio que se pode obter a partir de 1,00 g de b. 4,48 L e. 44,8 L
carbono? c. 5 L
São dadas as massas molares: H = 1; C = 12; Dado: volume molar nas CNTP = 22,4L/mol
O = 16; Na = 23 549. UEL-PR
a. 3,7 g d. 8,4 g O polímero (CF2)n pode ser obtido a partir do
b. 4,3 g e. 84,0 g polietileno, (CH2)n.
c. 7,0 g (CH2)n + 4nCoF3 → (CF2)n + 2nHF + 4nCoF2
547. UFPI 4nCoF2 + 2nF2 → 4nCoF3
Superóxido de potássio, KO2, é utilizado em O CoF3 é então reciclado.
aparelhos do tipo sistema fechado, para
respiração. O ar exalado contém dióxido de Nesse processo cíclico, para a produção de 100
carbono e água sendo ambos removidos, mols de (CF2)n, gastam-se 100 mols de polieti-
ocorrendo, simultaneamente, a geração de leno e quantos de F2?
oxigênio.
Dado: n = número grande
4 KO2(s) + 2 H2O( ) → 3 O2(g) + 4 KOH(s)
a. 10n d. 200n
KOH(s) + CO2(g) → KHCO3(s)
b. 50n e. 400n
Assinale a alternativa correta: c. 100n
a. Para a remoção de 1 mol de CO2(g) são
necessários 4 mols de KO2(s). 550. UERJ
PV-13-14

b. Para uma mesma quantidade de Uma das principais causas de poluição atmos-
KO2(s),remove-se maior massa de CO2(g) férica é a queima de óleos e carvão, que libera
que de água. para o ambiente gases sulfurados. A sequência
c. Para uma quantidade fixa de KO2(s) reacional abaixo demonstra um procedimento
remove-se maior número de mols de moderno de eliminação de anidrido sulfuroso,
H2O( ) que de CO2(g). que consiste em sua conversão a gesso.
d. Mais moléculas de substâncias no esta- SO2 + H2O → H+ + HSO3−
do gasoso são produzidas que consumi-
H+ + HSO3− + 1/2 O2 → 2H+ + SO24−
das, de acordo com as reações acima.
e. Este aparelho pode ser usado indefini- 2H+ + SO24− + Ca(OH)2 → CaSO4 ⋅ 2H2O
  
damente sem possibilidade de exaus- Gesso
tão do sistema. Dados: H = 1; O = 16; S = 32; Ca = 40

172
Química geral e estequiometria Química

Calcule a massa de gesso, em gramas, que 554. UFF-RJ


pode ser obtida a partir de 192g de anidrido Garimpeiros inexperientes, quando encon-
sulfuroso, considerando um rendimento de
tram pirita, pensam estar diante de ouro: por
100% no processo de conversão.
isso, a pirita é chamada “ouro dos tolos”.
551. Fuvest-SP
Entretanto, a pirita não é um mineral sem apli-
Polietileno pode ser obtido do etanol pela se- cação. O H2SO4, ácido muito utilizado nos labo-
quência de reações abaixo equacionadas: ratórios de química, pode ser obtido a partir
C2H5OH → C2H4 + H2O da pirita por meio do processo:
n C2H4 → (C2H4)n
4 FeS2 + 11 O2 →
 2 Fe2O3 + 8 SO2
Quantos milhares de litros de etanol são VO
necessários para a produção de 5,6 ton de 2 SO2 + O2  
2 5
→ 2 SO3
polietileno? Suponha reações com 100% de H SO
rendimento. SO3 + H2 O 
2 4 ( dil )
→ H2 SO4
Dados: densidade do etanol = 0,8 kg/L;
Assinale a opção que indica a massa de H2SO4 ob-
massa molar do C2H5OH = 46 g/mol; tida a partir de 60 kg de pirita, com 100% de pu-
massa molar de C2H4 = 28 g/mol.) reza, por meio do processo equacionado acima.
552. Fuvest-SP modificado a. 9,8 kg
O equipamento de proteção conhecido como b. 12,4 kg
air bag, usado em automóveis, contém subs- c. 49,0 kg
tâncias que se transformam, em determinadas d. 60,0 kg
condições, liberando N2 que infla um recipien-
te de plástico. As equações das reações envol- e. 98,0 kg
vidas no processo são:
555.
2 NaN3 → 2 Na + 3 N2
Uma fábrica multinacional foi acusada por um
10 Na + 2 KNO3 → K2O + 5 Na2O + N2 jornalista de estar lançando SO2 na atmosfera, e
Considerando que N2 é gerado nas duas re- com isso colaborando para a formação de chuva
ações, calcule a massa de azoteto de sódio ácida, que vem transformando o mármore dos
edifícios em gesso, pela seqüência de reações:
(NaN3) necessária para que sejam gerados 80 L
de nitrogênio, nas condições ambiente. 1/2
Dados: volume molar de gás nas condições am-
bientes: 25 L/mol; massa molar do NaN3: 65 g/mol
553. Fuvest-SP Mármore Gesso
Duas das reações que ocorrem na produção A fábrica publicou, então, sua defesa, afirman-
PV-13-14

do ferro são representadas por: do que a chuva ácida não é causada apenas pelo
2 C(g) + O2(g) → 2 CO(g) poluente SO2; que ela, na verdade, pode ser
considerada um fenômeno natural que ocorre
Fe2O3 + 3 CO(g) → 2 Fe(g) + 3 CO2(g) quando fortes descargas elétricas, normalmen-
O monóxido de carbono formado na primeira te em tempestades, fazem o gás nitrogênio rea-
reação é consumido na segunda. Consideran- gir com o gás oxigênio no ar, produzindo, numa
do apenas essas duas etapas do processo, cal- primeira etapa, monóxido de nitrogênio e, em
cule a massa aproximada, em quilogramas, de seguida, dióxido de nitrogênio, que, na presen-
carvão consumido na produção de 1 t de ferro. ça de água, produz ácido nítrico e ácido nitroso.
Dados: massas atômicas → Fe = 56; C = 12; O = 16. A fábrica concluiu sua defesa dizendo que o
ácido nítrico também ataca o mármore e es-
quematizou as reações a seguir:

173
Química Química geral e estequiometria

N2 + O2 → 2 NO ácido sulfúrico. A quantidade de H2SO4, em


2 NO + O2 → 2 NO2 kg, que cairá sobre o solo, como resultado da
queima do enxofre, é igual a:
2 NO2 + H2O → HNO3 + HNO2
Dados: H = 1 ; S = 32 ; O = 16
2 HNO3 + CaCO3 → Ca(NO3)2 + CO2 + H2O
a. 45,94
A respeito dessa polêmica, considerando para b. 22,97
as reações um rendimento igual a 100%, res-
ponda às questões a seguir. c. 68,90
d. 91,88
Dados: H = 1 ; O = 16 ; C = 12 ; S = 32 ; Ca = 40 ;
volume molar nas CNTP = 22,4 L/mol e. 114,85
a. Qual massa de gesso que pode ser for- 558. UFRGS-RS
mada pela reação de 2.240 L de SO2 lan-
çados na atmosfera, nas CNTP? Trataram-se 3,3g de uma mistura de CaCl2 e
NaCl com carbonato, a fim de precipitar todo o
b. Quantos mols de dióxido de nitrogênio
cálcio sob forma de CaCO3, que foi então aque-
devem reagir com água de chuva para
cido e transformado em CaO puro. A massa fi-
acabar produzindo uma massa de 2,46
nal do CaO obtida foi 0,56 g. A porcentagem
kg de nitrato de cálcio?
em massa de CaCl2 na mistura primitiva era de,
556. Fuvest-SP aproximadamente:
Um sólido S é decomposto por aquecimento e o a. 1,1%
produto sólido obtido, ao reagir com água, for- b. 3,3%
ma hidróxido de cálcio. Este reage com carbona-
to de sódio produzindo soda cáustica (NaOH) e c. 11,1%
regenerando o sólido S, que é reciclado. d. 33,3%
Qual a fórmula de S e sua respectiva massa e. 66,6%
necessária para iniciar um ciclo de produção
559. UFF-RJ
de soda cáustica a partir de 1,06 toneladas de
carbonato de sódio? Mirella, uma das técnicas de produção do
KMnO4 requer duas reações características. Na
Admita, em todas as etapas, um rendimento
primeira, o MnO2 é convertido a K2MnO4, por
de 100%.
reação com KOH fundido na presença de O2:
Dados: massas molares (g/mol): C = 12; O = 16;
Na = 23 e Ca = 40 MnO2 + 2 KOH + 1/2 O2 → K2MnO4 + H2O
a. CaO e 0,56 t Na segunda, K2MnO4 é convertido em KMnO4
b. CaO e 1,12 t por reação com Cl2:
c. Ca(OH)2 e 1,06 t K2MnO4 + 1/2 Cl2 → KMnO4 + KCl
d. CaCO3 e 1,00 t Que massa de Cl2 é necessária para produzir
PV-13-14

e. CaCO3 e 2,00 t KMnO4, partindo-se de 10,0 g de MnO2?


557. Unifacs-BA a. 4,1 g
Uma fábrica, para produzir ácido sulfúrico, b. 8,1 g
queima 0,5 t de enxofre por dia, sendo que c. 10,1 g
3,0% se perdem na atmosfera, sob a forma de
SO2. Este sofre oxidação, dando SO3, que reage d. 18,3 g
com água existente na atmosfera, produzindo e. 36,5 g
Dados: O = 16 ; Cl = 35,5 ; Mn = 55

174
Química geral e estequiometria Química

560. Fuvest-SP
A decomposição térmica por aquecimento gradual e contínuo (ao ar) do acetato de manganês (II)
tetraidratado, sólido, ocorre em duas etapas.
°C
Mn (CH3COO)2 ⋅ 4H2O (s) 130
 → Mn (CH3COO)2(s) + 4 H2O(g)

350 ° C
Mn (CH3COO)2(s) 
→ MnO(s) + (CH3 )2 CO(g) + CO2(g)

Certa massa do sal hidratado é aquecida nessas condições. Qual dos gráficos abaixo representa o
que ocorre com a massa (m) da fase sólida com o aumento da temperatura (t)?

m m m
a. c. e.

t t t

m m
d.
b.

t t
PV-13-14

175
Química Química geral e estequiometria

ANOTAÇÕES

PV-13-14

176
Química geral e estequiometria R: Química

GABARITO DOS EXERCÍCIOS PROPOSTOS


Capítulo 01 V T = 4/3 · π · r3T e VE = líquido e adiciona-se uma pe-
4/3 · π · r3E . Assim, V T/V E = quena quantidade de água.
01. A 04. D (rT/rE) 3. No tubo em que se forma a
02. C 05. B O raio do átomo na estre- mistura homogênea está o
la rE = (0,01/2) Å = 0,005 Å, en- metanol. No tubo em que se
03. B 06. E
quanto que rT = 0,77 Å (dado). formam duas camadas, com a
07. m = 360 + 8 + 1 + 100 + 90 + água na camada superior, está
Aplicando-se a equa-
+ 100 + 3 =662 g o tetracloreto de carbono,
ção, tem-se: VT/VE = (rT/rE)3 =
(0,77/0,005)3 = 3.652.264 = que é mais denso que a água.
m 3,65 · 106. 33. A 36. D
d=
v 34. B 37. B
18. 21 (01 + 04 + 16)
662
v= ≅ 601, 8 cm3
19. B 35. A 38. E
1,1
20. D
b. Quando misturamos 39. a. A fusão ocorre quando
os ingredientes da "massa", Capítulo 02 uma substância passa do esta-
ocorrem interações entre eles, do sólido para o estado líquido.
o que acarreta em mudança de 21. E O estado sólido é caracterizado
volume e, consequentemente, 22. D principalmente pelo arranjo es-
de densidade. pacial organizado de átomos ou
23. B
08. E moléculas, a uma distância rela-
24. C tivamente pequena. Já o estado
09. Líquido: clorofórmio, eta- líquido se caracteriza por uma
nol, fenol. 25. C
maior liberdade de movimento
Gasoso: éter etílico, pentano. 26. a. TA = TB = 100 °C. Re- das moléculas ou átomos numa
presentam a temperatura de distância média um pouco su-
10. B ebulição da água pura sob perior àquela do estado sólido.
11. A pressão de 1 atm. Portanto, para passar ao estado
12. C b. Béquer B, pois leva líquido, os átomos ou moléculas
mais tempo para ebulir. do sólido devem romper as for-
13. D c. Béquer A → 10 minu- ças de atração existentes. Quan-
14. C tos para ebulir 200 g de H2O. to mais fortes as interações,
15. A Béquer B → 15 minutos mais alta será a temperatura de
para ebulir x g de H2O. fusão. Como pode ser visto na
16. C tabela e nas figuras, o diamante
10 min 200 g 
PV-13-14

17. a. Sim, a analogia está  x = 200 g H2O é praticamente infusível, já que


correta. Por se tratar de 15 min x  a separação entre seus átomos
mudança de estado físico, envolve a quebra de ligações
a temperatura do sistema d. Béquer B, porque apre- químicas. Para a fusão do grafi-
água permanece constante senta maior quantidade de água. te, basta que as camadas planas
durante o processo de soli- 27. A de átomos de carbono se sepa-
dificação, assim como ocorre rem. Como entre as camadas há
para o caso das estrelas de 28. B apenas interações de Van der
diamante, conforme afirma o 29. C Waals e estas são mais fracas
texto. que ligações químicas propria-
b. Como o átomo é consi- 30. D
mente ditas, o processo pode
derado uma esfera, o seu volume 31. Verdadeiros: 0, 1 e 3 ocorrer a 4.600 K.
é dado por: V = 4/3 · π · r3. Apli-
cando-se a equação aos dois 32. Coloca-se em três tubos b. No diamante a hibri-
átomos, tem-se: de ensaio um pouco de cada dização dos átomos de carbo-

177
Química R: Química geral e estequiometria

no é sp3 e, no grafite, sp2. No caso do grafite, a hibridização sp2 58. D


permite a existência de ligações duplas alternadas com ligações 59. C
simples, de forma semelhante ao que ocorre no benzeno. Dize-
mos que as duplas ligações se conjugam ao longo das camadas 60. B
planas, permitindo o movimento praticamente livre dos elé- 61 13 (01 + 04 + 08)
trons nesses planos. Por isso observa-se corrente elétrica com
a aplicação de uma diferença de potencial. O mesmo não se ob- 62. a. Há cinco elementos
serva para o diamante, pois na estrutura do mesmo não existem químicos: um no balão I e qua-
elétrons pi capazes de se deslocar por toda a estrutura, o que faz tro no balão II, e há três substân-
dessa substância um isolante. cias simples: uma no balão I e
duas no balão II.
40. D b. Dois sistemas homo-
41. D gêneos, pois todos os compo-
nentes são gasosos.
42. E
63. 06 (02 + 04)
43. 15 (01 + 02 + 04 + 08)
44. 64 B 72. C
65. E 73. C
θ (°C)
66. D 74. E
Liquefação 67. D 75. D
68,2
68. A 76. B
Início da solificação
69. E 77. D
Solidificação
70. D 78. C
t (min) 71. D
45. 79. 30 (02 + 04 + 08 + 16)
θ (°C)
80. a. Tubo 1: Está presente
o CHCl3 (clorofórmio). Apre-
sentando as moléculas dessa
Fusão
Início da ebulição substância uma baixa pola-
192 ridade, na presença de água
tem-se um sistema heterogê-
t (min) neo (bifásico), no qual a água,
por apresentar uma menor
46. B densidade do que o solvente
47. D orgânico, estará presente na
PV-13-14

fase superior do tubo de en-


48. D saio, conforme representado
49. A na figura.
50. D Tubo 2: Está presente o
51. 12 (04 + 08) CH3CH2OH (etanol). Sendo as
moléculas dessa substância
52. A polares, e, portanto, poden-
53. 26 (02 + 08 + 16) do realizar ligações de hidro-
gênio com as moléculas de
54. C
água, esses dois compostos
55. 01 são miscíveis entre si, levan-
56. A do à formação de um sistema
homogêneo, conforme repre-
57. A sentado na figura.

178
Química geral e estequiometria R: Química

Tubo 3: Está presente a gasolina, CaCl2(aq) + (NH4)2C2O4(aq) + H2O() → CaC2O4 · H2O(s) + 2 NH4Cl(aq)
substância composta por molé-
CaC2O4 · H2O(s) → CaO(s) + CO2(g) + CO(g) + H2O(g)
culas de natureza apolar, que
formam com a água (moléculas Ou seja:
polares) um sistema bifásico. Em
virtude de sua baixa densidade, 1 mol CaCO3(s) 1 mol CaO(s)
a gasolina ocupará a parte supe- 100 g 56 g
rior do tubo, conforme repre- x 2g
sentado na figura.
100 ⋅ 2
b. Nem todos os líquidos x= ≅ 3, 6 g CaCO3
indicados são substâncias puras. 56
A gasolina é uma mistura de hi- 5g 100%
drocarbonetos obtidos a partir 3, 6 g y
da destilação do petróleo em y = 72%
determinado intervalo de tem-
peratura. 88. 02
81. B 89. C
82. C 90. B
83. a. Espaço 1: areia 91. D
Espaço 2: óleo 92. C
b. Espaço 1: sedimenta- 93.
ção (decantação)
Espaço 2: decantação NaCl, AgCl, PbCl2,
(s) (s) (s)
84. E
85. B Água fria
86. A filtração
87. a. 1a filtração: O resíduo
sólido retido no filtro é forma- PbCl2(s) NaCl(aq) (Evaporação)
do por substâncias constituin- AgCl(s)
tes da concha, que são quimi-
camente resistentes ao ataque Água quente
do HCl e insolúveis em sua so- Filtração
lução. O filtrado conterá uma
PV-13-14

solução aquosa de Ca2+, Cl– e


outros sais solúveis.
AgCl(s) PbCl2(aq) (Evaporação)
2a filtração: O resíduo
sólido será CaC2O4 · H2O, e o Devem ser seguidos os seguintes passos:
filtrado será uma solução con- 1 – adicionar água fria;
tendo íons NH4+ , Cl– e outros 2 – filtrar;
solúveis. 3 – vaporizar a água (obtém-se o NaCI);
b. CaC2O4 · H2O(s) → CaO(s) 4 – adicionar água quente;
+ CO2(g) + CO(g) + H2O(g) 5 – filtrar (separa-se o AgCI(s) da solução de PbCI2);
c. O processo pode ser 6 – vaporizar a água (obtém-se o PbCl2).
assim representado: 94. A
CaCO3(s) + 2 HCl(aq) → CaCl2(aq) + 95. Adicionamos inicialmente água à mistura, dissolvendo o NaCl,
H2O() + CO2(g) mas não a areia. Filtramos o material, retendo apenas a areia. Dei-

179
Química R: Química geral e estequiometria

xamos evaporar o solvente (H2O) para a obtenção do NaCl sólido. O álcool é solúvel em água
Os processos usados foram: dissolução fracionada e filtração. devido à sua parte polar
96. Adiciona-se água, agita-se. O nitrato de sódio dissolve-se. (– CH 2 – OH), ocorrendo pon-
Filtra-se. O carvão e o enxofre são retidos. Por vaporização, te de hidrogênio entre as
separa-se a água do nitrato de sódio. Adiciona-se dissulfeto de duas espécies.
carbono à mistura de carvão e enxofre. O enxofre se dissolve. A parte apolar do etanol
Filtra-se. O carvão fica retido. Por vaporização, separa-se o dis- (H3C – CH2) é responsável pela
sulfeto de carbono do enxofre. dissolução da fenolftaleína
97. a. Sólido por meio de forças de Van
der Waals entre as duas es-
b. Sim, pois a glicerina é solúvel em água e o eugenol não
pécies.
é solúvel em água.
A adição de água (bastante po-
98. A 100. D 102. A lar) na solução de fenolftaleína
99. A 101. C em etanol deixa o meio mais
polar, provocando a cristalização
103. a. Destilação da fenolftaleína. Esta é insolúvel
em solvente bastante polar.

Capítulo 03
b.
121. A
122. B
123 Massa de 23 átomos de
104. A 108. 19 (01 + 02 + 16)112. A oxigênio = 23 · 16
105. A 109. D 113. C Massa de 16 átomos de sódio
106. B 110. A 114. 01 = 16 · 23
107. D 111. D \ afirmação verdadeira.
124. B
115. a. Fermentação alcoólica ou fermentação. Destilação
fracionada. 125. E
b. A função da vidraria v1 (condensador) é condensar os 126. C
vapores que estão sendo destilados. O etanol forma menos pon- 127. E
tes de hidrogênio, comparado à água, fazendo com que o seu
ponto de ebulição seja menor e/ou a pressão de vapor e/ou a 128. a. Cl: p = 17
37

sua volatilidade sejam maiores. e = 17


116. A 117. A 118. A n = 20
PV-13-14

119. V, V, V, F, F, V, F b.
120. a. 100 g da mistura → 20 g f e 80 g l M ⋅ A1 ⋅ X + M ⋅ A 2 ⋅ Y
M⋅ A =
Lactose: 80 °C→ 80 g dissolvidos 100
25 °C → 25 g dissolvidos 34 , 97 ⋅ x + 36, 97 ⋅ (100 − x)
35, 45 =
100
Portanto, 55 g de lactose devem cristalizar. x = 76% (35 Cl)
b. Temos 20 g de fenolftaleína:
y = 24% (37 Cl)
100 mL — 6,7 g
350 mL — x 129. C

x = 23,45 g 130. 26 (02 + 08 + 16)

O volume de etanol é suficiente para dissolver toda a fenolf- 131. B


taleína. 132. C

180
Química geral e estequiometria R: Química

133. 30 (02 + 04 + 08 + 16) h. Al = 27 · 2 = 54 147. a. C = 12 · 1 = 12


134. D S = 32 · 3 = 96 O = 16 · 1 = 16
O = 16 · 12 = 192 N = 14 · 2 = 28
135.
FM = 342 u H=1·4=4
i. N = 14 · 3 = 42 MM = 60 u
u
H = 1 · 12 = 12 b. H = 1 · 2 = 2
136. C
P = 31 · 1 = 31 S = 32 · 2 = 64
137. E
O = 16 · 4 = 64 O = 16 · 3 = 48
138. B
MF = 149 u MM = 114 u
139.
j. Cu = 63,5 · 1 = 63,5 c. Na = 23 · 2 = 46
M.A1 ⋅ x + M.A2 ⋅ y + M.A 3 ⋅ z N = 14 · 2 = 28,0 B = 11 · 4 = 44
M.A =
100
O = 16 · 6 = 96,0 O = 16 · 17 = 272
80 ⋅ 60 + 84 ⋅ 20 + 88 ⋅ 20
M.A = MF = 187,5 u H = 1 · 20 = 20
100
M.A = 82, 4 u k. Fe = 56 · 7 = 392 MF = 382 u
C = 12 · 18 = 216
140. C 148. B 151. E
N = 14 · 18 = 252
141. a. C = 12 · 2 = 24 149. D 152. A
MF = 860 u
H=1·6=6 150. C 153. C
l. Na = 23 · 2 = 46
MM = 30 u 154. 0. Verdadeiro
S = 32 · 1 = 32
b. S = 32 · 1 = 32 1. Verdadeiro
O = 16 · 4 = 64 2. Falso: massas iguais
O = 16 · 3 = 48
H = 1 · 20 = 20 de diferentes elementos quí-
MM = 80 u micos contêm diferentes nú-
c. N = 14 · 1 = 14 O = 16 · 10 = 160
meros de átomos.
H=1·3=3 MF = 322 u 3. Verdadeiro: pois é o
m. H = 1 · 4 = 4 padrão.
MM = 17 u
d. S = 32 · 8 = 256 P = 31 · 2 = 62 155. E
MM = 256 u O = 16 · 7 = 112 156. A
e. H = 1 · 2 = 2 MM = 178 u 157. V, V, V, V
S = 32 · 1 = 32 n. Cu = 63,5 · 1 = 63,5 158. D
O = 16 · 4 = 64 S = 32 · 1 = 32,0 159. A
PV-13-14

MM = 98 u O = 16 · 9 = 144,0 160.
f. Ca = 40 · 1 = 40 H = 1 · 10 = 10,0
V = A ⋅ h ⇒ V = 8.000 ⋅ 10 = 8 ⋅ 104 km3
C = 12 · 1 = 12 MF = 249,5 u
V = 8 ⋅ 104 km3 = 8 ⋅ 1013 m3
O = 16 · 3 = 48 142. A 20 mol de CO = 12 ⋅ 1024 moléculas
MF = 100 u 143. I. Correta Portanto :
g. Na = 23 · 1 = 23 II. Correta
8 ⋅ 1013 m3 __________ 12 ⋅ 1024 moléculas
H=1·1=1 III. Correta
1 m3 __________ x
S = 32 · 1 = 32 144. A
x = 1, 5 ⋅ 1011 moléculas / m3
O = 16 · 4 = 64 145. A 161. A
MF = 120 u 146. E 162. C

181
Química R: Química geral e estequiometria

163. E 166. B 169. B 194. Cada átomo de cálcio seria substituído


164. A 167. C 170. C por um átomo de bário, pois eles apresen-
tam a mesma valência.
165. D 168. B 171. C 1,6 kg Ca:
172. C
1 mol Ca 40 g 6 ⋅ 1023 átomos
173. a. O elemento químico que aparece no de- 
tergente e não no sabão é o enxofre: S 1, 6103 g x 
b. C12H25O4XNa = 288 x = 2, 4 ⋅ 1025 átomos
12 · 12 + 25 · 1 + 4 · 16 + Mx + 1 · 23 = 288
Mx = 32 g/mol 1 mol Ba 137 g 6 ⋅ 1023 átomos

174. B y 2, 4 ⋅ 1025 
175. Supondo-se uma massa de 1 g: 137 ⋅ 2, 4 ⋅ 1025
y= 8 ⋅ 102 =
= 54 ,8
6 ⋅ 1023
1 mol H2O 18 g 3 ⋅ 6 ⋅ 1023 átomos 5, 48 ⋅ 103 g = 5, 48 kg
1g x
Massa do indivíduo sem cálcio = 70 kg – 1,6
x = 1 ⋅ 1023 átomos kg = 68,4 kg
1 mol C2H6 O 46 g 9 ⋅ 6 ⋅ 1023 átomos Massa com bário no lugar do cálcio:
1g y 68,4 + 5,48 = 73,9 kg
y = 1,2 ⋅ 1023 átomos 195. A
O frasco com álcool. 196. 1 mol H3C – N = C = O → 57g – 6 · 1023
moléculas
176. A 5·10-5g – x
177. D
x ≅ 5 · 1017 moléculas.
178.
197. D
1 mol C2H6 O 46 g 6 ⋅ 6 ⋅ 1023 átomos de H 198. C
100 g x 199. C
x = 7, 83 ⋅ 1024 átomos de hidrogênio 200. C

179. D 185. 19 (01 + 02 + 16)


Capítulo 04
180. A 186. A
201. a. 1 atm -------- 76 cmHg
181. B 187. D x --------19 cmHg x = 0,25 atm
PV-13-14

182. E 188. D b. 1 atm --------760 torr


183. E 189. E x -------- 1.140 torr x = 1,5 atm
c. 1 atm --------760 mmHg
184. E 190. B
x -------- 304 mmHg x = 0,4 atm
191. B d. 1 atm --------76 cmHg
192. x -------- 380 cmHg x = 5 atm
1 g ––– 100% e. 1 atm --------760 torr
x -------- 60,8 torr x = 0,08
x ––– 90% x = 0,9 g Pt atm
1 mol Pt ––– 195 g ––– 6 · 1023 Pt f. 1 atm --------760 mmHg
0,9 g ––– y y = 2,77 · 1021 Pt x -------- 152 mmHg x = 0,2 atm
193. B

182
Química geral e estequiometria R: Química

202. a. 1 m3 -------- 1.000 L 216. D


0,15 m -------- x x = 150 L
3 217. B
b. 1 L -------- 1.000 cm3 218. V1 V2 = V1 + 2/3
x -------- 280 cm3 x = 0,28 L V2 = 5/3 V1
c. 1 L -------- 1 dm3 P1 P2 = ?
x -------- 40 dm
3 x = 40 L P1 · V1 = P2 · 5/3 V1 ⇒ P2 = 3/5 P1
d. 1 L -------- 1.000 mL \ para que o volume sofra um
x -------- 25 mL x = 0,025 L aumento de 2/3, sua pressão
e. 1 m3 -------- 1.000 L deve reduzir para 3/5 da inicial.
5 · 10-3 m3 -------- x x=5L 219. D
f. 1 L -------- 1.000 cm3 220. D
x -------- 2 · 103 cm3 x = 2 L 221.
203. T (K) = T (ºC) + 273 V = constante
a. T(K) = 273 + 273 = 546 K
T1 = 73 °C + 273 = 200 K
b. T(K) = 0 + 273 = 273 K
c. T(K) = 727 + 273 = 1.000 K P1 = 600 mmHg
d. T(K) = – 23 + 273 = 250 K T2 = 77 °C + 273 = 350 K
e. T(K) = – 273 + 273 = 0 K
f. T(K) = – 100 + 273 = 173 K P2 = ?
204. D P1 P2
=
T1 T2
205. D
600 P
206. C = 2
200 350
207. a. v2 = 6L
P2 = 1.050 mmHg
v3 = 3L
222. B
Pressão (atm) 223. E
5
224. B
4
225. E
3
226. 27 °C = 300 K
2
1 177 °C = 450 K
PV-13-14

P0 = 1 atm P1 = ?
0 1 2 3 4 5 6 7 8 9 10 11 12 Volume
1 P
b. ≅ 0,5 mol = 1 ⇒ P1 = 1, 5 atm
300 450
208. C 210. A 212. B 227. C
209. D 211. C 228. C
213. a. Lei de Boyle – Mariotte – Transformação isotérmica 229. C
b. P1 V1 = P2 V2 230. B
1 · 10 = P2 · 2 231. E
P2 = 5 atm 232. B
214. D 233. A
234. D
215. D

183
Química R: Química geral e estequiometria

235. 20 248.
236. A P ⋅ V P' ⋅ V '
237. A =
T T'
238. C
239. 09 (01 + 08) 120 ⋅ 10 20 ⋅ 40
=
240. A 600 T'
241.
= 400 K = 127 °C
P ⋅ V P '⋅ V '
=
T T'
1 ⋅ 57 2 ⋅ V ' ∆T = T ' − T = 400 − 600 = 200 K (200 °C)
= V ' = 38 L
300 400 249.
242.
V0 = 6 L
P ⋅ V P '⋅ V ' P0 = 2 , 5 atm
=
T T' T0 = 300 K
1 ⋅ 500 P '⋅ 1 ⋅106
= V1 = V0 − 1 = 5 L (contração)
273 2.000
P1 = P0 + 0 , 5 = 3 atm (aumento de pressão)
243. E
T1 = ?
244.
2 ,5 ⋅ 6 3 ⋅ 5
= ⇒ T1 = 300 K
PT ⋅ VT PV ⋅ VV 300 T1
=
TT TV
1 ⋅ 24 , 6 100 ⋅ VV b. Transformação isotérmica
=
300 600 250.
245. C V0 = 60 mL V1 = ?
246. P0 = 0 , 8 atm P1 = 0 , 4 atm
P ⋅ V P' ⋅ V ' T0 = 200 K T1 = 400 K
=
T T' 0 , 8 ⋅ 60 0 , 4 ⋅ V1
=
P ⋅ 30 2 ⋅ P ⋅ V ' 200 400
=
2⋅T V1 = 240 mL
PV-13-14

T
V ' = 30 L
251. D
247. V0 = 10 L V1 = ?
P0 = 1 atm P1 = 0,25 atm
P ⋅ V P '⋅ V '
= T0 = 300 K T1 = 223 K
T T'
1 ⋅ 10 0,25 ⋅ V1
1 ⋅ 50 0 , 4 ⋅ V ' =
= 300 223
293 223
V1 = 29,7 L

V ' ≅ 95 L

184
Química geral e estequiometria R: Química

252. 256. D
P ⋅ V P′ ⋅ V ′ 3⋅P 5⋅P 257.
= P′ = P − =
T T′ 8 8 P1 ⋅ V1 P2 ⋅ V2
=
T1 T2
P ⋅ V 5 ⋅ P ⋅ V′ ⋅ 8 1, 5 ⋅ 40 4 ⋅ 20
= =
T 8⋅5⋅T 300 T2

V′ = V T2 = 400 K = 127 °C
258.
253.
O diagrama pressão × volume para as
V1 = 38 L
etapas do motor de Stirling pode ser de-
T1 = 127 °C = 400 K monstrado por:
P1 = 720 mmHg P
V2 = ?
2
T2 = 273 K
P2 = 760 mmHg 1 3
P1 ⋅ V1 P2 ⋅ V2
=
T1 T2 4
V
720 ⋅ 38 760 ⋅ V2
=
400 273
259. A
V2 = 24,57 L (CNTP) 260.
254. E
P ⋅ V P' ⋅ V '
T0 = 288 K T1 = 298 K =
T T'
P0 = 600 mmHg P1 = 760 mmHg 760 ⋅10 300 ⋅ V '
V0 = (alto da montanha) V1 = (pé da montanha) =
300 240
600 ⋅ V0 760 ⋅ V1 V ' ≅ 20, 31
=
288 298 261. A
V0 m 28 g –––
1 mol N2 ––– V 25 L
= 1,2
V1 x ––– 100 L · x = 112 g N2
255. 262. C
PV-13-14

P0 P1 263.
V0 v1 = v 0 + 1/ 4 ⋅ v 0 = 5/ 4 ⋅ V0 1 mol O2 32 g 22, 4 L (CNTP)

T0 T1 = T0 − 1/ 4 ⋅ T0 = 3/ 4 ⋅ T0 8 ⋅ 103 g x 
5 x = 5, 6 ⋅ 103 L = 5, 6 m3
P0 ⋅ V0 P1 ⋅ 4 ⋅ V0
=
T0 3 264. C
⋅T
4 0 265.
3
P1 ⋅ ⋅ P0 1 mol ___ 22,4 L (CNTP) ___ 6,0 · 1023
5 moléculas
\ sua pressão se reduz a 3/5 da inicial.
56 · 10-3 L ___ x
x = 1,5 · 1021 moléculas

185
Química R: Química geral e estequiometria

266. D 268. B m
P⋅ V = ·R· T
267. C 269. E M
270. m
1 · 4 , 92 · 103 = ⋅ 0, 082 ⋅ 300
32
1 mol CO2 _____ 44 g _____ 25 L (Cond. ambientes )
 x = 10 L m = 6,4 · 103 g O2
17, 6 g _____ x 
294. D
271. E 273. B 275. E 295. E
272. A 274. C 296. E
276. De acordo com o texto, se o mesmo número de moléculas 297.
apresenta a mesma massa, é porque os dois compostos apresen-
tam a mesma massa molar. m
P ⋅ V = ⋅R ⋅ T
M
MH S = MFosfina = 34 g · mol -1 85
2 2 ⋅ 10 = ⋅ 0, 082 ⋅ 300
M
277. D M = 104 , 5 g /mol
278. D
M CClF3 = 104,5 g/mol
279. De acordo com a lei de Avogadro, o número de móleculas
será o mesmo. Aquele que fornecerá maior quantidade de ener- M C2Cl3F3 = 187,5 g/mol
gia é o butano, pois é o que contém a maior energia para o mes- Portanto, o gás citado é o CClF3
mo número de mols.
298. D 300. E
280. E 283. D
299. E 301. A
281. C 284. B
302.
282. E 285. C
m
P ⋅ V = ⋅R ⋅ T ⇒
286. 66 km M
287. a. n = 0,5 mol m
⇒ 3 ⋅ 8,2 = ⋅ 0, 082 ⋅ 300
b. m = 16 g 44
288. M = 16,0 g · mol–1 m = 44 g CO2

Fórmula de Lewis (estrutura): 303.


H P · V = n · R · T ⇒ 6 · 8,2 = n ·
H CH 0,082 · 400
PV-13-14

H
n = 9 ⋅ 1023 moléculas
O átomo de carbono possui 4 elétrons na camada de valência e
o átomo de hidrogênio tem somente um elétron. 304. C
305. C
289. B 290. A 291. C
306.
292. C
293. a. O2(g) é utilizado na respiração. m
P⋅ V = ⋅ R⋅ T
M
O3(g) nos protege, filtrando os raios utravioleta prove- 0, 8
nientes do Sol. 2 ⋅ 1,12 = ⋅ 0, 082 ⋅ 546
M
b. VO = 0,2 · Var
2
M = 16 g · mol−1
VO = 0,2 · 24,6
2

VO = 4,92 m3 = 4,92 · 103 L MM = 16 u


2

186
Química geral e estequiometria R: Química

307. Dois gases, ocupando o mesmo volume


a. 3, 36 L O3 / m2 nas mesmas condições de pressão e tem-
peratura, apresentam o mesmo número de
P = 1 atm mols (moléculas).
CNTP VM = 22, 4 L / mol
T = 0 °C = 273 K
0,82 ⋅ 30 = nA ⋅ 0,082 ⋅ 300
22, 4 L _________ 1 mol / m2
nA = 1 mol H2 = 1 mol O2
3, 36 L _________ x

x = 0,15 mol / m2 mO = 32 g
2

313. A
b. 1 átomoCl _________ 106 moléculas O3
314. C
1 mol Cl _________ 10 5 molO
3
315. Antes do acidente: P1 · 12.300 =
35, 5 g _________ 105 mol n1 · 0,082 · 250
2 Após o acidente: P2 · 12.300 = n2 · 0,082 · 250
x _________ ⋅ 0,15 mol
3
O número de moléculas que escaparam
35, 5 · 2 · 0,15
x= exercia uma pressão de 0,6 atm (exatamen-
105 ⋅ 3 te a queda de pressão observada), Portan-
x = 3, 55 ⋅ 10 −5 g Cl to:
(P1 – P2) 12.300 = (n1 – n2) · 0,082 · 250
308. D
0,6 · 12.300 = (n1 – n2) · 0,082 · 250
309.
a. 1 cm3 -------- 0,81 g n1 – n2 = 360 mol (perdidos)
1000 cm3 -------- x x = 810 g N2 1 mol C2H6 30 g
360 mol x
b. P ⋅ V = m ⋅ R ⋅ T
M
x = 10.800 g ou 10,8 kg
810 ⋅ 0,082 ⋅ 300
P= = 23,7 atm 316. A
28 ⋅ 30
P ′ = 1 + 23,7 = 24,7 atm 317. C
318. B
310. A
319.
311.
PA · VA = nO · R · TA ⇒ PA · VA = 0,125 · R · TA VB = ?
2 1,6 g CH4
PV-13-14

4 3 8,2L
PB · VB = nN · R · TB ⇒ · PA · VA = nN · R · TA 300 K
2 5 4 2

4 P · V 0 , 125 · R · T

A A
=
A P ⋅ V = P′ ⋅ V ′
5 3 nN · R · TA 0, 3 ⋅ 8,2 = 0,2 ⋅ (8,2 + VB )
PA · VA
4
2

12 1, 64 + 0,2 ⋅ VB = 2, 46
nN = · 0,125
2 20 0,2 ⋅ VB = 0, 82
nN = 0, 075 mol = 2,1 g de N2 VB = 4 ,1 L
2

P ⋅ V = n⋅R ⋅ T
312.
m ⋅ R ⋅ T 1, 6 ⋅ 0, 082 ⋅ 300
PA ⋅ VA = nA ⋅ R ⋅ TA  P= = = 0, 3 atm
M⋅ V 16 ⋅ 8,2

PB ⋅ VB = nB ⋅ R ⋅ TB 

187
Química R: Química geral e estequiometria

320. a. Como na estratosfera a pressão é menor, uma mesma b.


quantidade de gás vai ocupar um volume maior.
MX 58
P·V=n·R·T dX , H = = = 29
2 MH 2
b. Cálculo considerando as condições na superfície terrestre: 2

P = 1 atm MX 58
V = área ⋅ altura = 150 ⋅ 106 km2 ⋅ 3 ⋅ 10 −6 km (3mm)
c. dX , ar = = =2
Mar 29
V = 450 km3 = 4 , 5 ⋅ 1014 L
P ⋅ V = n⋅R ⋅ T 324. T = 300 °C + 273 = 573 K
1 ⋅ 4 , 5 ⋅ 1014 = n ⋅ 0, 082 ⋅ 293 d = 0,78 g/cm3
n = 1, 87 ⋅ 1013 mol
1 mol O3 ______ 48 g 
 x ≅ 9 ⋅ 1014 g = 9 ⋅ 1011 kgO3
1, 87 ⋅ 1013 mol O3 ______ x 

321.

M 28
a. d = = = 1,25 g / L P ≅ 2.036 atm
22, 4 22, 4
325.
P ⋅ M 600 ⋅ 44
b. d = = = 1, 41 g / L P· V = n·R· T
R ⋅ T 62, 3 ⋅ 300
P· V 0, 004 · 4 ,1·104
n= =
MCH4 16 R· T 0, 082· 300
c. d = = =8
CH 4 , H2 MH2 2 n = 6, 67mols
1min_______ 0, 6 mol
MAr 40
d. dAr , ar = = = 1, 38 x _______ 6, 67mols
Mar 29
x = 11,1min
322. a. d = M M = 2, 5·22, 4 = 56 g/mol
22, 4 326. a. O gás metano apre-
MM = 56 u senta menor massa molar
P ·M 2· 56 (16,04 g · mol–1), assim, a sua
b. d = = = 2 , 5 g /L densidade relativa indicará
R· T 0, 082· 546
que ele é menos denso que o
MA 56 ar atmosférico e irá ascender
c. dA ,He = = = 14 na presença deste.
MHe 4
PV-13-14

dCH /dar = MCH4 / Mar


4
M 56 dCH4/dar = 16,04 / 28,8
d. dA , ar = A = = 1, 93
Mar 29 dCH4/dar = 0,56
b No ar atmosférico
323. a. (ambiente aberto), a mistu-
ra é possível, porém, como o
P ⋅M 0, 82 ⋅ M
d= ∴1, 45 = metano é menos denso, ele irá
R⋅T 0, 082 ⋅ 400 ascender, e a combinação ne-
M = 58 g/mol cessária para a explosão (de 5
a 15%) não será atingida.
MM = 58 u
327. 24 (08 + 16)

188
Química geral e estequiometria R: Química

328. Cálculo da densidade interna do gás: b. A combustão do gás hidrogênio libera


calor, logo é um processo exotérmico. Ao ana-
P ⋅M lisarmos a equação geral dos gases (P · V)/T
d= , sendo P = 1 atm; M = 28 g/mol;
R⋅T = constante), percebemos que o aumento da
R = 0,082 atm · L/K · mol e T = 373 K temperatura, num volume constante, provo-
d = 0,915 g/L ca o aumento de pressão. A explosão ocorre
quando a pressão interna provoca o rompi-
Cálculo da densidade externa do gás: mento da casca do ovo.
P ⋅M 339. A
d= , sendo P = 1 atm; M = 28 g/mol;
R⋅T 340. C
R = 0,082 atm · L/K · mol e T = 298 K
d = 1,15 g/L 341. E
Como a densidade do gás é menor no in- 342. PT · VT = PH2 · VH2 + Pco2 · Vco2
terior do balão, ele vai flutuar. PT · 7 = 4 · 2 + 3 · 6
329. 05 PT = 3,7 atm
330. 343. E
m P.V.M 344. E
a. P.V = . R .T T=
M m.R
1.90 . 10 .30
-3 345. P V = P V + P ⋅ V
T= = 439 K = 166ºC n n co co He
2 He2

75. 10 −3 . 0, 082 Pn ⋅ 10 = 2 ⋅ 5 + 4 ⋅ 10

P.V.M Pn = 5 atm
m
b. P.V = . R.T m=
M R.T
1.600 .10 −3 . 30 346. PV = ΣnR.T
m= = 0, 73 g
0, 082.300 PV 6.82
Σn = =
RT 0, 082.300
331. D
10mol CH4  m
332. A Σn = 20mol  n = M
10mol Etano (C H )
2 6 
333. a. Por que ocorre aquecimento do ar no
interior do balão, diminuindo a densidade.
CH4: m = 10 . 16 = 160 g
b. Não. Á medida que o balões sobem,
há explosão da bexiga, pois a pressão externa C2H6: m = 10 . 30 = 300 g
diminui. mT = 160 + 300 = 460 g
334. A 347.
PV-13-14

335. B a.
336. B
337. a. a massa molar do CO2 é maior que a
do ar.
b. x = 133 L
c. O aumento da concentração de oxigê-
nio desloca o equilíbrio no sentido de forma-
ção de maus produto.
338. a. A equação química correspondente é:
2 H2(g) + O2(g) → 2 H2O(v)

189
Química R: Química geral e estequiometria

b. Mistura dos dois: 357. Soma = 21


V = 10 L 01. Correto
02. Incorreto. O N2 é componente ma-
joritário do ar (≅ 78%).
04. Correto
08. Incorreto. 1 mol CO2 ___ 44 g CNTP
2 · 1 + 3 · 3 = Pm · 10 22,4 L

16. 358. B 360. B


Correto 359. A 361. A
348. B 362. C
pO2 0,2
349. 363. pO = PT ⋅ XO XO = = = 0,08
2 2 2 PT 2,5
364. PO2 = XO2 · P
0,21 = xO2 · 8,38
PV=nRT XO2 = 0,025
P · 8 = 0,3 · 0,082 · 300 %molar = 2,5%
P = 0,92 atm
365. V, F, F, V, F
350. B (V)
351. A (F) Tal afirmação refere-se à lei de Char-
352. B les – Gay Lussac.
353.
(F) A fração em volume do nitrogênio é
a. Total: 1 mol mistura gases CNTP 22,4 L
78
nHe + nH = 1,0 igual a .
2 100
x = 1 g H2 (V)
(F) R é denominado constante univer-
sal dos gases.
b.
366. C
354. C 367. D
355. D 368. V = 9 L
356. V, F, F, V, F P ⋅ V = ∑n ⋅ R ⋅ T
PV-13-14

(V) ∑n ⋅ R ⋅ T
V=
(F) Tal afirmação refere-se à lei de Charles – P
Gay Lussac. m
n=
(F) A fração em volume do nitrogênio é igual M
78 ∑n = nCO2 + nH2 = 1 + 3 = 4 mols
a
100
(V) 4 ⋅ 0, 082 ⋅ 300
(F) R é denominado constante universal dos V= = 12 L
8,2
gases.
3
VH = V ⋅ xH = 12 ⋅ = 9 L
2 2 4

190
Química geral e estequiometria R: Química

369. C 385. D
370. C 386. E
371. A 387. A
372. E 388. 26 (02 + 08 + 16)
373.D 389. A
374. D 390. B
375. E 391. A
376. B 392. B
377. E 393. B
378. D 394. a. 1,6
379. a. P(atmosférica) no interior da aeronave: b. NH3 borbulhado em
água destilada fará com que a
2,2 · 104 Pa = pressão de O2. mesma apresente caráter bá-
Valor aceitável: 1 · 104 Pa a 6 · 104 Pa sico, uma vez que forma-se o
Como a pressão do O2 no interior da aeronave está hidróxido de amônio.
dentro da faixa aceitável (1 · 104 Pa < P(O2) < 6 · 104 Pa), o CO2 borbulhado em água desti-
piloto poderá sobreviver. lada fará com que a mesma apre-
b. P(total) 40 m = 5 · P(atmosférica) na superfície sente caráter ácido, uma vez que
forma-se o ácido carbônico.
P(O2) superfície (pressão parcial = 2,1 · 10 Pa (21% em
4

Volume) 395. 19396.


21% Vol ––– 2,1 · 104 Pa 397.
100% Vol ––– P atm (superfície) Isótopo leve = V1
P atm (superfície) = 1 · 105 Pa Isótopo pesado = V2
Na profundidade de 40 m, P(total) = 5 · 10 Pa5 V1 M2
=
P(O2) (pressão parcial) ––– 2,1 · 10 Pa
4 V2 M1
X
5 · 105 Pa ––– 100% em volume 1,0043=
349
2,1 · 104 Pa ––– x X=352,0078 g/mol
x = 4,2% em volume
398. C
380. a. Se as moléculas não se atraem nem se repelem, então a
pressão exercida pelas moléculas de um gás não é afetada pela 399. E
PV-13-14

presença de outro gás. Consequentemente, a pressão total é 400. a. Pela lei de Graham, a
dada pela soma das pressões parciais dos gases, isto é, a pressão velocidade de difusão de um
total é o resultado da força exercida nas paredes pelo choque gás é inversamente proporcio-
das moléculas de todos os gases. nal à raiz quadrada de sua mas-
b. Um aumento da temperatura implica o aumento da ener- sa molar. Assim, como o anel
gia cinética média das moléculas. Então, se o gás for aquecido, as de sólido branco (NH4Cl) foi for-
mado a 6 cm da extremidade
moléculas irão colidir com as paredes do recipiente mais frequen- que apresenta o chumaço que
temente e com mais força e a pressão aumentará. liberava HCI(g), verifica-se que a
381. D amônia percorreu 9 cm ao lon-
go do tubo de vidro enquanto
382. D o HCl percorreu apenas 6 cm.
383. 23 (01 + 02 + 04 + 16) Conclui-se, então, que a amô-
nia tem maior velocidade de di-
384. D fusão por apresentar, portanto,
menor massa molar.

191
Química R: Química geral e estequiometria

b. A partir das equações da energia cinética dos gases: 413.


a. ½ N2 + O2 → NO2
m ⋅ v2
Ec = e Ec = x ⋅ k ⋅ T , em que x e ksão 14 g 32 g 46 g
2
fatores constantes b. N2 + 2O2 → N2O4
2 N2 + 4 O2 → 2N2O4
m · v2 ↑
x · k· T ↑= ou
2
N2 + 2 O2 → 2 NO2
Logo, repetindo o experimento, porém a uma temperatura mais 2 N2 + 4 O2 → 4 NO2
alta, a velocidade de difusão dos dois gases será maior, acarre-
tando em um tempo necessário menor para a formação do anel. c. 1/2 N2 + O2 → NO2
c. Caso o algodão embebido de solução aquosa de 14 g 32 g 46 g
NH3(g) seja colocado no tubo um pouco antes do algodão que
N2 + O2 → N2O2
libera HCl(g), isso possibilitará que o NH3(g) percorra uma dis-
tância maior até a formação do anel de sólido branco. Assim, 28 g 32 g 60 g
o experimento resultará em um dado de maior velocidade de 3/2 N2 + O2 → N3O2
difusão para a amônia. Como, pela lei de Graham, a veloci-
dade de difusão da amônia é diretamente proporcional à raiz 42 g 32 g 74 g
quadrada da massa molar do HCl, o valor obtido para a massa 414. E
molar do HCl será afetado. 415. D
VNH ↑ MMHCl ↑
3
= 416. a. C2H2(g) + 5/2 O2(g) → 2
VHCl MMNH
3 CO2(g) + H2O( )
b. Sabendo-se que as
Capítulo 05 condições de pressão e tempe-
ratura são as mesmas nos dois
401. C cilindros e que esses cilindros
402. D possuem a mesma capacida-
de, o número de mols também
403. C será o mesmo. Assim, o gás
404. E que apresentar a maior massa
molar possuirá a maior massa.
405. Fe(s) + S(s) → FeS(s)
Dessa forma, o cilindro
21 g + 12 g = 33 g  A possui a maior massa, pois o
 Lei de Lavoisier oxigênio puro (O2 – 32 g/mol)
28 g + 16 g = 44 g 
apresenta maior massa molar
21 12 33 3
} que o ar atmosférico (N2 + O2 –
PV-13-14

= = = Lei de Pr oust
28 16 44 4 média ponderada entre as mas-
sas: 28 g/mol e 32 g/mol).
c. Como a proporção es-
406. A referida observação não contraria a lei de Lavoisier, por- tequiométrica utilizada para os
que o sistema estava aberto. dois casos é a mesma, quando
407. D utilizamos a mistura oxigênio
408. As observações não violam a lei da conservação das mas- puro e acetileno, tem-se uma
sas, pois os sistemas não se encontram fechados. quantidade maior de oxigênio
do que na mistura ar atmos-
409. C férico (oxigênio + nitrogênio
410. D – incombustível) e acetileno.
Assim, quanto maior a quanti-
411. C
dade de oxigênio, maior o ca-
412. B lor liberado na combustão.

192
Química geral e estequiometria R: Química

417. F, V, V, F, V 435. a.
418. 02 Cx Hy Oz
419. D
12x + 1y + 16z = 32,4
420. C
↓ ↓ ↓ ↓
Capítulo 06 37, 5% 12,6% 49,9% 100%
421. D
422. B 32,4 _____ 100% 37, 5 ⋅ 32, 4
Cx  x= ∴ x=1
423. a. O sal orgânico apre- 12x
_____ 37,5% 100 ⋅12
senta cinco elementos quími-
cos diferentes.
b. 32,4 _____ 100% 32, 4 ⋅ 12, 6
Hy  y= ∴ y =4
 1y
_____ 12,6% 100
58, 5 g 100%
 x = 39, 3% Na
23 g x 
32,4 _____ 100% 32, 4 ⋅ 49, 9
424. A Oz  z= ∴ z=1
16z
_____ 49,9% 16 ⋅ 100
425. C
426. B
427. 06
428. D
429. C
430. B
431. D b.
432. a.
nO = nHC 436. D
2

m m 437. D
=
M M 438. A
1, 31 1, 72 439. D
=
32 M
440. B
M = 42 g / mol
PV-13-14

441. B
MM = 42u
b. 442. a. 1, 1, 1, 2
CH2 = C + 2 ⋅ H = 12 + 2 ⋅ 1 b. 2, 3, 1, 6
CH2 = 14 c. 3, 2, 1, 6
d. 1, 2, 2, 1, 1
42
F.moléc = ⋅ CH2 443. E
14
F.moléc = 3 ⋅ CH2 = C3H6 444. D
433. D 445. E
434. E 446.

193
Química R: Química geral e estequiometria

a. 3 CaO + P2 O5 → Ca3 (PO4 )2 b. 1 mol NH4NO3 1 mol N2O


80g NH4NO3 44g N2O
b. 2 Al(OH)3 + 3 H2 SO4 → 1 Al2 (SO4 )3 + 6 H2 O x 880g N2O
x = 1600g NH4NO3
447. a. 2 K3PO4 + 3 Ca(NO3)2 → 6 KNO3 + 1 Ca3 (PO4)2
b. 1 BaCl2 + 1 Na2SO4 → 2 NaCl + 1 BaSO4
462. C 465. E
448. E
463. E 466. B
449. D
464. A 467. C
450. B
451. B 468.
452. C 2 NaCl 1 Na2 SO4 2 HCl
453. E 2 mol 1 mol 2 mol
454. I. Fe2O3(s) + 3 H2SO4(aq) → Fe2(SO4)3(aq) + 3 H2O() 2 mol x y
II. 2 Al(OH)3(s) + 3 H2SO4(aq) → Al2(SO4)3(aq) + 6 H2O()
∴ x = 3, 5 mols de Na2 SO4
III. 3 CaO(s) + 2 H3PO4(aq) → Ca3(PO4)2(aq) + 2 H2O()
y = 7 mol de HCl
IV. MgCl2(aq) + Na2CO3(aq) → MgCO3(s) 2 NaCl(aq)
455. B 469. D
456. A
470. 1 C6H8 O7 _____ 1 Na3C6H5O7
457. a. O2(g) → 2[O](g)
1 mol _____ 1 mol
2 O2(g) + 2[O](g) → 2 O3(g)
192 g _____ 1 mol
b. De acordo com a equação do item A, a pro-
porção entre O2(g) e [O](g) é de 1 : 1, que se encontra no x _____ 8 mol
gráfico na altitude aproximada de 140 km. x = 1.536 g
458. D 471. E 473. C 475. C
459. E 472. B 474. C 476. E
460.
Reator: Pt 477. D
4 NH3+ 5 O2 → 4 NO + 6 H2O 478. V, V, F, F, F
Torre de oxidação: 479. D 482. D
PV-13-14

2 NO + O2 → 2 NO2 480. A 483. A


Torre de absorção: 481. A 484. E
2 NO2 + H2O → HNO3 + HNO2 (1) 485. a.
3 HNO2 → HNO3 + 2 NO + H2O (2)
1
Somando (1) e (2), temos a equação global: 2 C3H5N3O9 → 6 CO2(g ) + 5H2O(g ) + O2(g)
 2 
3 NO2 + H2O → 2 HNO3 + NO
2 mol ______ 14 , 5mol produto
Devido ao excesso de O2: NO + 0,5 O2 → NO2
2 ⋅ 227 g ______ 14 , 5mol
Capítulo 07 4 , 54 g ______ x
461.
x = 0,145mol produto
a.

194
Química geral e estequiometria R: Química

492.
b. P ⋅ V = n ⋅ R ⋅ T
a.
n ⋅ R ⋅ T 0,145 ⋅ 0, 082 ⋅ 773 C2H5OH() + 3 O2 →
V= =
P 1 → 2 CO2(g) + 3H2O(g)
V = 9,2 L
C 8 H18(l) + 25/2O2(g) →
486. D → 8 CO2(g) + 9 H2O(g)

487. P· V = n · R · T
mol
P· V b. Etanol : 14 L = 14 L · 17, 2 = 240, 8 mols
n= L
R· T C2H5OH → 2 CO2
 
2 · 50
n= = 4 , 06 mol N2 _____
0, 082 · 300 1mol 2 mols
_____
2 mol NaN3 ___ 3 mol N2 240, 8 x
x ___ 4 , 06 mol N2 ⇒ x = 481, 6 mols CO2
mol
x = 2, 7 mol (176 g) n − octano : 10 L = 10 L · 6,15
L
= 61, 5 mols
488. D C8H18 → 8 CO2
 
 
489.
a. CO2(g) + 2 Li(OH)(s) → Li2CO3(s)+H2O() 1mol _____ 8 mols
b. 61, 5 _____ y
⇒ y = 492 mols CO2
P· V = n·R· T Portanto, o n − octano é mais poluente.
P· V 2 · 1 · 60 · 103
n= = ≅ 50 mol CO2
R· T 100.0, 082 ⋅ 293
1mol CO2 ______ 2molsLiOH 493. PV = nRT
1mol CO2 ______ 2·24 g 150 · 82 = n · 0, 082 · 300
n = 500 mol de metano
50 mol CO2 ______ x
500 mol (CH4 ) ______ 100%
x = 2, 4 kgLiOH
x ______ 1%
490. A x = 5 mol de me tan o
CH4 + O2 → H2CO + H2O
491. detanol = 0, 8 g/cm3 = 0, 8 Kg / L
PV-13-14

1 mol ______ 1 mol


m
d= 5 mol ______5 mol
V
m nmetanal = 5 mol
0, 8 = ⇒ 40 Kg etanol
50 nmetanal = 5 · 30 g = 150 g
C 12 H22 O11 + H2O → 4 C2H5OH + 4 CO2
   

1 mol ______ 4 mol


342 g 40 Kg
______

x = 74 ,3
35Kg

195
Química R: Química geral e estequiometria

494. 506. a.
C6H12O6 + 6 O2 →
 6 CO2 + 6 H2O
1h _______ 50 km
1 mol 6 mols
2h ______ x
180 g 6 ⋅ 32 g
x = 100 km x 216 g
180 ⋅ 216
x=
1L etanol _____ 10 km 6 ⋅ 32
x = 202, 5 g < 216 g
y _____ 100 km
∴ C6H12O6 está em excesso.
y = 10 L etanol
O2 é o reagente limit an te.
m
d= b. Massa residual, em gramas, do reagen-
v
∴m = d· v te em excesso.
kg c. 1 C6H12O6 6 CO2
m = 0, 8 · 10 L
L
∴8 kg = 8.000 g 1 mol 6 mols
C2H5OH(l) + 302(gg) → 2 CO2(g) + 3H2O(l) 180 g 6 mols
202, 5 g y
202, 5 ⋅ 6
1molC2H5OH ______ 2molsCO2 y=
180
46 gC2H5OH _____ 2 · 25LCO2 y = 6, 75 mol CO2
8.000 gC2H5OH ____ z P ⋅ V = n⋅R ⋅ T
8.695, 6 P ⋅ 0,2 = 6, 75 ⋅ 0, 082 ⋅ 300
z = 8. 695, 6 L ⇒ v = ≅ 8, 7m3
1.000 P = 830,25 atm
495. D 499. A
507. 01 508. 11
496. A 500. D
509.
497. C 501. B
Ca5(PO4)3F + 5 H2SO4 → 3 H3PO4 + 5 CaSO4 + HF
498. C 502. A
1 mol 5 mol
503. 504 g ------- 5.98 g
Na + H2O → 1 2 H2 + NaOH 50,45 g ------ x
PV-13-14

NaOH + HCl → NaCl + H2 O 50, 45 ⋅ 5 ⋅ 98


x=
504
NaCl + AgNO3 → AgCl + NaNO3
x = 49, 04 g < 98,12
 HCl + AgNO3 → 2 H2 + AgCl
Na +  + NaNO3
1
∴ H2 SO4 está em excesso.
1 mol 1 mol
fluoroapatita (Ca5 (PO4)3F) é o reagente limi-
23 g 143, 5 g tante da reação.
x 14 , 35 g b. Ca5(PO4)3F ------- 3H3PO4
x = 2, 3 g 1 mol 3 mols
504 g ------- 3.98 g
504. A
50,45 g ------- y
505. B

196
Química geral e estequiometria R: Química

50, 45 ⋅ 3 ⋅ 98 537. C
y=
504 538. D
y = 29, 4 g H3PO4 539.

510. D 514. A
511. C 515. B
512. C 516. B
513. D 517. D
518. B
519. a. 3CaCO3(s) + 2H3PO4(aq) →
Ca3(PO4)2(s) 3H2O() + 3CO2(g)
b. CaCO3
540. A
c. 1 mol de Ca3(PO4)2(s)
541. A
520. A
542.
521. 17 kg de P
KIO3(aq) + 3 H2SO3(aq) → KI(aq) + 3 H2SO4(aq)
522. A 525. C
KI(aq) + AgNO3(aq) → AgI(s) + KNO3(aq)
523. A 526. B
524. C 527. D KIO3(aq) + 3 H2SO3(aq) + AgNO3(aq) →
 
528. 1 mol

1 mol NH4NO2 —— 1 mol N2 → AgI(s) + KNO3(aq) + 3 H2SO4(aq)



64 g —— 22,4 L 1 mol

0,8 · 12,8 g —— x 214 g 235 g


x = 4,48 L · 0,8 x 4,7 · 10–5 g
x = 3,584 L N2 x = 4,28 · 10–5 g KIO3/g sal
529. D 4,28 · 10-5 1 g de sal
530. A y 106 g de sal (1 t)
531. y = 4,28 · 101 = 42,8
532. 543. D
PV-13-14

544. B
545.
1 mol ZnS l 1 mol Zn
97 g l 65 g
x l 1 ton
x = 1,49 ton ZnS
533. A 546. C
534. B 547. B
535. D 548. A
536. A

197
Química R: Química geral e estequiometria

549. D 553.
550. A
(3x ) 6C(s) + 3 O2(g) → 6CO(g)
551.
n mol C2H5OH l 1 mol (C2H4)n (2 x ) 2Fe2 O3(s) + 6CO(g) → 4Fe(s) + 6CO2(g)
n ⋅ 46 g l 28 g ⋅ n 6C(s) + 3 O2(g) + 2Fe2 O3(s) → 4Fe(s) + 6CO2(g)
 
x l 5,6 ton 6 mols 4 mols

x = 9,2 ton C2H5OH 6 mol 4 mol


1 L l 0,8⋅103 g 6 ⋅ 12 g 4 ⋅ 56 g
y l 9,2⋅10 g x 1t
y = 11,5 mil litros de C2H5OH x = 0, 32 t = 320 kg
552. 554. E
a. (x 5) 10 NaN → 10 Na + 15 N 555. a. x = 13.600 g ou 13,6 kg
3 2
10 Na + 2 KNO3 → K2O + 5 Na2O + N2 b. x = 60 kg
10 NaN3 + 2 KNO3 → 16 N2 + K2 O + 5 Na2 O 556. D
557. B
10 mols ____ 16 mols 558. D
10 · 65 g ____ 16 · 25 L 559. A
x ____ 80 L 560. D
x = 130 g
b.
Na 2 O(s) + H 2 O() → 2NaOH (aq)
K 2 O(s) + H 2 O() → 2KOH (aq)

PV-13-14

198
Química geral e estequiometria R: Química

ANOTAÇÕES
PV-13-14

199
Química R: Química geral e estequiometria

ANOTAÇÕES

PV-13-14

200
Química geral e estequiometria R: Química

ANOTAÇÕES
PV-13-14

201
Química R: Química geral e estequiometria

ANOTAÇÕES

PV-13-14

202
Química geral e estequiometria R: Química

ANOTAÇÕES
PV-13-14

203
Química R: Química geral e estequiometria

ANOTAÇÕES

PV-13-14

204
Química geral e estequiometria R: Química

ANOTAÇÕES
PV-13-14

205
Química R: Química geral e estequiometria

ANOTAÇÕES

PV-13-14

206
Química geral e estequiometria R: Química

ANOTAÇÕES
PV-13-14

207
Química R: Química geral e estequiometria

ANOTAÇÕES

PV-13-14

208

Você também pode gostar